Vous êtes sur la page 1sur 110

G.R. No. L-18216 October 30, 1962 We agree with the opinion of these two officials.

A corporation is a
juridical person distinct from the members composing it. Properties
registered in the name of the corporation are owned by it as an entity
STOCKHOLDERS OF F. GUANZON AND SONS, INC., petitioners-
separate and distinct from its members. While shares of stock
appellants,
constitute personal property they do not represent property of the
vs.
corporation. The corporation has property of its own which consists
REGISTER OF DEEDS OF MANILA, respondent-appellee.
chiefly of real estate (Nelson v. Owen, 113 Ala., 372, 21 So. 75;
Morrow v. Gould, 145 Iowa 1, 123 N.W. 743). A share of stock only
Ramon C. Fernando for petitioners-appellants. typifies an aliquot part of the corporation's property, or the right to
Office of the Solicitor General for respondent-appellee. share in its proceeds to that extent when distributed according to law
and equity (Hall & Faley v. Alabama Terminal, 173 Ala 398, 56 So.,
235), but its holder is not the owner of any part of the capital of the
BAUTISTA ANGELO, J.:
corporation (Bradley v. Bauder 36 Ohio St., 28). Nor is he entitled to
the possession of any definite portion of its property or assets
On September 19, 1960, the five stockholders of the F. Guanzon and (Gottfried v. Miller, 104 U.S., 521; Jones v. Davis, 35 Ohio St., 474).
Sons, Inc. executed a certificate of liquidation of the assets of the The stockholder is not a co-owner or tenant in common of the
corporation reciting, among other things, that by virtue of a resolution corporate property (Halton v. Hohnston, 166 Ala 317, 51 So 992).
of the stockholders adopted on September 17, 1960, dissolving the
corporation, they have distributed among themselves in proportion to On the basis of the foregoing authorities, it is clear that the act of
their shareholdings, as liquidating dividends, the assets of said
liquidation made by the stockholders of the F. Guanzon and Sons, Inc.
corporation, including real properties located in Manila. of the latter's assets is not and cannot be considered a partition of
community property, but rather a transfer or conveyance of the title of
The certificate of liquidation, when presented to the Register of Deeds its assets to the individual stockholders. Indeed, since the purpose of
of Manila, was denied registration on seven grounds, of which the the liquidation, as well as the distribution of the assets of the
following were disputed by the stockholders: corporation, is to transfer their title from the corporation to the
stockholders in proportion to their shareholdings, — and this is in effect
the purpose which they seek to obtain from the Register of Deeds of
3. The number of parcels not certified to in the Manila, — that transfer cannot be effected without the corresponding
acknowledgment; deed of conveyance from the corporation to the stockholders. It is,
therefore, fair and logical to consider the certificate of liquidation as
5. P430.50 Reg. fees need be paid; one in the nature of a transfer or conveyance.

6. P940.45 documentary stamps need be attached to the WHEREFORE, we affirm the resolution appealed from, with costs
document; against appellants.

7. The judgment of the Court approving the dissolution and


directing the disposition of the assets of the corporation need
be presented (Rules of Court, Rule 104, Sec. 3).

Deciding the consulta elevated by the stockholders, the Commissioner


of Land Registration overruled ground No. 7 and sustained
requirements Nos. 3, 5 and 6.

The stockholders interposed the present appeal.

As correctly stated by the Commissioner of Land Registration, the


propriety or impropriety of the three grounds on which the denial of the
registration of the certificate of liquidation was predicated hinges on
whether or not that certificate merely involves a distribution of the
corporation's assets or should be considered a transfer or conveyance.

Appellants contend that the certificate of liquidation is not a


conveyance or transfer but merely a distribution of the assets of the
corporation which has ceased to exist for having been dissolved. This
is apparent in the minutes for dissolution attached to the document.
Not being a conveyance the certificate need not contain a statement of
the number of parcel of land involved in the distribution in the
acknowledgment appearing therein. Hence the amount of documentary
stamps to be affixed thereon should only be P0.30 and not P940.45, as
required by the register of deeds. Neither is it correct to require
appellants to pay the amount of P430.50 as registration fee.

The Commissioner of Land Registration, however, entertained a


different opinion. He concurred in the view expressed by the register of
deed to the effect that the certificate of liquidation in question, though it
involves a distribution of the corporation's assets, in the last analysis
represents a transfer of said assets from the corporation to the
stockholders. Hence, in substance it is a transfer or conveyance.

1
G.R. No. L-48627 It would appear from the above justification that the petitioners were
not really involved in the initial steps that finally led to the incorporation
of the Filipinas Orient Airways. Elsewhere in the decision, Barretto was
FERMIN Z. CARAM, JR. and ROSA O. DE CARAM, petitioners
described as "the moving spirit." The finding of the respondent court is
vs.
that the project study was undertaken by the private respondent at the
THE HONORABLE COURT OF APPEALS and ALBERTO V.
request of Barretto and Garcia who, upon its completion, presented it
ARELLANO, respondents.
to the petitioners to induce them to invest in the proposed airline. The
study could have been presented to other prospective investors. At any
rate, the airline was eventually organized on the basis of the project
study with the petitioners as major stockholders and, together with
Barretto and Garcia, as principal officers.
CRUZ, J.:
The following portion of the decision in question is also worth
considering:
We gave limited due course to this petition on the question of the
solidary liability of the petitioners with their co-defendants in the lower
court 1 because of the challenge to the following paragraph in the ... Since defendant Barretto was the moving spirit in the pre-
dispositive portion of the decision of the respondent court: * organization work of defendant corporation based on his
experience and expertise, hence he was logically
compensated in the amount of P200,000.00 shares of stock
1. Defendants are hereby ordered to jointly and severally pay
not as industrial partner but more for his technical services
the plaintiff the amount of P50,000.00 for the preparation of that brought to fruition the defendant corporation. By the
the project study and his technical services that led to the same token, We find no reason why the plaintiff should not
organization of the defendant corporation, plus P10,000.00
be similarly compensated not only for having actively
attorney's fees; 2 participated in the preparation of the project study for several
months and its subsequent revision but also in his having
The petitioners claim that this order has no support in fact and law been involved in the pre-organization of the defendant
because they had no contract whatsoever with the private respondent corporation, in the preparation of the franchise, in inviting the
regarding the above-mentioned services. Their position is that as mere interest of the financiers and in the training and screening of
subsequent investors in the corporation that was later created, they personnel. We agree that for these special services of the
should not be held solidarily liable with the Filipinas Orient Airways, a plaintiff the amount of P50,000.00 as compensation is
separate juridical entity, and with Barretto and Garcia, their co- reasonable. 5
defendants in the lower court, ** who were the ones who requested the
said services from the private respondent. 3 The above finding bolsters the conclusion that the petitioners were not
involved in the initial stages of the organization of the airline, which
We are not concerned here with the petitioners' co-defendants, who were being directed by Barretto as the main promoter. It was he who
have not appealed the decision of the respondent court and may, for was putting all the pieces together, so to speak. The petitioners were
this reason, be presumed to have accepted the same. For purposes of merely among the financiers whose interest was to be invited and who
resolving this case before us, it is not necessary to determine whether were in fact persuaded, on the strength of the project study, to invest in
it is the promoters of the proposed corporation, or the corporation itself the proposed airline.
after its organization, that shall be responsible for the expenses
incurred in connection with such organization. Significantly, there was no showing that the Filipinas Orient Airways
was a fictitious corporation and did not have a separate juridical
The only question we have to decide now is whether or not the personality, to justify making the petitioners, as principal stockholders
petitioners themselves are also and personally liable for such thereof, responsible for its obligations. As a bona fide corporation, the
expenses and, if so, to what extent. Filipinas Orient Airways should alone be liable for its corporate acts as
duly authorized by its officers and directors.
The reasons for the said order are given by the respondent court in its
decision in this wise: In the light of these circumstances, we hold that the petitioners cannot
be held personally liable for the compensation claimed by the private
respondent for the services performed by him in the organization of the
As to the 4th assigned error we hold that as to the corporation. To repeat, the petitioners did not contract such services. It
remuneration due the plaintiff for the preparation of the was only the results of such services that Barretto and Garcia
project study and the pre-organizational services in the presented to them and which persuaded them to invest in the proposed
amount of P50,000.00, not only the defendant corporation airline. The most that can be said is that they benefited from such
but the other defendants including defendants Caram should services, but that surely is no justification to hold them personally liable
be jointly and severally liable for this amount. As we above therefor. Otherwise, all the other stockholders of the corporation,
related it was upon the request of defendants Barretto and including those who came in later, and regardless of the amount of
Garcia that plaintiff handled the preparation of the project their share holdings, would be equally and personally liable also with
study which project study was presented to defendant the petitioners for the claims of the private respondent.
Caram so the latter was convinced to invest in the proposed
airlines. The project study was revised for purposes of
presentation to financiers and the banks. It was on the basis The petition is rather hazy and seems to be flawed by an ambiguous
of this study that defendant corporation was actually ambivalence. Our impression is that it is opposed to the imposition of
organized and rendered operational. Defendants Garcia and solidary responsibility upon the Carams but seems to be willing, in a
Caram, and Barretto became members of the Board and/or vague, unexpressed offer of compromise, to accept joint liability. While
officers of defendant corporation. Thus, not only the it is true that it does here and there disclaim total liability, the thrust of
defendant corporation but all the other defendants who were the petition seems to be against the imposition of solidary liability only
involved in the preparatory stages of the incorporation, who rather than against any liability at all, which is what it should have
caused the preparation and/or benefited from the project categorically argued.
study and the technical services of plaintiff must be liable. 4
Categorically, the Court holds that the petitioners are not liable at all,
jointly or jointly and severally, under the first paragraph of the

2
dispositive portion of the challenged decision. So holding, we find it
unnecessary to examine at this time the rules on solidary obligations,
which the parties-needlessly, as it turns out have belabored unto
death.

WHEREFORE, the petition is granted. The petitioners are declared not


liable under the challenged decision, which is hereby modified
accordingly. It is so ordered.

3
G.R. No. 124293 January 31, 2005 At the pre-bidding conference held on September 18, 1993, interested
bidders were given copies of the JVA between NIDC and KAWASAKI,
and of the Asset Specific Bidding Rules (ASBR) drafted for the
J.G. SUMMIT HOLDINGS, INC., petitioner,
National Government's 87.6% equity share in PHILSECO. The
vs.
provisions of the ASBR were explained to the interested bidders who
COURT OF APPEALS; COMMITTEE ON PRIVATIZATION, its
were notified that the bidding would be held on December 2, 1993. A
Chairman and Members; ASSET PRIVATIZATION TRUST; and
portion of the ASBR reads:
PHILYARDS HOLDINGS, INC., respondents.

1.0 The subject of this Asset Privatization Trust (APT) sale through
RESOLUTION
public bidding is the National Government's equity in PHILSECO
consisting of 896,869,942 shares of stock (representing 87.67% of
PUNO, J.: PHILSECO's outstanding capital stock), which will be sold as a whole
block in accordance with the rules herein enumerated.
For resolution before this Court are two motions filed by the petitioner,
J.G. Summit Holdings, Inc. for reconsideration of our Resolution dated xxx xxx xxx
September 24, 2003 and to elevate this case to the Court En Banc.
The petitioner questions the Resolution which reversed our Decision of
2.0 The highest bid, as well as the buyer, shall be subject to the final
November 20, 2000, which in turn reversed and set aside a Decision of
approval of both the APT Board of Trustees and the Committee on
the Court of Appeals promulgated on July 18, 1995.
Privatization (COP).

I. Facts
2.1 APT reserves the right in its sole discretion, to reject any or all bids.

The undisputed facts of the case, as set forth in our Resolution of


3.0 This public bidding shall be on an Indicative Price Bidding basis.
September 24, 2003, are as follows:
The Indicative price set for the National Government's 87.67% equity in
PHILSECO is PESOS: ONE BILLION THREE HUNDRED MILLION
On January 27, 1997, the National Investment and Development (₱1,300,000,000.00).
Corporation (NIDC), a government corporation, entered into a Joint
Venture Agreement (JVA) with Kawasaki Heavy Industries, Ltd. of
xxx xxx xxx
Kobe, Japan (KAWASAKI) for the construction, operation and
management of the Subic National Shipyard, Inc. (SNS) which
subsequently became the Philippine Shipyard and Engineering 6.0 The highest qualified bid will be submitted to the APT Board of
Corporation (PHILSECO). Under the JVA, the NIDC and KAWASAKI Trustees at its regular meeting following the bidding, for the purpose of
will contribute ₱330 million for the capitalization of PHILSECO in the determining whether or not it should be endorsed by the APT Board of
proportion of 60%-40% respectively. One of its salient features is the Trustees to the COP, and the latter approves the same. The APT shall
grant to the parties of the right of first refusal should either of them advise Kawasaki Heavy Industries, Inc. and/or its nominee,
decide to sell, assign or transfer its interest in the joint venture, viz: [PHILYARDS] Holdings, Inc., that the highest bid is acceptable to the
National Government. Kawasaki Heavy Industries, Inc. and/or
[PHILYARDS] Holdings, Inc. shall then have a period of thirty (30)
1.4 Neither party shall sell, transfer or assign all or any part of its
calendar days from the date of receipt of such advice from APT within
interest in SNS [PHILSECO] to any third party without giving the other
which to exercise their "Option to Top the Highest Bid" by offering a bid
under the same terms the right of first refusal. This provision shall not
equivalent to the highest bid plus five (5%) percent thereof.
apply if the transferee is a corporation owned or controlled by the
GOVERNMENT or by a KAWASAKI affiliate.
6.1 Should Kawasaki Heavy Industries, Inc. and/or [PHILYARDS]
Holdings, Inc. exercise their "Option to Top the Highest Bid," they shall
On November 25, 1986, NIDC transferred all its rights, title and interest
so notify the APT about such exercise of their option and deposit with
in PHILSECO to the Philippine National Bank (PNB). Such interests
APT the amount equivalent to ten percent (10%) of the highest bid plus
were subsequently transferred to the National Government pursuant to
five percent (5%) thereof within the thirty (30)-day period mentioned in
Administrative Order No. 14. On December 8, 1986, President
paragraph 6.0 above. APT will then serve notice upon Kawasaki Heavy
Corazon C. Aquino issued Proclamation No. 50 establishing the
Industries, Inc. and/or [PHILYARDS] Holdings, Inc. declaring them as
Committee on Privatization (COP) and the Asset Privatization Trust
the preferred bidder and they shall have a period of ninety (90) days
(APT) to take title to, and possession of, conserve, manage and
from the receipt of the APT's notice within which to pay the balance of
dispose of non-performing assets of the National Government.
their bid price.
Thereafter, on February 27, 1987, a trust agreement was entered into
between the National Government and the APT wherein the latter was
named the trustee of the National Government's share in PHILSECO. 6.2 Should Kawasaki Heavy Industries, Inc. and/or [PHILYARDS]
In 1989, as a result of a quasi-reorganization of PHILSECO to settle its Holdings, Inc. fail to exercise their "Option to Top the Highest Bid"
huge obligations to PNB, the National Government's shareholdings in within the thirty (30)-day period, APT will declare the highest bidder as
PHILSECO increased to 97.41% thereby reducing KAWASAKI's the winning bidder.
shareholdings to 2.59%.
xxx xxx xxx
In the interest of the national economy and the government, the COP
and the APT deemed it best to sell the National Government's share in
12.0 The bidder shall be solely responsible for examining with
PHILSECO to private entities. After a series of negotiations between
appropriate care these rules, the official bid forms, including any
the APT and KAWASAKI, they agreed that the latter's right of first
addenda or amendments thereto issued during the bidding period. The
refusal under the JVA be "exchanged" for the right to top by five
bidder shall likewise be responsible for informing itself with respect to
percent (5%) the highest bid for the said shares. They further agreed
any and all conditions concerning the PHILSECO Shares which may,
that KAWASAKI would be entitled to name a company in which it was
in any manner, affect the bidder's proposal. Failure on the part of the
a stockholder, which could exercise the right to top. On September 7,
bidder to so examine and inform itself shall be its sole risk and no relief
1990, KAWASAKI informed APT that Philyards Holdings, Inc.
for error or omission will be given by APT or COP. . . .
(PHI)1 would exercise its right to top.

4
At the public bidding on the said date, petitioner J.G. Summit Holdings, inequitable provisions thereof." Thus, this Court voided the transfer of
Inc.2 submitted a bid of Two Billion and Thirty Million Pesos the national government's 87.67% share in PHILSECO to Philyard[s]
(₱2,030,000,000.00) with an acknowledgment of Holdings, Inc., and upheld the right of JG Summit, as the highest
KAWASAKI/[PHILYARDS'] right to top, viz: bidder, to take title to the said shares, viz:

4. I/We understand that the Committee on Privatization (COP) has up WHEREFORE, the instant petition for review on certiorari is
to thirty (30) days to act on APT's recommendation based on the result GRANTED. The assailed Decision and Resolution of the Court of
of this bidding. Should the COP approve the highest bid, APT shall Appeals are REVERSED and SET ASIDE. Petitioner is ordered to pay
advise Kawasaki Heavy Industries, Inc. and/or its nominee, to APT its bid price of Two Billion Thirty Million Pesos
[PHILYARDS] Holdings, Inc. that the highest bid is acceptable to the (₱2,030,000,000.00), less its bid deposit plus interests upon the finality
National Government. Kawasaki Heavy Industries, Inc. and/or of this Decision. In turn, APT is ordered to:
[PHILYARDS] Holdings, Inc. shall then have a period of thirty (30)
calendar days from the date of receipt of such advice from APT within
(a) accept the said amount of ₱2,030,000,000.00 less bid
which to exercise their "Option to Top the Highest Bid" by offering a bid
deposit and interests from petitioner;
equivalent to the highest bid plus five (5%) percent thereof.

(b) execute a Stock Purchase Agreement with petitioner;


As petitioner was declared the highest bidder, the COP approved the
sale on December 3, 1993 "subject to the right of Kawasaki Heavy
Industries, Inc./[PHILYARDS] Holdings, Inc. to top JGSMI's bid by 5% (c) cause the issuance in favor of petitioner of the certificates
as specified in the bidding rules." of stocks representing 87.6% of PHILSECO's total
capitalization;
On December 29, 1993, petitioner informed APT that it was protesting
the offer of PHI to top its bid on the grounds that: (a) the (d) return to private respondent PHGI the amount of Two
KAWASAKI/PHI consortium composed of KAWASAKI, [PHILYARDS], Billion One Hundred Thirty-One Million Five Hundred
Mitsui, Keppel, SM Group, ICTSI and Insular Life violated the ASBR Thousand Pesos (₱2,131,500,000.00); and
because the last four (4) companies were the losing bidders thereby
circumventing the law and prejudicing the weak winning bidder; (b)
only KAWASAKI could exercise the right to top; (c) giving the same (e) cause the cancellation of the stock certificates issued to
PHI.
option to top to PHI constituted unwarranted benefit to a third party; (d)
no right of first refusal can be exercised in a public bidding or auction
sale; and (e) the JG Summit consortium was not estopped from SO ORDERED.
questioning the proceedings.
In separate Motions for Reconsideration, respondents submit[ted] three
On February 2, 1994, petitioner was notified that PHI had fully paid the basic issues for x x x resolution: (1) Whether PHILSECO is a public
balance of the purchase price of the subject bidding. On February 7, utility; (2) Whether under the 1977 JVA, KAWASAKI can exercise its
1994, the APT notified petitioner that PHI had exercised its option to right of first refusal only up to 40% of the total capitalization of
top the highest bid and that the COP had approved the same on PHILSECO; and (3) Whether the right to top granted to KAWASAKI
January 6, 1994. On February 24, 1994, the APT and PHI executed a violates the principles of competitive bidding.3 (citations omitted)
Stock Purchase Agreement. Consequently, petitioner filed with this
Court a Petition for Mandamus under G.R. No. 114057. On May 11,
1994, said petition was referred to the Court of Appeals. On July 18, In a Resolution dated September 24, 2003, this Court ruled in favor of
1995, the Court of Appeals denied the same for lack of merit. It ruled the respondents. On the first issue, we held that Philippine Shipyard
that the petition for mandamus was not the proper remedy to question and Engineering Corporation (PHILSECO) is not a public utility, as by
the constitutionality or legality of the right of first refusal and the right to nature, a shipyard is not a public utility4 and that no law declares a
top that was exercised by KAWASAKI/PHI, and that the matter must be shipyard to be a public utility.5 On the second issue, we found nothing
brought "by the proper party in the proper forum at the proper time and in the 1977 Joint Venture Agreement (JVA) which prevents Kawasaki
threshed out in a full blown trial." The Court of Appeals further ruled Heavy Industries, Ltd. of Kobe, Japan (KAWASAKI) from acquiring
that the right of first refusal and the right to top are prima facie legal more than 40% of PHILSECO’s total capitalization.6 On the final issue,
and that the petitioner, "by participating in the public bidding, with full we held that the right to top granted to KAWASAKI in exchange for its
knowledge of the right to top granted to KAWASAKI/[PHILYARDS] right of first refusal did not violate the principles of competitive bidding. 7
is…estopped from questioning the validity of the award given to
[PHILYARDS] after the latter exercised the right to top and had paid in On October 20, 2003, the petitioner filed a Motion for
full the purchase price of the subject shares, pursuant to the ASBR." Reconsideration8 and a Motion to Elevate This Case to the Court En
Petitioner filed a Motion for Reconsideration of said Decision which Banc.9 Public respondents Committee on Privatization (COP) and
was denied on March 15, 1996. Petitioner thus filed a Petition for Asset Privatization Trust (APT), and private respondent Philyards
Certiorari with this Court alleging grave abuse of discretion on the part Holdings, Inc. (PHILYARDS) filed their Comments on J.G. Summit
of the appellate court. Holdings, Inc.’s (JG Summit’s) Motion for Reconsideration and Motion
to Elevate This Case to the Court En Banc on January 29, 2004 and
On November 20, 2000, this Court rendered x x x [a] Decision ruling February 3, 2004, respectively.
among others that the Court of Appeals erred when it dismissed the
petition on the sole ground of the impropriety of the special civil action II. Issues
of mandamus because the petition was also one of certiorari. It further
ruled that a shipyard like PHILSECO is a public utility whose
capitalization must be sixty percent (60%) Filipino-owned. Based on the foregoing, the relevant issues to resolve to end this
Consequently, the right to top granted to KAWASAKI under the Asset litigation are the following:
Specific Bidding Rules (ASBR) drafted for the sale of the 87.67%
equity of the National Government in PHILSECO is illegal — not only 1. Whether there are sufficient bases to elevate the case at
because it violates the rules on competitive bidding — but more so, bar to the Court en banc.
because it allows foreign corporations to own more than 40% equity in
the shipyard. It also held that "although the petitioner had the
opportunity to examine the ASBR before it participated in the bidding, it
cannot be estopped from questioning the unconstitutional, illegal and

5
2. Whether the motion for reconsideration raises any new and whether they were complied with in the case at bar. 16 This Court
matter or cogent reason to warrant a reconsideration of this categorically ruled on the petitioner’s argument that PHILSECO, as a
Court’s Resolution of September 24, 2003. shipyard, is a public utility which should maintain a 60%-40% Filipino-
foreign equity ratio, as it was a pivotal issue. In doing so, we
recognized the impact of our ruling on the shipbuilding industry which
Motion to Elevate this Case to the
was beyond avoidance.17

Court En Banc
We reject petitioner’s argument that the present case may be
considered under the Supreme Court Resolution dated February 23,
The petitioner prays for the elevation of the case to the Court en 1984 which included among en banc cases those involving a novel
banc on the following grounds: question of law and those where a doctrine or principle laid down by
the court en banc or in division may be modified or reversed. The case
was resolved based on basic principles of the right of first refusal in
1. The main issue of the propriety of the bidding process
commercial law and estoppel in civil law. Contractual obligations
involved in the present case has been confused with the arising from rights of first refusal are not new in this jurisdiction and
policy issue of the supposed fate of the shipping industry have been recognized in numerous cases.18 Estoppel is too known a
which has never been an issue that is determinative of this
civil law concept to require an elongated discussion. Fundamental
case.10 principles on public bidding were likewise used to resolve the issues
raised by the petitioner. To be sure, petitioner leans on the right to top
2. The present case may be considered under the Supreme in a public bidding in arguing that the case at bar involves a novel
Court Resolution dated February 23, 1984 which included issue. We are not swayed. The right to top was merely a condition or a
among en banc cases those involving a novel question of reservation made in the bidding rules which was fully disclosed to all
law and those where a doctrine or principle laid down by the bidding parties. In Bureau Veritas, represented by Theodor H.
Court en banc or in division may be modified or reversed.11 Hunermann v. Office of the President, et al., 19 we dealt with this
conditionality, viz:
3. There was clear executive interference in the judicial
functions of the Court when the Honorable Jose Isidro x x x It must be stressed, as held in the case of A.C. Esguerra & Sons
Camacho, Secretary of Finance, forwarded to Chief Justice v. Aytona, et al., (L-18751, 28 April 1962, 4 SCRA 1245), that in an
Davide, a memorandum dated November 5, 2001, attaching "invitation to bid, there is a condition imposed upon the bidders to
a copy of the Foreign Chambers Report dated October 17, the effect that the bidding shall be subject to the right of the
2001, which matter was placed in the agenda of the Court government to reject any and all bids subject to its discretion. In
and noted by it in a formal resolution dated November 28, the case at bar, the government has made its choice and unless
2001.12 an unfairness or injustice is shown, the losing bidders have no
cause to complain nor right to dispute that choice. This is a well-
settled doctrine in this jurisdiction and elsewhere."
Opposing J.G. Summit’s motion to elevate the case en banc,
PHILYARDS points out the petitioner’s inconsistency in
previously opposing PHILYARDS’ Motion to Refer the Case to the The discretion to accept or reject a bid and award contracts is vested in
Court En Banc. PHILYARDS contends that J.G. Summit should now be the Government agencies entrusted with that function. The discretion
estopped from asking that the case be referred to the Court en banc. given to the authorities on this matter is of such wide latitude that the
PHILYARDS further contends that the Supreme Court en banc is not Courts will not interfere therewith, unless it is apparent that it is used as
an appellate court to which decisions or resolutions of its divisions may a shield to a fraudulent award (Jalandoni v. NARRA, 108 Phil. 486
be appealed citing Supreme Court Circular No. 2-89 dated February 7, [1960]). x x x The exercise of this discretion is a policy decision that
1989.13 PHILYARDS also alleges that there is no novel question of law necessitates prior inquiry, investigation, comparison, evaluation, and
involved in the present case as the assailed Resolution was based on deliberation. This task can best be discharged by the Government
well-settled jurisprudence. Likewise, PHILYARDS stresses that the agencies concerned, not by the Courts. The role of the Courts is to
Resolution was merely an outcome of the motions for reconsideration ascertain whether a branch or instrumentality of the Government has
filed by it and the COP and APT and is "consistent with the inherent transgressed its constitutional boundaries. But the Courts will not
power of courts to ‘amend and control its process and orders so as to interfere with executive or legislative discretion exercised within those
make them conformable to law and justice.’ (Rule 135, sec. boundaries. Otherwise, it strays into the realm of policy decision-
5)"14 Private respondent belittles the petitioner’s allegations regarding making.
the change in ponente and the alleged executive interference as
shown by former Secretary of Finance Jose Isidro Camacho’s
It is only upon a clear showing of grave abuse of discretion that the
memorandum dated November 5, 2001 arguing that these do not Courts will set aside the award of a contract made by a government
justify a referral of the present case to the Court en banc. entity. Grave abuse of discretion implies a capricious, arbitrary and
whimsical exercise of power (Filinvest Credit Corp. v. Intermediate
In insisting that its Motion to Elevate This Case to the Court En Appellate Court, No. 65935, 30 September 1988, 166 SCRA 155). The
Banc should be granted, J.G. Summit further argued that: its abuse of discretion must be so patent and gross as to amount to an
Opposition to the Office of the Solicitor General’s Motion to Refer is evasion of positive duty or to a virtual refusal to perform a duty
different from its own Motion to Elevate; different grounds are invoked enjoined by law, as to act at all in contemplation of law, where the
by the two motions; there was unwarranted "executive interference"; power is exercised in an arbitrary and despotic manner by reason of
and the change in ponente is merely noted in asserting that this case passion or hostility (Litton Mills, Inc. v. Galleon Trader, Inc., et al[.], L-
should be decided by the Court en banc.15 40867, 26 July 1988, 163 SCRA 489).

We find no merit in petitioner’s contention that the propriety of the The facts in this case do not indicate any such grave abuse of
bidding process involved in the present case has been confused with discretion on the part of public respondents when they awarded the
the policy issue of the fate of the shipping industry which, petitioner CISS contract to Respondent SGS. In the "Invitation to Prequalify and
maintains, has never been an issue that is determinative of this case. Bid" (Annex "C," supra), the CISS Committee made an express
The Court’s Resolution of September 24, 2003 reveals a clear and reservation of the right of the Government to "reject any or all
definitive ruling on the propriety of the bidding process. In discussing bids or any part thereof or waive any defects contained thereon
whether the right to top granted to KAWASAKI in exchange for its right and accept an offer most advantageous to the Government." It is
of first refusal violates the principles of competitive bidding, we made a well-settled rule that where such reservation is made in an
an exhaustive discourse on the rules and principles of public bidding Invitation to Bid, the highest or lowest bidder, as the case may be,

6
is not entitled to an award as a matter of right (C & C Commercial affect the right of first refusal itself, but only the manner of its
Corp. v. Menor, L-28360, 27 January 1983, 120 SCRA 112). Even the exercise.29 Also, PHILYARDS argues that if this Court takes
lowest Bid or any Bid may be rejected or, in the exercise of sound cognizance of J.G. Summit’s allegations of fact regarding PHILSECO’s
discretion, the award may be made to another than the lowest bidder landholding, it must also recognize PHILYARDS’ assertions that
(A.C. Esguerra & Sons v. Aytona, supra, citing 43 Am. Jur., 788). PHILSECO’s landholdings were sold to another corporation.30 As
(emphases supplied)1awphi1.nét regards the right of first refusal, private respondent explains that
KAWASAKI’s reduced shareholdings (from 40% to 2.59%) did not
translate to a deprivation or loss of its contractually granted right of first
Like the condition in the Bureau Veritas case, the right to top was a
refusal.31 Also, the bidding was valid because PHILYARDS exercised
condition imposed by the government in the bidding rules which was
the right to top and it was of no moment that losing bidders later joined
made known to all parties. It was a condition imposed on all bidders
PHILYARDS in raising the purchase price.32
equally, based on the APT’s exercise of its discretion in deciding
on how best to privatize the government’s shares in PHILSECO. It
was not a whimsical or arbitrary condition plucked from the ether and In cadence with the private respondent PHILYARDS, public
inserted in the bidding rules but a condition which the APT approved as respondents COP and APT contend:
the best way the government could comply with its contractual
obligations to KAWASAKI under the JVA and its mandate of getting the
1. The conversion of the right of first refusal into a right to top
most advantageous deal for the government. The right to top had its
by 5% does not violate any provision in the JVA between
history in the mutual right of first refusal in the JVA and was reached by
NIDC and KAWASAKI.
agreement of the government and KAWASAKI.

2. PHILSECO is not a public utility and therefore not


Further, there is no "executive interference" in the functions of this
governed by the constitutional restriction on foreign
Court by the mere filing of a memorandum by Secretary of Finance
ownership.
Jose Isidro Camacho. The memorandum was merely "noted" to
acknowledge its filing. It had no further legal significance. Notably
too, the assailed Resolution dated September 24, 2003 was 3. The petitioner is legally estopped from assailing the
decided unanimously by the Special First Division in favor of the validity of the proceedings of the public bidding as it
respondents. voluntarily submitted itself to the terms of the ASBR which
included the provision on the right to top.
Again, we emphasize that a decision or resolution of a Division is that
of the Supreme Court20 and the Court en banc is not an appellate court 4. The right to top was exercised by PHILYARDS as the
to which decisions or resolutions of a Division may be appealed. 21 nominee of KAWASAKI and the fact that PHILYARDS
formed a consortium to raise the required amount to exercise
the right to top the highest bid by 5% does not violate the
For all the foregoing reasons, we find no basis to elevate this case to
JVA or the ASBR.
the Court en banc.

5. The 60%-40% Filipino-foreign constitutional requirement


Motion for Reconsideration
for the acquisition of lands does not apply to PHILSECO
because as admitted by petitioner itself, PHILSECO no
Three principal arguments were raised in the petitioner’s Motion for longer owns real property.
Reconsideration. First, that a fair resolution of the case should be
based on contract law, not on policy considerations; the contracts do
6. Petitioner’s motion to elevate the case to the Court en
not authorize the right to top to be derived from the right of first
banc is baseless and would only delay the termination of this
refusal.22 Second, that neither the right of first refusal nor the right to
case.33
top can be legally exercised by the consortium which is not the proper
party granted such right under either the JVA or the Asset Specific
Bidding Rules (ASBR).23 Third, that the maintenance of the 60%-40% In a Consolidated Comment dated March 8, 2004, J.G. Summit
relationship between the National Investment and Development countered the arguments of the public and private respondents in this
Corporation (NIDC) and KAWASAKI arises from contract and from the wise:
Constitution because PHILSECO is a landholding corporation and
need not be a public utility to be bound by the 60%-40% constitutional
limitation.24 1. The award by the APT of 87.67% shares of PHILSECO to
PHILYARDS with losing bidders through the exercise of a
right to top, which is contrary to law and the constitution is
On the other hand, private respondent PHILYARDS asserts that J.G. null and void for being violative of substantive due process
Summit has not been able to show compelling reasons to warrant a and the abuse of right provision in the Civil Code.
reconsideration of the Decision of the Court.25 PHILYARDS denies that
the Decision is based mainly on policy considerations and points out
that it is premised on principles governing obligations and contracts a. The bidders[’] right to top was actually exercised
and corporate law such as the rule requiring respect for contractual by losing bidders.
stipulations, upholding rights of first refusal, and recognizing the
assignable nature of contracts rights.26 Also, the ruling that shipyards b. The right to top or the right of first refusal cannot
are not public utilities relies on established case law and fundamental co-exist with a genuine competitive bidding.
rules of statutory construction. PHILYARDS stresses that KAWASAKI’s
right of first refusal or even the right to top is not limited to the 40%
equity of the latter.27 On the landholding issue raised by J.G. Summit, c. The benefits derived from the right to top were
PHILYARDS emphasizes that this is a non-issue and even involves a unwarranted.
question of fact. Even assuming that this Court can take cognizance of
such question of fact even without the benefit of a trial, PHILYARDS 2. The landholding issue has been a legitimate issue since
opines that landholding by PHILSECO at the time of the bidding is the start of this case but is shamelessly ignored by the
irrelevant because what is essential is that ultimately a qualified entity respondents.
would eventually hold PHILSECO’s real estate properties.28 Further,
given the assignable nature of the right of first refusal, any applicable
nationality restrictions, including landholding limitations, would not a. The landholding issue is not a non-issue.

7
b. The landholding issue does not pose questions enter into business with in order to raise funds to purchase the shares
of fact. are basically its business. This is in contrast to a case involving a
contract for the operation of or construction of a government
infrastructure where the identity of the buyer/bidder or financier
c. That PHILSECO owned land at the time that the
constitutes an important consideration. In such cases, the government
right of first refusal was agreed upon and at the
would have to take utmost precaution to protect public interest by
time of the bidding are most relevant.
ensuring that the parties with which it is contracting have the ability to
satisfactorily construct or operate the infrastructure.
d. Whether a shipyard is a public utility is not the
core issue in this case.
On the landholding issue, J.G. Summit submits that since PHILSECO
is a landholding company, KAWASAKI could exercise its right of first
3. Fraud and bad faith attend the alleged conversion of an refusal only up to 40% of the shares of PHILSECO due to the
inexistent right of first refusal to the right to top. constitutional prohibition on landholding by corporations with more than
40% foreign-owned equity. It further argues that since KAWASAKI
already held at least 40% equity in PHILSECO, the right of first refusal
a. The history behind the birth of the right to top was inutile and as such, could not subsequently be converted into the
shows fraud and bad faith.
right to top. 37 Petitioner also asserts that, at present, PHILSECO
continues to violate the constitutional provision on landholdings as its
b. The right of first refusal was, indeed, "effectively shares are more than 40% foreign-owned.38 PHILYARDS admits that it
useless." may have previously held land but had already divested such
landholdings.39 It contends, however, that even if PHILSECO owned
land, this would not affect the right of first refusal but only the exercise
4. Petitioner is not legally estopped to challenge the right to thereof. If the land is retained, the right of first refusal, being a property
top in this case. right, could be assigned to a qualified party. In the alternative, the land
could be divested before the exercise of the right of first refusal. In the
a. Estoppel is unavailing as it would stamp validity case at bar, respondents assert that since the right of first refusal was
to an act that is prohibited by law or against public validly converted into a right to top, which was exercised not by
policy. KAWASAKI, but by PHILYARDS which is a Filipino corporation (i.e.,
60% of its shares are owned by Filipinos), then there is no violation of
the Constitution.40 At first, it would seem that questions of fact beyond
b. Deception was patent; the right to top was an cognizance by this Court were involved in the issue. However, the
attractive nuisance. records show that PHILYARDS admits it had owned land up until
the time of the bidding.41 Hence, the only issue is whether
c. The 10% bid deposit was placed in escrow. KAWASAKI had a valid right of first refusal over PHILSECO
shares under the JVA considering that PHILSECO owned land
until the time of the bidding and KAWASAKI already held 40% of
J.G. Summit’s insistence that the right to top cannot be sourced from PHILSECO’s equity.
the right of first refusal is not new and we have already ruled on the
issue in our Resolution of September 24, 2003. We upheld the mutual
right of first refusal in the JVA.34 We also ruled that nothing in the JVA We uphold the validity of the mutual rights of first refusal under the JVA
prevents KAWASAKI from acquiring more than 40% of PHILSECO’s between KAWASAKI and NIDC. First of all, the right of first refusal is a
total capitalization.35 Likewise, nothing in the JVA or ASBR bars the property right of PHILSECO shareholders, KAWASAKI and NIDC,
conversion of the right of first refusal to the right to top. In sum, nothing under the terms of their JVA. This right allows them to purchase the
new and of significance in the petitioner’s pleading warrants a shares of their co-shareholder before they are offered to a third
reconsideration of our ruling. party. The agreement of co-shareholders to mutually grant this
right to each other, by itself, does not constitute a violation of the
provisions of the Constitution limiting land ownership to Filipinos
Likewise, we already disposed of the argument that neither the right of and Filipino corporations. As PHILYARDS correctly puts it, if
first refusal nor the right to top can legally be exercised by the PHILSECO still owns land, the right of first refusal can be validly
consortium which is not the proper party granted such right under assigned to a qualified Filipino entity in order to maintain the 60%-40%
either the JVA or the ASBR. Thus, we held: ratio. This transfer, by itself, does not amount to a violation of the Anti-
Dummy Laws, absent proof of any fraudulent intent. The transfer could
The fact that the losing bidder, Keppel Consortium (composed of be made either to a nominee or such other party which the holder of
Keppel, SM Group, Insular Life Assurance, Mitsui and ICTSI), has the right of first refusal feels it can comfortably do business with.
joined PHILYARDS in the latter's effort to raise ₱2.131 billion Alternatively, PHILSECO may divest of its landholdings, in which case
necessary in exercising the right to top is not contrary to law, public KAWASAKI, in exercising its right of first refusal, can exceed 40% of
policy or public morals. There is nothing in the ASBR that bars the PHILSECO’s equity. In fact, it can even be said that if the foreign
losing bidders from joining either the winning bidder (should the right to shareholdings of a landholding corporation exceeds 40%, it is not
top is not exercised) or KAWASAKI/PHI (should it exercise its right to the foreign stockholders’ ownership of the shares which is
top as it did), to raise the purchase price. The petitioner did not allege, adversely affected but the capacity of the corporation to own
nor was it shown by competent evidence, that the participation of the land – that is, the corporation becomes disqualified to own land. This
losing bidders in the public bidding was done with fraudulent intent. finds support under the basic corporate law principle that the
Absent any proof of fraud, the formation by [PHILYARDS] of a corporation and its stockholders are separate juridical entities. In this
consortium is legitimate in a free enterprise system. The appellate vein, the right of first refusal over shares pertains to the shareholders
court is thus correct in holding the petitioner estopped from questioning whereas the capacity to own land pertains to the corporation. Hence,
the validity of the transfer of the National Government's shares in the fact that PHILSECO owns land cannot deprive stockholders of their
PHILSECO to respondent.36 right of first refusal. No law disqualifies a person from purchasing
shares in a landholding corporation even if the latter will exceed
the allowed foreign equity, what the law disqualifies is the
Further, we see no inherent illegality on PHILYARDS’ act in seeking corporation from owning land. This is the clear import of the
funding from parties who were losing bidders. This is a purely following provisions in the Constitution:
commercial decision over which the State should not interfere absent
any legal infirmity. It is emphasized that the case at bar involves the
disposition of shares in a corporation which the government sought to Section 2. All lands of the public domain, waters, minerals, coal,
privatize. As such, the persons with whom PHILYARDS desired to petroleum, and other mineral oils, all forces of potential energy,

8
fisheries, forests or timber, wildlife, flora and fauna, and other natural land arising from the separate juridical personalities of the corporation
resources are owned by the State. With the exception of agricultural and its shareholders.
lands, all other natural resources shall not be alienated. The
exploration, development, and utilization of natural resources shall be
We note that in its Motion for Reconsideration, J.G. Summit alleges
under the full control and supervision of the State. The State may
that PHILSECO continues to violate the Constitution as its foreign
directly undertake such activities, or it may enter into co-production,
equity is above 40% and yet owns long-term leasehold rights which
joint venture, or production-sharing agreements with Filipino citizens,
are real rights.45 It cites Article 415 of the Civil Code which includes in
or corporations or associations at least sixty per centum of
the definition of immovable property, "contracts for public works, and
whose capital is owned by such citizens. Such agreements may be
servitudes and other real rights over immovable property."46 Any
for a period not exceeding twenty-five years, renewable for not more
existing landholding, however, is denied by PHILYARDS citing its
than twenty-five years, and under such terms and conditions as may
recent financial statements.47 First, these are questions of fact, the
be provided by law. In cases of water rights for irrigation, water supply,
veracity of which would require introduction of evidence. The Court
fisheries, or industrial uses other than the development of water power,
needs to validate these factual allegations based on competent and
beneficial use may be the measure and limit of the grant.
reliable evidence. As such, the Court cannot resolve the questions they
pose. Second, J.G. Summit misreads the provisions of the Constitution
xxx xxx xxx cited in its own pleadings, to wit:

Section 7. Save in cases of hereditary succession, no private lands 29.2 Petitioner has consistently pointed out in the past that private
shall be transferred or conveyed except to individuals, respondent is not a 60%-40% corporation, and this violates the
corporations, or associations qualified to acquire or hold lands of Constitution x x x The violation continues to this day because under the
the public domain.42 (emphases supplied) law, it continues to own real property…

The petitioner further argues that "an option to buy land is void in itself xxx xxx xxx
(Philippine Banking Corporation v. Lui She, 21 SCRA 52 [1967]). The
right of first refusal granted to KAWASAKI, a Japanese corporation, is
32. To review the constitutional provisions involved, Section 14, Article
similarly void. Hence, the right to top, sourced from the right of first
XIV of the 1973 Constitution (the JVA was signed in 1977), provided:
refusal, is also void."43 Contrary to the contention of petitioner, the case
of Lui She did not that say "an option to buy land is void in itself," for
we ruled as follows: "Save in cases of hereditary succession, no private lands shall be
transferred or conveyed except to individuals, corporations, or
associations qualified to acquire or hold lands of the public domain."
x x x To be sure, a lease to an alien for a reasonable period is
valid. So is an option giving an alien the right to buy real
property on condition that he is granted Philippine citizenship. As 32.1 This provision is the same as Section 7, Article XII of the 1987
this Court said in Krivenko vs. Register of Deeds: Constitution.

[A]liens are not completely excluded by the Constitution from the use of 32.2 Under the Public Land Act, corporations qualified to acquire or
lands for residential purposes. Since their residence in the Philippines hold lands of the public domain are corporations at least 60% of
is temporary, they may be granted temporary rights such as a lease which is owned by Filipino citizens (Sec. 22, Commonwealth Act 141,
contract which is not forbidden by the Constitution. Should they desire as amended). (emphases supplied)
to remain here forever and share our fortunes and misfortunes, Filipino
citizenship is not impossible to acquire.
As correctly observed by the public respondents, the prohibition in the
Constitution applies only to ownership of land.48 It does not extend to
But if an alien is given not only a lease of, but also an option to immovable or real property as defined under Article 415 of the
buy, a piece of land, by virtue of which the Filipino owner cannot Civil Code. Otherwise, we would have a strange situation where the
sell or otherwise dispose of his property, this to last for 50 years, ownership of immovable property such as trees, plants and growing
then it becomes clear that the arrangement is a virtual transfer of fruit attached to the land49 would be limited to Filipinos and Filipino
ownership whereby the owner divests himself in stages not only corporations only.
of the right to enjoy the land (jus possidendi, jus utendi, jus
fruendi and jus abutendi) but also of the right to dispose of it (jus
disponendi) — rights the sum total of which make up ownership. III.
It is just as if today the possession is transferred, tomorrow, the
use, the next day, the disposition, and so on, until ultimately all WHEREFORE, in view of the foregoing, the petitioner’s Motion for
the rights of which ownership is made up are consolidated in an Reconsideration is DENIED WITH FINALITY and the decision
alien. And yet this is just exactly what the parties in this case did within appealed from is AFFIRMED. The Motion to Elevate This Case to the
this pace of one year, with the result that Justina Santos'[s] ownership Court En Banc is likewise DENIED for lack of merit.
of her property was reduced to a hollow concept. If this can be done,
then the Constitutional ban against alien landholding in the Philippines,
as announced in Krivenko vs. Register of Deeds, is indeed in grave SO ORDERED.
peril.44 (emphases supplied; Citations omitted)

In Lui She, the option to buy was invalidated because it amounted to a


virtual transfer of ownership as the owner could not sell or dispose of
his properties. The contract in Lui She prohibited the owner of the land
from selling, donating, mortgaging, or encumbering the property during
the 50-year period of the option to buy. This is not so in the case at bar
where the mutual right of first refusal in favor of NIDC and KAWASAKI
does not amount to a virtual transfer of land to a non-Filipino. In fact,
the case at bar involves a right of first refusal over shares of
stock while the Lui She case involves an option to buy the land
itself. As discussed earlier, there is a distinction between the
shareholder’s ownership of shares and the corporation’s ownership of

9
G.R. No. 111008 November 7, 1994 Hence, the instant petition.

TRAMAT MERCANTILE, INC. AND DAVID ONG, petitioners, We could find no reason to reverse the factual findings of both the trial
vs. court and the appellate court, particularly in holding that the contract
HON. COURT OF APPEALS AND MELCHOR DE LA between de la Cuesta and TRAMAT was one of absolute, not
CUESTA, respondents. conditional, sale of the tractor and that de la Cuesta did not violate any
warranty on the sale of the tractor to TRAMAT. The appellate court, in
its decision, adequately explained:
Emilio G. Abrogena for petitioners.

If the perfection of the sale was dependent upon


Constante B. Albano for private respondent.
acceptance by the MWSS of the subject tractor
why did the appellants issue a check in payment of
the item to the appellee? And long after MWSS
had complained about the defective tractor engine,
and after the appellee had failed to remedy the
VITUG, J.:
defect, why did the appellants still draw and deliver
a replacement check to the appellee for the
This petition for review on certiorari challenges the 04th March 1993 increased amount of P33,500.00?
decision of the Court of Appeals and its resolution of 01 July 1993
denying the motion for reconsideration.
These payments argue against the claim now
made by the defendants that the sale was
On 09 April 1984, Melchor de la Cuesta, doing business under the conditional.
name and style of "Farmers Machineries," sold to Tramat Mercantile,
Inc. ("Tramat"), one (1) unit HINOMOTO TRACTOR Model MB 1100D According to the appellee, the additional amount
powered by a 13 H.P. diesel engine. In payment, David Ong, Tramat's covered the cost of replacing the oil gasket of the
president and manager, issued a check for P33,500.00 (apparently
tractor engine when it was repaired in Soledad
replacing an earlier postdated check for P33,080.00). Tramat, in turn, Cac's gasoline station in Quezon City. The
sold the tractor, together with an attached lawn mower fabricated by it, appellants, on the other hand, claims the amount
to the Metropolitan Waterworks and Sewerage System ("NAWASA")
represented the freight charges for transporting
for P67,000.00. David Ong caused a "stop payment" of the check when the tractor from Cauayan, Isabela to Metro Manila.
NAWASA refused to pay the tractor and lawn mower after discovering
that, aside from some stated defects of the attached lawn mower, the
engine (sold by de la Cuesta) was a reconditioned unit. The appellants should have explained why they
failed to include the freight charges in the first
check. The tractor was transported from Isabela to
On 28 May 1985, de la Cuesta filed an action for the recovery of Metro Manila as early as April 1984, and the first
P33,500.00, as well as attorney's fees of P10,000.00, and the costs of check was drawn at about the same time. The
suit. Ong, in his answer, averred, among other things, that de la
freight charges cannot be said to have been
Cuesta had no cause of action; that the questioned transaction was incurred when the tractor engine was delivered
between plaintiff and Tramat Mercantile, Inc., and not with Ong in his back to the supplier for repairs. The appellants
personal capacity; and that the payment of the check was stopped
admitted that the engine was not brought back to
because the subject tractor had been priced as a brand new, not as a Isabela. The repairs were done at Soledad Cac's
reconditioned unit. gasoline station in Quezon City.

On 02 November 1989, after the reception of evidence, the trial court Anent the first assigned error, We sustain the trial
rendered a decision, the dispositive portions of which read: court's finding that at the time of the purchase, the
appellants did not reveal to the appellee the true
WHEREFORE, in view of the foregoing purpose for which the tractor would be used.
consideration, judgment is hereby rendered: Granting that the appellants informed the appellee
that they would be reselling the unit to the MWSS,
an entity admittedly not engaged in farming, and
1. Ordering the defendants, that they ordered the tractor without the power
jointly and severally, to pay tiller, an indispensable accessory if the tractor
the plaintiff the sum of would be used in farming, these in themselves
P33,500.00 with legal interest would not constitute the required implied notice to
thereon at the rate of 12% per the appellee as seller.
annum from July 7, 1984 until
fully paid; and
xxx xxx xxx
2. Ordering the defendants,
jointly and severally, to pay In regard to the second assigned error, We do not
the plaintiff the sum of agree that the appellee should have been held
P10,000.00 as attorney's fees, liable for the tractor's alleged hidden defects. . . .
and the costs of this suit.
It has to be noted in this regard that, to satisfy the
SO ORDERED. 1 requirements of the MWSS, the appellants
borrowed a lawn mower from the MWSS so they
could fabricate one such mower. The appellants'
An appeal was timely interposed by the defendants. On 04 March witness stated that the kind of mid-mounted lawn
1993, the Court of Appeals affirmed in toto the decision of the trial mower was being manufactured by their
court. Defendant-appellants' motion for reconsideration was denied. competitor, Alpha Machinery, which had by then
stopped supplying the same (tsn,

10
Nov. 29, 1988, pp. 73-74). There is no showing In the case at bench, there is no indication that petitioner David Ong
that the appellants had had any previous could be held personally accountable under any of the
experience in the fabrication of this lawn mower. In abovementioned cases.
fact, as aforesaid, they had to borrow one from the
MWSS which they could copy. But although they
WHEREFORE, the petition is given DUE COURSE and the decision of
made a copy with the same specifications and
the trial court, affirmed by the appellate court, is MODIFIED insofar as
design, there was no assurance that the copy
it holds petitioner David Ong jointly and severally liable with Tramat
would function as well as with the model.
Mercantile, Inc., which portion of the questioned judgment is SET
ASIDE. In all other respects, the decision appealed from is AFFIRMED.
xxx xxx xxx No costs.

Although the trial court discussed it in a different SO ORDERED.


light, We view the matter in the same way the trial
court did — that the lawn mower as fabricated by
the appellants was the root of the parties'
problems.

Having had no previous experience in the


manufacture of lawn mowers of the same type as
that in litigation, and in a possibly patent-infringing
effort to undercut their competition, the appellants
gathered enough daring to do the fabrication
themselves. But the product might have proved
too much for the subject tractor to power, and the
tractor's engine was strained beyond its limits,
causing it to overheat and damage its gaskets.

No wonder, then, it was a gasket Soledad Cac had


to replace, at a cost chargeable to the appellants.
No wonder, furthermore, the appellants' witness
declared that even after the replacement of that
one gasket, the engine still leaked oil after being
torture-tested. The integrity of the other engine
gaskets might have been impaired, too. Such was
the burden placed on the engine. The engine
malfunctioned not necessarily because the engine,
as alleged by the appellants, had been a
reconditioned, and not a brand new, one. It
malfunctioned because it was made to do what it
simply could not.2

It was, nevertheless, an error to hold David Ong jointly and severally


liable with TRAMAT to de la Cuesta under the questioned transaction.
Ong had there so acted, not in his personal capacity, but as an officer
of a corporation, TRAMAT, with a distinct and separate personality. As
such, it should only be the corporation, not the person acting for and on
its behalf, that properly could be made liable thereon.3

Personal liability of a corporate director, trustee or officer along


(although not necessarily) with the corporation may so validly attach,
as a rule, only when —

1. He assents (a) to a patently unlawful act of the corporation, or


(b) for bad faith, or gross negligence in directing its affairs, or (c) for
conflict of interest, resulting in damages to the corporation, its
stockholders or other persons;4

2. He consents to the issuance of watered stocks or who, having


knowledge thereof, does not forthwith file with the corporate secretary
his written objection thereto;5

3. He agrees to hold himself personally and solidarily liable with the


corporation;6 or

4. He is made, by a specific provision of law, to personally answer for


his corporate action.7

1
G.R. No. L-5081 February 24, 1954 Antonio Cristobal 45 " 45
P1,025
MARVEL BUILDING CORPORATION, ET AL., plaintiffs-appellees,
vs.
SATURNINO DAVID, in his capacity as Collector, Bureau of Maria B Castro was elected President and Maximo Cristobal,
Internal Revenue, defendant-appellant. Secretary-Treasurer (Exhibit A).

Assistant Solicitor General Francisco Carreon for appellant. The Wise Building was purchased on September 4, 1946, the
Antonio Quirino and Rosendo J. Tansinsin for appellees. purchase being made in the name of Dolores Trinidad, wife of Amado
A. Yatco (Exhibit V), and the Aguinaldo Building, on January 17, 1947,
in the name of Segundo Esguerra, Sr. (Exhibit M). Both building were
LABRADOR, J.:
purchased for P1,800,000, but as the corporation had only P1,025,000,
the balance of the purchase price was obtained as loans from the
This action was brought by plaintiffs as stockholders of the Marvel Insular Life Assurance Co., Ltd. and the Philippine Guaranty Co., Inc.
Building Corporation to enjoin the defendant Collector of Internal (Exhibit C).
Revenue from selling at public auction various properties described in
the complaint, including three parcels of land, with the buildings
Of the incorporators of the Marvel Building Corporation, Maximo
situated thereon, known as the Aguinaldo Building, the Wise Building,
Cristobal and Antonio Cristobal are half-brothers of Maria B. Castro,
and the Dewey Boulevard-Padre Faura Mansion, all registered in the
Maria Cristobal is a half-sister, and Segundo Esguerra, Sr. a brother-
name of the said corporation. Said properties were seized and
in-law, husband of Maria Cristobal, Maria B. Castro's half-sister.
distrained by defendant to collect war profits taxes assessed against
Maximo B. Cristobal did not file any income tax returns before the year
plaintiff Maria B. Castro (Exhibit B). Plaintiffs allege that the said three
1946, except for three years 1939 and 1940, but in these years he was
properties (lands and buildings) belong to Marvel Building Corporation
exempted from the tax. He has not filed any war profits tax return
and not to Maria B. Castro, while the defendant claims that Maria B.
(Exhibit 54). Antonio Cristobal, Segundo Esguerra, Sr. and Jose T.
Castro is the true and sole owner of all the subscribed stock of the
Lopez did not file any income tax returns for the years prior to 1946,
Marvel Building Corporation, including those appearing to have been
and neither did they file any war profits tax returns (Exhibit 52). Maria
subscribed and paid for by the other members, and consequently said
Cristobal filed income tax returns for the year 1929 to 1942, but they
Maria B. Castro is also the true and exclusive owner of the properties
were exempt from the tax (Exhibit 53). Benita A. Lamagna did not file
seized. The trial court held that the evidence, which is mostly
any income tax returns prior to 1945, except for 1942 which was
circumstantial, fails to show to its satisfaction that Maria B. Castro is
exempt. She did not file any was profits tax (Exhibit 55). Ramon M.
the true owner of all the stock certificates of the corporation, because
Sangalang did not file income tax returns up to 1945 except for the
the evidence is susceptible of two interpretations and an interpretation
years 1936, 1937, 1938, 1939 and 1940. He has not filed any war
may not be made which would deprive one of the property without due
profits tax return (Exhibit 57). Amada A. Yatco did not file income tax
process of law.
returns prior to 1945, except for the years 1937, 1938, 1939, 1941 and
1942, but these were exempt. He did not file any war profits tax return
It appears that on September 15, 1950, the Secretary of Finance, upon (Exhibit 58).
consideration of the report of a special committee assigned to study
the war profits tax case of Mrs. Maria B. Castro, recommended the
Antonio Cristobal's income in 1946 is P15,630, and in 1947, P4,550
collection of P3,593,950.78 as war profits taxes for the latter, and on
(Exhibits 59-60); Maximo B. Cristobal's income in 1946 is P19,759.10,
September 22, 1953 the President instructed the Collector that steps
in 1947, P9,773.47 (Exhibits 61-62); Segundo Esquerra's income in
be taken to collect the same (Exhibits 114, 114-A to 114-D). Pursuant
1946 is P5,500, in 1947, P7,754.32 (Exhibits 63-64); Jose T. Lopez's
thereto various properties, including the three above mentioned, were
income in 1946 is P20,785, in 1947, P14,302.77 (Exhibits 69-70);
seized by the Collector of Internal Revenue on October 31, 1950. On
Benita A. Lamagna's income in 1945 is P1,559, in 1946, P6,463.36, in
November 13, 1950, the original complaint in this case was filed. After
1947, P6,189.79 and her husband's income in 1947 is P10,825.53
trial, the Court of First Instance of Manila rendered judgment ordering
(Exhibits 65-68); Ramon M. Sangalang's income in 1945 is P5,500, in
the release of the properties mentioned, and enjoined the Collector of
1946, P18,300.00 (Exhibits 71-72); Santiago Tan's income in 1945 is
Internal Revenue from selling the same. The Collector of Internal
P456, in 1947 is P9,167.95, and in 1947, P7,620.11 (Exhibits 73-75);
Revenue has appealed to this Court against the judgment.
and Amado Yatco's income in 1945 is P12,600, in 1946, P23,960, and
in 1947, P11,160 (Exhibits 76-78).
The following facts are not disputed, or are satisfactorily proved by the
evidence:
In October, 1945 Maria B. Castro, Nicasio Yatco, Maxima Cristobal de
Esquerra, Maria Cristobal Lopez and Maximo Cristobal organized the
The Articles of Incorporation of the Marvel Building Corporation is Maria B. Castro, Inc. with capital stock of P100,000, of which Maria B.
dated February 12, 1947 and according to it the capital stock is Castro subscribed for P99,600 and all others for P100 each. This was
P2,000,000, of which P1,025,000 was (at the time of incorporation) increased in 1950 to P500,000 and Maria B. Castro subscribed
subscribed and paid for by the following incorporators: P76,000 and the others P1,000 each (Exhibit 126).

Maria B. Castro 250 shares It does not appear that the stockholders or the board of directors of the
P 250,000.00
Marvel Building Corporation have ever held a business meeting, for no
Amado A. Yatco 100 " 100,000.00
books thereof or minutes meeting were ever mentioned by the officers
Santiago Tan 100 " thereof or presented by them at the trial. The by-laws of the
100,000.00
corporation, if any had ever been approved, has not been presented.
Jose T. Lopez 90 " 90,000.00
Neither does it appear that any report of the affairs of the corporation
Benita Lamagna 90 " has been made, either of its transactions or accounts.
90,000.00
C.S. Gonzales 80 " 80,000.00
From the book of accounts of the corporation, advances to the Marvel
Maria Cristobal 70 " 70,000.00
Building Corporation of P125,000 were made by Maria B. Castro in
Segundo Esguerra, Sr. 75 " 1947, P102,916.05 in 1948 and P102,916.05 in 1948, and
75,000.00
P160,910.96 in 1949 (Exhibit 118).
Ramon Sangalang 70 " 70,000.00
Maximo Cristobal 55 " 55,000.00

12
The main issue involved in these proceedings is: Is Maria B. Castro the 1948 and 1949 the Llamados were trusted friends of Maria B. Castro,
owner of all the shares of stocks of Marvel Building Corporation and and this explains why they had knowledge of her secret transactions.
the other stockholders mere dummies of hers? The younger Llamado even made advances for the hand of Maria B.
Castro's daughter, and this at the time when as a bookkeeper he was
entrusted with checking up the certificates of stock. When the older
The most important evidence presented by the Collector of Internal
Llamado kept secret the existence of the endorsed certificates, the
Revenue to prove his claim that Maria B. Castro is the sole and
friendship between the two families was yet intact; hence, the
exclusive owner of the shares of stock of the Marvel Building
existence of the endorsed certificates must have been kept to himself
Corporation is supposed endorsement in blank of the shares of stock
by the older Llamado. All the above circumstances reinforce our belief
issued in the name of the other incorporators, and the possession
that the Llamados had personal knowledge of the facts they testify to,
thereof by Maria B. Castro. The existence of said endorsed certificates
and the existence of this knowledge in turn renders improbable
was testified to by witnesses Felipe Aquino, internal revenue examiner,
plaintiffs' claim that their testimonies were biased.
Antonio Mariano, examiner, and Crispin Llamado, Under Secretary of
Finance, who declared as follows: Towards the end of the year 1948
and about the beginning of the year 1949, while Aquino and Mariano Attempt was also made by the plaintiffs to show by expert evidence
were examining the books and papers were furnished by its secretary, that the endorsement could have been superimposed, i.e., that the
Maximo Cristobal, they came across an envelope containing eleven signatures made on other papers and these were pasted and
stock certificates, bound together by an Acco fastener, which thereafter the documents photographed. Judicial experience is to the
(certificates) corresponded in number and in amount on their face to effect that the expert witnesses can always be obtained for both sides
the subscriptions of the stockholders that all the certificates, except of an issue, most likely because expert witnesses are no longer
that in the name of Maria B. Castro, were endorsed in the bank by the impermeable to the influence of fees (II Wigmore, Sec. 563(2), p. 646).
subscribers; that as the two revenue agents could not agree what to do And if parties are capable of paying fees, expert opinion should be
with the certificates, Aquino brought them to Under-Secretary of received with caution. In the case at bar, the opinion on the supposed
Finance Llamado, who thereupon suggested that photostatic copies superimposition was merely a possibility, and we note various
thereof be taken; that this was done, and the photostatic copies taken circumstances which proved that the signatures were not
are (Exhibits 4, 5, 6, 7, 8, 9, 10, 11, 12 and 13; and in that July, 1950, superimposed and corroborate defendant's claim that they were
copy-cat copies of the above photostats were taken, and said copy-cat genuine. In the first place,, the printed endorsement contains a very
copies are Exhibits 40-49. heavy line at the bottom for the signature of the endorsee. This line in
almost all the endorsements is as clear as the printed letters above it,
and at the points where the letters of the signature extend down and
Julio Llamado, bookkeeper of the Marvel Building Corporation from
transversed it (the line), there is no indication that the line is covered by
1947 to May, 1948, also testified that he was the one who had
a superimposed paper. Again in these places both the signatures and
prepared the original certificates, putting therein the number of shares
the lines are clear and distinct where they cross one another. Had
in words in handprint; that the originals were given to him by Maria B.
there been superimpositions the above features could not have been
Castro for comparison with the articles of incorporation; that they were
possible. In the second place, Maria B. Castro admitted having signed
not yet signed by the President and by the Secretary-Treasurer when
25 stock certificates, but only eleven were issued (t. s. n., p. 662). No
he had the certificates; and that after the checking he returned all of
explanation is given by her why she had to sign as many as 25 forms
them to Mrs. Castro. He recognized the photostats, Exhibit 4 to 13 as
when there were only eleven subscribers and eleven forms to be filed.
photostats of the said originals. He also declared that he also prepared
This circumstances corroborate the young Llamado's declaration that
a set of stock certificates, similar to the certificates which were copied
two sets of certificates had been prepared. The nineteen issue must be
in the photostats, the number of shares, and the date issue, and that
Exhibits H, H-1 to H-7 and J, or Nos. 30 to 38, and the stock
the certificates he had prepared are Exhibits H, H-1 to H-7 and J
certificates endorsed whose photostatic copies are Exhibits 4 to 13. It
(Exhibits 30-38). This set of certificates was made by him first and the
is to be remembered also, that it is a common practice among
set of which photostats were taken, a few days later.
unscrupulous merchants to carry two sets of books, one set for
themselves and another to be shown to tax collectors. This practice
The plaintiffs offered a half-hearted denial of the existence of the could not have been unknown to Maria B. Castro, who apparently had
endorsed blank certificates, Maximo Cristobal, secretary of the been able to evade the payment of her war profits taxes. These
corporation, saying that no investigation was ever made by Aquino and circumstances, coupled with the testimony of Julio Llamado that two
Mariano in which said certificates were discovered by the latter. They, sets of certificates were given to him for checking, show to an impartial
however, vigorously attack the credibility of the witnesses for the mind the existence of the set of certificates endorsed in blank, thus
defendant, imputing to the Llamados, enmity against Maria B. Castro, confirming the testimonies of the defendant's witnesses, Aquino,
and to Aquino and Mariano, a very doubtful conduct in not divulging the Mariano and Crispin Llamado, and thus discrediting the obviously
existence of the certificates either Lobrin, Chief Income Tax Examiner, partial testimony of the expert presented by plaintiffs. The genuineness
or to the Collector of Internal Revenue, both their immediate chiefs. of the signatures on the endorsements is not disputed. How could the
Reliance is also placed on a certificate, Exhibit W, wherein Aquino and defendant have secured these genuine signatures? Plaintiffs offer no
others declare that the certificates (Exhibits 30 to 38, or H, H-1 to H-7 explanation for this, although they do not question them. It follows that
and J) were regular and were not endorsed when the same were the genuine signatures must have been made on the stock certificates
examined. In connection with this certificate, Exhibit W, we note that it themselves.
states that the certificates examined were Exhibits 30 to 38, the
existence or character of which are not disputed. But the statement
Next in importance among the evidence submitted by the defendant
contains nothing to the effect that the above certificates were the only
collector to prove his contention that Maria B. Castro is the sole owner
one existence, according to their knowledge. Again the certificate was
of the shares of stock of the Marvel Building Corporation, is the fact
issued for an examination on September 1949, not by Aquino and
that the other stockholders did not have incomes in such amounts,
Mariano at the end of 1948 or the beginning of 1949. The certificate,
during the time of the organization of the corporation in 1947, or
therefore, neither denies the existence of the endorsed certificates, nor
immediately thereto, as to enable them to pay in full for their supposed
that Aquino and Mariano had made an examination of the papers of
subscriptions. This fact is proved by their income tax returns, or the
the corporation at the end of the year 1948. It ca not, therefore,
absence thereof. Let us take Amado A. Yatco as an example. Before
discredit the testimonies of the defendant's witnesses.
1945 his returns were exempt from the tax, in 1945 he had P12,600
and in 1946, P23,000. He has four children. How could he have paid
As to the supposed enmity of the Llamados towards the plaintiff Maria P100,000 in 1945 and 1946? Santiago Tan who also contributed
B. Castro, we note that, supposing that there really was such enmity, it P100,000 had no appreciable income before 1946, and this year an
does not appear that it was of such magnitude or force as could have income of only P9,167.95. Jose T. Lopez also did not file any income
induced the Llamados and Maria B. Castro were close friends way tax returns before 1940 and 1946 he had an income of only P20,785,
back in 1947 and up to 1949; but that at the time of the trial the whereas he is supposed to have subscribed P90,000 worth of stock
friendship had been marred by misunderstandings. We believe that in early in 1947. Benita Lamagna had no returns either up to 1945,
13
except in 1942, which was exempt and in 1945 she had an income of undisputed fact that Maria B. Castro had made enormous profits and,
P1,550 and in 1946, P6,463.36. In the same situation are all the therefore, had a motive to hide them to evade the payment of taxes,
others, and besides, brothers and sisters and brother-in-law of Maria B. the fact that the other subscribers had no incomes of sufficient
Castro. On the other hand, Maria B. Castro had been found to have magnitude to justify their big subscriptions, the fact that the
made enormous gains or profits in her business such that the taxes subscriptions were not receipted for and deposited by the treasurer in
thereon were assessed at around P3,000,000. There was, therefore, the name of the corporation but were kept by Maria B. Castro herself,
a prima facie case out by the defendant collector that Maria B. Castro the fact that the stockholders or the directors never appeared to have
had furnished (& all the money that the Marvel Building Corporation ever met to discuss the business of the corporation, the fact that Maria
had. B. Castro advanced big sums of money to the corporation without any
previous arrangement or accounting, and the fact that the books of
accounts were kept as if they belonged to Maria B. Castro alone —
In order the meet the above evidence only three of the plaintiffs
these facts are of patent and potent significance. What are their
testified, namely, Maximo Cristobal, the corporation's secretary, who
necessary implications? Maria B. Castro would not have asked them to
made the general assertion on the witness stand that the other
endorse their stock certificates, or be keeping these in her possession,
stockholders paid for their shares in full, Maria B. Castro, who stated
if they were really the owners. They never would have consented that
that payments for the subscription were made to her, and C.S.
Maria B. Castro keep the funds without receipts or accounting, nor that
Gonzales, who admitted that Maria B. Castro, paid for his subscription.
she manages the business without their knowledge or concurrence,
After a careful study of the above testimonies, however, we find them
were they owners of the stocks in their own rights. Each and every one
subject to various objections. Maximo Cristobal declared that he issued
of the facts all set forth above, in the same manner, is inconsistent with
provincial receipts for the subscriptions supposedly paid to him in
the claim that the stockholders, other than Maria B. Castro, own their
1946; but none of the supposed receipts were presented. If the
shares in their own right. On the other hand, each and every one of
subscriptions were really received by him, big as the amounts were, he
them, and all of them, can point to no other conclusion than that Maria
would have been able to tell specifically, by dates and in fix amounts,
B. Castro was the sole and exclusive owner of the shares and that they
when and how the payments were made. The general assertion of
were only her dummies.
alleged payments, without the concrete days and amounts of
payments, are, according to our experience, positive identifications of
untruthfulness, for when a witness testified to a fact that actually In our opinion, the facts and circumstances duly set forth above, all of
occurs, the act is concretely stated and no generalization is made. which have been proved to our satisfaction, prove conclusively and
beyond reasonable doubt (section 89, Rule 123 of the Rules of Court
and section 42 of the Provisional law for the application of the Penal
With respect to Maria B. Castro's testimony, we find it to be as
Code) that Maria B. Castro is the sole and exclusive owner of all the
untruthful as that of Cristobal. She declared that payments of the
shares of stock of the Marvel Building Corporation and that the other
subscriptions took place between July and December, 1946, and that
partners are her dummies.
first payments were first deposited by her in the National City Bank of
New York. A study of her account in said bank (Exhibit 82), however,
fails to show the alleged deposit of the subscriptions during the year Wherefore, the judgment appealed from should be, as it hereby is,
1946 (See Exhibits 83-112). This fact completely belies her assertion. reversed and the action filed by plaintiffs-appellees, dismissed, with
As to the testimony of C.S. Gonzales that Maria B. Castro advanced costs against plaintiffs-appellees. So ordered.
his subscription, there is nothing in the evidence to corroborate it, and
the circumstances show otherwise. If he had really been a stockholder
and Maria B. Castro advanced his subscription, the agreement
between him and Castro should have been put in writing, the amount
advanced being quite considerable (P80,000), and it appearing further
that Gonzales is no close relative or confidant of Castro.

Lastly, it is significant that the plaintiffs, the supposed subscribers, who


should have come to court to assert that they actually paid for their
subscriptions, and are not mere dummies, did not do so. They could
not have afforded such a costly indifference, valued at from P70,000 to
P100,000 each, if they were not actual dummies. This failure on their
part to take the witness stand to deny or refute the charge that they
were mere dummies is to us of utmost significance. What could have
been easier to disprove the charge that they were dummies, than for
them to come to court and show their receipts and testify on the
payments they have paid on their subscriptions? This they, however
refused to do so. They had it in their power to rebut the charges, but
they chose to keep silent. The non-production of evidence that would
naturally have been produced by an honest and therefore fearless
claimant permits the inference that its tenor is unfavorable to the
party's cause (II Wigmore, Sec. 285, p.162). A party's silence to
adverse testimony is equivalent to an admission of its truth (Ibid, Sec.
289, p. 175).

Our consideration of the evidence submitted on both sides leads us to


a conclusion exactly opposite that arrived at by the trial court. In
general the evidence offered by the plaintiffs is testimonial and direct
evidence, easy of fabrication; that offered by defendant, documentary
and circumstantial, not only difficult of fabrication but in most cases
found in the possession of plaintiffs. There is very little room for choice
as between the two. The circumstantial evidence is not only
convincing; it is conclusive. The existence of endorsed certificates,
discovered by the internal revenue agents between 1948 and 1949 in
the possession of the Secretary-Treasurer, the fact that twenty-five
certificates were signed by the president of the corporation, for no
justifiable reason, the fact that two sets of certificates were issued, the
14
G.R. No. L-15121 August 31, 1962 On June 20, 1956, defendant filed its answer. By way of
affirmative defenses, it alleges (1) that complaint states no
cause of action against defendant, and (2) that the sale and
GREGORIO PALACIO, in his own behalf and in behalf of his minor
transfer of the jeep AC-687 by Isabelo Calingasan to the
child,
Fely Transportation was made on December 24, 1955, long
MARIO PALACIO, plaintiffs-appellants,
after the driver Alfredo Carillo of said jeep had been
vs.
convicted and had served his sentence in Criminal Case No.
FELY TRANSPORTATION COMPANY, defendant-appellee.
Q-1084 of the Court of First Instance of Quezon City, in
which both the civil and criminal cases were simultaneously
Antonio A. Saba for plaintiffs-appellants. tried by agreement of the parties in said case. In the
Mercado, Ver and Reyes for defendant-appellee. Counterclaim of the Answer, defendant alleges that in view
of the filing of this complaint which is a clearly unfounded
civil action merely to harass the defendant, it was compelled
REGALA, J.:
to engage the services of a lawyer for an agreed amount of
P500.00.
This is an appeal by the plaintiffs from the decision of the Court of First
Instance of Manila which dismissed their complaint.
During the trial, plaintiffs presented the transcript of the
stenographic notes of the trial of the case of "People of the
Originally taken to the Court of Appeals, this appeal was certified to Philippines vs. Alfredo Carillo, Criminal Case No. Q-1084," in
this Court on the ground that it raises purely questions of law. the Court of First Instance of Rizal, Quezon City (Branch IV),
as Exhibit "A".1äwphï1.ñët
The parties in this case adopt the following findings of fact of the lower
court: It appears from Exhibit "A" that Gregorio Palacio, one of the
herein plaintiffs, testified that Mario Palacio, the other
plaintiff, is his son; that as a result of the reckless driving of
In their complaint filed with this Court on May 15, 1954, accused Alfredo Carillo, his child Mario was injured and
plaintiffs allege, among other things, "that about December, hospitalized from December 24, 1952, to January 8, 1953;
1952, the defendant company hired Alfredo Carillo as driver that during all the time that his child was in the hospital, he
of AC-787 (687) (a registration for 1952) owned and watched him during the night and his wife during the day;
operated by the said defendant company; that on December that during that period of time he could not work as he slept
24, 1952, at about 11:30 a.m., while the driver Alfonso during the day; that before his child was injured, he used to
(Alfredo) Carillo was driving AC-687 at Halcon Street, earn P10.00 a day on ordinary days and on Sundays from
Quezon City, wilfully, unlawfully and feloniously and in a P20 to P50 a Sunday; that to meet his expenses he had to
negligent, reckless and imprudent manner, run over a child sell his compressor and electric drill for P150 only; and that
Mario Palacio of the herein plaintiff Gregorio Palacio; that on they could have been sold for P300 at the lowest price.
account of the aforesaid injuries, Mario Palacio suffered a
simple fracture of the right tenor (sic), complete third, thereby
hospitalizing him at the Philippine Orthopedic Hospital from During the trial of the criminal case against the driver of the
December 24, 1952, up to January 8, 1953, and continued to jeep in the Court of First Instance of Quezon City (Criminal
be treated for a period of five months thereafter; that the Case No. Q-1084) an attempt was unsuccessfully made by
plaintiff Gregorio Palacio herein is a welder by occupation the prosecution to prove moral damages allegedly suffered
and owner of a small welding shop and because of the by herein plaintiff Gregorio Palacio. Likewise an attempt was
injuries of his child he has abandoned his shop where he made in vain by the private prosecutor in that case to prove
derives income of P10.00 a day for the support of his big the agreed attorney's fees between him and plaintiff
family; that during the period that the plaintiff's (Gregorio Gregorio Palacio and the expenses allegedly incurred by the
Palacio's) child was in the hospital and who said child was herein plaintiffs in connection with that case. During the trial
under treatment for five months in order to meet the needs of of this case, plaintiff Gregorio Palacio testified substantially
his big family, he was forced to sell one air compressor to the same facts.
(heavy duty) and one heavy duty electric drill, for a sacrifice
sale of P150.00 which could easily sell at P350.00; that as a
The Court of First Instance of Quezon City in its decision in
consequence of the negligent and reckless act of the driver Criminal Case No. 1084 (Exhibit "2") determined and
Alfredo Carillo of the herein defendant company, the herein thoroughly discussed the civil liability of the accused in that
plaintiffs were forced to litigate this case in Court for an
case. The dispositive part thereof reads as follows:
agreed amount of P300.00 for attorney's fee; that the herein
plaintiffs have now incurred the amount of P500.00 actual
expenses for transportation, representation and similar IN VIEW OF THE FOREGOING, the Court finds the accused
expenses for gathering evidence and witnesses; and that Alfredo Carillo y Damaso guilty beyond reasonable doubt of
because of the nature of the injuries of plaintiff Mario Palacio the crime charged in the information and he is hereby
and the fear that the child might become a useless invalid, sentenced to suffer imprisonment for a period of Two Months
the herein plaintiff Gregorio Palacio has suffered moral & One Day of Arresto Mayor; to indemnify the offended
damages which could be conservatively estimated at party, by way of consequential damages, in the sum of
P1,200.00. P500.00 which the Court deems reasonable; with subsidiary
imprisonment in case of insolvency but not to exceed ¹/3 of
the principal penalty imposed; and to pay the costs.
On May 23, 1956, defendant Fely Transportation Co., filed a
Motion to Dismiss on the grounds (1) that there is no cause
of action against the defendant company, and (2) that the On the basis of these facts, the lower court held action is barred by the
cause of action is barred by prior judgment.. judgment in the criminal case and, that under Article 103 of the
Revised Penal Code, the person subsidiarily liable to pay damages is
Isabel Calingasan, the employer, and not the defendant corporation.
In its Order, dated June 8, 1956, this Court deferred the
determination of the grounds alleged in the Motion to
Dismiss until the trial of this case. Against that decision the plaintiffs appealed, contending that:

15
THE LOWER COURT ERRED IN NOT SUSTAINING THAT their claim for damages. This rules out the defense of res judicata,
THE DEFENDANT-APPELLEE IS SUBSIDIARILY LIABLE because such liability proceeds precisely from the judgment in the
FOR DAMAGES AS A RESULT OF CRIMINAL CASE NO. criminal action, where the accused was found guilty and ordered to pay
Q-1084 OF THE COURT OF FIRST INSTANCE OF an indemnity in the sum P500.00.
QUEZON CITY FOR THE REASON THAT THE
INCORPORATORS OF THE FELY TRANSPORTATION
WHEREFORE, the decision of the lower court is hereby reversed and
COMPANY, THE DEFENDANT-APPELLEE HEREIN, ARE
defendants Fely Transportation and Isabelo Calingasan are ordered to
ISABELO CALINGASAN HIMSELF, HIS SON AND
pay, jointly and severally, the plaintiffs the amount of P500.00 and the
DAUGHTERS;
costs.

THE LOWER COURT ERRED IN NOT CONSIDERING


THAT THE INTENTION OF ISABELO CALINGASAN IN
INCORPORATING THE FELY TRANSPORTATION
COMPANY, THE DEFENDANT-APPELLEE HEREIN, WAS
TO EVADE HIS CIVIL LIABILITY AS A RESULT OF THE
CONVICTION OF HIS DRIVER OF VEHICLE AC-687 THEN
OWNED BY HIM:

THE LOWER COURT ERRED IN HOLDING THAT THE


CAUSE OF ACTION OF THE PLAINTIFFS-APPELLANTS
IS BARRED BY PRIOR JUDGMENT.

With respect to the first and second assignments of errors, plaintiffs


contend that the defendant corporate should be made subsidiarily
liable for damages in the criminal case because the sale to it of the
jeep in question, after the conviction of Alfred Carillo in Criminal Case
No. Q-1084 of the Court of First Instance of Quezon City was merely
an attempt on the part of Isabelo Calingasan its president and general
manager, to evade his subsidiary civil liability.

The Court agrees with this contention of the plaintiffs. Isabelo


Calingasan and defendant Fely Transportation may be regarded as
one and the same person. It is evident that Isabelo Calingasan's main
purpose in forming the corporation was to evade his subsidiary civil
liability1 resulting from the conviction of his driver, Alfredo Carillo. This
conclusion is borne out by the fact that the incorporators of the Fely
Transportation are Isabelo Calingasan, his wife, his son, Dr.
Calingasan, and his two daughters. We believe that this is one case
where the defendant corporation should not be heard to say that it has
a personality separate and distinct from its members when to allow it to
do so would be to sanction the use of the fiction of corporate entity as a
shield to further an end subversive of justice. (La Campana Coffee
Factory, et al. v. Kaisahan ng mga Manggagawa, etc., et al., G.R. No.
L-5677, May 25, 1953) Furthermore, the failure of the defendant
corporation to prove that it has other property than the jeep (AC-687)
strengthens the conviction that its formation was for the purpose above
indicated.

And while it is true that Isabelo Calingasan is not a party in this case,
yet, is held in the case of Alonso v. Villamor, 16 Phil. 315, this Court
can substitute him in place of the defendant corporation as to the real
party in interest. This is so in order to avoid multiplicity of suits and
thereby save the parties unnecessary expenses and delay. (Sec. 2,
Rule 17, Rules of Court; Cuyugan v. Dizon. 79 Phil. 80; Quison v.
Salud, 12 Phil. 109.)

Accordingly, defendants Fely Transportation and Isabelo Calingasan


should be held subsidiarily liable for P500.00 which Alfredo Carillo was
ordered to pay in the criminal case and which amount he could not pay
on account of insolvency.

We also sustain plaintiffs' third assignment of error and hold that the
present action is not barred by the judgment of the Court of First
Instance of Quezon City in the criminal case. While there seems to be
some confusion on part of the plaintiffs as to the theory on which the is
based — whether ex-delito or quasi ex-delito (culpa aquiliana) — We
are convinced, from the discussion prayer in the brief on appeal, that
they are insisting the subsidiary civil liability of the defendant. As a
matter of fact, the record shows that plaintiffs merely presented the
transcript of the stenographic notes (Exhibit "A") taken at the hearing of
the criminal case, which Gregorio Palacio corroborated, in support of

16
G.R. No. L-20886 April 27, 1967 After due trial court rendered the appealed judgment. The appeal was
taken to the Court of Appeals, but on January 15, 1963 the latter
certified the case to us for final adjudication pursuant to sections 17
NATIONAL MARKETING CORPORATION (NAMARCO), plaintiff-
and 31 of the Judiciary Act of 1948, as amended, the amount involved
appellant,
being more than P200,000.00, exclusive of interests and cost.
vs.
ASSOCIATED FINANCE COMPANY, INC., and FRANCISCO
SYCIP, defendants. The only issue to be resolved is whether, upon the facts found by the
FRANCISCO SYCIP, defendant-appellee. trial court, — which, in our opinion, are fully supported by the evidence
— Francisco Sycip may be held liable, jointly and severally with his co-
defendant, for the sums of money adjudged in favor of NAMARCO.
Tomas P. Matic, Jr,. for plaintiff and appellant.
Francisco Sycip in his behalf as defendant and appellee.
The evidence of record shows that, of the capital stock of
ASSOCIATED, Sycip owned P60,000.00 worth of shares, while his
DIZON, J.:
wife — the second biggest stockholder — owned P20,000.00 worth of
shares; that the par value of the subscribed capital stock of
Appeal by the National Marketing Corporation — hereinafter referred to ASSOCIATED was only P105,000.00; that negotiations that lead to the
as NAMARCO, from the decision of the Court of First Instance of execution of the exchange agreement in question were conducted
Manila in Civil Case No. 45770 ordering the Associated Finance exclusively by Sycip on behalf of ASSOCIATED; that, as a matter of
Company, Inc. — hereinafter referred to as the ASSOCIATED — to fact, in the course of his testimony, Sycip referred to himself as the one
pay the NAMARCO the sum of P403,514.28, with legal interest thereon who contracted or transacted the business in his personal capacity,
from the date of filing of the action until fully paid, P80,702.26 as and asserted that the exchange agreement was his personal contract;
liquidated damages, P5,000.00 as attorney's fees, plus costs, but that it was Sycip who made personal representations and gave
dismissing the complaint insofar as defendant Francisco Sycip was assurances that ASSOCIATED was in actual possession of the 22,516
concerned, as well as the latter's counterclaim. The appeal is only from bags of "Victorias" and/or "National" refined sugar which the latter had
that portion of the decision dismissing the case as against Francisco agreed to deliver to NAMARCO, and that the same was ready for
Sycip. delivery; that, as a matter of fact, ASSOCIATED was at that time
already insolvent; that when NAMARCO made demands upon
ASSOCIATED to deliver the 22,516 bags of refined sugar it was under
On March 25, 1958, ASSOCIATED, a domestic corporation, through its obligation to deliver to the former, ASSOCIATED and Sycip, instead of
President, appellee Francisco Sycip, entered into an agreement to making delivery of the sugar, offered to pay its value at the rate of
exchange sugar with NAMARCO, represented by its then General P15.30 per bag — a clear indication that they did not have the sugar
Manager, Benjamin Estrella, whereby the former would deliver to the contracted for.1äwphï1.ñët
latter 22,516 bags (each weighing 100 pounds) of "Victorias" and/or
"National" refined sugar in exchange for 7,732.71 bags of "Busilak" and
17,285.08 piculs of "Pasumil" raw sugar belonging to NAMARCO, both The foregoing facts, fully established by the evidence, can lead to no
agreeing to pay liquidated damages equivalent to 20% of the other conclusion than that Sycip was guilty of fraud because through
contractual value of the sugar should either party fail to comply with the false representations he succeeded in inducing NAMARCO to enter
terms and conditions stipulated (Exhibit A). Pursuant thereto, on May into the aforesaid exchange agreement, with full knowledge, on his
19,1958, NAMARCO delivered to ASSOCIATED 7,732.71 bars of part, on the fact that ASSOCIATED whom he represented and over
"Busilak" and 17,285.08 piculs of "Pasumil" domestic raw sugar. As whose business and affairs he had absolute control, was in no position
ASSOCIATED failed to deliver to NAMARCO the 22,516 bags of to comply with the obligation it had assumed. Consequently, he can not
"Victoria" and/or "National" refined sugar agreed upon, the latter, on now seek refuge behind the general principle that a corporation has a
January 12, 1959, demanded in writing from the ASSOCIATED either personality distinct and separate from that of its stockholders and that
(a) immediate delivery thereof before January 20, or (b) payment of its the latter are not personally liable for the corporate obligations. To the
equivalent cash value amounting to P372,639.80. contrary, upon the proven facts, We feel perfectly justified in "piercing
the veil of corporate fiction" and in holding Sycip personally liable,
jointly and severally with his co-defendant, for the sums of money
On January 19, 1959, ASSOCIATED, through Sycip, offered to pay adjudged in favor of appellant. It is settled law in this and other
NAMARCO the value of 22,516 bags of refined sugar at the rate of jurisdictions that when the corporation is the mere alter ego of a
P15.30 per bag, but the latter rejected the offer. Instead, on January 21
person, the corporate fiction may be disregarded; the same being true
of the same year it demanded payment of the 7,732.71 bags of when the corporation is controlled, and its affairs are so conducted as
"Busilak" raw sugar at P15.30 per bag, amounting to P118,310.40. and to make it merely an instrumentality, agency or conduit of another
of the 17,285.08 piculs of "Pasumil" raw sugar at P16.50 per picul,
(Koppel Phils., etc. vs. Yatco, etc., 43 O.G. No. 11. Nov. 1947; Yutivo
amounting, to P285.203.82, or a total price of P403,514.28 for both Sons, etc. vs. Court of Tax Appeals, etc., G.R. No. L-13203,
kinds of sugar, based on the sugar quotations (Exh. H) as of March 20, promulgated on January 28, 1961).
1958 — the date when the exchange agreement was entered into.

Wherefore, the decision appealed from is modified by sentencing


As ASSOCIATED refused to deliver the raw sugar or pay for the defendant-appellee Francisco Sycip to pay, jointly and severally with
refined sugar delivered to it, inspite of repeated demands therefore,
the Associated Finance Company, Inc., the sum of money which the
NAMARCO instituted the present action in the lower court to recover trial court sentenced the latter to pay to the National Marketing
the sum of P403,514.28 in payment of the raw sugar received by Corporation, as follows: the sum of FOUR HUNDRED THREE
defendants from it; P80,702.86 as liquidated damages; P10,000.00 as
THOUSAND FIVE HUNDRED FOURTEEN PESOS, and TWENTY-
attorney's fees, expenses of litigation and exemplary damages, with EIGHT CENTAVOS P403,514.28), with interest at the legal rate from
legal interest thereon from the filing of the complaint until fully paid. the date of the filing of the action until fully paid plus an additional
amount of EIGHTY THOUSAND SEVEN HUNDRED TWO PESOS
In their amended answer defendants, by way of affirmative defenses, and EIGHTY-SIX CENTAVOS (P80,702.86) as liquidated damages
alleged that the correct value of the sugar delivered by NAMARCO to and P5,000.00 as attorney's fees and further to pay the costs. With
them was P259,451.09 or P13.30 per bag of 100 lbs. weight (quedan costs.
basis) and not P403,514.38 as claimed by NAMARCO. As
counterclaim they prayed for the award of P500,000.00 as moral
damages, P100,000.00 as exemplary damages and P10,000.00 as
attorney's fees.

17
G.R. No. L-41337 June 30, 1988 1975 (Ibid., pp. 64-69), ruled in favor of PADCO. The decretal portion
of the said order, reads:
TAN BOON BEE & CO., INC., petitioner,
vs. WHEREFORE, the sale of the 'Heidelberg cylinder
THE HONORABLE HILARION U. JARENCIO, PRESIDING JUDGE press executed by the Sheriff in favor of the
OF BRANCH XVIII of the Court of First Instance of Manila, plaintiff as well as the levy on the said property is
GRAPHIC PUBLISHING, INC., and PHILIPPINE AMERICAN CAN hereby set aside and declared to be without any
DRUG COMPANY, respondents. force and effect. The Sheriff is ordered to return
the said machinery to its owner, the Philippine
American Drug Co.
De Santos, Balgos & Perez Law Office for petitioner.

Petitioner filed a Motion For Reconsideration (Ibid., pp. 7093) and an


Araneta Mendoza & Papa Law Office for respondent Phil. American
Addendum to Motion for Reconsideration (Ibid., pp. 94-08), but in an
Drug Company.
Order dated August 13, 1975, the same was denied for lack of merit
(Ibid., p. 109). Hence, the instant petition.

In a Resolution dated September 12, 1975, the Second Division of this


PARAS, J.: Court resolved to require the respondents to comment, and to issue a
temporary restraining order (Rollo, p. 111 ). After submission of the
parties' Memoranda, the case was submitted for decision in the
This is a petition for certiorari, with prayer for preliminary injunction, to Resolution of November 28, 1975 (Ibid., p. 275).
annul and set aside the March 26, 1975 Order of the then Court of First
Instance of Manila, Branch XXIII, setting aside the sale of "Heidelberg"
cylinder press executed by the sheriff in favor of the herein petitioner, Petitioner, to support its stand, raised two (2) issues, to wit:
as well as the levy on the said property, and ordering the sheriff to
return the said machinery to its owner, herein private respondent
I
Philippine American Drug Company.

THE RESPONDENT JUDGE GRAVELY EXCEEDED, IF NOT ACTED


Petitioner herein, doing business under the name and style of Anchor
WITHOUT JURISDICTION WHEN HE ACTED UPON THE MOTION
Supply Co., sold on credit to herein private respondent Graphic
OF PADCO, NOT ONLY BECAUSE SECTION 17, RULE 39 OF THE
Publishing, Inc. (GRAPHIC for short) paper products amounting to
RULES OF COURT WAS NOT COMPLIED WITH, BUT ALSO
P55,214.73. On December 20, 1972, GRAPHIC made partial payment
BECAUSE THE CLAIMS OF PADCO WHICH WAS NOT A PARTY TO
by check to petitioner in the total amount of P24,848.74; and on
THE CASE COULD NOT BE VENTILATED IN THE CASE BEFORE
December 21, 1972, a promissory note was executed to cover the
HIM BUT IN INDEPENDENT PROCEEDING.
balance of P30,365.99. In the said promissory note, it was stipulated
that the amount will be paid on monthly installments and that failure to
pay any installment would make the amount immediately demandable II
with an interest of 12% per annum. On September 6, 1973, for failure
of GRAPHIC to pay any installment, petitioner filed with the then Court
THE RESPONDENT JUDGE GRAVELY ABUSED HIS DISCRETION
of First Instance of Manila, Branch XXIII, presided over by herein
WHEN HE REFUSED TO PIERCE THE PADCO'S (IDENTITY) AND
respondent judge, Civil Case No. 91857 for a Sum of Money (Rollo, pp.
DESPITE THE ABUNDANCE OF EVIDENCE CLEARLY SHOWING
36-38). Respondent judge declared GRAPHIC in default for failure to
THAT PADCO WAS CONVENIENTLY SHIELDING UNDER THE
file its answer within the reglementary period and plaintiff (petitioner
THEORY OF CORPORATE PETITION.
herein) was allowed to present its evidence ex parte. In a Decision
dated January 18, 1974 (Ibid., pp. 39-40), the trial court ordered
GRAPHIC to pay the petitioner the sum of P30,365.99 with 12% Petitioner contends that respondent judge gravely exceeded, if not,
interest from March 30, 1973 until fully paid, plus the costs of suit. On acted without jurisdiction, in nullifying the sheriffs sale not only because
motion of petitioner, a writ of execution was issued by respondent Section 17, Rule 39 of the Rules of Court was not complied with, but
judge; but the aforestated writ having expired without the sheriff finding more importantly because PADCO could not have litigated its claim in
any property of GRAPHIC, an alias writ of execution was issued on the same case, but in an independent civil proceeding.
July 2, 1974.
This contention is well-taken.
Pursuant to the said issued alias writ of execution, the executing sheriff
levied upon one (1) unit printing machine Identified as "Original
Heidelberg Cylinder Press" Type H 222, NR 78048, found in the In the case of Bayer Philippines, Inc. vs. Agana (63 SCRA 355, 366-
premises of GRAPHIC. In a Notice of Sale of Execution of Personal 367 [1975]), this Court categorically ruled as follows:
Property dated July 29, 1974, said printing machine was scheduled for
auction sale on July 26, 1974 at 10:00 o'clock at 14th St., Cor. Atlanta In other words, constitution, Section 17 of Rule 39
St., Port Area, Manila (lbid., p. 45); but in a letter dated July 19, 1974, of the Revised Rules of Court, the rights of third-
herein private respondent, Philippine American Drug Company party claimants over certain properties levied upon
(PADCO for short) had informed the sheriff that the printing machine is by the sheriff to satisfy the judgment should not be
its property and not that of GRAPHIC, and accordingly, advised the decided inthe action where the third-party claims
sheriff to cease and desist from carrying out the scheduled auction sale have been presented, but in the separate action
on July 26, 1974. Notwithstanding the said letter, the sheriff proceeded instituted by the claimants.
with the scheduled auction sale, sold the property to the petitioner, it
being the highest bidder, and issued a Certificate of Sale in favor of
petitioner (Rollo, p. 48). More than five (5) hours after the auction sale ... Otherwise stated, the court issuing a writ of
and the issuance of the certificate of sale, PADCO filed an "Affidavit of execution is supposed to enforce the authority only
Third Party Claim" with the Office of the City Sheriff (Ibid., p. 47). over properties of the judgment debtor, and should
Thereafter, on July 30,1974, PADCO filed with the Court of First a third party appeal- to claim the property levied
Instance of Manila, Branch XXIII, a Motion to Nullify Sale on Execution upon by the sheriff, the procedure laid down by the
(With Injunction) (Ibid., pp, 49-55), which was opposed by the petitioner Rules is that such claim should be the subject of a
(Ibid., pp. 5668). Respondent judge, in an Order dated March 26, separate and independent action.

18
xxx xxx xxx Capitol Publishing. That the said machine was allegedly leased by
PADCO to GRAPHIC on January 24, 1966, even before PADCO
purchased it from Capital Publishing on July 11, 1966, only serves to
... This rule is dictated by reasons of convenience,
show that PADCO's claim of ownership over the printing machine is
as "intervention is more likely to inject confusion
not only farce and sham but also unbelievable.
into the issues between the parties in the case . . .
with which the third-party claimant has nothing to
do and thereby retard instead of facilitate the Considering the aforestated principles and the circumstances
prompt dispatch of the controversy which is the established in this case, respondent judge should have pierced
underlying objective of the rules of pleading and PADCO's veil of corporate Identity.
practice." Besides, intervention may not be
permitted after trial has been concluded and a final
Respondent PADCO argues that if respondent judge erred in not
judgment rendered in the case.
piercing the veil of its corporate fiction, the error is merely an error of
judgment and not an error of jurisdiction correctable by appeal and not
However, the fact that petitioner questioned the jurisdiction of the court by certiorari.
during the initial hearing of the case but nevertheless actively
participated in the trial, bars it from questioning now the court's
To this argument of respondent, suffice it to say that the same is a
jurisdiction. A party who voluntarily participated in the trial, like the
mere technicality. In the case of Rubio vs. Mariano (52 SCRA 338, 343
herein petitioner, cannot later on raise the issue of the court's lack of
[1973]), this Court ruled:
jurisdiction (Philippine National Bank vs. Intermediate Appellate Court,
143 SCRA [1986]).
While We recognize the fact that these movants —
the MBTC, the Phillips spouses, the Phillips
As to the second issue (the non-piercing of PADCO's corporate
corporation and the Hacienda Benito, Inc.— did
Identity) the decision of respondent judge is as follows:
raise in their respective answers the issue as to
the propriety of the instant petition for certiorari on
The plaintiff, however, contends that the the ground that the remedy should have been
controlling stockholders of the Philippine American appeal within the reglementary period, We
Drug Co. are also the same controlling considered such issue as a mere technicality
stockholders of the Graphic Publishing, Inc. and, which would have accomplished nothing
therefore, the levy upon the said machinery which substantial except to deny to the petitioner the
was found in the premises occupied by the right to litigate the matters he raised ...
Graphic Publishing, Inc. should be upheld. This
contention cannot be sustained because the two
Litigations should, as much as possible, be decided on their merits and
corporations were duly incorporated under the
not on technicality (De las Alas vs. Court of Appeals, 83 SCRA 200,
Corporation Law and each of them has a juridical
216 [1978]). Every party-litigant must be afforded the amplest
personality distinct and separate from the other
opportunity for the proper and just determination of his cause, free from
and the properties of one cannot be levied upon to
the unacceptable plea of technicalities (Heirs of Ceferino Morales vs.
satisfy the obligation of the other. This legal
Court of Appeals, 67 SCRA 304, 310 [1975]).
preposition is elementary and fundamental.

PREMISES CONSIDERED, the March 26,1975 Order of the then


It is true that a corporation, upon coming into being, is invested by law
Court of First Instance of Manila, is ANNULLED and SET ASIDE, and
with a personality separate and distinct from that of the persons
the Temporary Restraining Order issued is hereby made permanent.
composing it as well as from any other legal entity to which it may be
related (Yutivo & Sons Hardware Company vs. Court of Tax Appeals, 1
SCRA 160 [1961]; and Emilio Cano Enterprises, Inc. vs. CIR, 13 SCRA SO ORDERED.
290 [1965]). As a matter of fact, the doctrine that a corporation is a
legal entity distinct and separate from the members and stockholders
who compose it is recognized and respected in all cases which are
within reason and the law (Villa Rey Transit, Inc. vs. Ferrer, 25 SCRA
845 [1968]). However, this separate and distinct personality is merely a
fiction created by law for convenience and to promote justice (Laguna
Transportation Company vs. SSS, 107 Phil. 833 [1960]). Accordingly,
this separate personality of the corporation may be disregarded, or the
veil of corporate fiction pierced, in cases where it is used as a cloak or
cover for fraud or illegality, or to work an injustice, or where necessary
to achieve equity or when necessary for the protection of creditors
(Sulo ng Bayan, Inc. vs. Araneta, Inc., 72 SCRA 347 [1976]).
Corporations are composed of natural persons and the legal fiction of a
separate corporate personality is not a shield for the commission of
injustice and inequity (Chenplex Philippines, Inc., et al. vs. Hon.
Pamatian et al., 57 SCRA 408 (19741). Likewise, this is true when the
corporation is merely an adjunct, business conduit or alter ego of
another corporation. In such case, the fiction of separate and distinct
corporation entities should be disregarded (Commissioner of Internal
Revenue vs. Norton & Harrison, 11 SCRA 714 [1964]).

In the instant case, petitioner's evidence established that PADCO was


never engaged in the printing business; that the board of directors and
the officers of GRAPHIC and PADCO were the same; and that PADCO
holds 50% share of stock of GRAPHIC. Petitioner likewise stressed
that PADCO's own evidence shows that the printing machine in
question had been in the premises of GRAPHIC since May, 1965, long
before PADCO even acquired its alleged title on July 11, 1966 from
19
G.R. No. 58168 December 19, 1989 shares in SUBIC cannot legally entitle them to intervene because
SUBIC has a personality separate and distinct from its stockholders.
CONCEPCION MAGSAYSAY-LABRADOR, SOLEDAD
MAGSAYSAY-CABRERA, LUISA MAGSAYSAY-CORPUZ, assisted On appeal, respondent Court of Appeals found no factual or legal
be her husband, Dr. Jose Corpuz, FELICIDAD P. MAGSAYSAY, justification to disturb the findings of the lower court. The appellate
and MERCEDES MAGSAYSAY-DIAZ, petitioners, court further stated that whatever claims the petitioners have against
vs. the late Senator or against SUBIC for that matter can be ventilated in a
THE COURT OF APPEALS and ADELAIDA RODRIGUEZ- separate proceeding, such that with the denial of the motion for
MAGSAYSAY, Special Administratrix of the Estate of the late intervention, they are not left without any remedy or judicial relief under
Genaro F. Magsaysay respondents. existing law.

Petitioners' motion for reconsideration was denied. Hence, the instant


recourse.
FERNAN, C.J.:
Petitioners anchor their right to intervene on the purported assignment
made by the late Senator of a certain portion of his shareholdings to
In this petition for review on certiorari, petitioners seek to reverse and
them as evidenced by a Deed of Sale dated June 20, 1978. 2 Such
set aside [1] the decision of the Court of Appeals dated July l3,
transfer, petitioners posit, clothes them with an interest, protected by
1981, 1 affirming that of the Court of First Instance of Zambales and
law, in the matter of litigation.
Olongapo City which denied petitioners' motion to intervene in an
annulment suit filed by herein private respondent, and [2] its resolution
dated September 7, 1981, denying their motion for reconsideration. Invoking the principle enunciated in the case of PNB v. Phil. Veg. Oil
Co., 49 Phil. 857,862 & 853 (1927), 3 petitioners strongly argue that
their ownership of 41.66% of the entire outstanding capital stock of
Petitioners are raising a purely legal question; whether or not
SUBIC entitles them to a significant vote in the corporate affairs; that
respondent Court of Appeals correctly denied their motion for
they are affected by the action of the widow of their late brother for it
intervention.
concerns the only tangible asset of the corporation and that it appears
that they are more vitally interested in the outcome of the case than
The facts are not controverted. SUBIC.

On February 9, 1979, Adelaida Rodriguez-Magsaysay, widow and Viewed in the light of Section 2, Rule 12 of the Revised Rules of Court,
special administratix of the estate of the late Senator Genaro this Court affirms the respondent court's holding that petitioners herein
Magsaysay, brought before the then Court of First Instance of have no legal interest in the subject matter in litigation so as to entitle
Olongapo an action against Artemio Panganiban, Subic Land them to intervene in the proceedings below. In the case of Batama
Corporation (SUBIC), Filipinas Manufacturer's Bank (FILMANBANK) Farmers' Cooperative Marketing Association, Inc. v. Rosal, 4 we held:
and the Register of Deeds of Zambales. In her complaint, she alleged "As clearly stated in Section 2 of Rule 12 of the Rules of Court, to be
that in 1958, she and her husband acquired, thru conjugal funds, a permitted to intervene in a pending action, the party must have a legal
parcel of land with improvements, known as "Pequena Island", covered interest in the matter in litigation, or in the success of either of the
by TCT No. 3258; that after the death of her husband, she discovered parties or an interest against both, or he must be so situated as to be
[a] an annotation at the back of TCT No. 3258 that "the land was adversely affected by a distribution or other disposition of the property
acquired by her husband from his separate capital;" [b] the registration in the custody of the court or an officer thereof ."
of a Deed of Assignment dated June 25, 1976 purportedly executed by
the late Senator in favor of SUBIC, as a result of which TCT No. 3258
To allow intervention, [a] it must be shown that the movant has legal
was cancelled and TCT No. 22431 issued in the name of SUBIC; and
interest in the matter in litigation, or otherwise qualified; and [b]
[c] the registration of Deed of Mortgage dated April 28, 1977 in the
consideration must be given as to whether the adjudication of the rights
amount of P 2,700,000.00 executed by SUBIC in favor of
of the original parties may be delayed or prejudiced, or whether the
FILMANBANK; that the foregoing acts were void and done in an
intervenor's rights may be protected in a separate proceeding or not.
attempt to defraud the conjugal partnership considering that the land is
Both requirements must concur as the first is not more important than
conjugal, her marital consent to the annotation on TCT No. 3258 was
the second. 5
not obtained, the change made by the Register of Deeds of the
titleholders was effected without the approval of the Commissioner of
Land Registration and that the late Senator did not execute the The interest which entitles a person to intervene in a suit between
purported Deed of Assignment or his consent thereto, if obtained, was other parties must be in the matter in litigation and of such direct and
secured by mistake, violence and intimidation. She further alleged that immediate character that the intervenor will either gain or lose by the
the assignment in favor of SUBIC was without consideration and direct legal operation and effect of the judgment. Otherwise, if persons
consequently null and void. She prayed that the Deed of Assignment not parties of the action could be allowed to intervene, proceedings will
and the Deed of Mortgage be annulled and that the Register of Deeds become unnecessarily complicated, expensive and interminable. And
be ordered to cancel TCT No. 22431 and to issue a new title in her this is not the policy of the law. 6
favor.
The words "an interest in the subject" mean a direct interest in the
On March 7, 1979, herein petitioners, sisters of the late senator, filed a cause of action as pleaded, and which would put the intervenor in a
motion for intervention on the ground that on June 20, 1978, their legal position to litigate a fact alleged in the complaint, without the
brother conveyed to them one-half (1/2 ) of his shareholdings in SUBIC establishment of which plaintiff could not recover. 7
or a total of 416,566.6 shares and as assignees of around 41 % of the
total outstanding shares of such stocks of SUBIC, they have a
Here, the interest, if it exists at all, of petitioners-movants is indirect,
substantial and legal interest in the subject matter of litigation and that
contingent, remote, conjectural, consequential and collateral. At the
they have a legal interest in the success of the suit with respect to
very least, their interest is purely inchoate, or in sheer expectancy of a
SUBIC.
right in the management of the corporation and to share in the profits
thereof and in the properties and assets thereof on dissolution, after
On July 26, 1979, the court denied the motion for intervention, and payment of the corporate debts and obligations.
ruled that petitioners have no legal interest whatsoever in the matter in
litigation and their being alleged assignees or transferees of certain

20
While a share of stock represents a proportionate or aliquot interest in
the property of the corporation, it does not vest the owner thereof with
any legal right or title to any of the property, his interest in the
corporate property being equitable or beneficial in nature.
Shareholders are in no legal sense the owners of corporate property,
which is owned by the corporation as a distinct legal person. 8

Petitioners further contend that the availability of other remedies, as


declared by the Court of appeals, is totally immaterial to the availability
of the remedy of intervention.

We cannot give credit to such averment. As earlier stated, that the


movant's interest may be protected in a separate proceeding is a factor
to be considered in allowing or disallowing a motion for intervention. It
is significant to note at this juncture that as per records, there are four
pending cases involving the parties herein, enumerated as follows: [1]
Special Proceedings No. 122122 before the CFI of Manila, Branch
XXII, entitled "Concepcion Magsaysay-Labrador, et al. v. Subic Land
Corp., et al.", involving the validity of the transfer by the late Genaro
Magsaysay of one-half of his shareholdings in Subic Land Corporation;
[2] Civil Case No. 2577-0 before the CFI of Zambales, Branch III,
"Adelaida Rodriguez-Magsaysay v. Panganiban, etc.; Concepcion
Labrador, et al. Intervenors", seeking to annul the purported Deed of
Assignment in favor of SUBIC and its annotation at the back of TCT
No. 3258 in the name of respondent's deceased husband; [3] SEC
Case No. 001770, filed by respondent praying, among other things that
she be declared in her capacity as the surviving spouse and
administratrix of the estate of Genaro Magsaysay as the sole
subscriber and stockholder of SUBIC. There, petitioners, by motion,
sought to intervene. Their motion to reconsider the denial of their
motion to intervene was granted; [4] SP No. Q-26739 before the CFI of
Rizal, Branch IV, petitioners herein filing a contingent claim pursuant to
Section 5, Rule 86, Revised Rules of Court. 9 Petitioners' interests are
no doubt amply protected in these cases.

Neither do we lend credence to petitioners' argument that they are


more interested in the outcome of the case than the corporation-
assignee, owing to the fact that the latter is willing to compromise with
widow-respondent and since a compromise involves the giving of
reciprocal concessions, the only conceivable concession the
corporation may give is a total or partial relinquishment of the corporate
assets. 10

Such claim all the more bolsters the contingent nature of petitioners'
interest in the subject of litigation.

The factual findings of the trial court are clear on this point. The
petitioners cannot claim the right to intervene on the strength of the
transfer of shares allegedly executed by the late Senator. The
corporation did not keep books and records. 11 Perforce, no transfer
was ever recorded, much less effected as to prejudice third parties.
The transfer must be registered in the books of the corporation to affect
third persons. The law on corporations is explicit. Section 63 of the
Corporation Code provides, thus: "No transfer, however, shall be valid,
except as between the parties, until the transfer is recorded in the
books of the corporation showing the names of the parties to the
transaction, the date of the transfer, the number of the certificate or
certificates and the number of shares transferred."

And even assuming arguendo that there was a valid transfer,


petitioners are nonetheless barred from intervening inasmuch as their
rights can be ventilated and amply protected in another proceeding.

WHEREFORE, the instant petition is hereby DENIED. Costs against


petitioners.

SO ORDERED.

21
G.R. No. 96490 February 3, 1992 The petitioner's contention was opposed by private respondent which
submits that it is a juridical entity separate and distinct from Acrylic.
INDOPHIL TEXTILE MILL WORKERS UNION-PTGWO, petitioner,
vs. The existing impasse led the petitioner and private respondent to enter
VOLUNTARY ARBITRATOR TEODORICO P. CALICA and into a submission agreement on September 6, 1990. The parties jointly
INDOPHIL TEXTILE MILLS, INC., respondents. requested the public respondent to act as voluntary arbitrator in the
resolution of the pending labor dispute pertaining to the proper
interpretation of the CBA provision.
Romeo C. Lagman for petitioner.

After the parties submitted their respective position papers and replies,
Borreta, Gutierrez & Leogardo for respondent Indophil Textile Mills,
the public respondent Voluntary Arbitrator rendered its award on
Inc.
December 8, 1990, the dispositive portion of which provides as follows:

PREMISES CONSIDERED, it would be a strained


interpretation and application of the questioned
MEDIALDEA, J.: CBA provision if we would extend to the
employees of Acrylic the coverage clause of
Indophil Textile Mills CBA. Wherefore, an award is
This is a petition for certiorari seeking the nullification of the award made to the effect that the proper interpretation
issued by the respondent Voluntary Arbitrator Teodorico P. Calica
and application of Sec. l, (c), Art. I, of the 1987
dated December 8, 1990 finding that Section 1 (c), Article I of the CBA do (sic) not extend to the employees of
Collective Bargaining Agreement between Indophil Textile Mills, Inc. Acrylic as an extension or expansion of Indophil
and Indophil Textile Mill Workers Union-PTGWO does not extend to
Textile Mills, Inc. (Rollo, p.21)
the employees of Indophil Acrylic Manufacturing Corporation as an
extension or expansion of Indophil Textile Mills, Incorporated.
Hence, this petition raising four (4) issues, to wit:
The antecedent facts are as follows:
1. WHETHER OR NOT THE
RESPONDENT
Petitioner Indophil Textile Mill Workers Union-PTGWO is a legitimate
ARBITRATOR ERRED IN
labor organization duly registered with the Department of Labor and INTERPRETING SECTION
Employment and the exclusive bargaining agent of all the rank-and-file 1(c), ART I OF THE CBA
employees of Indophil Textile Mills, Incorporated. Respondent BETWEEN PETITIONER
Teodorico P. Calica is impleaded in his official capacity as the UNION AND RESPONDENT
Voluntary Arbitrator of the National Conciliation and Mediation Board of COMPANY.
the Department of Labor and Employment, while private respondent
Indophil Textile Mills, Inc. is a corporation engaged in the manufacture,
sale and export of yarns of various counts and kinds and of materials 2. WHETHER OR NOT
of kindred character and has its plants at Barrio Lambakin. Marilao, INDOPHIL ACRYLIC IS A
Bulacan. SEPARATE AND DISTINCT
ENTITY FROM
RESPONDENT COMPANY
In April, 1987, petitioner Indophil Textile Mill Workers Union-PTGWO FOR PURPOSES OF UNION
and private respondent Indophil Textile Mills, Inc. executed a collective REPRESENTATION.
bargaining agreement effective from April 1, 1987 to March 31, 1990.

3. WHETHER OR NOT THE


On November 3, 1967 Indophil Acrylic Manufacturing Corporation was RESPONDENT
formed and registered with the Securities and Exchange Commission.
ARBITRATOR GRAVELY
Subsequently, Acrylic applied for registration with the Board of ABUSED HIS DISCRETION
Investments for incentives under the 1987 Omnibus Investments Code. AMOUNTING TO LACK OR
The application was approved on a preferred non-pioneer status.
IN EXCESS OF HIS
JURISDICTION.
In 1988, Acrylic became operational and hired workers according to its
own criteria and standards. Sometime in July, 1989, the workers of
4. WHETHER OR NOT THE
Acrylic unionized and a duly certified collective bargaining agreement RESPONDENT
was executed. ARBITRATOR VIOLATED
PETITIONER UNION'S
In 1990 or a year after the workers of Acrylic have been unionized and CARDINAL PRIMARY RIGHT
a CBA executed, the petitioner union claimed that the plant facilities TO DUE PROCESS. (Rollo,
built and set up by Acrylic should be considered as an extension or pp. 6-7)
expansion of the facilities of private respondent Company pursuant to
Section 1(c), Article I of the CBA, to wit,. The central issue submitted for arbitration is whether or not the
operations in Indophil Acrylic Corporation are an extension or
c) This Agreement shall apply expansion of private respondent Company. Corollary to the
to the Company's plant aforementioned issue is the question of whether or not the rank-and-
facilities and installations and file employees working at Indophil Acrylic should be recognized as part
to any extension and of, and/or within the scope of the bargaining unit.
expansion thereat. (Rollo, p.4)
Petitioner maintains that public respondent Arbitrator gravely erred in
In other words, it is the petitioner's contention that Acrylic is interpreting Section l(c), Article I of the CBA in its literal meaning
part of the Indophil bargaining unit. without taking cognizance of the facts adduced that the creation of the
aforesaid Indophil Acrylic is but a devise of respondent Company to
2
evade the application of the CBA between petitioner Union and which are not really essential in the actual production of Acrylic. It also
respondent Company. pointed out that the essential services are discharged exclusively by
Acrylic personnel under the control and supervision of Acrylic
managers and supervisors.
Petitioner stresses that the articles of incorporation of the two
corporations establish that the two entities are engaged in the same
kind of business, which is the manufacture and sale of yarns of various In sum, petitioner insists that the public respondent committed grave
counts and kinds and of other materials of kindred character or nature. abuse of discretion amounting to lack or in excess of jurisdiction in
erroneously interpreting the CBA provision and in failing to disregard
the corporate entity of Acrylic.
Contrary to petitioner's assertion, the public respondent through the
Solicitor General argues that the Indophil Acrylic Manufacturing
Corporation is not an alter ego or an adjunct or business conduit of We find the petition devoid of merit.
private respondent because it has a separate legitimate business
purpose. In addition, the Solicitor General alleges that the primary
Time and again, We stress that the decisions of voluntary arbitrators
purpose of private respondent is to engage in the business of
are to be given the highest respect and a certain measure of finality,
manufacturing yarns of various counts and kinds and textiles. On the
but this is not a hard and fast rule, it does not preclude judicial review
other hand, the primary purpose of Indophil Acrylic is to manufacture,
thereof where want of jurisdiction, grave abuse of discretion, violation
buy, sell at wholesale basis, barter, import, export and otherwise deal
of due process, denial of substantial justice, or erroneous interpretation
in yarns of various counts and kinds. Hence, unlike private respondent,
of the law were brought to our attention. (see Ocampo, et al. v.
Indophil Acrylic cannot manufacture textiles while private respondent
National Labor Relations Commission, G.R. No. 81677, 25 July 1990,
cannot buy or import yarns.
First Division Minute Resolution citing Oceanic Bic Division (FFW) v.
Romero, G.R. No. L-43890, July 16, 1984, 130 SCRA 392)
Furthermore, petitioner emphasizes that the two corporations have
practically the same incorporators, directors and officers. In fact, of the
It should be emphasized that in rendering the subject arbitral award,
total stock subscription of Indophil Acrylic, P1,749,970.00 which
the voluntary arbitrator Teodorico Calica, a professor of the U.P. Asian
represents seventy percent (70%) of the total subscription of
Labor Education Center, now the Institute for Industrial Relations,
P2,500,000.00 was subscribed to by respondent Company.
found that the existing law and jurisprudence on the matter, supported
the private respondent's contentions. Contrary to petitioner's assertion,
On this point, private respondent cited the case of Diatagon Labor public respondent cited facts and the law upon which he based the
Federation v. Ople, G.R. No. L-44493-94, December 3, 1980, 10l award. Hence, public respondent did not abuse his discretion.
SCRA 534, which ruled that two corporations cannot be treated as a
single bargaining unit even if their businesses are related. It submits
Under the doctrine of piercing the veil of corporate entity, when valid
that the fact that there are as many bargaining units as there are
grounds therefore exist, the legal fiction that a corporation is an entity
companies in a conglomeration of companies is a positive proof that a
with a juridical personality separate and distinct from its members or
corporation is endowed with a legal personality distinctly its own,
stockholders may be disregarded. In such cases, the corporation will
independent and separate from other corporations (see Rollo, pp. 160-
be considered as a mere association of persons. The members or
161).
stockholders of the corporation will be considered as the corporation,
that is liability will attach directly to the officers and stockholders. The
Petitioner notes that the foregoing evidence sufficiently establish that doctrine applies when the corporate fiction is used to defeat public
Acrylic is but an extension or expansion of private respondent, to wit: convenience, justify wrong, protect fraud, or defend crime, or when it is
made as a shield to confuse the legitimate issues, or where a
corporation is the mere alter ego or business conduit of a person, or
(a) the two corporations have
where the corporation is so organized and controlled and its affairs are
their physical plants, offices
so conducted as to make it merely an instrumentality, agency, conduit
and facilities situated in the
or adjunct of another corporation. (Umali et al. v. Court of Appeals,
same compound, at Barrio
G.R. No. 89561, September 13, 1990, 189 SCRA 529, 542)
Lambakin, Marilao, Bulacan;

In the case at bar, petitioner seeks to pierce the veil of corporate entity
(b) many of private
of Acrylic, alleging that the creation of the corporation is a devise to
respondent's own
evade the application of the CBA between petitioner Union and private
machineries, such as dyeing
respondent Company. While we do not discount the possibility of the
machines, reeling, boiler,
similarities of the businesses of private respondent and Acrylic, neither
Kamitsus among others, were
are we inclined to apply the doctrine invoked by petitioner in granting
transferred to and are now
the relief sought. The fact that the businesses of private respondent
installed and being used in the
and Acrylic are related, that some of the employees of the private
Acrylic plant;
respondent are the same persons manning and providing for auxilliary
services to the units of Acrylic, and that the physical plants, offices and
(c) the services of a number of facilities are situated in the same compound, it is our considered
units, departments or sections opinion that these facts are not sufficient to justify the piercing of the
of private respondent are corporate veil of Acrylic.
provided to Acrylic; and
In the same case of Umali, et al. v. Court of Appeals (supra), We
(d) the employees of private already emphasized that "the legal corporate entity is disregarded only
respondent are the same if it is sought to hold the officers and stockholders directly liable for a
persons manning and corporate debt or obligation." In the instant case, petitioner does not
servicing the units of Acrylic. seek to impose a claim against the members of the Acrylic.
(see Rollo, pp. 12-13)
Furthermore, We already ruled in the case of Diatagon Labor
Private respondent insists that the existence of a bonafide business Federation Local 110 of the ULGWP v. Ople (supra) that it is grave
relationship between Acrylic and private respondent is not a proof of abuse of discretion to treat two companies as a single bargaining unit
being a single corporate entity because the services which are when these companies are indubitably distinct entities with separate
supposedly provided by it to Acrylic are auxiliary services or activities juridical personalities.
23
Hence, the Acrylic not being an extension or expansion of private
respondent, the rank-and-file employees working at Acrylic should not
be recognized as part of, and/or within the scope of the petitioner, as
the bargaining representative of private respondent.

All premises considered, the Court is convinced that the public


respondent Voluntary Arbitrator did not commit grave abuse of
discretion in its interpretation of Section l(c), Article I of the CBA that
the Acrylic is not an extension or expansion of private respondent.

ACCORDINGLY, the petition is DENIED and the award of the


respondent Voluntary Arbitrator are hereby AFFIRMED.

SO ORDERED.

24
G.R. No. 80043 June 6, 1991 veil of corporate fiction and on the liability of herein petitioner are
overwhelmingly supported by the evidence.
ROBERTO A. JACINTO, petitioner,
vs. Insofar as material and relevant to the issues raised, the trial court
HONORABLE COURT OF APPEALS and METROPOLITAN BANK found and held:11
AND TRUST COMPANY, respondents.
As to [the] liability of [the] defendant Roberto A. Jacinto, it
Romeo G. Carlos for petitioner. would appear that he is in factetum (sic), or, in fact, the
Jorge, Perez & Associates for private respondents. corporation itself known as Inland Industries, Inc. Aside from
the fact that he is admittedly the President and General
Manager of the corporation and a substantial stockholders
(sic) thereof, it was defendant Roberto A. Jacinto who dealt
entirely with the plaintiff in those transactions. In the Trust
Receipts that he signed supposedly in behalf of Inland
DAVIDE, JR., J.: Industries, Inc., it is not even mentioned that he did so in this
official capacity.
This is an appeal by certiorari to partially set aside the Decision of the
Court of Appeals in C.A-G.R. CV No. 081531.promulgated on 19 xxx xxx xxx
August 1987, which affirmed in toto the decision of the Regional Trial
Court of Manila, Branch 11, in Civil Case No. 133164 entitled
In this case, the Court is satisfied that Roberto A. Jacinto
"Metropolitan Bank and Trust Co. vs. Inland Industries Inc. and
was practically the corporation itself, the Inland industries,
Roberto Jacinto," the dispositive portion of which reads:
Inc.

WHEREFORE, judgment is hereby rendered ordering


In a detailed fashion, the respondent Court of Appeals brushed aside
defendants to pay, jointly and severally, the plaintiff, the
the posturing of petitioner as follows:
principal obligation of P382,015.80 (Annex J-1 to J-3 of
Stipulation), with interest/charges thereon at the rate of 16
% per annum from January 1, 1979 up to the time the said Defendant Roberto Jacinto, tried to escape liability and shift
amount is fully paid, plus the sum of P20,000.00 as the entire blame under the trust receipts solely and
attorney's fees. Said defendants are further ordered to pay exclusively on defendant-appellant corporation. He asserted
in solidum the costs of this suit. that he cannot be held solidarily liable with the latter
(defendant corporation) because he just signed said
instruments in his official capacity as president of Inland
SO ORDERED.2
Industries, Inc. and the latter (defendant corporation) has a
juridical personality distinct and separate from its officers and
Petitioner's co-defendant in the courts below, Inland Industries Inc., stockholders. It is likewise asserted, citing an American
just as in the case of petitioner's motion to reconsider the questioned case, that the principle of piercing the fiction of corporate
decision,3 chose not to join him in this appeal. entity should be applied with great caution and not
precipitately, because a dual personality by a corporation
and its stockholders would defeat the principal purpose for
In Our resolution of 28 August 1988 We required the respondent to
which a corporation is formed. Upon the other hand, plaintiff-
comment on the petition. Respondent Metropolitan Bank and Trust Co. appellee reiterated its allegation in the complaint that
filed its comment4 on 12 October 1988. We required the petitioner to defendant corporation is just a mere alter ego of defendant
file a reply thereto,5 which he comment plied with on 20 December
Roberto Jacinto who is its President and General Manager,
1988.6 while the wife of the latter owns a majority of its shares of
stock.
We gave due course to the petition on 8 May 19897 and required the
parties to submit their respective memoranda. Defendants-appellants' assertion is plainly without legal
basis. This is shown by the undisputed fact that Roberto
Private respondent filed its memorandum on 29 June 19898 while Jacinto even admitted that he and his wife own 52% of the
petitioner asked leave to adopt his petition and reply as his stocks of defendant corporation (TSN, April 22, 1985, p. 6).
memorandum,9 which We granted on 14 June 1989.10 We cannot accept as true the assertion of defendant Jacinto
that he only acted in his official capacity as President and
General Manager of Inland Industries, Inc. when he signed
Petitioner submits the following issues: the aforesaid trust receipts. To Our mind the same is just a
clever ruse and a convenient ploy to thwart his personal
1. Whether or not the respondent Court of Appeals can liability therefor by taking refuge under the protective mantle
validly pierce the fiction of corporate identity of the defendant of the separate corporate personality of defendant
corporation Inland Industries, Inc. even if there is no corporation.
allegation in the complaint regarding the same, nor is there
anything in the prayer demanding the piercing of the As could be expected, Roberto Jacinto in his direct testimony
corporate veil of the corporation Inland Industries, Inc.; presented a different corporate scenario regarding Inland
Industries, Inc. and vehemently declared that it is Bienvenida
2. Whether or not the Court of Appeals can validly pierce the Catabas who is its President, while Aurora Heresa is its
fiction of corporate identity of the defendant Inland Chairman of the Board. His assertion on this point, however,
Industries, Inc. even if absolutely no proof was presented in is not convincing in view of his admission in the same breath,
court to serve as legal justification for the same. that his wife, Hedy U. Jacinto, own (sic) with him 52% of the
shares of stock of said corporation. Indeed, this
circumstance –– even if standing alone –– cannot but
We find this petition to be bereft of merit. The issues are basically engender in the most unprejudiced mind doubt and misgiving
factual and a careful scrutiny of the decisions of both courts below why Catabas and Heresa would be defendant corporation's
reveals that their findings and conclusions on the matter of piercing the

25
President and Chairman of the Board, respectively. Pertinent . . . (p. 3 of Stipulations of Facts and Formulation
portion of his testimony on this point is quoted hereunder: of Issues [p. 95, Records]).

Atty. Carlos Do you know the defendant Inland The conflicting statements by defendant Jacinto place in
Industries, Inc.? extreme doubt his credibility anent his alleged participation in
said transactions and We are thus persuaded to agree with
the findings of the lower court that the latter (Roberto
A Yes, sir. Because I am the General Manager of
Jacinto) was practically the corporation itself. Indeed, a
this corporation.
painstaking examination of the records show that there is no
clear-cut delimitation between the personality of Roberto
Q Aside from being the General Manager of the Jacinto as an individual and the personality of Inland
defendant corporation are you in any other way Industries, Inc. as a corporation.
connected with the same?
The circumstances aforestated lead Us to conclude that the
A I am also a stockholder. corporate veil that en-shrouds defendant Inland Industries,
Inc. could be validly pierced, and a host of cases decided by
our High Court is supportive of this view. Thus it held that
Q Does your corporation have a Board of "when the veil of corporate fiction is made as a shield to
Directors? perpetuate fraud and/or confuse legitimate issues, the same
should be pierced." (Republic vs. Razon, 20 SCRA 234; A.D.
A Yes, sir. Santos, Inc. vs. Vasquez, 22 SCRA 1156; Emilio Cano
Enterprises, Inc. vs. Court of Appeals, 13 SCRA 290).
Almost in the same vein is the dictum enunciated by the
Q By the way, who are the stockholders of this same court in the case of Commissioner of Internal Revenue
corporation? vs. Norton & Harrison Co., (11 SCRA 714), that "Where a
corporation is merely an adjunct, business conduit or alter
A Bienvenida Catabas, Aurora Heresa, Paz Yulo, ego, the fiction of separate and distinct corporate entity
Hedy Y. Jacinto and myself. should be disregarded."

Q Who is the President of the defendant In its resolution of 29 September 1987, the respondent Court of
corporation? Appeals, on the contention again of petitioner that the finding that
defendant corporation is his mere alter ego is not supported by the
evidence and has no legal justification, ruled that:
A Bienvenida Catabas.

The contention . . . is nothing but an empty assertion. A


Q Who is the Chairman of the Board? cursory perusal of the decision would at once readily show
on pages 11-13 of the same that said factual findings of the
A Aurora Heresa. court is well grounded as the same in fact even include a
portion of the very testimony of said defendant-appellant
admitting that he and his wife own 52% of the stocks of
Q Do you have any relation with Hedy Y. Jacinto? defendant corporation. The stipulation of facts also show
(sic) that appellant Roberto Jacinto acted in his capacity as
A She is my wife. President/General Manager of defendant corporation and
that "all the goods covered by the three (3) Letters of Credit
(Annexes "A", "B" & "C") and paid for under the Bills of
Q If you combine the stockholdings of your wife Exchange (Annexes "D", "E" & "F") were delivered to and
together with yours and percentage wise, how received by defendant Inland Industries, Inc. through its co-
much is your equity? defendant Roberto A. Jacinto, its President and General
Manager, who signed for and in behalf of defendant Inland
Atty. Dizon raised some objections. However, the and agreed to the terms and conditions of three (3) separate
Court allowed the same. trust receipts covering the same.

A About 52 % (Ibid., pp. 3-6) Petitioner, however, faults the courts below for piercing the veil of
corporate fiction despite the absence of any allegation in the complaint
questioning the separate identity and existence of Inland Industries,
Furthermore, a cursory perusal of the Stipulation of facts Inc. This is not accurate.1âwphi1 While on the face of the complaint
clearly shows that defendant Roberto Jacinto acted in his there is no specific allegation that the corporation is a mere alter ego of
capacity as President and General Manager of Inland petitioner, subsequent developments, from the stipulation of facts up to
Industries, Inc. when he signed said trust receipts. Pertinent the presentation of evidence and the examination of witnesses,
portion of his testimony are quoted below: unequivocally show that respondent Metropolitan Bank and Trust
Company sought to prove that petitioner and the corporation are one or
(d) All the goods covered by the three (3) Letters that he is the corporation. No serious objection was heard from
of Credit (Annexes "A", "B" & "C") and paid for petitioner. Section 5 of Rule 10 of the Rules of Court provides:
under the Bills of Exchange (Annexes "D", "E" &
"F") were delivered to and received by defendant Sec. 5. Amendment to conform to or authorize presentation
Inland Industries, Inc. through its co-defendant of evidence. –– When issues not raised by the pleadings are
Roberto A. Jacinto, its President and General tried by express or implied consent of the parties, they shall
Manager, who signed for and in behalf of be treated in all respects, as if they had been raised in the
defendant Inland and agreed to the terms and pleadings. Such amendment of the pleadings as may be
conditions of three (3) separate trust receipts necessary to cause them to conform to the evidence and to
covering the same and herein identified as follows: raise these issues may be made upon motion of any party at
any time, even after judgment; but failure so to amend does
26
not affect the trial of these issues. If the evidence is objected
to at the time of trial on the ground that it is not within the
issues made by the pleadings, the court may allow the
pleadings to be amended and shall do so freely when the
presentation of the merits of the action will be subserved
thereby and the objecting party fails to satisfy the court that
the admission of such evidence would prejudice him in
maintaining his action or defense upon the merits. The court
may grant continuance to enable the objecting party to meet
such evidence.

Pursuant thereto, "when evidence is presented by one party, with the


express or implied consent of the adverse party, as to issues not
alleged in the pleadings, judgment may be rendered validly as regards
those issues, which shall be considered as if they have been raised in
the pleadings. There is implied consent to the evidence thus presented
when the adverse party fails to object thereto.12

WHEREFORE, for lack of merit, the Petition is DISMISSED with costs


against petitioner.

SO ORDERED.

27
G.R. No. 108734 May 29, 1996 On October 16, 1986, the NLRC Research and Information
Department made the finding that private respondents' back wages
amounted to P199,800.00.3
CONCEPT BUILDERS, INC., petitioner,
vs.
THE NATIONAL LABOR RELATIONS COMMISSION, (First On October 29, 1986, the Labor Arbiter issued a writ of execution
Division); and Norberto Marabe; Rodolfo Raquel, Cristobal Riego, directing the sheriff to execute the Decision, dated December 19, 1984.
Manuel Gillego, Palcronio Giducos, Pedro Aboigar, Norberto The writ was partially satisfied through garnishment of sums from
Comendador, Rogelio Salut, Emilio Garcia, Jr., Mariano Rio, petitioner's debtor, the Metropolitan Waterworks and Sewerage
Paulina Basea, Alfredo Albera, Paquito Salut, Domingo Guarino, Authority, in the amount of P81,385.34. Said amount was turned over
Romeo Galve, Dominador Sabina, Felipe Radiana, Gavino to the cashier of the NLRC.
Sualibio, Moreno Escares, Ferdinand Torres, Felipe Basilan, and
Ruben Robalos, respondents.
On February 1, 1989, an Alias Writ of Execution was issued by the
Labor Arbiter directing the sheriff to collect from herein petitioner the
sum of P117,414.76, representing the balance of the judgment award,
and to reinstate private respondents to their former positions.
HERMOSISIMA, JR., J.:p
On July 13, 1989, the sheriff issued a report stating that he tried to
serve the alias writ of execution on petitioner through the security
The corporate mask may be lifted and the corporate veil may be
guard on duty but the service was refused on the ground that petitioner
pierced when a corporation is just but the alter ego of a person or of
no longer occupied the premises.
another corporation. Where badges of fraud exist; where public
convenience is defeated; where a wrong is sought to be justified
thereby, the corporate fiction or the notion of legal entity should come On September 26, 1986, upon motion of private respondents, the
to naught. The law in these instances will regard the corporation as a Labor Arbiter issued a second alias writ of execution.
mere association of persons and, in case of two corporations, merge
them into one.
The said writ had not been enforced by the special sheriff because, as
stated in his progress report, dated November 2, 1989:
Thus, where a sister corporation is used as a shield to evade a
corporation's subsidiary liability for damages, the corporation may not
1. All the employees inside petitioner's premises at 355 Maysan Road,
be heard to say that it has a personality separate and distinct from the
Valenzuela, Metro Manila, claimed that they were employees of Hydro
other corporation. The piercing of the corporate veil comes into play.
Pipes Philippines, Inc. (HPPI) and not by respondent;

This special civil action ostensibly raises the question of whether the
2. Levy was made upon personal properties he found in the premises;
National Labor Relations Commission committed grave abuse of
discretion when it issued a "break-open order" to the sheriff to be
enforced against personal property found in the premises of petitioner's 3. Security guards with high-powered guns prevented him from
sister company. removing the properties he had levied upon.4

Petitioner Concept Builders, Inc., a domestic corporation, with principal The said special sheriff recommended that a "break-open order" be
office at 355 Maysan Road, Valenzuela, Metro Manila, is engaged in issued to enable him to enter petitioner's premises so that he could
the construction business. Private respondents were employed by said proceed with the public auction sale of the aforesaid personal
company as laborers, carpenters and riggers. properties on November 7, 1989.

On November, 1981, private respondents were served individual On November 6, 1989, a certain Dennis Cuyegkeng filed a third-party
written notices of termination of employment by petitioner, effective on claim with the Labor Arbiter alleging that the properties sought to be
November 30, 1981. It was stated in the individual notices that their levied upon by the sheriff were owned by Hydro (Phils.), Inc. (HPPI) of
contracts of employment had expired and the project in which they which he is the Vice-President.
were hired had been completed.
On November 23, 1989, private respondents filed a "Motion for
Public respondent found it to be, the fact, however, that at the time of Issuance of a Break-Open Order," alleging that HPPI and petitioner
the termination of private respondent's employment, the project in corporation were owned by the same incorporator/stockholders. They
which they were hired had not yet been finished and completed. also alleged that petitioner temporarily suspended its business
Petitioner had to engage the services of sub-contractors whose operations in order to evade its legal obligations to them and that
workers performed the functions of private respondents. private respondents were willing to post an indemnity bond to answer
for any damages which petitioner and HPPI may suffer because of the
issuance of the break-open order.
Aggrieved, private respondents filed a complaint for illegal dismissal,
unfair labor practice and non-payment of their legal holiday pay,
overtime pay and thirteenth-month pay against petitioner. In support of their claim against HPPI, private respondents presented
duly certified copies of the General Informations Sheet, dated May 15,
1987, submitted by petitioner to the Securities Exchange Commission
On December 19, 1984, the Labor Arbiter rendered judgment 1 ordering
(SEC) and the General Information Sheet, dated May 25, 1987,
petitioner to reinstate private respondents and to pay them back wages
submitted by HPPI to the Securities and Exchange Commission.
equivalent to one year or three hundred working days.

The General Information Sheet submitted by the petitioner revealed the


On November 27, 1985, the National Labor Relations Commission
following:
(NLRC) dismissed the motion for reconsideration filed by petitioner on
the ground that the said decision had already become final and
executory.2 1. Breakdown of Subscribed Capital

Name of Stockholder Amount Subscribed

28
HPPI P 6,999,500.00 Virgilio O. Casino 100.00

Antonio W. Lim 2,900,000.00 2. Board of Directors

Dennis S. Cuyegkeng 300.00 Antonio W. Lim Chairman

Elisa C. Lim 100,000.00 Elisa C. Lim Member

Teodulo R. Dino 100.00 Dennis S. Cuyegkeng


Member
Virgilio O. Casino 100.00
Virgilio O. Casino Member
2. Board of Directors
Teodulo R. Dino Member
Antonio W. Lim Chairman
3. Corporate Officers
Dennis S. Cuyegkeng
Member Antonio W. Lim President

Elisa C. Lim Member Dennis S. Cuyegkeng


Assistant to the President
Teodulo R. Dino Member
Elisa C. Lim Treasurer
Virgilio O. Casino Member
Virgilio O. Casino Corporate
Secretary
3. Corporate Officers

4. Principal Office
Antonio W. Lim President

355 Maysan Road,


Dennis S. Cuyegkeng
Valenzuela, Metro Manila.6
Assistant to the President

On February 1, 1990, HPPI filed an Opposition to private respondents'


Elisa O. Lim Treasurer
motion for issuance of a break-open order, contending that HPPI is a
corporation which is separate and distinct from petitioner. HPPI also
Virgilio O. Casino Corporate alleged that the two corporations are engaged in two different kinds of
Secretary businesses, i.e., HPPI is a manufacturing firm while petitioner was then
engaged in construction.
4. Principal Office
On March 2, 1990, the Labor Arbiter issued an Order which denied
private respondents' motion for break-open order.
355 Maysan Road

Private respondents then appealed to the NLRC. On April 23, 1992,


Valenzuela, Metro Manila.5 the NLRC set aside the order of the Labor Arbiter, issued a break-open
order and directed private respondents to file a bond. Thereafter, it
On the other hand, the General Information Sheet of HPPI revealed the directed the sheriff to proceed with the auction sale of the properties
following: already levied upon. It dismissed the third-party claim for lack of merit.

1. Breakdown of Subscribed Capital Petitioner moved for reconsideration but the motion was denied by the
NLRC in a Resolution, dated December 3, 1992.
Name of Stockholder Amount
Subscribed Hence, the resort to the present petition.

Antonio W. Lim P 400,000.00 Petitioner alleges that the NLRC committed grave abuse of discretion
when it ordered the execution of its decision despite a third-party claim
on the levied property. Petitioner further contends, that the doctrine of
Elisa C. Lim 57,700.00 piercing the corporate veil should not have been applied, in this case,
in the absence of any showing that it created HPPI in order to evade its
AWL Trading 455,000.00 liability to private respondents. It also contends that HPPI is engaged in
the manufacture and sale of steel, concrete and iron pipes, a business
which is distinct and separate from petitioner's construction business.
Dennis S. Cuyegkeng Hence, it is of no consequence that petitioner and HPPI shared the
40,100.00 same premises, the same President and the same set of officers and
subscribers.7
Teodulo R. Dino 100.00
29
We find petitioner's contention to be unmeritorious. 3. The aforesaid control and breach of duty must
proximately cause the injury or unjust loss
complained of.
It is a fundamental principle of corporation law that a corporation is an
entity separate and distinct from its stockholders and from other
corporations to which it may be connected.8 But, this separate and The absence of any one of these elements
distinct personality of a corporation is merely a fiction created by law prevents "piercing the corporate veil." In applying
for convenience and to promote justice.9 So, when the notion of the "instrumentality" or "alter ego" doctrine, the
separate juridical personality is used to defeat public convenience, courts are concerned with reality and not form,
justify wrong, protect fraud or defend crime, or is used as a device to with how the corporation operated and the
defeat the labor laws,10 this separate personality of the corporation individual defendant's relationship to that
may be disregarded or the veil of corporate fiction pierced.11 This is operation.14
true likewise when the corporation is merely an adjunct, a business
conduit or an alter ego of another corporation.12
Thus the question of whether a corporation is a mere alter ego, a mere
sheet or paper corporation, a sham or a subterfuge is purely one of
The conditions under which the juridical entity may be disregarded vary fact.15
according to the peculiar facts and circumstances of each case. No
hard and fast rule can be accurately laid down, but certainly, there are
In this case, the NLRC noted that, while petitioner claimed that it
some probative factors of identity that will justify the application of the
ceased its business operations on April 29, 1986, it filed an Information
doctrine of piercing the corporate veil, to wit:
Sheet with the Securities and Exchange Commission on May 15, 1987,
stating that its office address is at 355 Maysan Road, Valenzuela,
1. Stock ownership by one or common ownership Metro Manila. On the other hand, HPPI, the third-party claimant,
of both corporations. submitted on the same day, a similar information sheet stating that its
office address is at 355 Maysan Road, Valenzuela, Metro Manila.
2. Identity of directors and officers.
Furthermore, the NLRC stated that:
3. The manner of keeping corporate books and
records. Both information sheets were filed by
the same Virgilio O. Casiño as the corporate
secretary of both corporations. It would also not be
4. Methods of conducting the business.13
amiss to note that both corporations had
the same president, the same board of directors,
The SEC en banc explained the "instrumentality rule" which the courts the same corporate officers, and substantially
have applied in disregarding the separate juridical personality of the same subscribers.
corporations as follows:
From the foregoing, it appears that, among other
Where one corporation is so organized and things, the respondent (herein petitioner) and the
controlled and its affairs are conducted so that it is, third-party claimant shared the same address
in fact, a mere instrumentality or adjunct of the and/or premises. Under this circumstances, (sic) it
other, the fiction of the corporate entity of the cannot be said that the property levied upon by the
"instrumentality" may be disregarded. The control sheriff were not of respondents.16
necessary to invoke the rule is not majority or even
complete stock control but such domination of
Clearly, petitioner ceased its business operations in order to evade the
instances, policies and practices that the
payment to private respondents of back wages and to bar their
controlled corporation has, so to speak, no
reinstatement to their former positions. HPPI is obviously a business
separate mind, will or existence of its own, and is
conduit of petitioner corporation and its emergence was skillfully
but a conduit for its principal. It must be kept in
orchestrated to avoid the financial liability that already attached to
mind that the control must be shown to have been
petitioner corporation.
exercised at the time the acts complained of took
place. Moreover, the control and breach of duty
must proximately cause the injury or unjust loss for The facts in this case are analogous to Claparols v. Court of Industrial
which the complaint is made. Relations, 17 where we had the occasion to rule:

The test in determining the applicability of the doctrine of piercing the Respondent court's findings that indeed the
veil of corporate fiction is as follows: Claparols Steel and Nail Plant, which ceased
operation of June 30, 1957, was SUCCEEDED by
the Claparols Steel Corporation effective the next
1. Control, not mere majority or complete stock
day, July 1, 1957, up to December 7, 1962, when
control, but complete domination, not only of
the latter finally ceased to operate, were not
finances but of policy and business practice in
disputed by petitioner. It is very clear that the latter
respect to the transaction attacked so that the
corporation was a continuation and successor of
corporate entity as to this transaction had at the
the first entity . . . . Both predecessors and
time no separate mind, will or existence of its own;
successor were owned and controlled by petitioner
Eduardo Claparols and there was no break in the
2. Such control must have been used by the succession and continuity of the same business.
defendant to commit fraud or wrong, to perpetuate This "avoiding-the-liability" scheme is very patent,
the violation of a statutory or other positive legal considering that 90% of the subscribed shares of
duty or dishonest and unjust act in contravention stock of the Claparols Steel Corporation (the
of plaintiff's legal rights; and second corporation) was owned by respondent . . .
Claparols himself, and all the assets of the
dissolved Claparols Steel and Nail plant were

30
turned over to the emerging Claparols Steel
Corporation.

It is very obvious that the second corporation


seeks the protective shield of a corporate fiction
whose veil in the present case could, and should,
be pierced as it was deliberately and maliciously
designed to evade its financial obligation to its
employees.

In view of the failure of the sheriff, in the case at bar, to effect a levy
upon the property subject of the execution, private respondents had no
other recourse but to apply for a break-open order after the third-party
claim of HPPI was dismissed for lack of merit by the NLRC. This is in
consonance with Section 3, Rule VII of the NLRC Manual of Execution
of Judgment which provides that:

Should the losing party, his agent or


representative, refuse or prohibit the Sheriff or his
representative entry to the place where the
property subject of execution is located or kept,
the judgment creditor may apply to the
Commission or Labor Arbiter concerned for a
break-open order.

Furthermore, our perusal of the records shows that the twin


requirements of due notice and hearing were complied with. Petitioner
and the third-party claimant were given the opportunity to submit
evidence in support of their claim.

Hence, the NLRC did not commit any grave abuse of discretion when it
affirmed the break-open order issued by the Labor Arbiter.

Finally, we do not find any reason to disturb the rule that factual
findings of quasi-judicial agencies supported by substantial evidence
are binding on this Court and are entitled to great respect, in the
absence of showing of grave abuse of a discretion. 18

WHEREFORE, the petition is DISMISSED and the assailed resolutions


of the NLRC, dated April 23, 1992 and December 3, 1992, are
AFFIRMED.

31
G.R. No. L-30822 July 31, 1975 which was reiterated by respondent Court in a subsequent order dated
November 10, 1964 (pp. 7-8, Brief for Respondents, p. 113, rec.).
EDUARDO CLAPAROLS, ROMULO AGSAM and/or CLAPAROLS
STEEL AND NAIL PLANT, petitioners, On December 14, 1964, respondent workers were accompanied by the
vs. Chief of Police of Talisay, Negros Occidental to the compound of
COURT OF INDUSTRIAL RELATIONS, ALLIED WORKERS' herein petitioner company to report for reinstatement per order of the
ASSOCIATION and/or DEMETRIO GARLITOS, ALFREDO court. Respondent workers were, however, refused reinstatement by
ONGSUCO, JORGE SEMILLANO, SALVADOR DOROTEO, company accountant Francisco Cusi for he had no order from plant
ROSENDO ESPINOSA, LUDOVICO BALOPENOS, ASER owner Eduardo Claparols nor from his lawyer Atty. Plaridel Katalbas, to
AMANCIO, MAXIMO QUIOYO, GAUDENCIO QUIOYO, and reinstate respondent workers.
IGNACIO QUIOYO, respondents.
Again, on December 15, 1964, respondent workers were accompanied
Ruben G. Bala for petitioners. by a police officer to the company compound, but then, they were
again refused reinstatement by Cusi on the same ground.
Rolando N. Medalla for private respondents.
On January 15, 1965, the CIR Chief Examiner Submitted his report
containing three computations, to wit:

The first computation covers the period February


MAKASIAR, J.:
1, 1957 to October 31, 1964. The second is up to
and including December 7, 1962, when the
A petition for certiorari to set aside the order of respondent Court of corporation stopped operations, while the third is
Industrial Relations dated May 30, 1969 directing petitioners to pay only up to June 30, 1957 when the Claparols Steel
back wages and bonuses to private respondents as well as its and Nail Plant ceased to operate (Annex B,
resolution of July 5, 1969 denying the motion for reconsideration of Petition for Review on Certiorari, p. 14, Brief for
said order in Case No. 32-ULP-Iloilo entitled "Allied Workers' appellees, p. 113, rec.).
Association, et. al., versus Eduardo Claparols, et. al.."
with the explanation that:
It appears that on August 6, 1957, a complaint for unfair labor practice
was filed by herein private respondent Allied Workers' Association,
6. Since the records of the Claparols Steel
respondent Demetrio Garlitos and ten (10) respondent workers against
Corporation show that it was established on July 1,
herein petitioners on account of the dismissal of respondent workers
1957 succeeding the Claparols Steel and Nail
from petitioner Claparols Steel and Nail Plant.
Plant which ceased operations on June 30, 1957,
and that the Claparols Steel Corporation stopped
On September 16, 1963, respondent Court rendered its decision operations on December 7, 1962, three (3)
finding "Mr. Claparols guilty of union busting and" of having "dismissed computations are presented herein for the
said complainants because of their union activities," and ordering consideration of this Honorable Court (p. 2, Report
respondents "(1) To cease and desist from committing unfair labor of Examiner, p. 29, rec.).
practices against their employees and laborers; (2) To reinstate said
complainants to their former or equivalent jobs, as soon as possible,
On January 23, 1965, petitioners filed an opposition alleging that under
with back wages from the date of their dismissal up to their actual
the circumstances presently engulfing the company, petitioner
reinstatement" (p. 12, Decision; p. 27, rec.).
Claparols could not personally reinstate respondent workers; that
assuming the workers are entitled to back wages, the same should
A motion to reconsider the above decision was filed by herein only be limited to three months pursuant to the court ruling in the case
petitioners, which respondent Court, sitting en banc, denied in a of Sta. Cecilia Sawmills vs. CIR (L-19273-74, February 20, 1964); and
resolution dated January 27, 1964. that since Claparols Steel Corporation ceased to operate on December
7, 1962, re-employment of respondent workers cannot go beyond
December 7, 1962.
On March 30, 1964, counsel for herein respondent workers
(complainants in the ULP case) filed a motion for execution of
respondent Court's September 16, 1963 decision. A reply to petitioner's opposition was filed by respondent workers,
alleging among others, that Claparols Steel and Nail Plant and
Claparols Steel and Nail Corporation are one and the same corporation
On May 14, 1964, respondent Court, in its order of September 16, controlled by petitioner Claparols, with the latter corporation
1963, granted execution and directed herein petitioners succeeding the former.

to reinstate the above complainants to their former On November 28, 1966, after conducting a series of hearings on the
or equivalent jobs within five (5) days after receipt report of the examiner, respondent Court issued an order, the
of a copy of this order. In order to implement the
dispositive portion of which reads:
award of back wages, the Chief of the Examining
Division or any of his assistants is hereby directed
to proceed to the office of the respondents at WHEREFORE, the Report of the. Examiner filed
Matab-ang, Talisay, Negros Occidental, and on January 15, 1965, is hereby approved subject
examine its payrolls and other pertinent records to the foregoing findings and dispositions.
and compute the back wages of the complainants Consequently, the Corporation Auditing Examiner
in accordance with the decision dated September is directed to recompute the back wages of
16, 1963, and, upon termination, to submit his complainants Demetrio Garlitos and Alfredo
report as soon as possible for further disposition Ongsuco on the basis of P200.00 and P270.00 a
(p. 7, Brief for Respondents, p. 113, rec.). month, respectively; to compute those of
complainant Ignacio Quioyo as aforesaid; to
compute the deductible earnings of complainants
Ongsuco, Jorge Semillano and Garlitos, as found
32
in the body of this order; and to compute the a copy of this Order (p. 28, Brief for Respondents,
bonuses of each and every complainant, except p. 113, rec.).
Honorato Quioyo. Thereafter, as soon as possible,
the Examiner should submit a report in compliance
Then on March 21, 1968, the Chief Examiner came out with his report,
herewith of the Court's further disposition (p. 24,
the disputed portion of which (regarding bonuses) reads:
Brief for Respondents, p. 113, rec.).

xxx xxx xxx


On December 7, 1966, a motion for reconsideration was filed by
petitioner, assailing respondent Court's ruling that (1) the ruling in the
case of Sta. Cecilia Sawmills Inc. CIR, et. al, does not apply in the 4. The yearly bonuses of the employees and
case at bar; and (2) that bonus should be included in the recoverable laborers of respondent corporation are given on
wages. the following basis:

On December 14, 1966, a counter-opposition was filed by private Basic Additional:


respondents alleging that petitioners' motion for reconsideration
was pro forma, it not making express reference to the testimony or
a. For every dependent 1% of
documentary evidence or to the provision of law alleged to be contrary
to such findings or conclusions of respondent Court. monthly salary

b. For every dependent in


On February 8, 1967, respondent Court of Industrial Relations
dismissed petitioners' motion for reconsideration for being pro forma. elementary grade 2% of
monthly salary

Whereupon, petitioners filed a petition for certiorari with this COURT in


G.R. No. L-27272 to set aside the November 28, 1966 order of c. For every dependent in high
respondent Court, as well as its February 8, 1967 resolution. school 3% of monthly salary
Petitioners assigned therein as errors of law the very same assignment
of errors it raises in the present case, to wit: d. For every dependent in
college 5% of monthly salary
I
xxx xxx xxx
THE RESPONDENT COURT ERRED AND/OR
ACTED WITH GRAVE ABUSE OF DISCRETION, 7. The computed ... bonuses after deducting the
AMOUNTING TO LACK OF JURISDICTION, IN earnings elsewhere of Messrs. Ongsuco, Garlitos
HOLDING IN THE ORDER UNDER REVIEW and Semillano are as follows:
THAT BONUSES SHOULD BE PAID TO THE
RESPONDENT WORKERS DESPITE THE FACT
THAT THE SAME WAS NOT ADJUDICATED IN Name x x x Bonuses x x x
ITS ORIGINAL DECISION.
1. Alfredo Ongsuco P1,620.00
II 2. Demetrio Garlitos 1,200.00
3. Ignacio Quioyo 455.23
4. Aser Abancio 461.00
THE RESPONDENT COURT ERRED AND/OR 5. Ludovico Belopeños 752.05
ACTED WITH GRAVE ABUSE OF DISCRETION, 6. Salvador Doroteo 714.70
AMOUNTING TO LACK OF JURISDICTION, IN 7. Rosendo Espinosa
NOT APPLYING THE DOCTRINE LAID DOWN 1,075.40
BY THIS HONORABLE TRIBUNAL IN THE CASE 8. Gaudencio Quioyo
OF "STA. CECILIA SAWMILLS, INC. VS. C.I.R., 1,167.92
ET. AL.," G.R. No. 9. Jorge Semillano 1,212.08
L-19273-74, PROMULGATED ON FEBRUARY 10. Maximo Quioyo 449.41
29, 1964 (pp. 10-11, rec.). Total P9,107.79

On April 27, 1967, the Supreme Court denied petitioners' petition (Pp. 30-31, Respondent's Brief, p. 113, rec.)
for certiorari (p. 77, rec. of L-27272), which was reiterated on May 19,
1967 (p. 27, Respondent's Brief, p. 113, rec.; p. 81, rec. of L-27272).
On April 16, 1968, petitioners filed their opposition to the report of the
Examiner dated March 21, 1968 on grounds already rejected by
On May 3, 1967, private respondents moved to have the workers' back respondent Court in its order dated November 28, 1966, and by the
wages properly recomputed. A motion to the same end was reiterated Supreme Court also in its ruling in G.R. No. L-27272.
by private respondents on June 14, 1967.
On May 4, 1968, a rejoinder to petitioners' opposition was filed by
On July 13, 1967, respondent Court directed a recomputation of the private respondents, alleging among others "that the grounds of
back wages of respondent workers in accordance with its order dated petitioners' opposition were the same grounds raised by them before
November 28, 1966. The said order in part reads: and passed upon by respondent Court and this Honorable Tribunal;
that this order of November 28, 1966 which passed upon these issues
became final and executory on June 3, 1967 from the Honorable
WHEREFORE, the Chief Auditing Examiner of the
Court or any of his assistants, is hereby directed to Supreme Court. (Order of respondent Court dated July 13, 1967). [p.
recompute the back wages of the workers involved 32, Brief for Respondents, p. 113, rec.].
in this case in accordance with the Order of
November 28, 1966 within 20 days from receipt of

3
On July 26, 1968, private respondents filed their motion for approval of WE uniformly held that "a bonus is not a demandable and enforceable
the Report of the Examiner submitted on March 21, 1968, alleging, obligation, except when it is a part of the wage or salary compensation"
among others, that petitioners, in their opposition, did not actually (Philippine Education Co. vs. CIR and the Union of Philippine Co.
dispute the data elicited by the Chief Examiner but rather harped on Employees [NLU], 92 Phil. 381; Ansay, et. al. vs. National
grounds which, as already stated, had already been turned down by Development Co., et. al., 107 Phil. 998, 999; Emphasis supplied).
the Supreme Court.
In Atok Big Wedge Mining Co. vs. Atok Big Wedge Mutual Benefit
On October 19, 1968, herein private respondents filed their Association (92 Phil. 754), this Court, thru Justice Labrador, held:
"Constancia", submitting the case for resolution of respondent Court of
Industrial Relations.
Whether or not bonus forms part of wages
depends upon the condition or circumstance for its
On May 30, 1969, respondent Court issued an order, subject of the payment. If it is an additional compensation
present appeal, the dispositive portion of which reads: WHICH THE EMPLOYER PROMISED AND
AGREED to give without any condition imposed
for its payment ... then it is part of the wage.
WHEREFORE, there being no proof offered to
(Emphasis supplied).1äwphï1.ñët
substantiate respondent Eduardo Claparols'
opposition, the Examiner's Report should be, and
it is hereby, APPROVED. Consequently, pursuant In Altomonte vs. Philippine American Drug Co. (106 Phil. 137), the
to the decision dated September 16, 1963, Supreme Court held that an employee is not entitled to bonus where
respondent ... (petitioners herein) are there is no showing that it had been granted by the employer to its
hereby directed to pay the respective back wages employees periodically or regularly as to become part of their wages or
and bonuses of the complainants (respondents salaries. The clear implication is that bonus is recoverable as part of
herein) ... (p. 35, Brief for Respondents; p. 113, the wage or salary where the employer regularly or periodically gives it
rec.; emphasis supplied).1äwphï1.ñët to employees.

On June 7, 1969, petitioners filed a motion for reconsideration on American jurisprudence equally regards bonuses as part of
practically the same grounds previously raised by them. compensation or recoverable wages.

On June 30, 1969, respondents filed an opposition to petitioners' Thus, it was held that "... it follows that in determining the regular rate
motion for reconsideration, with the following allegations: of pay, a bonus which in fact constitutes PART OF AN EMPLOYEE'S
compensation, rather than a true gift or gratuity, has to be taken into
consideration." (48 Am. Jur. 2d, Labor and Labor Relations, No. 1555,
1. The issues raised, namely, whether bonuses
citing the cases of Triple "AAA" Co. vs. Wirtz and Haber vs. Americana
should be included in the award for back wages
Corporation; Emphasis supplied). It was further held that "... the regular
had already been resolved by respondent court in
rate includes incentive bonuses paid to the employees in addition to
its orders dated November 28, 1966, and
the guaranteed base rates regardless of any contract provision to the
December 7, 1966, and in the Resolution of the
contrary and even though such bonuses could not be determined or
Honorable Supreme Court in G.R. No. L-27272
paid until such time after the pay day" (48 Am. Jur. 2d, Labor and
dated April 26, 1967 and May 19, 1967, and the
Labor Relations, No. 1555, citing the case of Walling vs. Harnischfeger
same is already a settled and final issue.
Corp., 325 US 427, 89 L Ed 1711, 65 S Ct. 1246; Emphasis
supplied).1äwphï1.ñët
2. Petitioners' motion for reconsideration is merely
a rehash of previous arguments, effete and
Petitioners in the present case do not dispute that as a matter of
unrejuvenated, pro forma, and intended merely to
tradition, the company has been doling out bonuses to employees. In
delay the proceedings.
fact, the company balance sheets for the years 1956 to 1962 contained
bonus and pension computations which were never repudiated or
As correctly contended by private respondents, the present petition is questioned by petitioners. As such, bonus for a given year earmarked
barred by Our resolutions of April 26, 1967 and May 19, 1967 in G.R. as a matter of tradition for distribution to employees has formed part of
No. L-27272 (Eduardo Claparols, et. al. vs. CIR, et. al.) [pp. 77-83, rec. their recoverable wages from the company. Moreover, with greater
of L- 27272], dismissing said case, wherein said petitioners invoked the reason, should recovery of bonuses as part of back wages be
applicability of the doctrine in Sta. Cecilia Sawmills, Inc. vs. CIR, et. al. observed in the present case since the company, in the light of the very
(L-19273-74, Feb. 29, 1964, 10 SCRA 433) and impugned the illegality admission of company accountant Francisco Cusi, distributes bonuses
of the order of respondent Court dated November 28, 1966 directing to its employees even if the company has suffered losses. Specifically,
the computation and payment of the bonuses, aside from back wages petitioner company has done this in 1962 (t.s.n., p. 149, Sept. 20,
on the ground that these bonuses were not included in the decision of 1965).
September 16, 1963, which had long become final.
Since bonuses are part of back wages of private respondents, the
The aforesaid resolutions in G.R. No. L-27272 constitute the law of the order of May 30, 1969, directing the payment of their bonuses, did not
instant case, wherein herein petitioners raised again practically the amend the decision of September 16, 1963 of respondent Court
same issues invoked in the abovementioned case. The denial of the directing payment of their wages, which has long become final and
petition in G.R. No. L-27272 suffices to warrant the denial of the executory, in the same way that the previous order of May 14, 1964
present petition; and We need not go any further. granting execution of said decision of September 16, 1963 also
directed the computation of the wages to be paid to private
respondents as decreed by the decision of September 16, 1963. All the
However, without lending a sympathetic ear to the obvious desire of orders of May 30, 1969, November 28, 1966 and May 14, 1964 merely
herein petitioners of this Court to re-examine — which would be an implement the already final and executory decision of September 16,
exercise in futility — the final ruling in G.R. No. L-27272, which as
1963.
above-stated is the law of the instant case, but solely to remind herein
petitioners, We reiterate the governing principles.
Petitioners insist that We adopt the ruling in the Sta. Cecilia Sawmills
case wherein the recoverable back wages were limited to only three (3)
months; because as in the Sta. Cecilia Sawmills case, the Claparols
34
Steel and Nail Plant ceased operations due to enormous business
reverses.

Respondent Court's findings that indeed the Claparols Steel and Nail
Plant, which ceased operation of June 30, 1957, was SUCCEEDED by
the Claparols Steel Corporation effective the next day, July 1, 1957 up
to December 7, 1962, when the latter finally ceased to operate, were
not disputed by petitioners. It is very clear that the latter corporation
was a continuation and successor of the first entity, and its emergence
was skillfully timed to avoid the financial liability that already attached
to its predecessor, the Claparols Steel and Nail Plant. Both
predecessors and successor were owned and controlled by the
petitioner Eduardo Claparols and there was no break in the succession
and continuity of the same business. This "avoiding-the-liability"
scheme is very patent, considering that 90% of the subscribed shares
of stocks of the Claparols Steel Corporation (the second corporation)
was owned by respondent (herein petitioner) Claparols himself, and all
the assets of the dissolved Claparols Steel and Nail Plant were turned
over to the emerging Claparols Steel Corporation.

It is very obvious that the second corporation seeks the protective


shield of a corporate fiction whose veil in the present case could, and
should, be pierced as it was deliberately and maliciously designed to
evade its financial obligation to its employees.

It is well remembering that in Yutivo & Sons Hardware Company vs.


Court of Tax Appeals (L-13203, Jan. 28, 1961, 1 SCRA 160), We held
that when the notion of legal entity is used to defeat public
convenience, justify wrong, protect fraud, or defend crime, the law will
regard the corporation as an association or persons, or, in the case of
two corporations, will merge them into one.

In Liddel & Company, Inc. vs. Collector of Internal Revenue (L-9687,


June 30, 1961, 2 SCRA 632), this Court likewise held that where a
corporation is a dummy and serves no business purpose and is
intended only as a blind, the corporate fiction may be ignored.

In Commissioner of Internal Revenue vs. Norton and Harrison


Company (L-17618, Aug. 31, 1964, 11 SCRA 714), We ruled that
where a corporation is merely an adjunct, business conduit or alter ego
of another corporation, the fiction of separate and distinct corporate
entities should be disregarded.

To the same uniform effect are the decisions in the cases of Republic
vs. Razon (L-17462, May 29, 1967, 20 SCRA 234) and A.D. Santos,
Inc. vs. Vasquez (L-23586, March 20, 1968, 22 SCRA 1156).

WE agree with respondent Court of Industrial Relations, therefore, that


the amount of back wages recoverable by respondent workers from
petitioners should be the amount accruing up to December 7, 1962
when the Claparols Steel Corporation ceased operations.

WHEREFORE, PETITION IS HEREBY DENIED WITH TREBLE


COSTS AGAINST PETITIONERS TO BE PAID BY THEIR COUNSEL.

35
G.R. No. L-23893 October 29, 1968 approval, with a prayer for the issuance of a provisional authority in
favor of the vendee Corporation to operate the service therein
involved.1 On May 19, 1959, the PSC granted the provisional permit
VILLA REY TRANSIT, INC., plaintiff-appellant,
prayed for, upon the condition that "it may be modified or revoked by
vs.
the Commission at any time, shall be subject to whatever action that
EUSEBIO E. FERRER, PANGASINAN TRANSPORTATION CO.,
may be taken on the basic application and shall be valid only during
INC. and PUBLIC SERVICE COMMISSION, defendants.
the pendency of said application." Before the PSC could take final
EUSEBIO E. FERRER and PANGASINAN TRANSPORTATION CO.,
action on said application for approval of sale, however, the Sheriff of
INC., defendants-appellants.
Manila, on July 7, 1959, levied on two of the five certificates of public
convenience involved therein, namely, those issued under PSC cases
PANGASINAN TRANSPORTATION CO., INC., third-party plaintiff- Nos. 59494 and 63780, pursuant to a writ of execution issued by the
appellant, Court of First Instance of Pangasinan in Civil Case No. 13798, in favor
vs. of Eusebio Ferrer, plaintiff, judgment creditor, against Valentin
JOSE M. VILLARAMA, third-party defendant-appellee. Fernando, defendant, judgment debtor. The Sheriff made and entered
the levy in the records of the PSC. On July 16, 1959, a public sale was
conducted by the Sheriff of the said two certificates of public
Chuidian Law Office for plaintiff-appellant.
convenience. Ferrer was the highest bidder, and a certificate of sale
Bengzon, Zarraga & Villegas for defendant-appellant / third-party was issued in his name.
plaintiff-appellant.
Laurea & Pison for third-party defendant-appellee.
Thereafter, Ferrer sold the two certificates of public convenience to
Pantranco, and jointly submitted for approval their corresponding
ANGELES, J.:
contract of sale to the PSC.2 Pantranco therein prayed that it be
authorized provisionally to operate the service involved in the
This is a tri-party appeal from the decision of the Court of First Instance said two certificates.
of Manila, Civil Case No. 41845, declaring null and void the sheriff's
sale of two certificates of public convenience in favor of defendant
The applications for approval of sale, filed before the PSC, by
Eusebio E. Ferrer and the subsequent sale thereof by the latter to Fernando and the Corporation, Case No. 124057, and that of Ferrer
defendant Pangasinan Transportation Co., Inc.; declaring the plaintiff and Pantranco, Case No. 126278, were scheduled for a joint hearing.
Villa Rey Transit, Inc., to be the lawful owner of the said certificates of In the meantime, to wit, on July 22, 1959, the PSC issued an order
public convenience; and ordering the private defendants, jointly and disposing that during the pendency of the cases and before a final
severally, to pay to the plaintiff, the sum of P5,000.00 as and for resolution on the aforesaid applications, the Pantranco shall be the one
attorney's fees. The case against the PSC was dismissed. to operate provisionally the service under the two certificates embraced
in the contract between Ferrer and Pantranco. The Corporation took
The rather ramified circumstances of the instant case can best be issue with this particular ruling of the PSC and elevated the matter to
understood by a chronological narration of the essential facts, to wit: the Supreme Court,3 which decreed, after deliberation, that until the
issue on the ownership of the disputed certificates shall have been
finally settled by the proper court, the Corporation should be the one to
Prior to 1959, Jose M. Villarama was an operator of a bus operate the lines provisionally.
transportation, under the business name of Villa Rey Transit, pursuant
to certificates of public convenience granted him by the Public Service
Commission (PSC, for short) in Cases Nos. 44213 and 104651, which On November 4, 1959, the Corporation filed in the Court of First
authorized him to operate a total of thirty-two (32) units on various Instance of Manila, a complaint for the annulment of the sheriff's sale
routes or lines from Pangasinan to Manila, and vice-versa. On January of the aforesaid two certificates of public convenience (PSC Cases
8, 1959, he sold the aforementioned two certificates of public Nos. 59494 and 63780) in favor of the defendant Ferrer, and the
convenience to the Pangasinan Transportation Company, Inc. subsequent sale thereof by the latter to Pantranco, against Ferrer,
(otherwise known as Pantranco), for P350,000.00 with the condition, Pantranco and the PSC. The plaintiff Corporation prayed therein that
among others, that the seller (Villarama) "shall not for a period of 10 all the orders of the PSC relative to the parties' dispute over the said
years from the date of this sale, apply for any TPU service identical or certificates be annulled.
competing with the buyer."
In separate answers, the defendants Ferrer and Pantranco averred
Barely three months thereafter, or on March 6, 1959: a corporation that the plaintiff Corporation had no valid title to the certificates in
called Villa Rey Transit, Inc. (which shall be referred to hereafter as the question because the contract pursuant to which it acquired them from
Corporation) was organized with a capital stock of P500,000.00 divided Fernando was subject to a suspensive condition — the approval of the
into 5,000 shares of the par value of P100.00 each; P200,000.00 was PSC — which has not yet been fulfilled, and, therefore, the Sheriff's
the subscribed stock; Natividad R. Villarama (wife of Jose M. levy and the consequent sale at public auction of the certificates
Villarama) was one of the incorporators, and she subscribed for referred to, as well as the sale of the same by Ferrer to Pantranco,
P1,000.00; the balance of P199,000.00 was subscribed by the brother were valid and regular, and vested unto Pantranco, a superior right
and sister-in-law of Jose M. Villarama; of the subscribed capital stock, thereto.
P105,000.00 was paid to the treasurer of the corporation, who was
Natividad R. Villarama. Pantranco, on its part, filed a third-party complaint against Jose M.
Villarama, alleging that Villarama and the Corporation, are one and the
In less than a month after its registration with the Securities and same; that Villarama and/or the Corporation was disqualified from
Exchange Commission (March 10, 1959), the Corporation, on April 7, operating the two certificates in question by virtue of the
1959, bought five certificates of public convenience, forty-nine buses, aforementioned agreement between said Villarama and Pantranco,
tools and equipment from one Valentin Fernando, for the sum of which stipulated that Villarama "shall not for a period of 10 years from
P249,000.00, of which P100,000.00 was paid upon the signing of the the date of this sale, apply for any TPU service identical or competing
contract; P50,000.00 was payable upon the final approval of the sale with the buyer."
by the PSC; P49,500.00 one year after the final approval of the sale;
and the balance of P50,000.00 "shall be paid by the BUYER to the Upon the joinder of the issues in both the complaint and third-party
different suppliers of the SELLER." complaint, the case was tried, and thereafter decision was rendered in
the terms, as above stated.
The very same day that the aforementioned contract of sale was
executed, the parties thereto immediately applied with the PSC for its
36
As stated at the beginning, all the parties involved have appealed from A. Because sometimes she uses my money, and
the decision. They submitted a joint record on appeal. sometimes the money given to her she gives to me and I
deposit the money.
Pantranco disputes the correctness of the decision insofar as it holds
that Villa Rey Transit, Inc. (Corporation) is a distinct and separate Q. Actually, aside from your wife, you were also the
entity from Jose M. Villarama; that the restriction clause in the contract custodian of some of the incorporators here, in the
of January 8, 1959 between Pantranco and Villarama is null and void; beginning?
that the Sheriff's sale of July 16, 1959, is likewise null and void; and the
failure to award damages in its favor and against Villarama.
A. Not necessarily, they give to my wife and when my
wife hands to me I did not know it belonged to the
Ferrer, for his part, challenges the decision insofar as it holds that the incorporators.
sheriff's sale is null and void; and the sale of the two certificates in
question by Valentin Fernando to the Corporation, is valid. He also
Q. It supposes then your wife gives you some of the
assails the award of P5,000.00 as attorney's fees in favor of the
money received by her in her capacity as treasurer of the
Corporation, and the failure to award moral damages to him as prayed
corporation?
for in his counterclaim.

A. Maybe.
The Corporation, on the other hand, prays for a review of that portion
of the decision awarding only P5,000.00 as attorney's fees, and
insisting that it is entitled to an award of P100,000.00 by way of Q. What did you do with the money, deposit in a regular
exemplary damages. account?

After a careful study of the facts obtaining in the case, the vital issues A. Deposit in my account.
to be resolved are: (1) Does the stipulation between Villarama and
Pantranco, as contained in the deed of sale, that the former "SHALL
Q. Of all the money given to your wife, she did not
NOT FOR A PERIOD OF 10 YEARS FROM THE DATE OF THIS
SALE, APPLY FOR ANY TPU SERVICE IDENTICAL OR receive any check?
COMPETING WITH THE BUYER," apply to new lines only or does it
include existing lines?; (2) Assuming that said stipulation covers all A. I do not remember.
kinds of lines, is such stipulation valid and enforceable?; (3) In the
affirmative, that said stipulation is valid, did it bind the Corporation?
Q. Is it usual for you, Doctor, to be given Fifty Thousand
Pesos without even asking what is this?
For convenience, We propose to discuss the foregoing issues by
starting with the last proposition.
xxx xxx xxx

The evidence has disclosed that Villarama, albeit was not an


incorporator or stockholder of the Corporation, alleging that he did not JUDGE: Reform the question.
become such, because he did not have sufficient funds to invest, his
wife, however, was an incorporator with the least subscribed number of Q. The subscription of your brother-in-law, Mr. Reyes, is
shares, and was elected treasurer of the Corporation. The finances of Fifty-Two Thousand Pesos, did your wife give you Fifty-two
the Corporation which, under all concepts in the law, are supposed to Thousand Pesos?
be under the control and administration of the treasurer keeping them
as trust fund for the Corporation, were, nonetheless, manipulated and
disbursed as if they were the private funds of Villarama, in such a way A. I have testified before that sometimes my wife gives
and extent that Villarama appeared to be the actual owner-treasurer of me money and I do not know exactly for what.
the business without regard to the rights of the stockholders. The
following testimony of Villarama,4 together with the other evidence on The evidence further shows that the initial cash capitalization of the
record, attests to that effect: corporation of P105,000.00 was mostly financed by Villarama. Of the
P105,000.00 deposited in the First National City Bank of New York,
Q. Doctor, I want to go back again to the incorporation of representing the initial paid-up capital of the Corporation, P85,000.00
the Villa Rey Transit, Inc. You heard the testimony presented was covered by Villarama's personal check. The deposit slip for the
here by the bank regarding the initial opening deposit of said amount of P105,000.00 was admitted in evidence as Exh. 23,
ONE HUNDRED FIVE THOUSAND PESOS, of which which shows on its face that P20,000.00 was paid in cash and
amount Eighty-Five Thousand Pesos was a check drawn by P85,000.00 thereof was covered by Check No. F-50271 of the First
yourself personally. In the direct examination you told the National City Bank of New York. The testimonies of Alfonso
Court that the reason you drew a check for Eighty-Five Sancho5 and Joaquin Amansec,6 both employees of said bank, have
Thousand Pesos was because you and your wife, or your proved that the drawer of the check was Jose Villarama himself.
wife, had spent the money of the stockholders given to her
for incorporation. Will you please tell the Honorable Court if Another witness, Celso Rivera, accountant of the Corporation, testified
you knew at the time your wife was spending the money to that while in the books of the corporation there appears an entry that
pay debts, you personally knew she was spending the the treasurer received P95,000.00 as second installment of the paid-in
money of the incorporators? subscriptions, and, subsequently, also P100,000.00 as the first
installment of the offer for second subscriptions worth P200,000.00
A. You know my money and my wife's money are one. from the original subscribers, yet Villarama directed him (Rivera) to
We never talk about those things. make vouchers liquidating the sums.7 Thus, it was made to appear that
the P95,000.00 was delivered to Villarama in payment for equipment
purchased from him, and the P100,000.00 was loaned as advances to
Q. Doctor, your answer then is that since your money the stockholders. The said accountant, however, testified that he was
and your wife's money are one money and you did not know not aware of any amount of money that had actually passed hands
when your wife was paying debts with the incorporator's among the parties involved,8 and actually the only money of the
money? corporation was the P105,000.00 covered by the deposit slip Exh. 23,

37
of which as mentioned above, P85,000.00 was paid by Villarama's time general manager. They show beyond doubt that the Corporation
personal check. is his alter ego.

Further, the evidence shows that when the Corporation was in its initial It is significant that not a single one of the acts enumerated above as
months of operation, Villarama purchased and paid with his personal proof of Villarama's oneness with the Corporation has been denied by
checks Ford trucks for the Corporation. Exhibits 20 and 21 disclose him. On the contrary, he has admitted them with offered excuses.
that the said purchases were paid by Philippine Bank of Commerce
Checks Nos. 992618-B and 993621-B, respectively. These checks
Villarama has admitted, for instance, having paid P85,000.00 of the
have been sufficiently established by Fausto Abad, Assistant
initial capital of the Corporation with the lame excuse that "his wife had
Accountant of Manila Trading & Supply Co., from which the trucks
requested him to reimburse the amount entrusted to her by the
were purchased9 and Aristedes Solano, an employee of the Philippine
incorporators and which she had used to pay the obligations of Dr.
Bank of Commerce,10 as having been drawn by Villarama.
Villarama (her husband) incurred while he was still the owner of Villa
Rey Transit, a single proprietorship." But with his admission that he
Exhibits 6 to 19 and Exh. 22, which are photostatic copies of ledger had received P350,000.00 from Pantranco for the sale of
entries and vouchers showing that Villarama had co-mingled his the two certificates and one unit,24 it becomes difficult to accept
personal funds and transactions with those made in the name of the Villarama's explanation that he and his wife, after consultation, 25 spent
Corporation, are very illuminating evidence. Villarama has assailed the the money of their relatives (the stockholders) when they were
admissibility of these exhibits, contending that no evidentiary value supposed to have their own money. Even if Pantranco paid the
whatsoever should be given to them since "they were merely P350,000.00 in check to him, as claimed, it could have been easy for
photostatic copies of the originals, the best evidence being the Villarama to have deposited said check in his account and issued his
originals themselves." According to him, at the time Pantranco offered own check to pay his obligations. And there is no evidence adduced
the said exhibits, it was the most likely possessor of the originals that the said amount of P350,000.00 was all spent or was insufficient to
thereof because they were stolen from the files of the Corporation and settle his prior obligations in his business, and in the light of the
only Pantranco was able to produce the alleged photostat copies stipulation in the deed of sale between Villarama and Pantranco that
thereof. P50,000.00 of the selling price was earmarked for the payments of
accounts due to his creditors, the excuse appears unbelievable.
Section 5 of Rule 130 of the Rules of Court provides for the requisites
for the admissibility of secondary evidence when the original is in the On his having paid for purchases by the Corporation of trucks from the
custody of the adverse party, thus: (1) opponent's possession of the Manila Trading & Supply Co. with his personal checks, his reason was
original; (2) reasonable notice to opponent to produce the original; (3) that he was only sharing with the Corporation his credit with some
satisfactory proof of its existence; and (4) failure or refusal of opponent companies. And his main reason for mingling his funds with that of the
to produce the original in court.11 Villarama has practically admitted the Corporation and for the latter's paying his private bills is that it would
second and fourth requisites.12 As to the third, he admitted their be more convenient that he kept the money to be used in paying the
previous existence in the files of the Corporation and also that he had registration fees on time, and since he had loaned money to the
seen some of them.13 Regarding the first element, Villarama's theory is Corporation, this would be set off by the latter's paying his bills.
that since even at the time of the issuance of the subpoena duces Villarama admitted, however, that the corporate funds in his
tecum, the originals were already missing, therefore, the Corporation possession were not only for registration fees but for other important
was no longer in possession of the same. However, it is not necessary obligations which were not specified.26
for a party seeking to introduce secondary evidence to show that the
original is in the actual possession of his adversary. It is enough that
Indeed, while Villarama was not the Treasurer of the Corporation but
the circumstances are such as to indicate that the writing is in his
was, allegedly, only a part-time manager,27 he admitted not only having
possession or under his control. Neither is it required that the party
held the corporate money but that he advanced and lent funds for the
entitled to the custody of the instrument should, on being notified to
Corporation, and yet there was no Board Resolution allowing it.28
produce it, admit having it in his possession.14 Hence, secondary
evidence is admissible where he denies having it in his possession.
The party calling for such evidence may introduce a copy thereof as in Villarama's explanation on the matter of his involvement with the
the case of loss. For, among the exceptions to the best evidence rule is corporate affairs of the Corporation only renders more credible
"when the original has been lost, destroyed, or cannot be produced in Pantranco's claim that his control over the corporation, especially in the
court."15 The originals of the vouchers in question must be deemed to management and disposition of its funds, was so extensive and
have been lost, as even the Corporation admits such loss. Viewed intimate that it is impossible to segregate and identify which money
upon this light, there can be no doubt as to the admissibility in belonged to whom. The interference of Villarama in the complex affairs
evidence of Exhibits 6 to 19 and 22. of the corporation, and particularly its finances, are much too
inconsistent with the ends and purposes of the Corporation law, which,
precisely, seeks to separate personal responsibilities from corporate
Taking account of the foregoing evidence, together with Celso Rivera's
undertakings. It is the very essence of incorporation that the acts and
testimony,16 it would appear that: Villarama supplied the organization
conduct of the corporation be carried out in its own corporate name
expenses and the assets of the Corporation, such as trucks and
because it has its own personality.
equipment;17 there was no actual payment by the original subscribers
of the amounts of P95,000.00 and P100,000.00 as appearing in the
books;18 Villarama made use of the money of the Corporation and The doctrine that a corporation is a legal entity distinct and separate
deposited them to his private accounts;19 and the Corporation paid his from the members and stockholders who compose it is recognized and
personal accounts.20 respected in all cases which are within reason and the law. 29 When the
fiction is urged as a means of perpetrating a fraud or an illegal act or as
a vehicle for the evasion of an existing obligation, the circumvention of
Villarama himself admitted that he mingled the corporate funds with his
statutes, the achievement or perfection of a monopoly or generally the
own money.21 He also admitted that gasoline purchases of the
perpetration of knavery or crime,30 the veil with which the law covers
Corporation were made in his name22 because "he had existing
and isolates the corporation from the members or stockholders who
account with Stanvac which was properly secured and he wanted the
compose it will be lifted to allow for its consideration merely as an
Corporation to benefit from the rebates that he received."23
aggregation of individuals.

The foregoing circumstances are strong persuasive evidence showing


Upon the foregoing considerations, We are of the opinion, and so hold,
that Villarama has been too much involved in the affairs of the
that the preponderance of evidence have shown that the Villa Rey
Corporation to altogether negative the claim that he was only a part-
Transit, Inc. is an alter ego of Jose M. Villarama, and that the restrictive

38
clause in the contract entered into by the latter and Pantranco is also may continue to tread the old footpath to his door and
enforceable and binding against the said Corporation. For the rule is maintain with him the business relations enjoyed by the
that a seller or promisor may not make use of a corporate entity as a seller.
means of evading the obligation of his covenant.31 Where the
Corporation is substantially the alter ego of the covenantor to the
... In order to be well assured of this, he obtains and pays for
restrictive agreement, it can be enjoined from competing with the
the seller's promise not to reopen business in competition
covenantee.32
with the business sold.

The Corporation contends that even on the supposition that Villa Rey
As to whether or not such a stipulation in restraint of trade is valid, our
Transit, Inc. and Villarama are one and the same, the restrictive clause
jurisprudence on the matter37says:
in the contract between Villarama and Pantranco does not include the
purchase of existing lines but it only applies to application for the new
lines. The clause in dispute reads thus: The law concerning contracts which tend to restrain business
or trade has gone through a long series of changes from
time to time with the changing condition of trade and
(4) The SELLER shall not, for a period of ten (10) years from
commerce. With trifling exceptions, said changes have been
the date of this sale apply for any TPU service identical or
a continuous development of a general rule. The early cases
competing with the BUYER. (Emphasis supplied)
show plainly a disposition to avoid and annul all contract
which prohibited or restrained any one from using a lawful
As We read the disputed clause, it is evident from the context thereof trade "at any time or at any place," as being against the
that the intention of the parties was to eliminate the seller as a benefit of the state. Later, however, the rule became well
competitor of the buyer for ten years along the lines of operation established that if the restraint was limited to "a certain time"
covered by the certificates of public convenience subject of their and within "a certain place," such contracts were valid and
transaction. The word "apply" as broadly used has for frame of not "against the benefit of the state." Later cases, and we
reference, a service by the seller on lines or routes that would compete think the rule is now well established, have held that a
with the buyer along the routes acquired by the latter. In this contract in restraint of trade is valid providing there is a
jurisdiction, prior authorization is needed before anyone can operate a limitation upon either time or place. A contract, however,
TPU service,33whether the service consists in a new line or an old one which restrains a man from entering into business or trade
acquired from a previous operator. The clear intention of the parties without either a limitation as to time or place, will be held
was to prevent the seller from conducting any competitive line for 10 invalid.
years since, anyway, he has bound himself not to apply for
authorization to operate along such lines for the duration of such
The public welfare of course must always be considered and
period.34
if it be not involved and the restraint upon one party is not
greater than protection to the other requires, contracts like
If the prohibition is to be applied only to the acquisition of new the one we are discussing will be sustained. The general
certificates of public convenience thru an application with the Public tendency, we believe, of modern authority, is to make the
Service Commission, this would, in effect, allow the seller just the test whether the restraint is reasonably necessary for the
same to compete with the buyer as long as his authority to operate is protection of the contracting parties. If the contract is
only acquired thru transfer or sale from a previous operator, thus reasonably necessary to protect the interest of the parties, it
defeating the intention of the parties. For what would prevent the seller, will be upheld. (Emphasis supplied.)
under the circumstances, from having a representative or dummy apply
in the latter's name and then later on transferring the same by sale to
Analyzing the characteristics of the questioned stipulation, We find that
the seller? Since stipulations in a contract is the law between the
although it is in the nature of an agreement suppressing competition, it
contracting parties,
is, however, merely ancillary or incidental to the main agreement which
is that of sale. The suppression or restraint is only partial or limited:
Every person must, in the exercise of his rights and in the first, in scope, it refers only to application for TPU by the seller in
performance of his duties, act with justice, give everyone his competition with the lines sold to the buyer; second, in duration, it is
due, and observe honesty and good faith. (Art. 19, New Civil only for ten (10) years; and third, with respect to situs or territory, the
Code.) restraint is only along the lines covered by the certificates sold. In view
of these limitations, coupled with the consideration of P350,000.00 for
just two certificates of public convenience, and considering,
We are not impressed of Villarama's contention that the re-wording of
furthermore, that the disputed stipulation is only incidental to a main
the two previous drafts of the contract of sale between Villarama and
agreement, the same is reasonable and it is not harmful nor obnoxious
Pantranco is significant in that as it now appears, the parties intended
to public service.38 It does not appear that the ultimate result of the
to effect the least restriction. We are persuaded, after an examination
clause or stipulation would be to leave solely to Pantranco the right to
of the supposed drafts, that the scope of the final stipulation, while not
operate along the lines in question, thereby establishing monopoly or
as long and prolix as those in the drafts, is just as broad and
predominance approximating thereto. We believe the main purpose of
comprehensive. At most, it can be said that the re-wording was done
the restraint was to protect for a limited time the business of the buyer.
merely for brevity and simplicity.

Indeed, the evils of monopoly are farfetched here. There can be no


The evident intention behind the restriction was to eliminate the sellers
danger of price controls or deterioration of the service because of the
as a competitor, and this must be, considering such factors as the
close supervision of the Public Service Commission.39 This Court had
good will35 that the seller had already gained from the riding public and
stated long ago,40 that "when one devotes his property to a use in
his adeptness and proficiency in the trade. On this matter, Corbin, an
which the public has an interest, he virtually grants to the public an
authority on Contracts has this to say.36
interest in that use and submits it to such public use under reasonable
rules and regulations to be fixed by the Public Utility Commission."
When one buys the business of another as a going concern,
he usually wishes to keep it going; he wishes to get the
Regarding that aspect of the clause that it is merely ancillary or
location, the building, the stock in trade, and the customers.
incidental to a lawful agreement, the underlying reason sustaining its
He wishes to step into the seller's shoes and to enjoy the
validity is well explained in 36 Am. Jur. 537-539, to wit:
same business relations with other men. He is willing to pay
much more if he can get the "good will" of the business,
meaning by this the good will of the customers, that they
39
... Numerous authorities hold that a covenant which is into the shoes of the judgment debtor. Of the same principle is the
incidental to the sale and transfer of a trade or business, and provision of Article 1544 of the Civil Code, that "If the same thing
which purports to bind the seller not to engage in the same should have been sold to different vendees, the ownership shall be
business in competition with the purchaser, is lawful and transferred to the person who may have first taken possession thereof
enforceable. While such covenants are designed to prevent in good faith, if it should be movable property."
competition on the part of the seller, it is ordinarily neither
their purpose nor effect to stifle competition generally in the
There is no merit in Pantranco and Ferrer's theory that the sale of the
locality, nor to prevent it at all in a way or to an extent
certificates of public convenience in question, between the Corporation
injurious to the public. The business in the hands of the
and Fernando, was not consummated, it being only a conditional sale
purchaser is carried on just as it was in the hands of the
subject to the suspensive condition of its approval by the Public
seller; the former merely takes the place of the latter; the
Service Commission. While section 20(g) of the Public Service Act
commodities of the trade are as open to the public as they
provides that "subject to established limitation and exceptions and
were before; the same competition exists as existed before;
saving provisions to the contrary, it shall be unlawful for any public
there is the same employment furnished to others after as
service or for the owner, lessee or operator thereof, without the
before; the profits of the business go as they did before to
approval and authorization of the Commission previously had ... to sell,
swell the sum of public wealth; the public has the same
alienate, mortgage, encumber or lease its property, franchise,
opportunities of purchasing, if it is a mercantile business; and
certificates, privileges, or rights or any part thereof, ...," the same
production is not lessened if it is a manufacturing plant.
section also provides:

The reliance by the lower court on tile case of Red Line Transportation
... Provided, however, That nothing herein contained shall be
Co. v. Bachrach41 and finding that the stipulation is illegal and void
construed to prevent the transaction from being negotiated
seems misplaced. In the said Red Line case, the agreement therein
or completed before its approval or to prevent the sale,
sought to be enforced was virtually a division of territory between two
alienation, or lease by any public service of any of its
operators, each company imposing upon itself an obligation not to
property in the ordinary course of its business.
operate in any territory covered by the routes of the other. Restraints of
this type, among common carriers have always been covered by the
general rule invalidating agreements in restraint of trade. 42 It is clear, therefore, that the requisite approval of the PSC is not a
condition precedent for the validity and consummation of the sale.
Neither are the other cases relied upon by the plaintiff-appellee
applicable to the instant case. In Pampanga Bus Co., Inc. v. Anent the question of damages allegedly suffered by the parties, each
Enriquez,43the undertaking of the applicant therein not to apply for the of the appellants has its or his own version to allege.
lifting of restrictions imposed on his certificates of public convenience
was not an ancillary or incidental agreement. The restraint was the
Villa Rey Transit, Inc. claims that by virtue of the "tortious acts" of
principal objective. On the other hand, in Red Line Transportation Co.,
Inc. v. Gonzaga,44 the restraint there in question not to ask for defendants (Pantranco and Ferrer) in acquiring the certificates of public
extension of the line, or trips, or increase of equipment — was not an convenience in question, despite constructive and actual knowledge on
their part of a prior sale executed by Fernando in favor of the said
agreement between the parties but a condition imposed in the
certificate of public convenience itself. corporation, which necessitated the latter to file the action to annul the
sheriff's sale to Ferrer and the subsequent transfer to Pantranco, it is
entitled to collect actual and compensatory damages, and attorney's
Upon the foregoing considerations, Our conclusion is that the fees in the amount of P25,000.00. The evidence on record, however,
stipulation prohibiting Villarama for a period of 10 years to "apply" for does not clearly show that said defendants acted in bad faith in their
TPU service along the lines covered by the certificates of public acquisition of the certificates in question. They believed that because
convenience sold by him to Pantranco is valid and reasonable. Having the bill of sale has yet to be approved by the Public Service
arrived at this conclusion, and considering that the preponderance of Commission, the transaction was not a consummated sale, and,
the evidence have shown that Villa Rey Transit, Inc. is itself the alter therefore, the title to or ownership of the certificates was still with the
ego of Villarama, We hold, as prayed for in Pantranco's third party seller. The award by the lower court of attorney's fees of P5,000.00 in
complaint, that the said Corporation should, until the expiration of the favor of Villa Rey Transit, Inc. is, therefore, without basis and should be
1-year period abovementioned, be enjoined from operating the line set aside.
subject of the prohibition.
Eusebio Ferrer's charge that by reason of the filing of the action to
To avoid any misunderstanding, it is here to be emphasized that the annul the sheriff's sale, he had suffered and should be awarded moral,
10-year prohibition upon Villarama is not against his application for, or exemplary damages and attorney's fees, cannot be entertained, in
purchase of, certificates of public convenience, but merely the view of the conclusion herein reached that the sale by Fernando to the
operation of TPU along the lines covered by the certificates sold by him Corporation was valid.
to Pantranco. Consequently, the sale between Fernando and the
Corporation is valid, such that the rightful ownership of the disputed
certificates still belongs to the plaintiff being the prior purchaser in good Pantranco, on the other hand, justifies its claim for damages with the
allegation that when it purchased ViIlarama's business for
faith and for value thereof. In view of the ancient rule of caveat
emptor prevailing in this jurisdiction, what was acquired by Ferrer in the P350,000.00, it intended to build up the traffic along the lines covered
sheriff's sale was only the right which Fernando, judgment debtor, had by the certificates but it was rot afforded an opportunity to do so since
barely three months had elapsed when the contract was violated by
in the certificates of public convenience on the day of the sale. 45
Villarama operating along the same lines in the name of Villa Rey
Transit, Inc. It is further claimed by Pantranco that the underhanded
Accordingly, by the "Notice of Levy Upon Personalty" the manner in which Villarama violated the contract is pertinent in
Commissioner of Public Service was notified that "by virtue of an Order establishing punitive or moral damages. Its contention as to the proper
of Execution issued by the Court of First Instance of Pangasinan, the measure of damages is that it should be the purchase price of
rights, interests, or participation which the defendant, VALENTIN A. P350,000.00 that it paid to Villarama. While We are fully in accord with
FERNANDO — in the above entitled case may have in the following Pantranco's claim of entitlement to damages it suffered as a result of
realty/personalty is attached or levied upon, to wit: The rights, interests Villarama's breach of his contract with it, the record does not
and participation on the Certificates of Public Convenience issued to sufficiently supply the necessary evidentiary materials upon which to
Valentin A. Fernando, in Cases Nos. 59494, etc. ... Lines — Manila to base the award and there is need for further proceedings in the lower
Lingayen, Dagupan, etc. vice versa." Such notice of levy only shows court to ascertain the proper amount.
that Ferrer, the vendee at auction of said certificates, merely stepped

40
PREMISES CONSIDERED, the judgment appealed from is hereby
modified as follows:

1. The sale of the two certificates of public convenience in question by


Valentin Fernando to Villa Rey Transit, Inc. is declared preferred over
that made by the Sheriff at public auction of the aforesaid certificate of
public convenience in favor of Eusebio Ferrer;

2. Reversed, insofar as it dismisses the third-party complaint filed by


Pangasinan Transportation Co. against Jose M. Villarama, holding that
Villa Rey Transit, Inc. is an entity distinct and separate from the
personality of Jose M. Villarama, and insofar as it awards the sum of
P5,000.00 as attorney's fees in favor of Villa Rey Transit, Inc.;

3. The case is remanded to the trial court for the reception of evidence
in consonance with the above findings as regards the amount of
damages suffered by Pantranco; and

4. On equitable considerations, without costs. So ordered.

41
[G.R. NO. 160039 : June 29, 2004] 5.The sum of P50,000.00 as exemplary
damages;chanroblesvirtuallawlibrary
RAYMUNDO ODANI SECOSA, EL BUENASENSO SYand
DASSAD WAREHOUSINGand PORT SERVICES, 6.The sum of P50,000.00 as attorneys fees plus cost of suit.
INCORPORATED, Petitioners, v. HEIRS OF ERWIN
SUAREZ FRANCISCO, Respondents.
SO ORDERED.

DECISION
Petitioners appealed the decision to the Court of Appeals,
which affirmed the appealed decision in toto.4 cralawred
YNARES-SANTIAGO, J.:
Hence the present petition, based on the following
This is a Petition for Review under Rule 45 of the Rules of arguments:
Court seeking the reversal of the decision1 of the Court of
Appeals dated February 27, 2003 in CA-G.R. CV No. 61868,
I.
which affirmed in toto the June 19, 1998 decision2 of Branch
20 of the Regional Trial Court of Manila in Civil Case No. 96-
79554. THE COURT OF APPEALS SERIOUSLY ERRED WHEN IT
AFFIRMED THE DECISION OF THE TRIAL COURT THAT
PETITIONER DASSAD DID NOT EXERCISE THE DILIGENCE OF
The facts are as follows:chanroblesvirtua1awlibrary
A GOOD FATHER OF A FAMILY IN THE SELECTION AND
SUPERVISION OF ITS EMPLOYEES WHICH IS NOT IN
On June 27, 1996, at around 4:00 p.m., Erwin Suarez ACCORDANCE WITH ARTICLE 2180 OF THE NEW CIVIL CODE
Francisco, an eighteen year old third year physical therapy AND RELATED JURISPRUDENCE ON THE MATTER.
student of the Manila Central University, was riding a
motorcycle along Radial 10 Avenue, near the Veteran
II.
Shipyard Gate in the City of Manila.At the same time,
Petitioner, Raymundo Odani Secosa, was driving an Isuzu
cargo truck with plate number PCU-253 on the same road. THE COURT OF APPEALS SERIOUSLY ERRED WHEN IT
The truck was owned by petitioner, Dassad Warehousing and AFFIRMED THE DECISION OF THE TRIAL COURT IN HOLDING
Port Services, Inc. PETITIONER EL BUENASENSO SY SOLIDARILY LIABLE WITH
PETITIONERS DASSAD AND SECOSA IN VIOLATION OF THE
CORPORATION LAW AND RELATED JURISPRUDENCE ON THE
Traveling behind the motorcycle driven by Francisco was a
MATTER.
sand and gravel truck, which in turn was being tailed by the
Isuzu truck driven by Secosa.The three vehicles were
traversing the southbound lane at a fairly high speed.When III.
Secosa overtook the sand and gravel truck, he bumped the
motorcycle causing Francisco to fall.The rear wheels of the THE JUDGMENT OF THE TRIAL COURT AS AFFIRMED BY THE
Isuzu truck then ran over Francisco, which resulted in his COURT OF APPEALS AWARDING P500,000.00 AS MORAL
instantaneous death.Fearing for his life, petitioner Secosa left DAMAGES IS MANIFESTLY ABSURD, MISTAKEN AND
his truck and fled the scene of the collision.3 cralawred UNJUST.5 cralawred

Respondents, the parents of Erwin Francisco, thus filed an The petition is partly impressed with merit.
action for damages against Raymond Odani Secosa, Dassad
Warehousing and Port Services, Inc. and Dassads president,
El Buenasucenso Sy.The complaint was docketed as Civil Case On the issue of whether petitioner Dassad Warehousing and
No.96-79554 of the RTC of Manila, Branch 20. Port Services, Inc. exercised the diligence of a good father of
a family in the selection and supervision of its employees, we
find the assailed decision to be in full accord with pertinent
On June 19, 1998, after a full-blown trial, the court a provisions of law and established jurisprudence.
quo rendered a decision in favor of herein respondents, the
dispositive portion of which states:chanroblesvirtua1awlibrary
Article 2176 of the Civil Code
provides:chanroblesvirtua1awlibrary
WHEREFORE, premised on the foregoing, judgment is hereby
rendered in favor of the plaintiffs ordering the defendants to
pay plaintiffs jointly and severally:chanroblesvirtua1awlibrary Whoever by act or omission causes damage to another, there
being fault or negligence, is obliged to pay for the damage
done.Such fault or negligence, if there is no pre-existing
1.The sum of P55,000.00 as actual and compensatory contractual relation between the parties, is called a quasi-
damages;chanroblesvirtuallawlibrary delict and is governed by the provisions of this Chapter.

2.The sum of P20,000.00 for the repair of the On the other hand, Article 2180, in pertinent part,
motorcycle;chanroblesvirtuallawlibrary states:chanroblesvirtua1awlibrary

3.The sum of P100,000.00 for the loss of earning The obligation imposed by article 2176 is demandable not
capacity;chanroblesvirtuallawlibrary only for ones own acts or omissions, but also for those of
persons for whom one is responsible x x x.
4.The sum of P500,000.00 as moral
damages;chanroblesvirtuallawlibrary
42
Employers shall be liable for the damages caused by their a valid defense to the legal presumption of negligence on the
employees and household helpers acting within the scope of part of an employer or master whose employee has by his
their assigned tasks, even though the former are not engaged negligence, caused damage to another. x x x (R) educing the
in any business or industry x x x. testimony of Albert to its proper proportion, we do not have
enough trustworthy evidence left to go by. We are of the
considered opinion, therefore, that the believable evidence on
The responsibility treated of in this article shall cease when
the degree of care and diligence that has been exercised in
the persons herein mentioned prove that they observed all
the selection and supervision of Roberto Leon y Salazar, is not
the diligence of a good father of a family to prevent damage.
legally sufficient to overcome the presumption of negligence
against the defendant company.
Based on the foregoing provisions, when an injury is caused
by the negligence of an employee, there instantly arises a
The above-quoted ruling was reiterated in a recent case again
presumption that there was negligence on the part of the
involving the Metro Manila Transit
employer either in the selection of his employee or in the
Corporation,12 thus:chanroblesvirtua1awlibrary
supervision over him after such selection.The presumption,
however, may be rebutted by a clear showing on the part of
the employer that it exercised the care and diligence of a In the selection of prospective employees, employers are
good father of a family in the selection and supervision of his required to examine them as to their qualifications,
employee.Hence, to evade solidary liability for quasi-delict experience, and service records.13 On the other hand, with
committed by an employee, the employer must adduce respect to the supervision of employees, employers should
sufficient proof that it exercised such degree of formulate standard operating procedures, monitor their
care.6 cralawred implementation, and impose disciplinary measures for
breaches thereof. To establish these factors in a trial involving
the issue of vicarious liability, employers must submit
How does an employer prove that he indeed exercised the
concrete proof, including documentary evidence.
diligence of a good father of a family in the selection and
supervision of his employee? The case of Metro Manila Transit
Corporation v. Court of Appeals7 is In this case, MMTC sought to prove that it exercised the
instructive:chanroblesvirtua1awlibrary diligence of a good father of a family with respect to the
selection of employees by presenting mainly testimonial
evidence on its hiring procedure. According to MMTC,
In fine, the party, whether plaintiff or defendant, who asserts
applicants are required to submit professional driving
the affirmative of the issue has the burden of presenting at
licenses, certifications of work experience, and clearances
the trial such amount of evidence required by law to obtain a
from the National Bureau of Investigation; to undergo tests of
favorable judgment8 . .. In making proof in its or his case, it is
their driving skills, concentration, reflexes, and vision; and, to
paramount that the best and most complete evidence is
complete training programs on traffic rules, vehicle
formally entered.9 cralawred
maintenance, and standard operating procedures during
emergency cases.
Coming now to the case at bar, while there is no rule which
requires that testimonial evidence, to hold sway, must be
xxx
corroborated by documentary evidence, inasmuch as the
witnesses testimonies dwelt on mere generalities, we cannot
consider the same as sufficiently persuasive proof that there Although testimonies were offered that in the case of Pedro
was observance of due diligence in the selection and Musa all these precautions were followed, the records of his
supervision of employees. Petitioners attempt to prove interview, of the results of his examinations, and of his
its deligentissimi patris familias in the selection and service were not presented.. . [T]here is no record that Musa
supervision of employees through oral evidence must fail as it attended such training programs and passed the said
was unable to buttress the same with any other evidence, examinations before he was employed. No proof was
object or documentary, which might obviate the apparent presented that Musa did not have any record of traffic
biased nature of the testimony.10 cralawred violations. Nor were records of daily inspections, allegedly
conducted by supervisors, ever presented.. . The failure of
MMTC to present such documentary proof puts in doubt the
Our view that the evidence for petitioner MMTC falls short of
credibility of its witnesses.
the required evidentiary quantum as would convincingly and
undoubtedly prove its observance of the diligence of a good
father of a family has its precursor in the underlying rationale Jurisprudentially, therefore, the employer must not merely
pronounced in the earlier case of Central Taxicab Corp. v. Ex- present testimonial evidence to prove that he observed the
Meralco Employees Transportation Co., et al.,11 set amidst an diligence of a good father of a family in the selection and
almost identical factual setting, where we held supervision of his employee, but he must also support such
that:chanroblesvirtua1awlibrary testimonial evidence with concrete or documentary evidence.
The reason for this is to obviate the biased nature of the
employers testimony or that of his witnesses.14 cralawred
The failure of the defendant company to produce in court any
record or other documentary proof tending to establish that it
had exercised all the diligence of a good father of a family in Applying the foregoing doctrines to the present case, we hold
the selection and supervision of its drivers and buses, that petitioner Dassad Warehousing and Port Services, Inc.
notwithstanding the calls therefor by both the trial court and failed to conclusively prove that it had exercised the requisite
the opposing counsel, argues strongly against its pretensions. diligence of a good father of a family in the selection and
supervision of its employees.
We are fully aware that there is no hard-and-fast rule on the
quantum of evidence needed to prove due observance of all Edilberto Duerme, the lone witness presented by Dassad
the diligence of a good father of a family as would constitute Warehousing and Port Services, Inc. to support its position

43
that it had exercised the diligence of a good father of a family Having both found Raymundo Secosa and Dassad
in the selection and supervision of its employees, testified Warehousing and Port Services, Inc. liable for negligence for
that he was the one who recommended petitioner Raymundo the death of Erwin Francisco on June 27, 1996, we now
Secosa as a driver to Dassad Warehousing and Port Services, consider the question of moral damages which his parents,
Inc.; that it was his duty to scrutinize the capabilities of herein respondents, are entitled to recover.Petitioners assail
drivers; and that he believed petitioner to be physically and the award of moral damages of P500,000.00 for being
mentally fit for he had undergone rigid training and attended manifestly absurd, mistaken and unjust.We are not
the PPA safety seminar.15 cralawred persuaded.

Petitioner Dassad Warehousing and Port Services, Inc. failed Under Article 2206, the spouse, legitimate and illegitimate
to support the testimony of its lone witness with documentary descendants and ascendants of the deceased may demand
evidence which would have strengthened its claim of due moral damages for mental anguish for the death of the
diligence in the selection and supervision of its deceased. The reason for the grant of moral damages has
employees.Such an omission is fatal to its position, on been explained in this wise:chanroblesvirtua1awlibrary
account of which, Dassad can be rightfully held solidarily
liable with its co-petitioner Raymundo Secosa for the
.. . the award of moral damages is aimed at a restoration,
damages suffered by the heirs of Erwin Francisco.
within the limits possible, of the spiritual status quo ante; and
therefore, it must be proportionate to the suffering inflicted.
However, we find that petitioner El Buenasenso Sy cannot be The intensity of the pain experienced by the relatives of the
held solidarily liable with his co-petitioners. While it may be victim is proportionate to the intensity of affection for him and
true that Sy is the president of petitioner Dassad bears no relation whatsoever with the wealth or means of the
Warehousing and Port Services, Inc., such fact is not by itself offender.22 cralawred
sufficient to hold him solidarily liable for the liabilities
adjudged against his co-petitioners.
In the instant case, the spouses Francisco presented evidence
of the searing pain that they felt when the premature loss of
It is a settled precept in this jurisdiction that a corporation is their son was relayed to them. That pain was highly evident in
invested by law with a personality separate from that of its the testimony of the father who was forever deprived of a
stockholders or members.16 It has a personality separate and son, a son whose untimely death came at that point when the
distinct from those of the persons composing it as well as latter was nearing the culmination of every parents wish to
from that of any other entity to which it may be related. Mere educate their children.The death of Francis has indeed left a
ownership by a single stockholder or by another corporation void in the lives of the respondents. Antonio Francisco
of all or nearly all of the capital stock of a corporation is not in testified on the effect of the death of his son, Francis, in this
itself sufficient ground for disregarding the separate corporate manner:chanroblesvirtua1awlibrary
personality.17 A corporations authority to act and its liability
for its actions are separate and apart from the individuals who
Q: (Atty. Balanag) :What did you do when you learned that
own it.18 cralawred
your son was killed on June 27,
1996?chanroblesvirtualawlibrary
The so-called veil of corporation fiction treats as separate and
distinct the affairs of a corporation and its officers and
A: (ANTONIO FRANCISCO) :I boxed the door and pushed the
stockholders. As a general rule, a corporation will be looked
image of St. Nio telling why this happened to us.
upon as a legal entity, unless and until sufficient reason to the
contrary appears.When the notion of legal entity is used to
defeat public convenience, justify wrong, protect fraud, or Q: Mr. Witness, how did you feel when you learned of the
defend crime, the law will regard the corporation as an untimely death of your son, Erwin Suares (sic)
association of persons.19 Also, the corporate entity may be ?chanroblesvirtualawlibrary
disregarded in the interest of justice in such cases as fraud
that may work inequities among members of the corporation A: Masakit po ang mawalan ng anak. Its really hard for me,
internally, involving no rights of the public or third persons.In the thought that my son is dead.
both instances, there must have been fraud and proof of it.
For the separate juridical personality of a corporation to be
disregarded, the wrongdoing must be clearly and convincingly xxx
established.20 It cannot be presumed.21 cralawred
Q: How did your family react to the death of Erwin Suarez
The records of this case are bereft of any evidence tending to Francisco?chanroblesvirtualawlibrary
show the presence of any grounds enumerated above that will
justify the piercing of the veil of corporate fiction such as to A: All of my family and relatives were felt (sic) sorrow
hold the president of Dassad Warehousing and Port Services, because they knew that my son is (sic) good.
Inc. solidarily liable with it.

Q: We know that it is impossible to put money terms(s) [on]


The Isuzu cargo truck which ran over Erwin Francisco was the life of [a] human, but since you are now in court and if
registered in the name of Dassad Warehousing and Port you were to ask this court how much would you and your
Services, Inc., and not in the name of El Buenasenso family compensate? (sic)
Sy.Raymundo Secosa is an employee of Dassad Warehousing
and Port Services, Inc. and not of El Buenasenso Sy.All these
things, when taken collectively, point toward El Buenasenso A: Even if they pay me millions, they cannot remove the
Sys exclusion from liability for damages arising from the anguish of my son (sic). 23 cralawred
death of Erwin Francisco.

4
Moral damages are emphatically not intended to enrich a
plaintiff at the expense of the defendant. They are awarded to
allow the former to obtain means, diversion or amusements
that will serve to alleviate the moral suffering he has
undergone due to the defendants culpable action and must,
perforce, be proportional to the suffering inflicted.24 We have
previously held as proper an award of P500,000.00 as moral
damages to the heirs of a deceased family member who died
in a vehicular accident. In our 2002 decision in Metro Manila
Transit Corporation v. Court of Appeals, et al.,25 we affirmed
the award of moral damages of P500,000.00 to the heirs of
the victim, a mother, who died from injuries she sustained
when a bus driven by an employee of the petitioner hit her.In
the case at bar, we likewise affirm the portion of the assailed
decision awarding the moral damages.

Since the petitioners did not question the other damages


adjudged against them by the court a quo, we affirm the
award of these damages to the Respondents.

WHEREFORE, the petition is DENIED.The assailed decision is


AFFIRMED with the MODIFICATION that petitioner El
Buenasenso Sy is ABSOLVED from any liability adjudged
against his co-petitioners in this case.

Costs against petitioners.

SO ORDERED.

45
G.R. No. 97212 June 30, 1993 fees, against Jade Mountain, Mr. Willy Co and the other private
respondents. The partnership and Willy Co denied petitioner's charges,
contending in the main that Benjamin Yu was never hired as an
BENJAMIN YU, petitioner,
employee by the present or new partnership.4
vs.
NATIONAL LABOR RELATIONS COMMISSION and JADE
MOUNTAIN PRODUCTS COMPANY LIMITED, WILLY CO, In due time, Labor Arbiter Nieves Vivar-De Castro rendered a decision
RHODORA D. BENDAL, LEA BENDAL, CHIU SHIAN JENG and holding that petitioner had been illegally dismissed. The Labor Arbiter
CHEN HO-FU, respondents. decreed his reinstatement and awarded him his claim for unpaid
salaries, backwages and attorney's fees.5
Jose C. Guico for petitioner.
On appeal, the National Labor Relations Commission ("NLRC")
reversed the decision of the Labor Arbiter and dismissed petitioner's
Wilfredo Cortez for private respondents.
complaint in a Resolution dated 29 November 1990. The NLRC held
that a new partnership consisting of Mr. Willy Co and Mr. Emmanuel
Zapanta had bought the Jade Mountain business, that the new
partnership had not retained petitioner Yu in his original position as
Assistant General Manager, and that there was no law requiring the
FELICIANO, J.:
new partnership to absorb the employees of the old partnership.
Benjamin Yu, therefore, had not been illegally dismissed by the new
Petitioner Benjamin Yu was formerly the Assistant General Manager of partnership which had simply declined to retain him in his former
the marble quarrying and export business operated by a registered managerial position or any other position. Finally, the NLRC held that
partnership with the firm name of "Jade Mountain Products Company Benjamin Yu's claim for unpaid wages should be asserted against the
Limited" ("Jade Mountain"). The partnership was originally organized original members of the preceding partnership, but these though
on 28 June 1984 with Lea Bendal and Rhodora Bendal as general impleaded had, apparently, not been served with summons in the
partners and Chin Shian Jeng, Chen Ho-Fu and Yu Chang, all citizens proceedings before the Labor Arbiter.6
of the Republic of China (Taiwan), as limited partners. The partnership
business consisted of exploiting a marble deposit found on land owned Petitioner Benjamin Yu is now before the Court on a Petition
by the Sps. Ricardo and Guillerma Cruz, situated in Bulacan Province, for Certiorari, asking us to set aside and annul the Resolution of the
under a Memorandum Agreement dated 26 June 1984 with the Cruz
NLRC as a product of grave abuse of discretion amounting to lack or
spouses. 1 The partnership had its main office in Makati, Metropolitan excess of jurisdiction.
Manila.

The basic contention of petitioner is that the NLRC has overlooked the
Benjamin Yu was hired by virtue of a Partnership Resolution dated 14 principle that a partnership has a juridical personality separate and
March 1985, as Assistant General Manager with a monthly salary of distinct from that of each of its members. Such independent legal
P4,000.00. According to petitioner Yu, however, he actually received personality subsists, petitioner claims, notwithstanding changes in the
only half of his stipulated monthly salary, since he had accepted the identities of the partners. Consequently, the employment contract
promise of the partners that the balance would be paid when the firm
between Benjamin Yu and the partnership Jade Mountain could not
shall have secured additional operating funds from abroad. Benjamin have been affected by changes in the latter's membership.7
Yu actually managed the operations and finances of the business; he
had overall supervision of the workers at the marble quarry in Bulacan
and took charge of the preparation of papers relating to the exportation Two (2) main issues are thus posed for our consideration in the case at
of the firm's products. bar: (1) whether the partnership which had hired petitioner Yu as
Assistant General Manager had been extinguished and replaced by a
new partnerships composed of Willy Co and Emmanuel Zapanta; and
Sometime in 1988, without the knowledge of Benjamin Yu, the general (2) if indeed a new partnership had come into existence, whether
partners Lea Bendal and Rhodora Bendal sold and transferred their petitioner Yu could nonetheless assert his rights under his employment
interests in the partnership to private respondent Willy Co and to one
contract as against the new partnership.
Emmanuel Zapanta. Mr. Yu Chang, a limited partner, also sold and
transferred his interest in the partnership to Willy Co. Between Mr.
Emmanuel Zapanta and himself, private respondent Willy Co acquired In respect of the first issue, we agree with the result reached by the
the great bulk of the partnership interest. The partnership now NLRC, that is, that the legal effect of the changes in the membership of
constituted solely by Willy Co and Emmanuel Zapanta continued to use the partnership was the dissolution of the old partnership which had
the old firm name of Jade Mountain, though they moved the firm's main hired petitioner in 1984 and the emergence of a new firm composed of
office from Makati to Mandaluyong, Metropolitan Manila. A Supplement Willy Co and Emmanuel Zapanta in 1987.
to the Memorandum Agreement relating to the operation of the marble
quarry was entered into with the Cruz spouses in February of
The applicable law in this connection — of which the NLRC seemed
1988.2 The actual operations of the business enterprise continued as
quite unaware — is found in the Civil Code provisions relating to
before. All the employees of the partnership continued working in the
partnerships. Article 1828 of the Civil Code provides as follows:
business, all, save petitioner Benjamin Yu as it turned out.

Art. 1828. The dissolution of a partnership is the


On 16 November 1987, having learned of the transfer of the firm's
change in the relation of the partners caused by
main office from Makati to Mandaluyong, petitioner Benjamin Yu
any partner ceasing to be associated in the
reported to the Mandaluyong office for work and there met private
carrying on as distinguished from the winding up of
respondent Willy Co for the first time. Petitioner was informed by Willy
the business. (Emphasis supplied)
Co that the latter had bought the business from the original partners
and that it was for him to decide whether or not he was responsible for
the obligations of the old partnership, including petitioner's unpaid Article 1830 of the same Code must also be noted:
salaries. Petitioner was in fact not allowed to work anymore in the Jade
Mountain business enterprise. His unpaid salaries remained unpaid. 3
Art. 1830. Dissolution is caused:

On 21 December 1988. Benjamin Yu filed a complaint for illegal


(1) without violation of the agreement between the
dismissal and recovery of unpaid salaries accruing from November
partners;
1984 to October 1988, moral and exemplary damages and attorney's
46
xxx xxx xxx the affairs of the partnership. In the case at bar, it is important to
underscore the fact that the business of the old partnership was simply
continued by the new partners, without the old partnership undergoing
(b) by the
the procedures relating to dissolution and winding up of its business
express
affairs. In other words, the new partnership simply took over the
will of any
business enterprise owned by the preceeding partnership, and
partner,
continued using the old name of Jade Mountain Products Company
who must
Limited, without winding up the business affairs of the old partnership,
act in
paying off its debts, liquidating and distributing its net assets, and then
good
re-assembling the said assets or most of them and opening a new
faith,
business enterprise. There were, no doubt, powerful tax considerations
when no
which underlay such an informal approach to business on the part of
definite
the retiring and the incoming partners. It is not, however, necessary to
term or
inquire into such matters.
particular
undertaki
ng is What is important for present purposes is that, under the above
specified; described situation, not only the retiring partners (Rhodora Bendal, et
al.) but also the new partnership itself which continued the business of
the old, dissolved, one, are liable for the debts of the preceding
xxx xxx xxx
partnership. In Singson, et al. v. Isabela Saw Mill, et al,8 the Court held
that under facts very similar to those in the case at bar, a withdrawing
(2) in partner remains liable to a third party creditor of the old
contraven partnership.9 The liability of the new partnership, upon the other hand,
tion of the in the set of circumstances obtaining in the case at bar, is established
agreemen in Article 1840 of the Civil Code which reads as follows:
t between
the
Art. 1840. In the following cases creditors of the
partners,
dissolved partnership are also creditors of the
where the
person or partnership continuing the business:
circumsta
nces do
not permit (1) When any new partner is admitted into an
a existing partnership, or when any partner retires
dissolutio and assigns (or the representative of the deceased
n under partner assigns) his rights in partnership property
any other to two or more of the partners, or to one or more of
provision the partners and one or more third persons, if the
of this business is continued without liquidation of the
article, by partnership affairs;
the
express
(2) When all but one partner retire and assign (or
will of any
the representative of a deceased partner assigns)
partner at
their rights in partnership property to the remaining
any time;
partner, who continues the business without
liquidation of partnership affairs, either alone or
xxx xxx xxx with others;

(Emphasi (3) When any Partner retires or dies and the


s business of the dissolved partnership is
supplied) continued as set forth in Nos. 1 and 2 of this
Article, with the consent of the retired partners or
the representative of the deceased partner, but
In the case at bar, just about all of the partners had sold their
without any assignment of his right in partnership
partnership interests (amounting to 82% of the total partnership
property;
interest) to Mr. Willy Co and Emmanuel Zapanta. The record does not
show what happened to the remaining 18% of the original partnership
interest. The acquisition of 82% of the partnership interest by new (4) When all the partners or their representatives
partners, coupled with the retirement or withdrawal of the partners who assign their rights in partnership property to one or
had originally owned such 82% interest, was enough to constitute a more third persons who promise to pay the debts
new partnership. and who continue the business of the dissolved
partnership;
The occurrence of events which precipitate the legal consequence of
dissolution of a partnership do not, however, automatically result in the (5) When any partner wrongfully causes a
termination of the legal personality of the old partnership. Article 1829 dissolution and remaining partners continue the
of the Civil Code states that: business under the provisions of article 1837,
second paragraph, No. 2, either alone or with
others, and without liquidation of the partnership
[o]n dissolution the partnership is not terminated,
affairs;
but continues until the winding up of partnership
affairs is completed.
(6) When a partner is expelled and the remaining
partners continue the business either alone or with
In the ordinary course of events, the legal personality of the expiring
others without liquidation of the partnership affairs;
partnership persists for the limited purpose of winding up and closing of

47
The liability of a third person becoming a partner in notify him of the change in ownership of the business, the relocation of
the partnership continuing the business, under this the main office of Jade Mountain from Makati to Mandaluyong and the
article, to the creditors of the dissolved partnership assumption by Mr. Willy Co of control of operations. The treatment
shall be satisfied out of the partnership property (including the refusal to honor his claim for unpaid wages) accorded to
only, unless there is a stipulation to the contrary. Assistant General Manager Benjamin Yu was so summary and cavalier
as to amount to arbitrary, bad faith treatment, for which the new Jade
Mountain may legitimately be required to respond by paying moral
When the business of a partnership after
damages. This Court, exercising its discretion and in view of all the
dissolution is continued under any conditions set
circumstances of this case, believes that an indemnity for moral
forth in this article the creditors of the retiring or
damages in the amount of P20,000.00 is proper and reasonable.
deceased partner or the representative of the
deceased partner, have a prior right to any claim
of the retired partner or the representative of the In addition, we consider that petitioner Benjamin Yu is entitled to
deceased partner against the person or interest at the legal rate of six percent (6%) per annum on the amount
partnership continuing the business on account of of unpaid wages, and of his separation pay, computed from the date of
the retired or deceased partner's interest in the promulgation of the award of the Labor Arbiter. Finally, because the
dissolved partnership or on account of any new Jade Mountain compelled Benjamin Yu to resort to litigation to
consideration promised for such interest or for his protect his rights in the premises, he is entitled to attorney's fees in the
right in partnership property. amount of ten percent (10%) of the total amount due from private
respondent Jade Mountain.
Nothing in this article shall be held to modify any
right of creditors to set assignment on the ground WHEREFORE, for all the foregoing, the Petition for Certiorari is
of fraud. GRANTED DUE COURSE, the Comment filed by private respondents
is treated as their Answer to the Petition for Certiorari, and the Decision
of the NLRC dated 29 November 1990 is hereby NULLIFIED and SET
xxx xxx xxx
ASIDE. A new Decision is hereby ENTERED requiring private
respondent Jade Mountain Products Company Limited to pay to
(Emphasis supplied) petitioner Benjamin Yu the following amounts:

Under Article 1840 above, creditors of the old Jade Mountain are also (a) for unpaid wages which,
creditors of the new Jade Mountain which continued the business of as found by the Labor Arbiter,
the old one without liquidation of the partnership affairs. Indeed, a shall be computed at the rate
creditor of the old Jade Mountain, like petitioner Benjamin Yu in of P2,000.00 per month
respect of his claim for unpaid wages, is entitled to priority vis-a-vis any multiplied by thirty-six (36)
claim of any retired or previous partner insofar as such retired partner's months (November 1984 to
interest in the dissolved partnership is concerned. It is not necessary December 1987) in the total
for the Court to determine under which one or mare of the above six (6) amount of P72,000.00;
paragraphs, the case at bar would fall, if only because the facts on
record are not detailed with sufficient precision to permit such
(b) separation pay computed
determination. It is, however, clear to the Court that under Article 1840
at the rate of P4,000.00
above, Benjamin Yu is entitled to enforce his claim for unpaid salaries,
monthly pay multiplied by
as well as other claims relating to his employment with the previous
three (3) years of service or a
partnership, against the new Jade Mountain.
total of P12,000.00;

It is at the same time also evident to the Court that the new partnership
(c) indemnity for moral
was entitled to appoint and hire a new general or assistant general
damages in the amount of
manager to run the affairs of the business enterprise take over. An
P20,000.00;
assistant general manager belongs to the most senior ranks of
management and a new partnership is entitled to appoint a top
manager of its own choice and confidence. The non-retention of (d) six percent (6%) per
Benjamin Yu as Assistant General Manager did not therefore constitute annum legal interest
unlawful termination, or termination without just or authorized cause. computed on items (a) and (b)
We think that the precise authorized cause for termination in the case above, commencing on 26
at bar was redundancy. 10 The new partnership had its own new December 1989 and until fully
General Manager, apparently Mr. Willy Co, the principal new owner paid; and
himself, who personally ran the business of Jade Mountain. Benjamin
Yu's old position as Assistant General Manager thus became
(e) ten percent (10%)
superfluous or redundant. 11 It follows that petitioner Benjamin Yu is
attorney's fees on the total
entitled to separation pay at the rate of one month's pay for each year
amount due from private
of service that he had rendered to the old partnership, a fraction of at
respondent Jade Mountain.
least six (6) months being considered as a whole year.

Costs against private respondents.


While the new Jade Mountain was entitled to decline to retain petitioner
Benjamin Yu in its employ, we consider that Benjamin Yu was very
shabbily treated by the new partnership. The old partnership certainly SO ORDERED.
benefitted from the services of Benjamin Yu who, as noted, previously
ran the whole marble quarrying, processing and exporting enterprise.
His work constituted value-added to the business itself and therefore,
the new partnership similarly benefitted from the labors of Benjamin
Yu. It is worthy of note that the new partnership did not try to suggest
that there was any cause consisting of some blameworthy act or
omission on the part of Mr. Yu which compelled the new partnership to
terminate his services. Nonetheless, the new Jade Mountain did not

48
G.R. No. L-33172 October 18, 1979 May, 1961 apparently on the eve of the expiry of
the three (3) year period provided by the law for
the liquidation of corporations, the board of
ERNESTO CEASE, CECILIA CEASE, MARION CEASE, TERESA
liquidators of Tiaong Milling executed an
CEASE-LACEBAL and the F.L. CEASE PLANTATION CO., INC. as
assignment and conveyance of properties and
Trustee of properties of the defunct TIAONG MILLING &
trust agreement in favor of F.L. Cease Plantation
PLANTATION CO., petitioners,
Co. Inc. as trustee of the Tiaong Milling and
vs.
Plantation Co. so Chat upon motion of the plaintiffs
HONORABLE COURT OF APPEALS, (Special Seventh Division),
trial Judge ordered that this alleged trustee be also
HON. MANOLO L. MADDELA, Presiding Judge, Court of First
included as party defendant; now this being the
Instance of Quezon, BENJAMIN CEASE and FLORENCE
situation, it will be remembered that there were
CEASE, respondents.
thus two (2) proceedings pending in the Court of
First Instance of Quezon namely Civil Case No.
6326 and Special Proceeding No. 3893 but both of
these were assigned to the Honorable Respondent
Judge Manolo L. Maddela p. 43 and the case was
GUERRERO, J:
finally heard and submitted upon stipulation of
facts pp, 34-110, rollo; and trial Judge by decision
Appeal by certiorari from the decision of the Court of Appeals in CA- dated 27 December 1969 held for the plaintiffs
G.R. No. 45474, entitled "Ernesto Cease, et al. vs. Hon. Manolo L. Benjamin and Florence, the decision containing
Maddela, Judge of the Court of First Instance of Quezon, et al." 1 which the following dispositive part:
dismissed the petition for certiorari, mandamus, and prohibition
instituted by the petitioners against the respondent judge and the
VIEWED IN THE LIGHT OF
private respondents. ALL THE FOREGOING,
judgment is hereby rendered
The antecedents of the case, as found by the appellate court, are as in favor of plaintiffs and
follows: against the defendants
declaring that:
IT RESULTING: That the antecedents are not
difficult to understand; sometime in June 1908, 1) The assets or properties of
one Forrest L. Cease common predecessor in the defunct Tiaong Milling and
interest of the parties together with five (5) other Plantation Company now
American citizens organized the Tiaong Milling appearing under the name of
and Plantation Company and in the course of its F.L. Cease Plantation
corporate existence the company acquired various Company as Trustee, is the
properties but at the same time all the other estate also of the deceased
original incorporators were bought out by Forrest Forrest L. Cease and ordered
L. Cease together with his children namely Ernest, divided, share and share
Cecilia, Teresita, Benjamin, Florence and one alike, among his six children
Bonifacia Tirante also considered a member of the the plaintiffs and the
family; the charter of the company lapsed in June defendants in accordance with
1958; but whether there were steps to liquidate it, Rule 69, Rules of Court;
the record is silent; on 13 August 1959, Forrest L.
Cease died and by extrajudicial partition of his 2) The Resolution to Sell
shares, among the children, this was disposed of dated October 12, 1959 and
on 19 October 1959; it was here where the trouble
the Transfer and Conveyance
among them came to arise because it would with Trust Agreement is
appear that Benjamin and Florence wanted an hereby set aside as improper
actual division while the other children wanted
and illegal for the purposes
reincorporation; and proceeding on that, these and effect that it was intended
other children Ernesto, Teresita and Cecilia and and, therefore, null and void;
aforementioned other stockholder Bonifacia
Tirante proceeded to incorporate themselves into
the F.L. Cease Plantation Company and registered 3) That F.L. Cease Plantation
it with the Securities and Exchange Commission Company is removed as
on 9 December, 1959; apparently in view of that, 'Trustee for interest against
Benjamin and Florence for their part initiated a the estate and essential to the
Special Proceeding No. 3893 of the Court of First protection of plaintiffs' rights
Instance of Tayabas for the settlement of the and is hereby ordered to
estate of Forest L. Cease on 21 April, 1960 and deliver and convey all the
one month afterwards on 19 May 1960 they filed properties and assets of the
Civil Case No. 6326 against Ernesto, Teresita and defunct Tiaong Milling now
Cecilia Cease together with Bonifacia Tirante under its name, custody and
asking that the Tiaong Milling and Plantation control to whomsoever be
Corporation be declared Identical to F.L. Cease appointed as Receiver -
and that its properties be divided among his disqualifying and of the parties
children as his intestate heirs; this Civil Case was herein - the latter to act
resisted by aforestated defendants and accordingly upon proper
notwithstanding efforts of the plaintiffs to have the assumption of office; and
properties placed under receivership, they were
not able to succeed because defendants filed a
4) Special Proceedings No.
bond to remain as they have remained in
3893 for administration is
possession; after that and already, during the
terminated and dismissed; the
pendency of Civil Case No. 6326 specifically on 21
49
instant case to proceed but on I. IN SANCTIONING THE WRONGFUL EXERCISE OF
issues of damages only and JURISDICTION BEYOND THE LIMITS OF AUTHORITY
for such action inherently CONFERRED BY LAW UPON THE LOWER COURT, WHEN IT
essential for partition. PROCEEDED TO HEAR, ADJUDGE AND ADJUDICATE -

SO ORDERED. (a) Special Proceedings No. 3893 for the


settlement of the Estate of Forrest L. Cease,
simultaneously and concurrently with -
Lucena City, December 27,
1969., pp. 122-a-123, rollo.
(b) Civil Case No. 6326, wherein the lower Court
ordered Partition under Rule 69, Rules of Court -
upon receipt of that, defendants there filled a
notice of appeal p. 129, rollo together with an
appeal bond and a record on appeal but the THE ISSUE OF LEGAL OWNERSHIP OF THE PROPERTIES
plaintiffs moved to dismiss the appeal on the COMMONLY INVOLVED IN BOTH ACTIONS HAVING BEEN RAISED
ground that the judgment was in fact interlocutory AT THE OUTSET BY THE TIAONG MILLING AND PLANTATION
and not appealable p. 168 rollo and this position of COMPANY, AS THE REGISTERED OWNER OF SUCH
defendants was sustained by trial Judge, His PROPERTIES UNDER ACT 496.
Honor ruling that
II. IN AFFIRMING - UNSUPPORTED BY ANY EVIDENCE
IN VIEW OF THE WHATSOEVER NOR CITATION OF ANY LAW TO JUSTIFY - THE
FOREGOING, the appeal UNWARRANTED CONCLUSION THAT SUBJECT PROPERTIES,
interposed by plaintiffs is FOUND BY THE LOWER COURT AND THE COURT OF APPEALS
hereby dismissed as AS ACTUALLY REGISTERED IN THE NAME OF PETITIONER
premature and the Record on CORPORATION AND/OR ITS PREDECESSOR IN INTEREST, THE
Appeal is necessarily TIAONG MILLING AND PLANTATION COMPANY, DURING ALL THE
disapproved as improper at 50 YEARS OF ITS CORPORATE EXISTENCE "ARE ALSO
this stage of the proceedings. PROPERTIES OF THE ESTATE OF FOREST L. CEASE."

SO ORDERED. III. IN AFFIRMING THE ARBITRARY CONCLUSION OF THE LOWER


COURT THAT ITS DECISION OF DECEMBER 27,1969 IS AN
"INTERLUCUTORY DECISION." IN DISMISSED NG THE PETITION
Lucena City, April 27, 1970.
FOR WRIT OF MANDAMUS, AND IN AFFIRMING THE MANIFESTLY
UNJUST JUDGMENT RENDERED WHICH CONTRADICTS THE
and so it was said defendants brought the matter FINDINGS OF ULTIMATE FACTS THEREIN CONTAINED.
first to the Supreme Court, on mandamus on 20
May, 1970 to compel the appeal and certiorari and
During the period that ensued after the filing in this Court of the
prohibition to annul the order of 27 April, 1970 on
respective briefs and the subsequent submission of the case for
the ground that the decision was "patently
decision, some incidents had transpired, the summary of which may be
erroneous" p. 16, rollo; but the Supreme Court
stated as follows:
remanded the case to this Court of Appeals by
resolution of 27 May 1970, p. 173, and this Court
of Appeals on 1 July 1970 p. 175 dismissed the 1. Separate from this present appeal, petitioners filed a petition for
petition so far as the mandamus was concerned certiorari and prohibition in this Court, docketed as G.R. No. L-35629
taking the view that the decision sought to be (Ernesto Cease, et al. vs. Hon. Manolo L. Maddela, et al.) which
appealed dated 27 December, 1969 was challenged the order of respondent judge dated September 27, 1972
interlocutory and not appealable but on motion for appointing his Branch Clerk of Court, Mr. Eleno M. Joyas, as receiver
reconsideration of petitioners and since there was of the properties subject of the appealed civil case, which order,
possible merit so far as its prayer for certiorari and petitioners saw as a virtual execution of the lower court's judgment (p.
prohibition was concerned, by resolution of the 92, rollo). In Our resolution of November 13, 1972, issued in G.R. No.
Court on 19 August, 1970, p. 232, the petition was L-35629, the petition was denied since respondent judge merely
permitted to go ahead in that capacity; and it is the appointed an auxilliary receiver for the preservation of the properties as
position of petitioners that the decision of 27 well as for the protection of the interests of all parties in Civil Case No.
December, 1969 as well as the order of 27 April, 6326; but at the same time, We expressed Our displeasure in the
1970 suffered of certain fatal defects, which appointment of the branch clerk of court or any other court personnel
respondents deny and on their part raise the for that matter as receiver. (p. 102, rollo).
preliminary point that this Court of Appeals has no
authority to give relief to petitioners because not
2. Meanwhile, sensing that the appointed receiver was making some
attempts to take possession of the properties, petitioners filed in this
in aid of its appellate jurisdiction, present appeal an urgent petition to restrain proceedings in the lower
court. We resolved the petition on January 29, 1975 by issuing a
corresponding temporary restraining order enjoining the court a quo
and that the questions presented cannot be raised
from implementing its decision of December 27, 1969, more
for the first time before this Court of Appeals;
particularly, the taking over by a receiver of the properties subject of
the litigation, and private respondents Benjamin and Florence Cease
Respondent Court of Appeals in its decision promulgated December 9, from proceeding or taking any action on the matter until further orders
1970 dismissed the petition with costs against petitioners, hence the from this Court (pp. 99-100, rollo). Private respondents filed a motion
present petition to this Court on the following assignment of errors: for reconsideration of Our resolution of January 29, 1975. After
weighing the arguments of the parties and taking note of Our resolution
in G.R. No. L-35629 which upheld the appointment of a receiver, We
THE COURT OF APPEALS ERRED - issued another resolution dated April 11, 1975 lifting effective
immediately Our previous temporary restraining order which enforced
the earlier resolution of January 29, 1975 (pp. 140-141, rollo).
50
3. On February 6, 1976, private respondents filed an urgent petition to Where the estate has no debts, recourse may be
restrain proceedings below in view of the precipitate replacement of the had to an administration proceeding only if the
court appointed receiver Mayor Francisco Escueta (vice Mr. Eleno M. heirs have good reasons for not resorting to an
Joyas) and the appointment of Mr. Guillermo Lagrosa on the eve of action for partition. Where partition is possible,
respondent Judge Maddela's retirement (p. 166, rollo). The urgent either in or out of court, the estate should not be
petition was denied in Our resolution of February 18, 1976 (p. 176, burdened with an administration proceeding
rollo). without good and compelling reasons. (Intestate
Estate of Mercado vs. Magtibay, 96 Phil. 383)
4. Several attempts at a compromise agreement failed to materialize. A
Tentative Compromise Agreement dated July 30, 1975 was presented In the records of this case, We find no indication of any indebtedness
to the Court on August 6, 1976 for the signature of the parties, but of the estate. No creditor has come up to charge the estate within the
respondents "unceremoniously" repudiated the same by leaving the two-year period after the death of Forrest L. Cease, hence, the
courtroom without the permission of the court (Court of First Instance presumption under Section 1, Rule 74 that the estate is free from
of Quezon, Branch 11) as a result of which respondents and their creditors must apply. Neither has the status of the parties as legal
counsel were cited for contempt (p. 195, 197, rollo) that respondents' heirs, much less that of respondents, been raised as an issue.
reason for the repudiation appears to be petitioners' failure to render Besides, extant in the records is the stipulation of the parties to submit
an audited account of their administration covering the period from May the pleadings and contents of the administration proceedings for the
31, 1961 up to January 29, 1974, plus the inclusion of a provision on cognizance of the trial judge in adjudicating the civil case for partition
waiver and relinquishment by respondents of whatever rights that may (Respondents' Brief, p, 20, rollo). As respondents observe, the parties
have accrued to their favor by virtue of the lower court's decision and in both cases are the same, so are the properties involved; that actual
the affirmative decision of the appellate court. division is the primary objective in both actions; the theory and defense
of the respective parties are likewise common; and that both cases
have been assigned to the same respondent judge. We feel that the
We go now to the alleged errors committed by the respondent Court of
unifying effect of the foregoing circumstances invites the wholesome
Appeals.
exception to the structures of procedural rule, thus allowing, instead,
room for judicial flexibility. Respondent judge's dismissal of the
As can be gleaned from petitioners' brief and the petition itself, two administration proceedings then, is a judicious move, appreciable in
contentions underlie the first assigned error. First, petitioners argue today's need for effective and speedy administration of justice. There
that there was an irregular and arbitrarte termination and dismissal of being ample reason to support the dismissal of the special proceedings
the special proceedings for judicial administration simultaneously in this appealed case, We cannot see in the records any compelling
ordered in the lower court . s decision in Civil Case No. 6326 reason why it may not be dismissed just the same even if considered in
adjudicating the partition of the estate, without categorically, reasoning a separate action. This is inevitably certain specially when the subject
the opposition to the petition for administration Second, that the issue property has already been found appropriate for partition, thus
of ownership had been raised in the lower court when Tiaong Milling reducing the petition for administration to a mere unnecessary
asserted title over the properties registered in its corporate name solicitation.
adverse to Forrest L. Cease or his estate, and that the said issue was
erroneously disposed of by the trial court in the partition proceedings
The second point raised by petitioners in their first assigned error is
when it concluded that the assets or properties of the defunct company
equally untenable. In effect, petitioners argue that the action for
is also the estate of the deceased proprietor.
partition should not have prospered in view of the repudiation of the co-
ownership by Tiaong Milling and Plantation Company when, as early in
The propriety of the dismissal and termination of the special the trial court, it already asserted ownership and corporate title over the
proceedings for judicial administration must be affirmed in spite of its properties adverse to the right of ownership of Forrest L. Cease or his
rendition in another related case in view of the established estate. We are not unmindful of the doctrine relied upon by petitioners
jurisprudence which favors partition when judicial administration in Rodriguez vs. Ravilan, 17 Phil. 63 wherein this Court held that in an
become, unnecessary. As observed by the Court of Appeals, the action for partition, it is assumed that the parties by whom it is
dismissal at first glance is wrong, for the reason that what was actually prosecuted are all co-owners or co-proprietors of the property to be
heard was Civil Case No. 6326. The technical consistency, however, it divided, and that the question of common ownership is not to be
is far less importance than the reason behind the doctrinal rule against argued, not the fact as to whether the intended parties are or are not
placing an estate under administration. Judicial rulings consistently the owners of the property in question, but only as to how and in what
hold the view that where partition is possible, either judicial or manner and proportion the said property of common ownership shall
extrajudicial, the estate should not be burdened with an administration be distributed among the interested parties by order of the Court.
proceeding without good and compelling reason. When the estate has Consistent with this dictum, it has been field that if any party to a suit
no creditors or pending obligations to be paid, the beneficiaries in for partition denies the pro-indiviso character of the estate whose
interest are not bound to submit the property to judicial administration partition is sought, and claims instead, exclusive title thereto the action
which is always long and costly, or to apply for the appointment of an becomes one for recovery of property cognizable in the courts of
administrator by the court, especially when judicial administration is ordinary jurisdiction. 2
unnecessary and superfluous. Thus -
Petitioners' argument has only theoretical persuasion, to say the least,
When a person dies without leaving pending rather apparent than real. It must be remembered that when Tiaong
obligations to be paid, his heirs, whether of age or Milling adduced its defense and raised the issue of ownership, its
not, are bound to submit the property to a judicial corporate existence already terminated through the expiration of its
administration, which is always long and costly, or charter. It is clear in Section 77 of Act No. 1459 (Corporation Law) that
to apply for the appointment of an administrator by upon the expiration of the charter period, the corporation ceases to
the court. It has been uniformly held that in such exist and is dissolved ipso facto except for purposes connected with
case the judicial administration and the the winding up and liquidation. The provision allows a three year,
appointment of an administrator are superfluous period from expiration of the charter within which the entity gradually
and unnecessary proceedings (Ilustre vs. Alaras settles and closes its affairs, disposes and convey its property and to
Frondosa, 17 Phil., 321; Malahacan vs. Ignacio, 19 divide its capital stock, but not for the purpose of continuing the
Phil, 434; Bondad vs. Bondad, 34 Phil., 232; business for which it was established. At this terminal stage of its
Baldemor vs. Malangyaon, 34 Phil., 367; Fule vs. existence, Tiaong Milling may no longer persist to maintain adverse
Fule, 46 Phil., 317). Syllabus, Intestate estate of title and ownership of the corporate assets as against the prospective
the deceased Luz Garcia. Pablo G. Utulo vs. distributees when at this time it merely holds the property in trust, its
Leona Pasion Viuda de Garcia, 66 Phil. 302. assertion of ownership is not only a legal contradiction, but more so, to

51
allow it to maintain adverse interest would certainly thwart the very We do not agree. In reposing ownership to the estate of Forrest L.
purpose of liquidation and the final distribute loll of the assets to the Cease, the trial court indeed found strong support, one that is based on
proper, parties. a well-entrenched principle of law. In sustaining respondents' theory of
"merger of Forrest L. Cease and The Tiaong Milling as one
personality", or that "the company is only the business conduit and
We agree with the Court of Appeals in its reasoning that substance is
alter ego of the deceased Forrest L. Cease and the registered
more important than form when it sustained the dismissal of Special
properties of Tiaong Milling are actually properties of Forrest L. Cease
Proceedings No. 3893, thus -
and should be divided equally, share and share alike among his six
children, ... ", the trial court did aptly apply the familiar exception to the
a) As to the dismissal of Special Proceedings No. general rule by disregarding the legal fiction of distinct and separate
3893, of course, at first glance, this was wrong, for corporate personality and regarding the corporation and the individual
the reason that the case trial had been heard was member one and the same. In shredding the fictitious corporate veil,
Civil Case No. 6326; but what should not be the trial judge narrated the undisputed factual premise, thus:
overlooked either is Chat respondent Judge was
the same Judge that had before him in his own
While the records showed that originally its
sala, said Special Proceedings No. 3893, p. 43
incorporators were aliens, friends or third-parties in
rollo, and the parties to the present Civil Case No.
relation of one to another, in the course of its
6326 had themselves asked respondent Judge to
existence, it developed into a close family
take judicial notice of the same and its contents
corporation. The Board of Directors and
page 34, rollo; it is not difficult to see that when
stockholders belong to one family the head of
respondent Judge in par. 4 of the dispositive part
which Forrest L. Cease always retained the
of his decision complained of, ordered that,
majority stocks and hence the control and
management of its affairs. In fact, during the
4) Special Proceedings No. reconstruction of its records in 1947 before the
3893 for administration is Security and Exchange Commission only 9
terminated and dismissed; the nominal shares out of 300 appears in the name of
instant case to proceed but on his 3 eldest children then and another person
issues of damages only and close to them. It is likewise noteworthy to observe
for such action inherently that as his children increase or perhaps become of
essential or partition. p. 123, age, he continued distributing his shares among
rollo, them adding Florence, Teresa and Marion until at
the time of his death only 190 were left to his
name. Definitely, only the members of his family
in truth and in fact, His Honor was issuing that
benefited from the Corporation.
order also within Civil Case No. 632 but in
connection with Special Proceedings No. 389:3:
for substance is more important Chan form, the The accounts of the corporation and therefore its
contending par ties in both proceedings being operation, as well as that of the family appears to
exactly the same, but not only this, let it not be be indistinguishable and apparently joined
forgotten that when His Honor dismissed Special together. As admitted by the defendants
Proceedings No. 3893, that dismissal precisely (Manifestation of Compliance with Order of March
was a dismissal that petitioners herein had 7, 1963 [Exhibit "21"] the corporation 'never' had
themselves sought and solicited from respondent any account with any banking institution or if any
Judge as petitioners themselves are in their account was carried in a bank on its behalf, it was
present petition pp. 5-6, rollo; this Court must find in the name of Mr. Forrest L. Cease. In brief, the
difficulty in reconciling petitioners' attack with the operation of the Corporation is merged with those
fact that it was they themselves that had insisted of the majority stockholders, the latter using the
on that dismissal; on the principle that not he who former as his instrumentality and for the exclusive
is favored but he who is hurt by a judicial order is benefits of all his family. From the foregoing
he only who should be heard to complain and indication, therefore, there is truth in plaintiff's
especially since extraordinary legal remedies are allegation that the corporation is only a business
remedies in extermies granted to parties ' who conduit of his father and an extension of his
have been the victims not merely of errors but of personality, they are one and the same thing.
grave wrongs, and it cannot be seen how one who Thus, the assets of the corporation are also the
got what he had asked could be heard to claim estate of Forrest L. Cease, the father of the parties
that he had been the victim of a wrong, petitioners herein who are all legitimate children of full blood.
should not now complain of an order they had
themselves asked in order to attack such an order
A rich store of jurisprudence has established the rule known as the
afterwards; if at all, perhaps, third parties,
doctrine of disregarding or piercing the veil of corporate fiction.
creditors, the Bureau of Internal Revenue, might
Generally, a corporation is invested by law with a personality separate
have been prejudiced, and could have had the
and distinct from that of the persons composing it as well as from that
personality to attack that dismissal of Special
of any other legal entity to which it may be related. By virtue of this
Proceedings No. 3893, but not petitioners herein,
attribute, a corporation may not, generally, be made to answer for acts
and it is not now for this Court of Appeals to
or liabilities of its stockholders or those of the legal entities to which it
protect said third persons who have not come to
may be connected, and vice versa. This separate and distinct
the Court below or sought to intervene herein;
personality is, however, merely a fiction created by law for convenience
and to promote the ends of justice (Laguna Transportation Company
On the second assigned error, petitioners argue that no evidence has vs. Social Security System, L-14606, April 28, 1960; La Campana
been found to support the conclusion that the registered properties of Coffee Factory, Inc. vs. Kaisahan ng mga Manggagawa sa La
Tiaong Milling are also properties of the estate of Forrest L. Cease; Campana, L-5677, May 25, 1953). For this reason, it may not be used
that on the contrary, said properties are registered under Act No. 496 or invoked for ends subversive of the policy and purpose behind its
in the name of Tiaong Milling as lawful owner and possessor for the creation (Emiliano Cano Enterprises, Inc. vs. CIR, L-20502, Feb. 26,
last 50 years of its corporate existence. 1965) or which could not have been intended by law to which it owes
its being McConnel vs. Court of Appeals, L- 10510, March 17, 1961, 1

52
SCRA 722). This is particularly true where the fiction is used to defeat reversed in the Fuentebella case which, in our
public convenience, justify wrong, protect fraud, defend crime (Yutivo opinion, expresses the correct view, considering
Sons Hardware Company vs. Court of Tax Appeals, L-13203, Jan. 28, that a decision or order directing partition is not
1961, 1 SCRA 160), confuse legitimate legal or judicial issues (R. F. final because it leaves something more to be done
Sugay & Co. vs. Reyes, L-20451, Dec. 28, 1964), perpetrate deception in the trial court for the complete disposition of the
or otherwise circumvent the law (Gregorio Araneta, Inc. vs. reason de case, namely, the appointment of commissioners,
Paterno, L-2886, Aug. 22, 1952, 49 O.G. 721). This is likewise true the proceedings to be had before them, the
where the corporate entity is being used as an alter ego, adjunct, or submission of their report which, according to law,
business conduit for the sole benefit of the stockholders or of another must be set for hearing. In fact, it is only after said
corporate entity (McConnel vs. Court of Appeals, supra; Commissioner hearing that the court may render a final judgment
of Internal Revenue vs. Norton Harrison Co., L-7618, Aug. 31, 1964). finally disposing of the action (Rule 71, section 7,
Rules of Court). (1 SCRA at page 1193).
In any of these cases, the notion of corporate entity will be pierced or
disregarded, and the corporation will be treated merely as an It should be noted, however, that the said ruling in Zaldarriaga as
association of persons or, where there are two corporations, they will based on Fuentebella vs. Carrascoso, XIV Lawyers Journal 305 (May
be merged as one, the one being merely regarded as part or the 27, 1942), has been expressly abandoned by the Court in Miranda vs.
instrumentality of the otter (Koppel [Phil.] Inc. vs. Yatco, 77 Phil. 496, Court of Appeals, 71 SCRA 295; 331-333 (June 18, 1976) wherein Mr.
Yutivo Sons Hardware Company vs. Court of Tax Appeals, supra). Justice Teehankee, speaking for the Court, laid down the following
doctrine:
So must the case at bar add to this jurisprudence. An indubitable
deduction from the findings of the trial court cannot but lead to the The Court, however, deems it proper for the
conclusion that the business of the corporation is largely, if not wholly, guidance of the bench and bar to now declare as
the personal venture of Forrest L. Cease. There is not even a shadow is clearly indicated from the compelling reasons
of a showing that his children were subscribers or purchasers of the and considerations hereinabove stated:
stocks they own. Their participation as nominal shareholders emanated
solely from Forrest L. Cease's gratuitous dole out of his own shares to
- that the Court considers the better rule to be that
the benefit of his children and ultimately his family.
stated in H. E. Heacock Co. vs. American Trading
Co., to wit, that where the primary purpose of a
Were we sustain the theory of petitioners that the trial court acted in case is to ascertain and determine who between
excess of jurisdiction or abuse of discretion amounting to lack of plaintiff and defendant is the true owner and
jurisdiction in deciding Civil Case No. 6326 as a case for partition when entitled to the exclusive use of the disputed
the defendant therein, Tiaong Milling and Plantation Company, Inc. as property, "the judgment . . . rendered by the lower
registered owner asserted ownership of the assets and properties court [is] a judgment on the merits as to those
involved in the litigation, which theory must necessarily be based on questions, and [that] the order of the court for
the assumption that said assets and properties of Tiaong Milling and an accounting was based upon, and is incidental
Plantation Company, Inc. now appearing under the name of F. L. to the judgment on the merits. That is to say, that
Cease Plantation Company as Trustee are distinct and separate from the judgment . . . [is] a final judgment ... that in this
the estate of Forrest L. Cease to which petitioners and respondents as kind of a case an accounting is a mere incident to
legal heirs of said Forrest L. Cease are equally entitled share and the judgment; that an appeal lies from the rendition
share alike, then that legal fiction of separate corporate personality of the judgment as rendered ... "(as is widely held
shall have been used to delay and ultimately deprive and defraud the by a great number of judges and members of the
respondents of their successional rights to the estate of their deceased bar, as shown by the cases so decided and filed
father. For Tiaong Milling and Plantation Company shall have been and still pending with the Court) for the
able to extend its corporate existence beyond the period of its charter fundamental reasons therein stated that "this is
which lapsed in June, 1958 under the guise and cover of F. L, Cease more in harmony with the administration of
Plantation Company, Inc. as Trustee which would be against the law, justice and the spirit and intent of the [Rules]. If on
and as Trustee shall have been able to use the assets and properties appeal the judgment of the lower court is affirmed,
for the benefit of the petitioners, to the great prejudice and it would not in the least work an injustice to any of
defraudation. of private respondents. Hence, it becomes necessary the legal rights of [appellee]. On the other hand, if
and imperative to pierce that corporate veil. for any reason this court should reverse the
judgment of the lower court, the accounting would
be a waste of time and money, and might work a
Under the third assigned error, petitioners claim that the decision of the
material injury to the [appellant]; and
lower court in the partition case is not interlocutory but rather final for it
consists of final and determinative dispositions of the contentions of the
parties. We find no merit in petitioners' stand. - that accordingly, the contrary ruling
in Fuentebella vs. Carrascoso which expressly
reversed the Heacock case and a line of similar
Under the 1961 pronouncement and ruling of the Supreme Court
decisions and ruled that such a decision for
in Vda. de Zaldarriaga vs. Enriquez, 1 SCRA 1188 (and the sequel
recovery of property with accounting "is not final
case of Vda. de Zaldarriaga vs. Zaldarriaga, 2 SCRA 356), the lower
but merely interlocutory and therefore not
court's dismissal of petitioners' proposed appeal from its December 27,
appealable" and subsequent cases adhering to the
1969 judgment as affirmed by the Court of Appeals on the ground of
same must be now in turn abandoned and set
prematurity in that the judgment was not final but interlocutory was in
aside.
order. As was said in said case:

Fuentebella adopted instead the opposite line of


It is true that in Africa vs. Africa, 42 Phil. 934 and
conflicting decisions mostly in partition
other cases it was held - contrary to the rule laid
proceedings and exemplified by Ron vs. Mojica 8
down in Ron vs. Mojica, 8 Phil. 328; Rodriguez vs.
Phil. 928 (under the old Code of Civil Procedure)
Ravilan, 17 Phil. 63 - that in a partition case where
that an order for partition of real property is not
defendant relies on the defense of exclusive
final and appealable until after the actual
ownership, the action becomes one for title and
partition of the property as reported by the court
the decision or order directing partition is final, but
appointed commissioners and approved by the
the ruling to this effect has been expressly
53
court in its judgment accepting the report. lt must company's assertion of ownership of the properties is a legal
be especially noted that such rule governing contradiction and would but thwart the liquidation and final distribution
partitions is now so expressly provided and spelled and partition of the properties among the parties hereof as children of
out in Rule 69 of the Rules of Court, with special their deceased father Forrest L. Cease. There is therefore no further
reference to Sections 1, 2, 3, 6, 7 and 11, to wit, hindrance to effect the partition of the properties among the parties in
that there must first be a preliminar, order for implementation of the appealed judgment.
partition of the real estate (section 2) and where
the parties-co-owners cannot agree, the court
One last consideration. Parties are brothers and sisters, legal heirs of
appointed commissioners make a plan of actual
their deceased father, Forrest L. Cease. By all rights in law and
partition which must first be passed upon and
jurisprudence, each is entitled to share and share alike in the estate,
accepted by the trial court and embodied in a
which the trial court correctly ordained and sustained by the appellate
judgment to be rendered by it (sections 6 and 11).
court. Almost 20 years have lapsed since the filing of Special
In partition cases, it must be further borne in mind
Proceedings No. 3893 for the administration of the Estate of Forrest L.
that Rule 69, section 1 refers to "a person having
Cease and Civil Case No. 6326 for liquidation and partition of the
the right to compel the partition of real estate," so
assets of the defunct Tiaong Milling and Plantation Co., Inc. A
that the general rule of partition that an appeal will
succession of receivers were appointed by the court to take, keep in
not lie until the partition or distribution proceedings
possession, preserve and manage properties of the corporation which
are terminated will not apply where appellant
at one time showed an income of P386,152.90 and expenses of
claims exclusive ownership of the whole property
P308,405.01 for the period covering January 1, 1960 to August 31,
and denies the adverse party's right to any
1967 as per Summary of Operations of Commissioner for Finance
partition, as was the ruling in Villanueva vs.
appointed by the Court (Brief for Respondents, p. 38). In the meantime,
Capistrano and Africa vs .Africa, supra,
ejectment cases were filed by and against the heirs in connection with
Fuentebellas express rehearsal of these cases
the properties involved, aggravating the already strained relations of
must likewise be deemed now also abandoned in
the parties. A prudent and practical realization of these circumstances
view of the Court's expressed preference for the
ought and must constrain the parties to give each one his due in law
rationale of the Heacock case.
and with fairness and dispatch that their basic rights be enjoyed. And
by remanding this case to the court a quo for the actual partition of the
The Court's considered opinion is properties, the substantial rights of everyone of the heirs have not been
that imperative considerations of public policy and impaired, for in fact, they have been preserved and maintained.
of sound practice in the courts and adherence to
the constitutional mandate of simplified, just,
WHEREFORE, IN VIEW OF THE FOREGOING, the judgment
speedy and inexpensive determination of every
appealed from is hereby AFFIRMED with costs against the petitioners.
action call for considering such judgments for
recovery of property with accounting as
final judgments which are duly appealable (and SO ORDERED.
would therefore become final and executory if not
appealed within the reglementary period) with
the accounting as a mere incident of the judgment
to be rendered during the course of the appeal as
provided in Rule 39, section 4 or to be
implemented at the execution stage upon final
affirmance on appeal of the judgment (as in Court
of Industrial Relations unfair labor practice cases
ordering the reinstatement of the worker with
accounting, computation and payment of his
backwages less earnings elsewhere during his
layoff) and that the only reason given in
Fuentebelia for the contrary ruling, viz, "the
general harm that would follow from throwing the
door open to multiplicity of appeals in a single
case" of lesser import and consequence.
(Emphasis copied).

The miranda ruling has since then been applied as the new rule by a
unanimous Court in Valdez vs. Bagasao, 82 SCRA 22 (March 8, 1978).

If there were a valid genuine claim of Exclusive ownership of the


inherited properties on the part of petitioners to respondents' action for
partition, then under the Miranda ruling, petitioners would be sustained,
for as expressly held therein " the general rule of partition that an
appeal will not lie until the partition or distribution proceedings are
terminated will not apply where appellant claims exclusive ownership of
the whole property and denies the adverse party's right to any
partition."

But this question has now been rendered moot and academic for the
very issue of exclusive ownership claimed by petitioners to deny and
defeat respondents' right to partition - which is the very core of their
rejected appeal - has been squarely resolved herein against them, as if
the appeal had been given due course. The Court has herein expressly
sustained the trial court's findings, as affirmed by the Court of Appeals,
that the assets or properties of the defunct company constitute the
estate of the deceased proprietor (supra at page 7) and the defunct
54
G.R. No. L-69259 January 26, 1988 ACCORDINGLY, the judgment is hereby rendered
declaring the valid existence of the plaintiffs
preferential right to acquire the subject property
DELPHER TRADES CORPORATION, and DELPHIN
(right of first refusal) and ordering the defendants
PACHECO, petitioners,
and all persons deriving rights therefrom to convey
vs.
the said property to plaintiff who may offer to
INTERMEDIATE APPELLATE COURT and HYDRO PIPES
acquire the same at the rate of P14.00 per square
PHILIPPINES, INC., respondents.
meter, more or less, for Lot 1095 whose area is
27,169 square meters only. Without
pronouncement as to attorney's fees and costs.
(Appendix I; Rec., pp. 246- 247). (Appellant's Brief,
pp. 1-2; p. 134, Rollo)
GUTIERREZ, JR., J.:

The lower court's decision was affirmed on appeal by the Intermediate


The petitioners question the decision of the Intermediate Appellate Appellate Court.
Court which sustained the private respondent's contention that the
deed of exchange whereby Delfin Pacheco and Pelagia Pacheco
conveyed a parcel of land to Delpher Trades Corporation in exchange The defendants-appellants, now the petitioners, filed a petition for
for 2,500 shares of stock was actually a deed of sale which violated a certiorari to review the appellate court's decision.
right of first refusal under a lease contract.
We initially denied the petition but upon motion for reconsideration, we
Briefly, the facts of the case are summarized as follows: set aside the resolution denying the petition and gave it due course.

In 1974, Delfin Pacheco and his sister, Pelagia The petitioners allege that:
Pacheco, were the owners of 27,169 square
meters of real estate Identified as Lot. No. 1095,
The denial of the petition will work great injustice
Malinta Estate, in the Municipality of Polo (now
to the petitioners, in that:
Valenzuela), Province of Bulacan (now Metro
Manila) which is covered by Transfer Certificate of
Title No. T-4240 of the Bulacan land registry. 1. Respondent Hydro Pipes Philippines, Inc,
("private respondent") will acquire from petitioners
a parcel of industrial land consisting of 27,169
On April 3, 1974, the said co-owners leased to
square meters or 2.7 hectares (located right after
Construction Components International Inc. the
the Valenzuela, Bulacan exit of the toll
same property and providing that during the
expressway) for only P14/sq. meter, or a total
existence or after the term of this lease the lessor
of P380,366, although the prevailing value thereof
should he decide to sell the property leased shall
is approximately P300/sq. meter or P8.1 Million;
first offer the same to the lessee and the letter has
the priority to buy under similar conditions
(Exhibits A to A-5) 2. Private respondent is allowed to exercise its
right of first refusal even if there is no "sale" or
transfer of actual ownership interests by
On August 3, 1974, lessee Construction
petitioners to third parties; and
Components International, Inc. assigned its rights
and obligations under the contract of lease in favor
of Hydro Pipes Philippines, Inc. with the signed 3. Assuming arguendo that there has been a
conformity and consent of lessors Delfin Pacheco transfer of actual ownership interests, private
and Pelagia Pacheco (Exhs. B to B-6 inclusive) respondent will acquire the land not under "similar
conditions" by which it was transferred to petitioner
Delpher Trades Corporation, as provided in the
The contract of lease, as well as the assignment of
same contractual provision invoked by private
lease were annotated at he back of the title, as per
respondent. (pp. 251-252, Rollo)
stipulation of the parties (Exhs. A to D-3 inclusive)

The resolution of the case hinges on whether or not the "Deed of


On January 3, 1976, a deed of exchange was
Exchange" of the properties executed by the Pachecos on the one
executed between lessors Delfin and Pelagia
hand and the Delpher Trades Corporation on the other was meant to
Pacheco and defendant Delpher Trades
be a contract of sale which, in effect, prejudiced the private
Corporation whereby the former conveyed to the
respondent's right of first refusal over the leased property included in
latter the leased property (TCT No.T-4240)
the "deed of exchange."
together with another parcel of land also located in
Malinta Estate, Valenzuela, Metro Manila (TCT
No. 4273) for 2,500 shares of stock of defendant Eduardo Neria, a certified public accountant and son-in-law of the late
corporation with a total value of P1,500,000.00 Pelagia Pacheco testified that Delpher Trades Corporation is a family
(Exhs. C to C-5, inclusive) (pp. 44-45, Rollo) corporation; that the corporation was organized by the children of the
two spouses (spouses Pelagia Pacheco and Benjamin Hernandez and
spouses Delfin Pacheco and Pilar Angeles) who owned in common the
On the ground that it was not given the first option to buy the leased
parcel of land leased to Hydro Pipes Philippines in order to perpetuate
property pursuant to the proviso in the lease agreement, respondent
their control over the property through the corporation and to avoid
Hydro Pipes Philippines, Inc., filed an amended complaint for
taxes; that in order to accomplish this end, two pieces of real estate,
reconveyance of Lot. No. 1095 in its favor under conditions similar to
including Lot No. 1095 which had been leased to Hydro Pipes
those whereby Delpher Trades Corporation acquired the property from
Philippines, were transferred to the corporation; that the leased
Pelagia Pacheco and Delphin Pacheco.
property was transferred to the corporation by virtue of a deed of
exchange of property; that in exchange for these properties, Pelagia
After trial, the Court of First Instance of Bulacan ruled in favor of the and Delfin acquired 2,500 unissued no par value shares of stock which
plaintiff. The dispositive portion of the decision reads: are equivalent to a 55% majority in the corporation because the other
5
owners only owned 2,000 shares; and that at the time of incorporation, to the extent of 100/1,000 or 1/10. Thus, by
he knew all about the contract of lease of Lot. No. 1095 to Hydro Pipes removing the par value of shares, the attention of
Philippines. In the petitioners' motion for reconsideration, they refer to persons interested in the financial condition of a
this scheme as "estate planning." (p. 252, Rollo) corporation is focused upon the value of assets
and the amount of its debts. (Agbayani,
Commentaries and Jurisprudence on the
Under this factual backdrop, the petitioners contend that there was
Commercial Laws of the Philippines, Vol. III, 1980
actually no transfer of ownership of the subject parcel of land since the
Edition, p. 107).
Pachecos remained in control of the property. Thus, the petitioners
allege: "Considering that the beneficial ownership and control of
petitioner corporation remained in the hands of the original co-owners, Moreover, there was no attempt to state the true or current market
there was no transfer of actual ownership interests over the land when value of the real estate. Land valued at P300.00 a square meter was
the same was transferred to petitioner corporation in exchange for the turned over to the family's corporation for only P14.00 a square meter.
latter's shares of stock. The transfer of ownership, if anything, was
merely in form but not in substance. In reality, petitioner corporation is
It is to be stressed that by their ownership of the 2,500 no par shares of
a mere alter ego or conduit of the Pacheco co-owners; hence the
stock, the Pachecos have control of the corporation. Their equity
corporation and the co-owners should be deemed to be the same,
capital is 55% as against 45% of the other stockholders, who also
there being in substance and in effect an Identity of interest." (p. 254,
belong to the same family group.
Rollo)

In effect, the Delpher Trades Corporation is a business conduit of the


The petitioners maintain that the Pachecos did not sell the property.
Pachecos. What they really did was to invest their properties and
They argue that there was no sale and that they exchanged the land
change the nature of their ownership from unincorporated to
for shares of stocks in their own corporation. "Hence, such transfer is
incorporated form by organizing Delpher Trades Corporation to take
not within the letter, or even spirit of the contract. There is a sale when
control of their properties and at the same time save on inheritance
ownership is transferred for a price certain in money or its equivalent
taxes.
(Art. 1468, Civil Code) while there is a barter or exchange when one
thing is given in consideration of another thing (Art. 1638, Civil Code)."
(pp. 254-255, Rollo) As explained by Eduardo Neria:

On the other hand, the private respondent argues that Delpher Trades xxx xxx xxx
Corporation is a corporate entity separate and distinct from the
Pachecos. Thus, it contends that it cannot be said that Delpher Trades
Corporation is the Pacheco's same alter ego or conduit; that petitioner ATTY. LINSANGAN:
Delfin Pacheco, having treated Delpher Trades Corporation as such a
separate and distinct corporate entity, is not a party who may allege Q Mr. Neria, from the point of
that this separate corporate existence should be disregarded. It view of taxation, is there any
maintains that there was actual transfer of ownership interests over the benefit to the spouses
leased property when the same was transferred to Delpher Trades Hernandez and Pacheco in
Corporation in exchange for the latter's shares of stock. connection with their
execution of a deed of
exchange on the properties
We rule for the petitioners.
for no par value shares of the
defendant corporation?
After incorporation, one becomes a stockholder of a corporation by
subscription or by purchasing stock directly from the corporation or
from individual owners thereof (Salmon, Dexter & Co. v. Unson, 47 A Yes, sir.
Phil, 649, citing Bole v. Fulton [1912], 233 Pa., 609). In the case at bar,
in exchange for their properties, the Pachecos acquired 2,500 original COURT:
unissued no par value shares of stocks of the Delpher Trades
Corporation. Consequently, the Pachecos became stockholders of the
corporation by subscription "The essence of the stock subscription is Q What do you mean by
an agreement to take and pay for original unissued shares of a "point of view"?
corporation, formed or to be formed." (Rohrlich 243, cited in Agbayani,
Commentaries and Jurisprudence on the Commercial Laws of the A To take advantage for both
Philippines, Vol. III, 1980 Edition, p. 430) It is significant that the spouses and corporation in
Pachecos took no par value shares in exchange for their properties. entering in the deed of
exchange.
A no-par value share does not purport to represent
any stated proportionate interest in the capital ATTY. LINSANGAN:
stock measured by value, but only an aliquot part
of the whole number of such shares of the issuing
corporation. The holder of no-par shares may see Q (What do you mean by
from the certificate itself that he is only an aliquot "point of view"?) What are
sharer in the assets of the corporation. But this these benefits to the spouses
character of proportionate interest is not hidden of this deed of exchange?
beneath a false appearance of a given sum in
money, as in the case of par value shares. The A Continuous control of the
capital stock of a corporation issuing only no-par property, tax exemption
value shares is not set forth by a stated amount of benefits, and other inherent
money, but instead is expressed to be divided into benefits in a corporation.
a stated number of shares, such as, 1,000 shares.
This indicates that a shareholder of 100 such
shares is an aliquot sharer in the assets of the Q What are these advantages
corporation, no matter what value they may have, to the said spouses from the

56
point of view of taxation in the spouse the property will
entering in the deed of be tied up in succession
exchange? proceedings and the
consequential payments of
estate and inheritance taxes
A Having fulfilled the
when an owner dies.
conditions in the income tax
law, providing for tax free
exchange of property, they Q Now what advantage is this
were able to execute the deed continuity in relation to
of exchange free from income ownership by a particular
tax and acquire a corporation. person of certain properties in
respect to taxation?
Q What provision in the
income tax law are you A The property is not
referring to? subjected to taxes on
succession as the corporation
does not die.
A I refer to Section 35 of the
National Internal Revenue
Code under par. C-sub-par. Q So the benefit you are
(2) Exceptions regarding the talking about are inheritance
provision which I quote: "No taxes?
gain or loss shall also be
recognized if a person
A Yes, sir. (pp. 3-5, tsn.,
exchanges his property for
December 15, 1981)
stock in a corporation of which
as a result of such exchange
said person alone or together The records do not point to anything wrong or objectionable about this
with others not exceeding four "estate planning" scheme resorted to by the Pachecos. "The legal right
persons gains control of said of a taxpayer to decrease the amount of what otherwise could be his
corporation." taxes or altogether avoid them, by means which the law permits,
cannot be doubted." (Liddell & Co., Inc. v. The collector of Internal
Revenue, 2 SCRA 632 citing Gregory v. Helvering, 293 U.S. 465, 7 L.
Q Did you explain to the
ed. 596).
spouses this benefit at the
time you executed the deed of
exchange? The "Deed of Exchange" of property between the Pachecos and
Delpher Trades Corporation cannot be considered a contract of sale.
There was no transfer of actual ownership interests by the Pachecos to
A Yes, sir
a third party. The Pacheco family merely changed their ownership from
one form to another. The ownership remained in the same hands.
Q You also, testified during Hence, the private respondent has no basis for its claim of a light of
the last hearing that the first refusal under the lease contract.
decision to have no par value
share in the defendant
WHEREFORE, the instant petition is hereby GRANTED, The
corporation was for the
questioned decision and resolution of the then Intermediate Appellate
purpose of flexibility. Can you
Court are REVERSED and SET ASIDE. The amended complaint in
explain flexibility in connection
Civil Case No. 885-V-79 of the then Court of First Instance of Bulacan
with the ownership of the
is DISMISSED. No costs.
property in question?

SO ORDERED.
A There is flexibility in using
no par value shares as the
value is determined by the
board of directors in
increasing capitalization. The
board can fix the value of the
shares equivalent to the
capital requirements of the
corporation.

Q Now also from the point of


taxation, is there any flexibility
in the holding by the
corporation of the property in
question?

A Yes, since a corporation


does not die it can continue to
hold on to the property
indefinitely for a period of at
least 50 years. On the other
hand, if the property is held by

57
173 F. Supp. 915 (1959) matters are not controlling and to sustain its position emphasizes the
W. H. GARRETT following facts:
v.
SOUTHERN RAILWAY COMPANY.
That although Lenoir sells the majority of its products to Southern or its
Civ. A. No. 3465.
affiliates, it does not sell to them exclusively and in the fifteen-year
period prior to the suit sold twelve percent to other customers; that
United States District Court E. D. Tennessee, N. D. Lenoir maintains its offices and business in Lenoir City, Tennessee,
May 8, 1959. although it concedes that the corporate and accounting offices are in
Washington in a building owned by Southern; that the management of
Lenoir is vested in a manager, Henry Marius, who has held the position
*916 Hodges & Doughty, Knoxville, Tenn., for plaintiff.
continuously since July 1, 1945, is paid by Lenoir, and although he
holds and votes the proxy of Southern at the annual stockholders
Key & Lee, Knoxville, Tenn., for defendant. meeting, he has no other connection with Southern; that never has any
individual served at the same time as a Director of Lenoir and
Southern; that the work of Lenoir is a specialty and in the last 20 years
ROBERT L. TAYLOR, District Judge. no official or supervising employee of Southern has by training, or
experience, been in a position to direct or supervise the operation of
The plaintiff in this case was employed as a wheel moulder by Lenoir such a business; that Southern has not purchased all its wheels, steel
Car Works, a Tennessee corporation. He claims injuries from silicosis and brass castings from Lenoir, but has bought substantial amounts
contracted from silica dust permeating the foundry. But the questions from others; that Southern has followed the custom in the trade as to
whether he had silicosis, and the amount of injury, are not reached in price quotations and supplying scrap material in its dealings with
this phase of the case. Lenoir, and all sales to Southern have been upon the basis of a bid
price or negotiated price; that in every case Southern obtains bids from
several manufacturers and purchases from the lowest bidder; that the
The sole question here is whether defendant, Southern Railway manager of Lenoir establishes the prices of all prospective purchases,
Company (hereinafter referred to as Southern), which acquired the including that of Southern; that the manager expects to make a profit
entire capital stock of Lenoir Car Works (referred to hereinafter as and is not informed by Southern of bids of its competitors even when
Lenoir) in 1904, the year of the latter's organization, so completely the prices are negotiated; that bids are revised sometimes, but never
dominated Lenoir that the latter was but an adjunct, or instrumentality, with knowledge of the bids of competitors; that all sales are the result
of Southern. If it did, then the complainant would, for the purpose of of the business judgment of the Manager and not the result of threats,
this case, be an employee of Southern and would be entitled to recover persuasion or intimidation by Southern; that at no time has any
under the Federal Employers' Liability Act, 45 U.S.C.A. § 51 et seq. individual occupied the same official position in both companies with
Because there is a real question that Lenoir is but an instrumentality of the exception of corporate and financial officials; that Lenior has since
Southern, the plaintiff in order to protect his rights, has also brought 1919 been a duly qualified employer under the Tennessee Workmen's
suit in the State courts against Lenoir for Workmen's Compensation. Compensation Law, T.C.A. § 50-901 et seq., and with the exception of
this and several similar suits has settled all claims made under and
The general rule is that stock ownership alone by one corporation of covered by the Tennessee Workmen's Compensation Law; that Lenoir
the stock of another does not thereby render the dominant corporation maintains a separate bank account and has never intermingled its
liable for the torts of the subsidiary unless the separate corporate funds with those of Southern; that the companies keep separate books
existence of the subsidiary is a mere sham, or unless the control of the and Lenoir pays its own taxes; that Southern issues no passes to the
subsidiary is such that it is but an instrumentality or adjunct of the manager or any other employee of Lenoir; that Lenoir owns no track or
dominant corporation. Sheridan v. Pan-American Refining Co., rolling stock, publishes no tariffs, files no reports with the Interstate
D.C.N.Y. 123 F. Supp. 81. The general rule is also stated in an Commerce Commission and uses no facilities or property jointly with
Annotation appearing in 50 A.L.R. 611. See also Kentucky Electric Southern; that claims of employees are investigated and settled by the
Power Co. v. Norton Coal Mining Co., 6 Cir., 93 F.2d 923, 926. Claim Department of the Southern Railway System, which likewise
Whether the subsidiary is an instrumentality of the owner corporation is defends claims litigation; that the general accounting of Lenoir is
a question to be determined from all the surrounding circumstances. handled *918 by the Accounting Department of the Southern Railway
Fletcher Cyclopedia Corporations, Sec. 43 (p. 157), Sec. 6222; Berkey System, but local accounting is handled by Lenoir employees at Lenoir
v. Third Ave. R. Co., 244 N.Y. 84, 155 N.E. 58, 50 A.L.R. 599 (opinion City; that salaries of employees of the System doing this general work
by Cardozo); Costan v. Manila Electric Co., 2 Cir., 24 F.2d 383, 384. are paid by Southern, but reimbursement of its fair proportionate part
Items to be considered in reaching such a determination are common thereof is made by Lenoir and other members of the System at monthly
directorships, common officers, common fiscal set up, etc. intervals, and that the share of Lenoir is $2,000 monthly; that Lenoir
makes separate collective bargaining agreements with its employees
and there is no interchange of seniority between operations of Lenoir
The plaintiff relies upon the following circumstances: and the railroad, or vice versa; that Lenoir has its own legal counsel in
Lenoir City, selected by its manager in addition to the use it makes of
That all directors and officers of Lenoir are employees of Southern and the Legal Department of the System.
live in Washington; that Southern owns all the *917 stock of Lenoir
except five qualifying shares; that between 1942-1957 Lenoir sold to Counsel have been commendably frank and candid in the pre-trial
Southern, or its affiliates, over 30 million dollars worth of its products conference, and in answering interrogatories and filing exhibits. The
while selling approximately four and a half million to outside factual data before the Court is not in conflict. The Court's
purchasers; that all profits of Lenoir went to Southern; that all claims of responsibility is to determine whether Lenoir is operated as a sham for
Lenoir employees for accidents are handled by the claim's office of Southern, or as the instrumentality, or as an adjunct of its operation.
Southern; that all litigation against Lenoir is handled by Southern's
attorneys; that general accounting of Lenoir is handled in Washington
by personnel of Southern; that the Railroad Retirement Board decided The facts set forth above outline with singular sharpness the relation
that employees of Lenoir were entitled to benefits under the Railroad between the two companies. The Court finds the existence of two
Retirement Act, 45 U.S.C.A. § 215 et seq., because of the relationship distinct operations. There is no evidence that Southern dictated the
between Lenoir and Southern; that Lenoir has the power of eminent management of Lenoir. In fact, the evidence indicates that Henry
domain; and that on May 13, 1904, the Board of Directors of Southern Marius was in full control of the operation. He established prices. He
authorized the purchase of the entire capital stock of Lenoir. handled all negotiations in collective bargaining agreements. Lenoir
paid local taxes, had local counsel, maintained Workmen's
Compensation. There is no evidence that Lenoir was run solely for the
Our question is whether these factors would establish Lenoir as the benefit of Southern. In fact a substantial part of its requirements in the
mere creature of Southern. The defendant railroad says that these
58
field of operation of Lenior were bought elsewhere. Lenoir sold unfair advantage of the subsidiary or using it as a mere adjunct to the
substantial quantities to other companies. It operated no rolling stock main corporation or as a subterfuge to justify wrongdoing, their identity
and had nothing to do with the transportation business. as separate corporations will not be disregarded but their respective
rights when dealing with each other in respect to their separate
property will be recognized and maintained. The extent of stock
The situation here is not like that in Southern Pacific Co. v. Gileo, 351
ownership and mere potential control of one company over another
U.S. 493, 76 S. Ct. 952, 100 L. Ed. 1357, where the railroad itself
has never been regarded as the determining factor in the consideration
operated a wheel foundry. The only question there as to employee
of such cases. Something must be disclosed to indicate the exercise of
Aranda was whether his duties as a wheel moulder served to further
undue domination or influence resulting in an infringement upon the
interstate commerce. The Supreme Court decided he was entitled to
rights of the subservient corporation for the benefit of the dominant
the benefit of the Federal Employers' Liability Act.
one. Otherwise, the rights of the separate corporations in respect to
their corporate property must be governed by the rules applicable in
But before the Gileo decision applies here we must first determine ordinary cases." [93 F.2d 926.]
whether Lenoir was operated so tightly by Southern that it was an
agency or instrumentality of Southern. In our opinion it was not. Policy
This excerpt from the Kentucky Electric Power case was quoted with
decisions and pricings remained in the hands of Marius. Certain
approval in a footnote to the case of Taylor v. Standard Gas & Electric
accounting and claims work was done in Washington to eliminate
Co., 10 Cir., 96 F.2d 693, 704. In that case the Court in considering the
duplication. But there is no evidence that policy decisions of Lenoir
indicia of control quoted at length from Powell on Parent and
were made or dictated by Southern. Marius operated the business as a
Subsidiary Corporations:
going concern in the fields it was equipped to handle. Where Southern,
or the Southern Railway System, did work for Lenoir there was careful
accounting and adjustment of costs at the end of each month. The
facts do not reveal the intimacy and inseparability of control which "There is respectable authority for the proposition that to justify the
would lead the Court to hold that Southern and Lenoir were one and application of the instrumentality rule between parent and subsidiary
the same. There is no such control here as in Costan v. Manila Electric corporation, there must be present in addition to the elements of
Co., supra. control through stock ownership and common directorates and officers,
elements of fraud or wrongdoing on the part of the parent corporation
to the detriment of the susbidiary and third persons in their relations
Lenoir was not in the words of the Act a "common carrier by railroad." It
with the subsidiary.
was not performing what have been called non-delegable duties of the
railroad. It was not an operator of a terminal, performed no switching or
transportation functions at all. It was a manufacturer and plaintiff was "Sections 5 and 6 of Powell on Parent and Subsidiary Corporations, in
one of its employees. It was hence not an "agent" of Southern in the part, read:
sense used in some of the cases cited by the plaintiff, since it
performed no common carrier operations. Chicago, M. & St. P. Ry. Co.
"`The Instrumentality Rule, in its shortest form, may now be stated:
v. Minnesota Civic & Commerce Ass'n, 247 U.S. 490, 38 S. Ct. 553, 62
L. Ed. 1229; Davis v. Alexander, 269 U.S. 114, 46 S. Ct. 34, 70 L. Ed.
186; Sinkler v. Missouri Pacific R. Co., 356 U.S. 326, 78 S. Ct. 758, 2 "`So far as the question of control alone is concerned, the parent
L. Ed. 2d 799. corporation will be responsible for the obligations of its subsidiary when
its control has been exercised to such *920 a degree that the
Defendant manufactured car wheels, bearings of various kinds and subsidiary has become its mere instrumentality.
items of a related nature. Southern could produce such items for its
own use and if it did *919 so would be liable in tort under the F.E. L.A. "`The Instrumentality Rule is recognized in all jurisdictions in this
for accidents occurring in the activity. But Southern did not country and our problem therefore is to determine the circumstances
manufacture these things in its own plants. Although necessary to keep which render the subsidiary an "instrumentality" within the meaning of
its rolling stock in good repair, the items could be purchased anywhere. the decisions. This is primarily a question of fact and of degree.'
In fact the evidence indicates that substantial purchases of these
products were made elsewhere. If purchased of a completely
independent company, no claim could be made that such company "`The circumstances rendering the subsidiary an instrumentality. It is
was an instrumentality of Southern. manifestly impossible to catalogue the infinite variations of fact that can
arise but there are certain common circumstances which are important
and which, if present in the proper combination, are controlling.
The question only arises because of the ownership of Lenoir by
Southern and because of the other relationships listed above. We
come back to the question whether the control was such as to make "These are as follows:
Lenoir a mere adjunct of Southern. We repeat again that it was not.
Friedman v. Vandalia R. Co., 8 Cir., 254 F. 292, 294; Atlantic Coast "`(a) The parent corporation owns all or most of the capital stock of the
Line R. Co. v. Shields, 5 Cir., 220 F.2d 242, 246. subsidiary.

In a case not involving a tort the Court of Appeals for the Sixth Circuit "`(b) The parent and subsidiary corporations have common directors or
has set down the following guide lines. Kentucky Electric Power Co. v. officers.
Norton Coal Mining Co., supra:

"`(c) The parent corporation finances the subsidiary.


"On the other hand, it is likewise well settled that a corporation is
ordinarily an entity, separate and apart from its stockholders, and mere "`(d) The parent corporation subscribes to all the capital stock of the
ownership of all the stock of one corporation by another, and the subsidiary or otherwise causes its incorporation.
identity of officers of one with officers of another, are not alone
sufficient to create identity of corporate interest between the two "`(e) The subsidiary has grossly inadequate capital.
companies or to create the relation of principal and agent or to create a
representative or fiduciary relationship between the two. If such stock
ownership and potential control be resorted to only for the purpose of "`(f) The parent corporation pays the salaries and other expenses or
normally participating in the affairs of the subsidiary corporation in a losses of the subsidiary.
manner usual to stockholders and not for the purpose of taking some

59
"`(g) The subsidiary has substantially no business except with the
parent corporation or no assets except those conveyed to it by the
parent corporation.

"`(h) In the papers of the parent corporation or in the statements of its


officers, the subsidiary is described as a department or division of the
parent corporation, or its business or financial responsibility is referred
to as the parent corporation's own.

"`(i) The parent corporation uses the property of the subsidiary as its
own.

"`(j) The directors or executives of the subsidiary do not act


independently in the interest of the subsidiary but take their orders from
the parent corporation in the latter's interest.

"`(k) The formal legal requirements of the subsidiary are not


observed.'"

In the case at bar only two of the eleven listed indicia occur, namely,
the ownership of most of the capital stock of Lenoir by Southern, and
possibly subscription by Southern to the capital stock of Lenoir. In our
opinion the principles of the Kentucky Electric Power case apply here,
and when applied we conclude that the control of Southern over Lenoir
was not such as to constitute the latter an adjunct of Southern. The
complaint must be dismissed.

60
[G.R. NO. 151438 July 15, 2005] two
brand new units of each 10-tons capacity Fedders unitary
packaged air conditioners with Fedders USA's technology
JARDINE DAVIES, INC., Petitioners, v. JRB REALTY,
perfected rotary compressors to always deliver 30,000 kcal or
INC., Respondent.
120,000 BTUH to the second floor of the Blanco Center
building at 119 Alfaro St., Salcedo Village, Makati, Metro
DECISION Manila;

CALLEJO, SR., J.: 2. Ordering defendants to jointly and severally reimburse


plaintiff not only the sums of P415,118.95 for unsaved
Before us is a Petition for Review of the Decision1 of the Court electricity from 21st October 1981 to 7th January 1990
of Appeals (CA) in CA-G.R. CV No. 54201 affirming in and P99,287.77 for repair costs of the two service units from
toto that of the Regional Trial Court (RTC) in Civil Case No. 7th March 1987 to 11th January 1990, with legal interest
90-237 for specific performance; and the Resolution dated thereon from the filing of this Complaint until fully
January 11, 2002 denying the motion for reconsideration reimbursed, but also like unsaved electricity costs and like
thereof. repair costs therefrom until Prayer No. 1 above shall have
been complied with;

The facts are as follows:


3. Ordering defendants to jointly and severally pay
plaintiff's P150,000.00 attorney's fees and other costs of
In 1979-1980, respondent JRB Realty, Inc. built a nine-storey litigation, as well as exemplary damages in an amount not
building, named Blanco Center, on its parcel of land located at less than or equal to Prayer 2 above; andcralawlibrary
119 Alfaro St., Salcedo Village, Makati City. An air
conditioning system was needed for the Blanco Law Firm
housed at the second floor of the building. On March 13, 4. Granting plaintiff such other and further relief as shall be
1980, the respondent's Executive Vice-President, Jose R. just and equitable in the premises.7
Blanco, accepted the contract quotation of Mr. A.G. Morrison,
President of Aircon and Refrigeration Industries, Inc. (Aircon), Of the four defendants, only the petitioner filed its Answer.
for two (2) sets of Fedders Adaptomatic 30,000 kcal (Code: The court did not acquire jurisdiction over Aircon because the
10-TR) air conditioning equipment with a net total selling latter ceased operations, as its corporate life ended on
price of P99,586.00.2 Thereafter, two (2) brand new packaged December 31, 1986.8 Upon motion, defendants Fedders Air
air conditioners of 10 tons capacity each to deliver 30,000 Conditioning USA and Maxim were declared in default.9
kcal or 120,000 BTUH3 were installed by Aircon. When the
units with rotary compressors were installed, they could not
On May 17, 1996, the RTC rendered its Decision, the
deliver the desired cooling temperature. Despite several
dispositive portion of which reads:
adjustments and corrective measures, the respondent
conceded that Fedders Air Conditioning USA's technology for
rotary compressors for big capacity conditioners like those WHEREFORE, judgment is hereby rendered ordering
installed at the Blanco Center had not yet been perfected. The defendants Jardine Davies, Inc., Fedders Air Conditioning
parties thereby agreed to replace the units with USA, Inc. and Maxim Industrial and Merchandising
reciprocating/semi-hermetic compressors instead. In a Letter Corporation, jointly and severally:
dated March 26, 1981,4 Aircon stated that it would be
replacing the units currently installed with new ones using
1. To deliver, install and place into operation the two (2)
rotary compressors, at the earliest possible time. Regrettably,
brand new units of Fedders unitary packaged airconditioning
however, it could not specify a date when delivery could be
units each of 10 tons capacity with rotary compressors to
effected.
deliver 30,000 kcal or 120,000 BTUH to the second floor of
the Blanco Center building, or to pay plaintiff the current price
TempControl Systems, Inc. (a subsidiary of Aircon until 1987) for two such units;
undertook the maintenance of the units, inclusive of parts and
services. In October 1987, the respondent learned, through
2. To reimburse plaintiff the amount of P556,551.55 as and
newspaper ads,5 that Maxim Industrial and Merchandising
for the unsaved electricity bills from October 21, 1981 up to
Corporation (Maxim, for short) was the new and exclusive
April 30, 1995; and another amount of P185,951.67 as and
licensee of Fedders Air Conditioning USA in the Philippines for
for repair costs;
the manufacture, distribution, sale, installation and
maintenance of Fedders air conditioners. The respondent
requested that Maxim honor the obligation of Aircon, but the 3. To pay plaintiff P50,000.00 as and for attorney's fees;
latter refused. Considering that the ten-year period of andcralawlibrary
prescription was fast approaching, to expire on March 13,
1990, the respondent then instituted, on January 29, 1990, 4. Cost of suit.10
an action for specific performance with damages against
Aircon & Refrigeration Industries, Inc., Fedders Air
Conditioning USA, Inc., Maxim Industrial & Merchandising The petitioner filed its notice of appeal with the CA, alleging
Corporation and petitioner Jardine Davies, Inc.6 The latter was that the trial court erred in holding it liable because it was not
impleaded as defendant, considering that Aircon was a a party to the contract between JRB Realty, Inc. and Aircon,
subsidiary of the petitioner. The respondent prayed that and that it had a personality separate and distinct from that
judgment be rendered, as follows: of Aircon.

1. Ordering the defendants to jointly and severally at their On March 23, 2000, the CA affirmed the trial court's ruling in
account and expense deliver, install and place in operation toto; hence, this petition.

61
The petitioner raises the following assignment of errors: 1982 (Exhibits "E," "F" and "L") also show that defendant
Jardine publicly represented Aircon to be its subsidiary.
I.
Records from the Securities and Exchange Commission (SEC)
also reveal that as per Jardine's December 31, 1986 and 1985
THE COURT OF APPEALS ERRED IN HOLDING JARDINE LIABLE
Financial Statements that "The company acts as general
FOR THE ALLEGED CONTRACTUAL BREACH OF
manager of its subsidiaries" (Exhibit "P"). Jardine's
AIRCON SOLELY BECAUSE THE LATTER WAS FORMERLY
Consolidated Balance Sheet as of December 31, 1979 filed
JARDINE'S SUBSIDIARY.
with the SEC listed Aircon as its subsidiary by owning 94.35%
of Aircon (Exhibit "P-1"). Also, Aircon's reportorial General
II. Information Sheet as of April 1980 and April 1981 filed with
the SEC show that Jardine was 94.34% owner of Aircon
ASSUMING ARGUENDO THAT AIRCON MAY BE CONSIDERED (Exhibits "Q" and "R") and that out of seven members of the
AS JARDINE'S MERE ALTER EGO, THE COURT OF APPEALS Board of Directors of Aircon, four (4) are also of Jardine.
ERRED IN NOT DECLARING AIRCON'S OBLIGATION TO
DELIVER THE TWO (2) AIRCONDITIONING UNITS TO JRB AS Defendant Jardine's witness, Atty. Fe delos Santos-Quiaoit
HAVING BEEN SUBSTANTIALLY COMPLIED WITH IN GOOD admitted that defendant Aircon, renamed Aircon &
FAITH. Refrigeration Industries, Inc. "is one of the subsidiaries of
Jardine Davies" (TSN, September 22, 1995, p. 12). She also
III. testified that Jardine nominated, elected, and appointed the
controlling majority of the Board of Directors and the highest
officers of Aircon (Ibid, pp. 10,13-14).
ASSUMING ARGUENDO THAT AIRCON MAY BE CONSIDERED
AS JARDINE'S MERE ALTER EGO, THE COURT OF APPEALS
ERRED IN NOT DECLARING JRB'S CAUSES OF ACTION AS The foregoing circumstances provide justifiable basis for this
HAVING BEEN BARRED BY LACHES. Court to disregard the fiction of corporate entity and treat
defendant Aircon as part of the instrumentality of co-
defendant Jardine.12
IV.

The respondent court arrived at the same conclusion basing


ASSUMING ARGUENDO THAT AIRCON MAY BE CONSIDERED its ruling on the following documents, to wit:
AS JARDINE'S MERE ALTER EGO, THE COURT OF APPEALS
ERRED IN FINDING JRB ENTITLED TO RECOVER ALLEGED
UNSAVED ELECTRICITY EXPENSES. (a) Contract/Quotation #78-No. 80-1639 dated March 03,
1980 (Exh. D-1);

V.
(b) Newspaper Advertisements (Exhs. E-1 and F-1);

THE COURT OF APPEALS ERRED IN HOLDING JARDINE LIABLE


TO PAY ATTORNEY'S FEES. (c) Letter dated March 26, 1981 of A.G. Morrison, President of
Aircon, to Atty. J.R. Blanco (Exh. J);

VI.
(d) News items of Bulletin Today dated August 30, 1982 (Exh.
L);
THE COURT OF APPEALS ERRED IN NOT HOLDING JRB LIABLE
TO JARDINE FOR DAMAGES.11
(e) Balance Sheet of Jardine Davies, Inc. as of December 31,
1979 listing Aircon as one of its subsidiaries (Exh. P);
It is the well-settled rule that factual findings of the trial
court, as affirmed by the CA, are accorded high respect, even
finality at times. However, considering that the factual (f) Financial Statement of Aircon as of December 31, 1982
findings of the CA and the RTC were based on speculation and and 1981 (Exh. S);
conjectures, unsupported by substantial evidence, the Court
finds that the instant case falls under one of the excepted (g) Financial Statement of Aircon as of December 31, 1981
instances. There is, thus, a need to correct the error. (Exh. S-1).13

The trial court ruled that Aircon was a subsidiary of the Applying the doctrine of piercing the veil of corporate fiction,
petitioner, and concluded, thus: both the respondent and trial courts conveniently held the
petitioner liable for the alleged omissions of Aircon,
Plaintiff's documentary evidence shows that at the time it considering that the latter was its instrumentality or corporate
contracted with Aircon on March 13, 1980 (Exhibit "D") and alter ego. The petitioner is now before us, reiterating its
on the date the revised agreement was reached on March 26, defense of separateness, and the fact that it is not a party to
1981, Aircon was a subsidiary of Jardine. The phrase "A the contract.
subsidiary of Jardine Davies, Inc." was printed on Aircon's
letterhead of its March 13, 1980 contract with plaintiff (Exhibit We find merit in the petition.
"D-1"), as well as the Aircon's letterhead of Jardine's Director
and Senior Vice-President A.G. Morrison and Aircon's
It is an elementary and fundamental principle of corporation
President in his March 26, 1981 letter to plaintiff (Exhibit "J-
law that a corporation is an artificial being invested by law
2") confirming the revised agreement. Aircon's newspaper ads
with a personality separate and distinct from its stockholders
of April 12 and 26, 1981 and a press release on August 30,
and from other corporations to which it may be connected.
62
While a corporation is allowed to exist solely for a lawful The existence of interlocking directors, corporate officers and
purpose, the law will regard it as an association of persons or shareholders, which the respondent court considered, is not
in case of two corporations, merge them into one, when this enough justification to pierce the veil of corporate fiction, in
corporate legal entity is used as a cloak for fraud or the absence of fraud or other public policy
illegality.14 This is the doctrine of piercing the veil of corporate considerations.24 But even when there is dominance over the
fiction which applies only when such corporate fiction is used affairs of the subsidiary, the doctrine of piercing the veil of
to defeat public convenience, justify wrong, protect fraud or corporate fiction applies only when such fiction is used to
defend crime.15 The rationale behind piercing a corporation's defeat public convenience, justify wrong, protect fraud or
identity is to remove the barrier between the corporation from defend crime.25 To warrant resort to this extraordinary
the persons comprising it to thwart the fraudulent and illegal remedy, there must be proof that the corporation is being
schemes of those who use the corporate personality as a used as a cloak or cover for fraud or illegality, or to work
shield for undertaking certain proscribed activities.16 injustice.26 Any piercing of the corporate veil has to be done
with caution.27 The wrongdoing must be clearly and
convincingly established. It cannot just be presumed.28
While it is true that Aircon is a subsidiary of the petitioner, it
does not necessarily follow that Aircon's corporate legal
existence can just be disregarded. In Velarde v. Lopez, In the instant case, there is no evidence that Aircon was
Inc.,17 the Court categorically held that a subsidiary has an formed or utilized with the intention of defrauding its creditors
independent and separate juridical personality, distinct from or evading its contracts and obligations. There was nothing
that of its parent company; hence, any claim or suit against fraudulent in the acts of Aircon in this case. Aircon, as a
the latter does not bind the former, and vice versa. In manufacturing firm of air
applying the doctrine, the following requisites must be conditioners, complied with its obligation of providing two air
established: (1) control, not merely majority or complete conditioning units for the second floor of the Blanco Center in
stock control; (2) such control must have been used by the good faith, pursuant to its contract with the respondent.
defendant to commit fraud or wrong, to perpetuate the Unfortunately, the performance of the air conditioning units
violation of a statutory or other positive legal duty, or did not satisfy the respondent despite several adjustments
dishonest acts in contravention of plaintiff's legal rights; and and corrective measures. In a Letter29 dated October 22,
(3) the aforesaid control and breach of duty must proximately 1980, the respondent even conceded that Fedders Air
cause the injury or unjust loss complained of.18 Conditioning USA has not yet perhaps perfected its
technology of rotary compressors, and agreed to change the
compressors with the semi-hermetic type. Thus, Aircon
The records bear out that Aircon is a subsidiary of the
substituted the units with serviceable ones which delivered
petitioner only because the latter acquired Aircon's majority of
the cooling temperature needed for the law office. After
capital stock. It, however, does not exercise complete control
enjoying ten (10) years of its cooling power, respondent
over Aircon; nowhere can it be gathered that the petitioner
cannot now complain about the performance of these units,
manages the business affairs of Aircon. Indeed, no
nor can it demand a replacement thereof.
management agreement exists between the petitioner and
Aircon, and the latter is an entirely different entity from the
petitioner.19 Moreover, it was reversible error to award the respondent the
amount of P556,551.55 representing the alleged 30%
unsaved electricity costs and P185,951.67 as maintenance
Jardine Davies, Inc., incorporated as early as June 28,
cost without showing any basis for such award. To justify a
1946,20 is primarily a financial and trading company. Its
grant of actual or compensatory damages, it is necessary to
Articles of Incorporation states among many others that the
prove with a reasonable degree of certainty, premised upon
purposes for which the said corporation was formed, are as
competent proof and on the best evidence obtainable by the
follows:
injured party, the actual amount of loss.30 The respondent
merely based its cause of action on Aircon's alleged
(a) To carry on the business of merchants, commission representation that Fedders air conditioners with rotary
merchants, brokers, factors, manufacturers, and agents; compressors can save as much as 30% on electricity
compared to other brands. Offered in evidence were
(b) Upon complying with the requirements of law applicable newspaper advertisements published on April 12 and 26,
thereto, to act as agents of companies and underwriters doing 1981. The respondent then recorded its electricity
and engaging in any and all kinds of insurance business.21 consumption from October 21, 1981 up to April 3, 1995 and
computed 30% thereof, which amounted to P556,551.55. The
Court rules that this amount is highly speculative and merely
On the other hand, Aircon, incorporated on December 27, hypothetical, and for which the petitioner can not be held
1952,22 is a manufacturing firm. Its Articles of Incorporation accountable.
states that its purpose is mainly -

First. The respondent merely relied on the newspaper


To carry on the business of manufacturers of commercial and advertisements showing the Fedders window-type air
household appliances and accessories of any form, conditioners, which are far different from the big capacity air
particularly to manufacture, purchase, sell or deal in air conditioning units installed at Blanco Center.
conditioning and refrigeration products of every class and
description as well as accessories and parts thereof, or other
kindred articles; and to erect, or buy, lease, manage, or Second. After such print advertisements, the respondent
otherwise acquire manufactories, warehouses, and depots for informed Aircon that it was going to install an electric meter
manufacturing, assemblage, repair and storing, buying, to register its electric consumption so as to determine the
selling, and dealing in the aforesaid appliances, accessories electric costs not saved by the presently installed units with
and products. '23 semi-hermetic compressors. Contrary to the allegations of the
respondent that this was in pursuance to their Revised
Agreement, no proof was adduced that Aircon agreed to the
respondent's proposition. It was a unilateral act on the part of

63
the respondent, which Aircon did not oblige or commit itself to
pay.

Third. Needless to state, the amounts computed are mere


estimates representing the respondent's self-serving claim of
unsaved electricity cost, which is too speculative and
conjectural to merit consideration. No other proofs, reports or
bases of comparison showing that Fedders Air Conditioning
USA could indeed cut down electricity cost by 30% were
adduced.

Likewise, there is no basis for the award of P185,951.67


representing maintenance cost. The respondent merely
submitted a schedule31 prepared by the respondent's
accountant, listing the alleged repair costs from March 1987
up to June 1994. Such evidence is self-serving and can not
also be given probative weight, considering that there are no
proofs of receipts, vouchers, etc., which would substantiate
the amounts paid for such services. Absent any more
convincing proof, the Court finds that the respondent's claims
are without basis, and cannot, therefore, be awarded.

We sustain the petitioner's separateness from that of Aircon


in this case. It bears stressing that the petitioner was never a
party to the contract. Privity of contracts take effect only
between parties, their successors-in-interest, heirs and
assigns.32 The petitioner, which has a
separate and distinct legal personality from that of Aircon,
cannot, therefore, be held liable.

IN VIEW OF THE FOREGOING, the petition is GRANTED.


The assailed decision of the Court of Appeals, affirming the
decision of the Regional Trial Court is REVERSED and SET
ASIDE. The complaint of the respondent is DISMISSED.
Costs against the respondent.

SO ORDERED.

64
G.R. No. L-47673 October 10, 1946 quotation.) Plaintiff then cabled for the quotation desired for
Koppel Industrial Car and Equipment Company. A sample of
the pertinent cable is hereto attached and made a part
KOPPEL (PHILIPPINES), INC., plaintiff-appellant,
hereof as Exhibit B. Koppel Industrial Car and Equipment
vs.
Company answered by cable quoting its cost price, usually
ALFREDO L. YATCO, Collector of Internal Revenue, defendant-
A. C. I. F. Manila cost price, which was later followed by a
appellee.
letter of confirmation. A sample of the said cable quotation
and of the letter of confirmation are hereto attached and
Padilla, Carlos and Fernando for appellant. made a part hereof as Exhibits C and C-1. Plaintiff, however,
Office of the Solicitor General Ozaeta, First Assistant Solicitor General quoted by Koppel Industrial Car and Equipment Company.
Reyes and. Copy of the plaintiff's letter to purchaser is hereto attached
Office of the Solicitor General Reyes and Solicitor Cañizanes for and made a part hereof as Exhibit D. On the basis of these
appellee. quotations, orders were placed by the local purchasers,
copies of which orders are hereto attached as Exhibits E and
E-1.

A cable was then sent to Koppel Industrial Car and


Equipment company giving instructions to ship the
HILADO, J.:
merchandise to Manila forwarding the customer's order.
Sample of said cable is hereto attached as Exhibit F. The
This is an appeal by Koppel (Philippines), Inc., from the judgment of bills of lading were usually made to "order" and indorsed in
the Court of First Instance of Manila in civil case No. 51218 of said blank with notation to the effect that the buyer be notified of
court dismissing said corporation's complaint for the recovery of the the shipment of the goods covered in the bills of lading;
sum of P64,122.51 which it had paid under protest to the Collector of commercial invoices were issued by Koppel Industrial Car
Internal Revenue on October 30, 1936, as merchant sales tax. The and Equipment Company in the names of the purchasers
main facts of the case were stipulated in the court below as follows: and certificates of insurance were likewise issued in their
names, or in the name of Koppel Industrial Car and
Equipment Company but indorsed in blank and attached to
AGREED STATEMENT OF FACTS drafts drawn by Koppel Industrial Car and Equipment
Company on the purchasers, which were forwarded through
Now come the plaintiff by attorney Eulogio P. Revilla and the foreign banks to local banks. Samples of the bills of lading
defendant by the Solicitor General and undersigned are hereto attached as Exhibits F-1, I-1, I-2 and I-3. Bills of
Assistant Attorney of the Bureau of Justice and, with leave of ladings, Exhibits I-1, I-2 and I-3, may equally have been
this Honorable Court, hereby respectfully stipulated and employed, but said Exhibits I-1, I-2 and I-3 have no
agree to the following facts, to wit: connection with the transaction covered by Exhibits B to G,
inclusive. The purchasers secured the shipping papers by
arrangement with the banks, and thereupon received and
I. That plaintiff is a corporation duly organized and existing cleared the shipments. If the merchandise were of European
under and by virtue of the laws of the Philippines, with origin, and if there was not sufficient time to forward the
principal office therein at the City of Manila, the capital stock documents necessary for clearance, through foreign banks
of which is divided into thousand (1,000) shares of P100 to local banks, to the purchasers, the Koppel Industrial Car
each. The Koppel Industrial Car and Equipment company, a and Equipment company did, in many cases, send the
corporation organized and existing under the laws of the documents directly from Europe to plaintiff with instructions
State of Pennsylvania, United States of America, and not to turn these documents over to the purchasers. In many
licensed to do business in the Philippines, owned nine cases, where sales was effected on the basis of C. I. F.
hundred and ninety-five (995) shares out of the total capital Manila, duty paid, plaintiff advanced the sums required for
stock of the plaintiff from the year 1928 up to and including the payment of the duty, and these sums, so advanced, were
the year 1936, and the remaining five (5) shares only were in every case reimbursed to plaintiff by Koppel Industrial Car
and are owned one each by officers of the plaintiff and Equipment Company. The price were payable by drafts
corporation. agreed upon in each case and drawn by Koppel Industrial
Car and Equipment Company on respective purchasers
II. That plaintiff, at all times material to this case, was and through local banks, and payments were made to the banks
now is duly licensed to engage in business as a merchant by the purchasers on presentation and delivery to them of
and commercial broker in the Philippines; and was and is the the above-mentioned shipping documents or copies thereof.
holder of the corresponding merchant's and commercial A sample of said drafts is hereto attached as Exhibit G.
broker's privilege tax receipts. Plaintiff received by way of compensation a percentage of
the profits realized on the above transactions as fixed in
paragraph 6 of the plaintiff's contract with Koppel Industrial
III. That the defendant Collector of Internal revenue is now Car and Equipment Company, which contract is hereto
Mr. Bibiano L. Meer in lieu of Mr. Alfredo L. Yatco. attached as Exhibit H, and suffered its corresponding share
in the losses resulting from some of the transactions.
IV. That during the period from January 1, 1929, up to and
including December 31, 1932, plaintiff transacted business in That the total gross sales from January 1, 1929, up to and
the Philippines in the following manner, with the exception of including December 31, 1932, effected in the foregoing
the transactions which are described in paragraphs V and VI manner and under the above specified conditions, amount to
of this stipulation: P3, 596,438.84.

When a local buyer was interested in the purchase of railway V. That when a local sugar central was interested in the
materials, machinery, and supplies, it asked for price purchase of railway materials, machinery and supplies, it
quotations from plaintiff. Atypical form of such request is secured quotations from, and placed the corresponding
attached hereto and made a part hereof as Exhibit A. orders with, the plaintiff in substantially the same manner as
(Exhibit A represents typical transactions arising from written outlined in paragraph IV of this stipulation, with the only
requests for quotations, while Exhibits B to G, inclusive, are difference that the purchase orders which were agreed to by
typical transactions arising from verbal requests for
65
the central and the plaintiff are similar to the sample hereto Manila, Philippines, December 26, 1939
attached and made a part hereof as Exhibit I. Typical
samples of the bills of lading covering the herein transaction
(Sgd.) ROMAN OZAETA
are hereto attached and made a part hereto as Exhibits I-1,
Solicitor General
I-2 and I-3. The value of the sales carried out in the manner
mentioned in this paragraph is P133,964.98.
(Sgd.) ANTONIO CAÑIZARES
Assistant Attorney
VI. That sometime in February, 1929, Miguel J. Ossorio, of
Manila, Philippines, placed an option with Koppel Industrial
Car and Equipment Company, through plaintiff, to purchase (Sgd.) E. P. REVILLA
within three months a pair of Atlas-Diesel Marine Engines. Attorney for the Plaintiff
Koppel Industrial Car and Equipment Company purchased 3rd Floor, Perez Samanillo Bldg.,
said Diesel Engines in Stockholm, Sweden, for $16,508.32. Manila
The suppliers drew a draft for the amount of $16,508.32 on
the Koppel Industrial Car and Equipment Company, which
paid the amount covered by the draft. Later, Miguel J. Both parties adduced some oral evidence in clarification of or
addition to their agreed statement of facts. A preponderance
Ossorio definitely called the deal off, and as Koppel
Industrial Car and Equipment Company could not ship to or of evidence has established, besides the facts thus
draw on said Mr. Miguel J. Ossorio, it in turn drew another stipulated, the following:
draft on plaintiff for the same amount at six months sight,
with the understanding that Koppel Industrial Car and (a) The shares of stock of plaintiff corporation were
Equipment Company would reimburse plaintiff when said and are all owned by Koppel Industries Car and
engines were disposed of. Plaintiff honored the draft and Equipment Company of Pennsylvania, U. S. A.,
debited the said sum of $16,508.32 to merchandise account. exceptive which were necessary to qualify the
The engines were left stored at Stockholm, Sweden. On April Board of Directors of said plaintiff corporation;
1, 1930, a new local buyer, Mr. Cesar Barrios, of Iloilo,
Philippines, was found and the same engines were sold to
him for $21,000 (P42,000) C. I. F. Hongkong. The engines (b) In the transactions involved herein the plaintiff
were shipped to Hongkong and a draft for $21,000 was corporation acted as the representative of Koppel
drawn by Koppel Industrial Car and Equipment Company on Industrial Car and Equipment Company only, and
Mr. Cesar Barrios. After the draft was fully paid by Mr. not as the agent of both the latter company and
Barrios, Koppel Industrial Car and Equipment Company the respective local purchasers — plaintiff's
reimbursed plaintiff with cost price of $16,508.32 and principal witness, A.H. Bishop, its resident Vice-
credited it with $1,152.95 as its share of the profit on the President, in his testimony invariably referred to
transaction. Exhibits J and J-1 are herewith attached and Koppel Industrial Car and Equipment Co. as "our
made integral parts of this stipulation with particular principal" 9 t. s. n., pp. 10, 11, 12, 19, 75), except
reference to paragraph VI hereof. that at the bottom of page 10 to the top of page 11,
the witness stated that they had "several principal"
abroad but that "our principal abroad was, for the
VII. That plaintiff's share in the profits realized out of these years in question, Koppel Industrial Car and
transactions described in paragraphs IV, V and VI hereof Equipment Company," and on page 68, he
totaling P3,772,403.82, amounts to P132,201.30; and that testified that what he actually said was ". . . but
plaintiff within the time provided by law returned the our principal abroad" and not "our principal
aforesaid amount P132,201.30 for the purpose of the abroad" — as to which it is very significant that
commercial broker's 4 per cent tax and paid thereon the sum neither this witness nor any other gave the name
P5,288.05 as such tax. of even a single other principal abroad of the
plaintiff corporation;
VIII. That defendant demanded of the plaintiff the sum of
P64,122.51 as the merchants' sales tax of 1% per cent on (c) The plaintiff corporation bore alone incidental
the amount of P3,772,403.82, representing the total gross expenses — as, for instance, cable expenses-not
value of the sales mentioned in paragraphs IV, V and VI only those of its own cables but also those of its
hereof, including the 25 per cent surcharge for the late "principal" (t.s.n., pp. 52, 53);
payment of the said tax, which tax and surcharge were
determined after the amount of P5,288.05 mentioned in
paragraph VI hereof was deducted. (d) the plaintiff's "share in the profits" realized from
the transactions in which it intervened was left
virtually in the hands of Koppel Industrial Car and
IX. That plaintiff, on October 30, 1936, paid under protest Equipment Company (t.s.n., p. 51);
said sum of P64,122.51 in order to avoid further penalties,
levy and distraint proceedings.
(e) Where drafts were not paid by the purchasers,
the local banks were instructed not to protest them
X. That defendant, on November 10, 1936, overruled but to refer them to plaintiff which was fully
plaintiff's protest, and defendant has failed and refused and empowered by Koppel Industrial Car and
still fails and refuses, notwithstanding demands by plaintiff, Equipment company to instruct the banks with
to return to the plaintiff said sum of P64,122.51 or any part regards to disposition of the drafts and documents
thereof. (t.s.n., p. 50; Exhibit G);lawphil.net

xxx xxx xxx (f) Where the goods were European origin,
consular invoices, bill of lading, and, in general,
That the parties hereby reserve the right the documents necessary for clearance were sent
to present additional evidence in support directly to plaintiff (t.s.n., p. 14);
of their respective contentions.

6
(g) If the plaintiff had in stock the merchandise 3. the court a quo erred in not holding that a character of a
desired by local buyers, it immediately filled the broker is determined by the nature of the transaction and not
orders of such local buyers and made delivery in by the basis or measure of his compensation;
the Philippines without the necessity of cabling its
principal in America either for price quotations or
4. The court a quo erred in not holding that appellant acted
confirmation or rejection of that agreed upon
as a commercial broker in the transactions covered under
between it and the buyer (t.s.n., pp. 39-43);
paragraph VI of the agreed statement of facts;

(h) Whenever the deliveries made by Koppel


5. The court a quo erred in not holding that appellant acted
Industrial Car and Equipment Company were
as a commercial broker in the transactions covered under
incomplete or insufficient to fill the local buyer's
paragraph v of the agreed statement of facts;
orders, plaintiff used to make good the deficiencies
by deliveries from its own local stock, but in such
cases it charged its principal only the actual cost of 6. The court a quo erred in not holding that appellant acted
the merchandise thus delivered by it from its stock as a commercial broker in the sole transaction covered
and in such transactions plaintiff did not realize under paragraph VI of the agreed statement of facts;
any profit (t.s.n., pp. 53-54);
7. the court a quo erred in dismissing appellant's complaint.
(i) The contract of sale involved herein were all
perfected in the Philippines.
The lower court found and held that Koppel (Philippines), Inc. is a mere
dummy or brach ("hechura") of Koppel industrial Car and Equipment
Those described in paragraph IV of the agreed statement of Company. The lower court did not deny legal personality to Koppel
facts went through the following process: (1) "When a local (Philippines), Inc. for any and all purposes, but in effect its conclusion
buyer was interested in the purchase of railway materials, was that, in the transactions involved herein, the public interest and
machinery, and supplies, it asked for price quotations from convenience would be defeated and what would amount to a tax
plaintiff"; (2) "Plaintiff then cabled for the quotation desired evasion perpetrated, unless resort is had to the doctrine of "disregard
from Koppel Industrial Car and Equipment Company"; (3) of the corporate fiction."
"Plaintiff, however, quoted to the purchaser a selling price
above the figures quoted by Koppel Industrial Car and
Equipment Company"; (4) "On the basis of these quotations, I. In its first assignment of error appellant submits that the trial court
orders were placed by the local purchasers . . ." erred in not holding that it is a domestic corporation distinct and
separate from and not a mere branch of Koppel Industrial Car and
Equipment Company. It contends that its corporate existence as
Those described in paragraph V of said agreed statement of Philippine corporation can not be collaterally attacked and that the
facts were transacted "in substantially the same manner as Government is estopped from so doing. As stated above, the lower
outlined in paragraph IV." court did not deny legal personality to appellant for any and all
purposes, but held in effect that in the transaction involved in this case
the public interest and convenience would be defeated and what would
As to the single transaction described in paragraph VI of the
amount to a tax evasion perpetrated, unless resort is had to the
same agreed statement of facts, discarding the Ossorio
doctrine of "disregard of the corporate fiction." In other words, in
option which anyway was called off, "On April 1, 1930, a new
looking through the corporate form to the ultimate person or
local buyer, Mr. Cesar Barrios, of Iloilo, Philippines, was
corporation behind that form, in the particular transactions which were
found and the same engines were sold to him for
involved in the case submitted to its determination and judgment, the
$21,000(P42,000) C.I.F. Hongkong." (Emphasis supplied.).
court did so in order to prevent the contravention of the local internal
revenue laws, and the perpetration of what would amount to a tax
(j) Exhibit H contains the following paragraph: evasion, inasmuch as it considered — and in our opinion, correctly —
that appellant Koppel (Philippines), Inc. was a mere branch or agency
or dummy ("hechura") of Koppel Industrial Car and Equipment Co. The
It is clearly understood that the intent of this contract is that
court did not hold that the corporate personality of Koppel (Philippines),
the broker shall perform only the functions of a broker as set
Inc., would also be disregarded in other cases or for other purposes. It
forth above, and shall not take possession of any of the
would have had no power to so hold. The courts' action in this regard
materials or equipment applying to said orders or perform
must be confined to the transactions involved in the case at bar "for the
any acts or duties outside the scope of a broker; and in no
purpose of adjudging the rights and liabilities of the parties in the case.
sense shall this contract be construed as granting to the
They have no jurisdiction to do more." (1 Flethcer, Cyclopedia of
broker the power to represent the principal as its agent or to
Corporation, Permanent ed., p. 124, section 41.)
make commitments on its behalf.

A leading and much cited case puts it as follows:


The Court of First Instance held for the defendant and dismissed
plaintiff's complaint with costs to it.
If any general rule can be laid down, in the present state of
authority, it is that a corporation will be looked upon as a
Upon this appeal, seven errors are assigned to said judgment as
legal entity as a general rule, and until sufficient reason to
follows:.
the contrary appears; but, when the notion of legal entity is
used to defeat public convinience, justify wrong, protect
1. That the court a quo erred in not holding that appellant is a fraud, or defend crime, the law will regard the corporation as
domestic corporation distinct and separate from, and not a an association of persons. (1 Fletcher Cyclopedia of
mere branch of Koppel Industrial Car and Equipment Co.; Corporation [Permanent Edition], pp. 135, 136; United States
vs. Milwaukee Refrigeration Transit Co., 142 Fed., 247, 255,
per Sanborn, J.)
2. the court a quo erred in ignoring the ruling of the Secretary
of Finance, dated January 31, 1931, Exhibit M;
In his second special defense appellee alleges "that the plaintiff was
and is in fact a branch or subsidiary of Koppel Industrial Car and
Equipment Co., a Pennsylvania corporation not licensed to do

67
business in the Philippines but actually doing business here through corporation, and commingling its affairs in administration with
the plaintiff; that the said foreign corporation holds 995 of the 1,000 the affairs of the railroad company, so as to make the two
shares of the plaintiff's capital stock, the remaining five shares being corporations virtually one, brings the railroad company so
held by the officers of the plaintiff herein in order to permit the voluntarily acting as to such producing, etc., corporation
incorporation thereof and to enable its aforesaid officers to act as within the prohibitions of the commodities clause. In other
directors of the plaintiff corporation; and that plaintiff was organized as words, that by operation and effect of the commodities
a Philippine corporation for the purpose of evading the payment by its clause there is duty cast upon a railroad company proposing
parent foreign corporation of merchants' sales tax on the transactions to carry in interstate commerce the product of a producing,
involved in this case and others of similar nature." etc., corporation in which it has a stock interest, not to abuse
such power so as virtually to do by indirection that which the
commodities clause prohibits, — a duty which plainly would
By most courts the entity is normally regarded but is
be violated by the unnecessary commingling of the affairs of
disregarded to prevent injustice, or the distortion or hiding of
the producing company with its own, so as to cause them to
the truth, or to let in a just defense. (1 Fletcher, Cyclopedia
be one and inseparable.
of Corporation, Permanent Edition, pp. 139,140; emphasis
supplied.)
Corrobarative authorities can be cited in support of the same
proposition, which we deem unnecessary to mention here.
Another rule is that, when the corporation is the mere alter
ego, or business conduit of a person, it may de disregarded."
(1 Fletcher, Cyclopedia of Corporation, Permanent Edition, From the facts hereinabove stated, as established by a preponderance
p. 136.) of the evidence , particularly those narrated in paragraph (a), (b), (c),
(d), (e),(f), (h), (i), and (j) after the agreed statement of facts, we find
that, in so far as the sales involved herein are concerned, Koppel
Manifestly, the principle is the same whether the "person" be natural or
(Philippines), Inc., and Koppel Industrial Car and Equipment company
artificial.
are to all intents and purposes one and the same; or, to use another
mode of expression, that, as regards those transactions, the former
A very numerous and growing class of cases wherein the corporation is a mere branch, subsidiary or agency of the latter. To our
corporate entity is disregarded is that (it is so organized and mind, this is conclusively borne out by the fact, among others, that the
controlled, and its affairs are so conducted, as to make it amount of he so-called "share in the profits" of Koppel (Philippines),
merely an instrumentality, agency, conduit or adjunct of Inc., was ultimately left to the sole, unbridled control of Koppel
another corporation)." (1 Fletcher, Cyclopedia of Industrial Car and Equipment Company. If, in their relations with each
Corporation, Permanent ed., pp. 154, 155.) other, Koppel (Philippines), Inc., was considered and intended to
function as a bona fide separate corporation, we can not conceive how
this arrangement could have been adopted, for if there was any factor
While we recognize the legal principle that a corporation
in its business as to which it would in that case naturally have been
does not lose its entity by the ownership of the bulk or even opposed to being thus controlled, it must have been precisely the
the whole of its stock, by another corporation (Monongahela amount of profit which it could endeavor and hope to earn. No group of
Co. vs. Pittsburg Co., 196 Pa., 25; 46 Atl., 99; 79 Am. St.
businessmen could be expected to organize a mercantile corporation
Rep., 685) yet it is equally well settled and ignore corporate — the ultimate end of which could only be profit — if the amount of that
forms." (Colonial Trust Co. vs. Montello Brick Works, 172 profit were to be subjected to such a unilateral control of another
Fed., 310.)
corporation, unless indeed the former has previously been designed by
the incorporators to serve as a mere subsidiary, branch or agency of
Where it appears that two business enterprises are owned, the latter. Evidently, Koppel Industrial Car and Equipment Company
conducted and controlled by the same parties, both law and made us of its ownership of the overwhelming majority — 99.5% — of
equity will, when necessary to protect the rights of third the capital stock of the local corporation to control the operations of the
persons, disregard the legal fiction that two corporations are latter to such an extent that it had the final say even as to how much
distinct entities, and treat them as identical. (Abney vs. should be allotted to said local entity in the so-called sharing in the
Belmont Country Club Properties, Inc., 279 Pac., 829.) profits. We can not overlook the fact that in the practical working of
corporate organizations of the class to which these two entities belong,
the holder or holders of the controlling part of the capital stock of the
. . . the legal fiction of distinct corporate existence will be corporation, particularly where the control is determined by the virtual
disregarded in a case where a corporation is so organized ownership of the totality of the shares, dominate not only the selection
and controlled and its affairs are so conducted, as to make it of the Board of Directors but, more often than not, also the action of
merely an instrumentality or adjunct of another corporation. that Board. Applying this to the instant case, we can not conceive how
(Hanter vs. Baker Motor Vehicle Co., 190 Fed., 665.) the Philippine corporation could effectively go against the policies,
decisions, and desires of the American corporation with regards to the
In United States vs. Lehigh Valley R. Co. 9220 U.S., 257; 55 Law. ed., scheme which was devised through the instrumentality of the contract
458, 464), the Supreme Court of the United States disregarded the Exhibit H, as well as all the other details of the system which was
artificial personality of the subsidiary coal company in order to avoid adopted in order to avoid paying the 1½ per cent merchants sales tax.
that the parent corporation, the Lehigh Valley R. Co., should be able, Neither can we conceive how the Philippine corporation could avoid
through the fiction of that personality, to evade the prohibition of the following the directions of the American corporation held 99.5 per cent
Hepburn Act against the transportation by railroad companies of the of the capital stock of the Philippine corporation. In the present
articles and commodities described therein. instance, we note that Koppel (Philippines), Inc., was represented in
the Philippines by its "resident Vice-President." This fact necessarily
leads to the inference that the corporation had at least a Vice-
Chief Justice White, speaking for the court, said: President, and presumably also a President, who were not resident in
the Philippines but in America, where the parent corporation is
. . . Coming to discharge this duty it follows, in view of the domiciled. If Koppel (Philippines), Inc., had been intended to operate
express prohibitions of the commodities clause, it must be as a regular domestic corporation in the Philippines, where it was
held that while the right of a railroad company as a formed, the record and the evidence do not disclose any reason why
stockholder to use its stock ownership for the purpose of all its officers should not reside and perform their functions in the
a bona fide separate administration of the affairs of a Philippines.
corporation in which it has a stock interest may not be
denied, the use of such stock ownership in substance for the
purpose of destroying the entity of a producing, etc.,
68
Other facts appearing from the evidence, and presently to be stated, invitable, and have to be effected in the ordinary course of business
strengthen our conclusion, because they can only be explained if the enterprise extends its trade to another land through a branch office, or
local entity is considered as a mere subsidiary, branch or agency of the through another scheme amounting to the same thing.
parent organization. Plaintiff charged the parent corporation no more
than actual cost — without profit whatsoever — for merchandise
If plaintiff were to act as broker in the Philippines for any other
allegedly of its own to complete deficiencies of shipments made by
corporation, entity or person, distinct from Koppel Industrial Car and
said parent corporation (t.s.n., pp. 53, 54) — a fact which could not
Equipment company, an entirely different question will arise, which,
conceivably have been the case if plaintiff had acted in such
however, we are not called upon, nor in a position, to decide.
transactions as an entirely independent entity doing business — for
profit, of course — with the American concern. There has been no
attempt even to explain, if the latter situation really obtained, why these As stated above, Exhibit H contains to the following paragraph:
two corporations should have thus departed from the ordinary course
of business. Plaintiff was charged by the American corporation with the
It is clearly understood that the intent of this contract is that
cost even of the latter's cable quotations — from ought that appears
from the evidence, this can only be comprehended by considering the broker shall perform only the functions of a broker as set
plaintiff as such a subsidiary, branch or agency of the parent entity, in forth above, and shall not take possession of any of the
materials or equipment applying to said orders or perform
which case it would be perfectly understandable that for convenient
accounting purposes and the easy determination of the profits or any acts or duties outside the scope of a broker; and in no
losses of the parent corporation's Philippines should be charged sense shall this contract be construed as granting to the
broker the power to represent the principal as its agent or to
against the Philippine office and set off against its receipts, thus
separating the accounts of said branch from those which the central make commitments on its behalf.
organization might have in other countries. The reference to plaintiff by
local banks, under a standing instruction of the parent corporation, of The foregoing paragraph, construed in the light of other facts noted
unpaid drafts drawn on Philippine customers by said parent elsewhere in this decision, betrays, we think a deliberate intent,
corporation, whenever said customers dishonored the drafts, and the through the medium of a scheme devised with great care, to avoid the
fact that the American corporation had previously advised said banks payment of precisely the 1½ per cent merchants' sales tax in force in
that plaintiff in those cases was "fully empowered to instruct (the the Philippines before, at the time of, and after, the making of the said
banks) with regard to the disposition of the drafts and documents" contract Exhibit H. If this were to be allowed, the payment of a tax,
(t.s.n., p. 50), in the absence of any other satisfactory explanation which directly could not have been avoided, could be evaded by
naturally give rise to the inference that plaintiff was a subsidiary, indirection, consideration being had of the aforementioned peculiar
branch or agency of the American concern, rather than an independent relations between the said American and local corporations. Such
corporation acting as a broker. For, without such positive explanation, evasion, involving as it would, a violation of the former Internal
this delegation of power is indicative of the relations between central Revenue Law, would even fall within the penal sanction of section
and branch offices of the same business enterprise, with the latter 2741 of the Revised Administrative Code. Which only goes to show the
acting under instructions already given by the former. Far from illegality of the whole scheme. We are not here concerned with the
disclosing a real separation between the two entities, particularly in impossibility of collecting the merchants' sales tax, as a mere incidental
regard to the transactions in question, the evidence reveals such consequence of transactions legal in themselves and innocent in their
commongling and interlacing of their activities as to render even purpose. We are dealing with a scheme the primary, not to say the
incomprehensible certain accounting operations between them, except sole, object of which the evasion of the payment of such tax. It is this
upon the basis that the Philippine corporation was to all intents and aim of the scheme that makes it illegal.
purposes a mere subsidiary, branch, or agency of the American parent
entity. Only upon this basis can it be comprehended why it seems not
to matter at all how much profit would be allocated to plaintiff, or even We have said above that the contracts of sale involved herein were all
that no profit at all be so allocated to it, at any given time or after any perfected in the Philippines. From the facts stipulated in paragraph IV
given period. of the agreed statement of facts, it clearly appears that the Philippine
purchasers had to wait for Koppel Industrial Car and Equipment
Company to communicate its cost prices to Koppel (Philippines), Inc.,
As already stated above, under the evidence the sales in the were perfected in the Philippines. In those cases where no such price
Philippines of the railway materials, machinery and supplies imported quotations from the American corporation were needed, of course, the
here by Koppel Industrial Car and Equipment Company could have sales effected in those cases described in paragraph V of the agreed
been as conviniently and efficiently transacted and handled — if not statement of facts were, as expressed therein, transacted "in
more so — had said corporation merely established a branch or substantially the same manner as outlined in paragraph VI." Even the
agency in the Philippines and obtained license to do business locally; single transaction described in paragraph VI of the agreed statement of
and if it had done so and said sales had been effected by such branch facts was also perfected in the Philippines, because the contracting
or agency, there seems to be no dispute that the 1½ per cent parties were here and the consent of each was given here. While it is
merchants' sales tax then in force would have been collectible. So far true that when the contract was thus perfected in the Philippines the
as we can discover, there would be only one, but very important, pair of Atlas-Diesel Marine Engines were in Sweden and the
difference between the two schemes — a difference in tax liability on agreement was to deliver them C.I.F. Hongkong, the contract of sale
the ground that the sales were made through another and distinct being consensual — perfected by mere consent — (Civil Code, article
corporation, as alleged broker, when we have seen that this latter 1445; 10 Manresa, 4th ed., p. 11), the location of the property and the
corporation is virtually owned by the former, or that they practically one place of delivery did not matter in the question of where the agreement
and the same, is to sanction a circumvention of our tax laws, and was perfected.
permit a tax evasion of no mean proportions and the consequent
commission of a grave injustice to the Government. Not only this; it
would allow the taxpayer to do by indirection what the tax laws In said paragraph VI, we read the following, as indicating where the
prohibited to be done directly (non-payment of legitimate taxes), contract was perfected, considering beforehand that one party, Koppel
paraphrasing the United States Supreme Court in United States vs. (Philippines),Inc., which in contemplation of law, as to that transaction,
Lehigh Valley R. Co., supra. was the same Koppel Industrial Car Equipment Co., was in the
Philippines:

The act of one corporation crediting or debiting the other for certain
items, expenses or even merchandise sold or disposed of, is perfectly . . . on April 1, 1930, a new local buyer Mr. Cesar Barrios,
compatible with the idea of the domestic entity being or acting as a of Iloilo, Philippines, was found and the same engines were
mere branch, agency or subsidiary of the parent organization. Such sold to him for $21,000 (P42,000) C.I.F. Hongkong . . .
operations were called for any way by the exigencies or convenience (Emphasis supplied.)
of the entire business. Indeed, accounting operation such as these are
69
Under the revenue law in force when the sales in question took place,
the merchants' sales tax attached upon the happening of the
respective sales of the "commodities, goods, wares, and merchandise"
involved, and we are clearly of opinion that such "sales" took place
upon the perfection of the corresponding contracts. If such perfection
took place in the Philippines, the merchants' sales tax then in force
here attached to the transactions.

Even if we should consider that the Philippine buyers in the cases


covered by paragraph IV and V of the agreed statement of facts,
contracted with Koppel Industrial Car and Equipment company, we will
arrive at the same final result. It can not be denied in that case that
said American corporation contracted through Koppel (Philippines),
Inc., which was in the Philippines. The real transaction in each case of
sale, in final effect, began with an offer of sale from the seller, said
American corporation, through its agent, the local corporation, of the
railway materials, machinery, and supplies at the prices quoted, and
perfected or completed by the acceptance of that offer by the local
buyers when the latter, accepting those prices, placed their orders. The
offer could not correctly be said to have been made by the local buyers
when they asked for price quotations, for they could not rationally be
taken to have bound themselves to buy before knowing the prices. And
even if we should take into consideration the fact that the american
corporation contracted, at least partly, through correspondence,
according to article 54 of the Code of Commerce, the respective
contracts were completed from the time of the acceptance by the local
buyers, which happened in the Philippines.

Contracts executed through correspondence shall be


completed from the time an answer is made accepting the
proposition or the conditions by which the latter may be
modified." (Code of Commerce, article 54; emphasis
supplied.)

A contract is as a rule considered as entered into at the


place where the place it is performed. So where delivery is
regarded as made at the place of delivery." (13 C. J., 580-81,
section 581.)

(In the consensual contract of sale delivery is not needed for


its perfection.)

II. Appellant's second assignment of error can be summarily disposed


of. It is clear that the ruling of the Secretary of Finance, Exhibit M, was
not binding upon the trial court, much less upon this tribunal, since the
duty and power of interpreting the laws is primarily a function of the
judiciary. (Ortua vs. Singson Encarnacion, 59 Phil., 440, 444.) Plaintiff
cannot be excused from abiding by this legal principle, nor can it
properly be heard to say that it relied on the Secretary's ruling and that,
therefore, the courts should not now apply an interpretation at variance
therewith. The rule of stare decisis is undoubtedly entitled to more
respect in the construction of statutes than the interpretations given by
officers of the administrative branches of the government, even those
entrusted with the administration of particular laws. But this court, in
Philippine Trust Company and Smith, Bell and Co. vs. Mitchell(59 Phil.,
30, 36), said:

. . . The rule of stare decisis is entitled to respect. Stability in


the law, particularly in the business field, is desirable. But
idolatrous reverence for precedent, simply as precedent, no
longer rules. More important than anything else is that court
should be right. . . .

III. In the view we take of the case, and after the disposition made
above of the first assignment of error, it becomes unnecessary to make
any specific ruling on the third, fourth, fifth, sixth, and seventh
assignments of error, all of which are necessarily disposed of
adversely to appellant's contention.

Wherefore, he judgment appealed from is affirmed, with costs of both


instances against appellant. So ordered.
70
G.R. No. L-9687 June 30, 1961 At the annual meeting of stockholders of Liddell & Co. held
on March 9, 1948, a 100% stock dividend was declared,
thereby increasing the issued capital stock of aid corporation
LIDDELL & CO., INC., petitioner-appellant,
from P1,000.000 to P 3,000,000 which increase was duly
vs.
approved by the Securities and Exchange Commission on
THE COLLECTOR OF INTERNAL REVENUE, respondent-appellee.
June 7, 1948. Frank Liddell subscribed to and paid 20% of
the increase of P400,000. He paid 25% thereof in the
Ozaeta, Lichauco and Picazo for petitioner-appellant. amount of P100,000 and the balance of P3,000,000 was
Office of the Solicitor General for respondent-appellee. merely debited to Frank Liddell-Drawing Account and
credited to Subscribed Capital Stock on December 11, 1948.
BENGZON, C.J.:
On March 8, 1949, stock dividends were again issued by
Liddell & Co. and in accordance with the agreements,
Statement. This is an appeal from the decision of the Court of Tax
Exhibits A, B, and C, the stocks of said company stood as
Appeals imposing a tax deficiency liability of P1,317,629.61 on Liddell follows:
& Co., Inc.

Said Company lists down several issues which may be boiled to the No. of
Name Amount Per Cent
following: Shares
Frank Liddell 13,688 P1,368,800 72.00
(a) Whether or not Judge Umali of the Tax Court below could Irene Liddell 1 100 .01
validly participate in the making of the decision;
Mercedes Vecin 1 100 .01

(b) Whether or not Liddell & Co. Inc., and the Liddell Motors, Charles Kurz 1,225 122,500 6.45
Inc. are (practically) identical corporations, the latter being E.J. Darras 1,225 122,500 6.45
merely .the alter ego of the former;
Angel Manzano 1,150 115,000 6.06
Julian Serrano 710 71,000 3.74
(c) Whether or not, granting the identical nature of the
corporations, the assessment of tax liability, including the E. Hasim 500 50,000 2.64
surcharge thereon by the Court of Tax Appeals, is correct.
G. W. Kernot 500 50,000 2.64
19,000 P1,900,000 100.00
Undisputed Facts. The parties submitted a partial stipulation of facts,
each reserving the right to present additional evidence.
On November 15, 1948, in accordance with a resolution of a special
Said undisputed facts are substantially as follows: meeting of the Board of Directors of Liddell & Co., stock dividends
were again declared. As a result of said declaration and in accordance
with the agreements, Exhibits, A, B, and C, the stockholdings in the
The petitioner, Liddell & Co. Inc., (Liddell & Co. for short) is a company appeared to be:
domestic corporation establish in the Philippines on February
1, 1946, with an authorized capital of P100,000 divided into
1000 share at P100 each. Of this authorized capital, 196 No. of
Name Amount Per Cent
shares valued at P19,600 were subscribed and paid by Shares
Frank Liddell while the other four shares were in the name of
Frank Liddell 19,738 P1,973,800 65.791
Charles Kurz, E.J. Darras, Angel Manzano and Julian
Serrano at one shares each. Its purpose was to engage in Irene Liddell 1 100 .003
the business of importing and retailing Oldsmobile and
Mercedes Vecin 1 100 .003
Chevrolet passenger cars and GMC and Chevrolet trucks..
Charles Kurz 2,215 221,500 7.381
On January 31, 1947, with the limited paid-in capital of E.J. Darras 2,215 221,500 7.381
P20,000, Liddell & Co. was able to declare a 90% stock Angel Manzano 1,810 181,000 6.031
dividend after which declaration on, Frank Liddells holding in
the Company increased to 1,960 shares and the employees, Julian Serrano 1,700 170,000 5.670
Charles Kurz E.J. Darras, Angel Manzano and Julian E. Hasim 830 83,000 2.770
Serrano at 10 share each. The declaration of stock dividend
was followed by a resolution increasing the authorized G. W. Kernot 1,490 149,000 4.970
capital of the company to P1,000.000 which the Securities & 30,000 P3,000,000 100.000
Exchange Commission approved on March 3, 1947. Upon
such approval, Frank Liddell subscribed to 3,000 additional
shares, for which he paid into the corporation P300,000 so On the basis of the agreement Exhibit A, (May, 1947) "40%" of the
that he had in his own name 4,960 shares. earnings available for dividends accrued to Frank Liddell although at
the time of the execution of aid instrument, Frank Liddell owned all of
On May 24, 1957, Frank Liddell, on one hand and Messrs. the shares in said corporation. 45% accrued to the employees, parties
Kurz, Darras, Manzano and Serrano on the other, executed thereto; Kurz 12-1/2%; Darras 12-1/2%; A. Manzano 12-1/2% and
an agreement (Exhibit A) which was further supplemented by Julian Serrano 7-1/2%. The agreement Exhibit A was also made
two other agreements (Exhibits B and C) dated May 24, retroactive to 1946. Frank Liddell reserved the right to reapportion the
1947 and June 3, 1948, wherein Frank Liddell transferred 45% dividends pertaining to the employees in the future for the
(On June 7, 1948) to various employees of Liddell & Co. purpose of including such other faithful and efficient employees as he
shares of stock. may subsequently designate. (As a matter of fact, Frank Liddell did so
designate two additional employees namely: E. Hasim and G. W.
Kernot). It was for such inclusion of future faithful employees that
Exhibits B-1 and C were executed. As per Exhibit C, dated May 13,

71
1948, the 45% given by Frank Liddell to his employees was adopted, and now here repeats, the ruling that the mere participation of
reapportioned as follows: C. Kurz — 12,%; E. J. Darras — 12%; A. a judge in prior proceedings relating to the subject in the capacity of an
Manzano — l2%; J. Serrano — 3-1/2%; G. W. Kernot — 2%. administrative official does not necessarily disqualify him from acting
as judge.2
Exhibit B contains the employees' definition in detail of the manner by
which they sought to prevent their share-holdings from being Appellant also contends that Judge Umali signed the said decision
transferred to others who may be complete strangers to the business contrary to the provision of Section 13, Republic Act No. 1125;3 that
on Liddell & Co. whereas the case was submitted for decision of the Court of Tax
Appeals on July 12, 1955, and the decision of Associate Judge
Luciano and Judge Nable were both signed on August 11, 1955 (that
From 1946 until November 22, 1948 when the purpose clause of the
is, on the last day of the 30-day period provided for in Section 13,
Articles of Incorporation of Liddell & Co. Inc., was amended so as to
Republic Act No. 1125), Judge Umali signed the decision August 31,
limit its business activities to importations of automobiles and trucks,
1955 or 20 days after the lapse of the 30-day period allotted by law.
Liddell & Co. was engaged in business as an importer and at the same
time retailer of Oldsmobile and Chevrolet passenger cars and GMC
and Chevrolet trucks. By analogy it may be said that inasmuch as in Republic Act No. 1125
(law creating the Court of Tax Appeals) like the law governing the
procedure in the court of Industrial Relations, there is no provision
On December 20, 1948, the Liddell Motors, Inc. was organized and
invalidating decisions rendered after the lapse of 30 days, the
registered with the Securities and Exchange Commission with an
requirement of Section 13, Republic Act No. 1125 should be construed
authorized capital stock of P100,000 of which P20,000 was subscribed
as directory.4
and paid for as follows: Irene Liddell wife of Frank Liddell 19,996
shares and Messrs. Marcial P. Lichauco, E. K. Bromwell, V. E. del
Rosario and Esmenia Silva, 1 share each. Besides as pointed out by appellee, the third paragraph of Section 13
of Republic Act No. 1125 (quoted in the margin)5 confirms this view;
because in providing for two thirty-day periods, the law means that
At about the end of the year 1948, Messrs. Manzano, Kurz and Kernot
decision may still be rendered within the second period of thirty days
resigned from their respective positions in the Retail Dept. of Liddell &
(Judge Umali signed his decision within that period).
Co. and they were taken in and employed by Liddell Motors, Inc.: Kurz
as Manager-Treasurer, Manzano as General Sales Manager for cars
and Kernot as General Sales Manager for trucks. B. Identity of the two corporations: On the question whether or not
Liddell Motors, Inc. is the alter ego of Liddell & Co. Inc., we are fully
convinced that Liddell & Co. is wholly owned by Frank Liddell. As of the
Beginning January, 1949, Liddell & Co. stopped retailing cars and
time of its organization, 98% of the capital stock belonged to Frank
trucks; it conveyed them instead to Liddell Motors, Inc. which in turn
Liddell. The 20% paid-up subscription with which the company began
sold the vehicles to the public with a steep mark-up. Since then, Liddell
its business was paid by him. The subsequent subscriptions to the
& Co. paid sales taxes on the basis of its sales to Liddell Motors Inc.
capital stock were made by him and paid with his own money.
considering said sales as its original sales.

These stipulations and conditions appear in Exhibit A: (1) that Frank


Upon review of the transactions between Liddell & Co. and Liddell
Liddell had the authority to designate in the future the employee who
Motors, Inc. the Collector of Internal Revenue determined that the latter
could receive earnings of the corporation; to apportion among the stock
was but an alter ego of Liddell & Co. Wherefore, he concluded, that for
holders the share in the profits; (2) that all certificates of stock in the
sales tax purposes, those sales made by Liddell Motors, Inc. to the
names of the employees should be deposited with Frank Liddell duly
public were considered as the original sales of Liddell & Co.
indorsed in blank by the employees concerned; (3) that each employee
Accordingly, the Collector of Internal Revenue assessed against Liddell
was required to sign an agreement with the corporation to the effect
& Co. a sales tax deficiency, including surcharges, in the amount of
that, upon his death or upon his retirement or separation for any cause
P1,317,629.61. In the computation, the gross selling price of Liddell
whatsoever from the corporation, the said corporation should, within a
Motors, Inc. to the general public from January 1, 1949 to September
period of sixty days therefor, have the absolute and exclusive option to
15, 1950, was made the basis without deducting from the selling price,
purchase and acquire the whole of the stock interest of the employees
the taxes already paid by Liddell & Co. in its sales to the Liddell Motors
so dying, resigning, retiring or separating.
Inc.

These stipulations in our opinion attest to the fact that Frank Liddell
The Court of Tax Appeals upheld the position taken by the Collector of
also owned it. He supplied the original his complete control over the
Internal Revenue.
corporation.

A. Judge Umali: Appellant urges the disqualification on of Judge


As to Liddell Motors, Inc. we are fully persuaded that Frank Liddell also
Roman M. Umali to participate in the decision of the instant case
owned it. He supplied the original capital funds.6 It is not proven that
because he was Chief of the Law Division, then Acting Deputy
his wife Irene, ostensibly the sole incorporator of Liddell Motors, Inc.
Collector and later Chief Counsel of the Bureau of Internal Revenue
had money of her own to pay for her P20,000 initial subscription.7 Her
during the time when the assessment in question was made.1 In
income in the United States in the years 1943 and 1944 and the
refusing to disqualify himself despite admission that had held the
savings therefrom could not be enough to cover the amount of
aforementioned offices, Judge Umali stated that he had not in any way
subscription, much less to operate an expensive trade like the retail of
participated, nor expressed any definite opinion, on any question
motor vehicles. The alleged sale of her property in Oregon might have
raised by the parties when this case was presented for resolution
been true, but the money received therefrom was never shown to have
before the said bureau. Furthermore, after careful inspection of the
been saved or deposited so as to be still available at the time of the
records of the Bureau, he (Judge Umali as well as the other members
organization of the Liddell Motors, Inc.
of the court below), had not found any indication that he had expressed
any opinion or made any decision that would tend to disqualify him
from participating in the consideration of the case in the Tax Court. The evidence at hand also shows that Irene Liddell had scant
participation in the affairs of Liddell Motors, Inc. She could hardly be
said to possess business experience. The income tax forms record no
At this juncture, it is well to consider that petitioner did not question the
independent income of her own. As a matter of fact, the checks that
truth of Judge Umali's statements. In view thereof, this Tribunal is not
represented her salary and bonus from Liddell Motors, Inc. found their
inclined to disqualify said judge. Moreover, in furtherance of the
way into the personal account of Frank Liddell. Her frequent absences
presumption of the judge's moral sense of responsibility this Court has

72
from the country negate any active participation in the affairs of the intended only as a blind, the corporate form may be ignored for the law
Motors company. cannot countenance a form that is bald and a mischievous fiction."

There are quite a series of conspicuous circumstances that militate Consistently with this view, the United States Supreme Court14 held
against the separate and distinct personality of Liddell Motors, Inc. that "a taxpayer may gain advantage of doing business thru a
from Liddell & Co.8 We notice that the bulk of the business of Liddell & corporation if he pleases, but the revenue officers in proper cases, may
Co. was channeled through Liddell Motors, Inc. On the other hand, disregard the separate corporate entity where it serves but as a shield
Liddell Motors, Inc. pursued no activities except to secure cars, trucks, for tax evasion and treat the person who actually may take the benefits
and spare parts from Liddell & Co. Inc. and then sell them to the of the transactions as the person accordingly taxable."
general public. These sales of vehicles by Liddell & Co. to Liddell
Motors, Inc. for the most part were shown to have taken place on the
Thus, we repeat: to allow a taxpayer to deny tax liability on the ground
same day that Liddell Motors, Inc. sold such vehicles to the public. We
that the sales were made through an other and distinct corporation
may even say that the cars and trucks merely touched the hands of
when it is proved that the latter is virtually owned by the former or that
Liddell Motors, Inc. as a matter of formality.
they are practically one and the same is to sanction a circumvention of
our tax laws.15
During the first six months of 1949, Liddell & Co. issued ten (10)
checks payable to Frank Liddell which were deposited by Frank Liddell
C. Tax liability computation: In the Yutivo case16 the same question
in his personal account with the Philippine National Bank. During this
involving the computation of the alleged deficiency sales tax has been
time also, he issued in favor of Liddell Motors, Inc. six (6) checks
raised. In accordance with our ruling in said case we hold as correctly
drawn against his personal account with the same bank. The checks
stated by Judge Nable in his concurring and dissenting opinion on this
issued by Frank Liddell to the Liddell Motors, Inc. were significantly for
case, that the deficiency sales tax should be based on the selling price
the most part issued on the same day when Liddell & Co. Inc. issued
obtained by Liddell Motors, Inc. to the public AFTER DEDUCTING
the checks for Frank Liddell9 and for the same amounts.
THE TAX ALREADY PAID BY LIDDELL & CO., INC. in its sales to
Liddell Motors, Inc.
It is of course accepted that the mere fact that one or more
corporations are owned and controlled by a single stockholder is not of
On the imposition of the 50% surcharge by reason of fraud, we see
itself sufficient ground for disregarding separate corporate entities.
that the transactions between Liddell Motors Inc. and Liddell & Co., Inc.
Authorities10 support the rule that it is lawful to obtain a corporation
have always been embodied in proper documents, constantly subject
charter, even with a single substantial stockholder, to engage in a
to inspection by the tax authorities. Liddell & Co., Inc. have always
specific activity, and such activity may co-exist with other private
made a full report of its income and receipts in its income tax returns.
activities of the stockholder. If the corporation is a substantial one,
conducted lawfully and without fraud on another, its separate identity is
to be respected. Paraphrasing our decision in the Yutivo case, we may now say, in filing
its return on the basis of its sales to Liddell Motors, Inc. and not on
those by the latter to the public, it cannot be held that the Liddell & Co.,
Accordingly, the mere fact that Liddell & Co. and Liddell Motors, Inc.
Inc. deliberately made a false return for the purpose of defrauding the
are corporations owned and controlled by Frank Liddell directly or
government of its revenue, and should suffer a 50% surcharge. But
indirectly is not by itself sufficient to justify the disregard of the separate
penalty for late payment (25%) should be imposed.
corporate identity of one from the other. There is, however, in this
instant case, a peculiar consequence of the organization and activities
of Liddell Motors, Inc. In view of the foregoing, the decision appealed from is hereby
modified: Liddell & Co., Inc. is declared liable only for the amount of
P426,811.67 with 25% surcharge for late payment and 6% interest
Under the law in force at the time of its incorporation the sales tax on
thereon from the time the judgment becomes final.
original sales of cars (sections 184, 185 and 186 of the National
Internal Revenue Code), was progressive, i.e. 10% of the selling price
of the car if it did not exceed P5000, and 15% of the price if more than As it appears that, during the pendency of this litigation appellant paid
P5000 but not more than P7000, etc. This progressive rate of the sales under protest to the Government the total amount assessed by the
tax naturally would tempt the taxpayer to employ a way of reducing the Collector, the latter is hereby required to return the excess to the
price of the first sale. And Liddell Motors, Inc. was the medium created petitioner. No costs.
by Liddell & Co. to reduce the price and the tax liability.

Let us illustrate: a car with engine motor No. 212381 was sold by
Liddell & Co. Inc. to Liddell Motors, Inc. on January 17, 1948 for
P4,546,000.00 including tax; the price of the car was P4,133,000.23,
the tax paid being P413.22, at 10%. And when this car was later sold
(on the same day) by Liddell Motors, Inc. to P.V. Luistro for P5500, no
more sales tax was paid.11 In this price of P5500 was included the
P413.32 representing taxes paid by Liddell & Co. Inc. in the sale to
Liddell Motors, Inc. Deducting P413.32 representing taxes paid by
Liddell & Co., Inc. the price of P5500, the balance of P5,087.68 would
have been the net selling price of Liddell & Co., Inc. to the general
public (had Liddell Motors, Inc. not participated and intervened in the
sale), and 15% sales tax would have been due. In this transaction,
P349.68 in the form of taxes was evaded. All the other transactions
(numerous) examined in this light will inevitably reveal that the
Government coffers had been deprived of a sizeable amount of taxes.

As opined in the case of Gregory v. Helvering,12 "the legal right of a


taxpayer to decrease the amount of what otherwise would be his taxes,
or altogether avoid them by means which the law permits, cannot be
doubted." But, as held in another case,13 "where a corporation is a
dummy, is unreal or a sham and serves no business purpose and is

73
G.R. No. L-5677 May 25, 1953 2. That the workers of the "La Campana Coffee Factory,
Inc." are less than thirty-one;
LA CAMPANA FACTORY, INC., and TAN TONG doing business
under the trial name "LA CAMPANA GAUGAU 3. That the petitioning union has no legal capacity to sue,
PACKING", petitioners, because its registration as an organized union has been
vs. revoked by the Department of Labor on September 5, 1951;
KAISAHAN NG MGA MANGGAGAWA SA LA CAMPANA (KKM) and
and THE COURT OF INDUSTRIAL RELATIONS, respondents.
4. That there is an existing valid contract between the
Ceferino de los Santos, R., Ceferino de los Santos, Jr. and Manuel V. respondent "La Campana Gaugau Packing" and the
Roxas for petitioners. intervenor PLOW, where-in the petitioner's members are
Carlos E. Santiago for respondent union. contracting parties bound by said contract.

REYES, J.: Several hearings were held on the above motions, in the course of
which ocular inspections were also made, and on the basis of the
evidence received and the facts observed in the ocular inspections, the
Tan Tong, one of the herein petitioners, has since 1932 been engaged
Court of Industrial Relations denied the said motions in its order of
in the business of buying and selling gaugau under the trade name La
January 14, 1952, because if found as a fact that:
Campana Gaugau Packing with an establishment in Binondo, Manila,
which was later transferred to España Extension, Quezon City. But on
July 6, 1950, Tan Tong, with himself and members of his family A. While the coffee corporation is a family corporation with
corporation known as La Campana Factory Co., Inc., with its principal Mr. Tan Tong, his wife, and children as the incorporations
office located in the same place as that of La Campana Gaugau and stockhelders (Exhibit 1), the La Campana Gaugau
Packing. Packing is merely a business name (Exhibit 4).

About a year before the formation of the corporation, or on July 11, B. According to the contract of lease (Exhibit 23), Mr. Tan
1949, Tan Tong had entered into a collective bargaining agreement Tong., propriety and manager of the Ka Campana Gaugau
with the Philippine Legion of Organized Workers, known as PLOW for Factory, leased a space of 200 square meters in the bodega
short, to which the union of Tan Tong's employees headed by Manuel housing the gaugau factory to his son Tan Keng Lim,
E. Sadde was then affiliated. Seceding, however, from the PLOW, Tan manager of the La Campana Coffee Factory. But the lease
Tong's employees later formed their own organization known was executed only on September 1, 1951, while the dispute
as Kaisahan Ng Mga Manggagawa Sa La Campana, one of the herein between the parties was pending before the Court.
respondents, and applied for registration in the Department of Labor as
an independent entity. Pending consideration of this application, the
C. There is only one entity La Campana Starch and Coffee
Department gave the new organization legal standing by issuing it a
Factory, as shown by the signboard (Exhibit 1), the
permit as an affiliate to the Kalipunan Ng Mga Manggagawa.
advertisement in the delivery trucks (Exhibit I-1), the
packages of gaugau(Exhibit K), and delivery forms (Exhibits
On July 19, 1951, the Kaisahan Ng Mga Manggagawa Sa La J, J-1, and J-2).
Campana, hereinafter to be referred to as the respondent Kaisahan,
which, as of that date, counted with 66 members — workers all of them
D. All the laborers working in the gaugau or in the coffee
of both La Campana Gaugau Packing and La Campana Coffee Factory
factory receive their pay from the same person, the cashier,
Co., Inc. — presented a demand for higher wages and more privileges,
Miss Natividad Garcia, secretary of Mr. Tan Tong; and they
the demand being addressed to La Campana Starch and Coffee
are transferred from the gaugau to the coffee and vice-versa
Factory, by which name they sought to designate, so it appears, the La
as the management so requires.
Campana Gaugau Packing and the La Campana Coffee Factory Co.,
Inc. As the demand was not granted and an attempt at settlement
through the mediation of the Conciliation Service of the Department of E. There has been only one payroll for the entire La
Labor had given no result, the said Department certified the dispute to Campana personnel and only one person preparing the
the Court of Industrial Relations on July 17, 1951, the case being there same — Miss Natividad Garcia, secretary of Mr. Tan Tong.
docketed as Case No. 584-V. But after the case at bar was certified to this Court on July
17, 1951, the company began making separate payrolls for
the coffee factory (Exhibits M-2 and M-3, and for the gaugau
With the case already pending in the industrial court, the Secretary of
factory (Exhibits O-2, O-3 and O-4). It is to be noted that
Labor, on September 5, 1951, revoked the Kalipunan Ng Mga
before July 21, 1951, the coffee payrolls all began with
Kaisahang Manggagawa's permit as a labor union on the strength of
number "41-Maria Villanueva" with 24 or more laborers
information received that it was dominated by subversive elements,
(Exhibits M and M-1), whereas beginning July 21, 1951, the
and, in consequence, on the 20th of the same month, also suspended
payrolls for the coffee factory began with No. 1-Loreta
the permit of its affiliate, the respondent Kaisahan.
Bernabe with only 14 laborers (Exhibits M-2 and M-3).

We have it from the court's order of January 15, 1952, which forms one
F. During the ocular inspection made in the factory on
of the annexes to the present petition, that following the revocation of
August 26, 1951 the Court has found the following:
the Kaisahan's permit, "La Campana Gaugau and Coffee Factory"
(obviously the combined name of La Campana Gaugau Packing and
La Campana Coffee Factory Co., Inc,) and the PLOW, which had been In the ground floor and second floor of the gaugau factory
allowed to intervene as a party having an interest in the dispute, filed there were hundreds of bags of raw coffee behind the pile of
separate motions for the dismissal of the case on the following gaugau sacks. There were also women employees working
grounds: paper wrappers for gaugau, and, in the same place there
were about 3,000 cans to be used as containers for coffee.
1. That the action is directed against two different entities
with distinct personalities, with "La Campana Starch Factory" The Court found out also that there were 16 trucks used both
and the "La Campana Coffee Factory, Inc."; for the delivery of coffee and gaugau. To show that those
trucks carried both coffee and gaugau, the union president

74
invited the Court to examine the contents of delivery truck . . . A subsidiary or auxiliary corporation which is created by
No. T-582 parked in a garage between the gaugau building a parent corporation merely as an agency for the latter may
and the coffee factory, and upon examination, there were sometimes be regarded as identical with the parent
found inside the said truck boxes of gaugau and cans of corporation, especially if the stockholders or officers of the
coffee, two corporations are substantially the same or their system
of operation unified. (Ibid. 162; see Annotation 1 A. L. R.
612, s. 34 A. L. R. 599.)
and held that:

In the present case Tan Tong appears to be the owner of


. . . there is only one management for the business of
the gaugau factory. And the coffee factory, though an incorporated
gaugau and coffee with whom the laborers are dealing
business, is in reality owned exclusively by Tan Tong and his family.
regarding their work. Hence, the filing of action against the
As found by the Court of industrial Relations, the two factories have but
Ka Campana Starch and Coffee Factory is proper and
one office, one management and one payroll, except after July 17, the
justified.
day the case was certified to the Court of Industrial Relations, when the
person who was discharging the office of cashier for both branches of
With regards to the alleged lack of personality, it is to be the business began preparing separate payrolls for the two. And above
noted that before the certification of the case to this Court on all, it should not be overlooked that, as also found by the industrial
July 17, 1951, the petitioner Kaisahan Ng Mga Manggagawa court, the laborers of the gaugau factory and the coffee factory were
Sa La Campana, had a separate permit from the Department interchangeable, that is, the laborers from the gaugau factory were
of Labor. This permit was suspended on September 30, sometimes transferred to the coffee factory and vice-versa. In view of
1951. (Exhibit M-Intervenor, page 55, of the record). It is not all these, the attempt to make the two factories appears as two
true that, on July 17, 1951, when this case forwarded to this separate businesses, when in reality they are but one, is but a device
Court, the petitioner's permit, as an independent union, had to defeat the ends of the law (the Act governing capital and labor
not yet been issued, for the very Exhibit MM-Intervenor relations) and should not be permitted to prevail.
regarding the permit, conclusively shows the preexistence of
said permit. (Annex G.)
The second point raised by petitioners is likewise with-out merit. In the
first place, there being more than 30 laborers involved and the
Their motion for reconsideration of the above order having been Secretary of Labor having certified the dispute to the Court of Industrial
denied, Tan Tong and La Campana Coffee Factory, Inc. (same as La Relations, that court duly acquired jurisdiction over the case
Campana Coffee Factory Co., Inc.), later joined by the PLOW, filed the (International Oil Factory vs. NLU, Inc. 73 Phil., 401; section 4, C. A.
present petition for certiorari on the grounds that the Court of Industrial 103). This jurisdiction was not when the Department of Labor
Relations had no jurisdiction to take cognizance of the case, for the suspended the permit of the respondent Kaisahan as a labor
reason, according to them, "(1) that the petitioner La Campana Coffee organization. For once jurisdiction is acquired by the Court of Industrial
Factory, Inc. has only 14 employees, only 5 of whom are members of Relations it is retained until the case is completely decided. (Manila
the respondent union and therefore the absence of the jurisdictional Hotel Employees Association vs. Manila Hotel Co. et al., 73 Phil., 374.)
number (30) as provided by sections 1 and 4 of Commonwealth Act
No. 103; and, (2) that the suspension of respondent union's permit by
In view of the foregoing, the petition is denied, with costs against the
the Secretary of Labor has the effect of taking away the union's right to
petitioner.
collective bargaining under section 2 of Commonwealth Act No. 213
and consequently, its personality to sue for ad in behalf of its
members."

As to the first ground, petitioners obviously do not question the fact that
the number of employees of the La Campana Gaugau Packing
involved in the case is more than the jurisdictional number (31)
required bylaw, but they do contend that the industrial court has no
jurisdiction to try the case as against La Campana Coffee Factory, Inc.
because the latter has allegedly only 14 laborers and only of these are
members of the respondent Kaisahan. This contention loses force
when it is noted that, as found by the industrial court — and this finding
is conclusive upon us — La Campana Gaugau Packing and La
Campana Coffee Factory Co. Inc., are operating under one single
management, that is, as one business though with two trade names.
True, the coffee factory is a corporation and, by legal fiction, an entity
existing separate and apart fro the persons composing it, that is, Tan
Tong and his family. But it is settled that this fiction of law, which has
been introduced as a matter of convenience and to subserve the ends
of justice cannot be invoked to further an end subversive of that
purpose.

Disregarding Corporate Entity. — The doctrine that a


corporation is a legal entity existing separate and apart from
the person composing it is a legal theory introduced for
purposes of convenience and to subserve the ends of
justice. The concept cannot, therefore, be extended to a
point beyond its reason and policy, and when invoked in
support of an end subversive of this policy, will be
disregarded by the courts. Thus, in an appropriate case and
in furtherance of the ends of justice, a corporation and the
individual or individuals owning all its stocks and assets will
be treated as identical, the corporate entity being
disregarded where used as a cloak or cover for fraud or
illegality. (13 Am. Jur., 160-161.)
75
G.R. No. 182770 September 17, 2014 In her prayer, the respondent sought indemnification in the amount of
₱112,876.60 plus interest at 12%per annum from June 18, 1990 until
fully paid; and 20% of the award as attorney’s fees. She likewise
WPM INTERNATIONAL TRADING, INC. and WARLITO P.
prayed that an award of ₱100,000.00 as moral damages and
MANLAPAZ, Petitioners,
₱20,000.00 as attorney’s fees be paid to her.
vs.
FE CORAZON LABAYEN, Respondent.
In his defense, Manlapaz claims that it was his fellow
incorporator/director Edgar Alcansajewho was in-charge with the daily
DECISION
operations of the Quickbite outlets; that when Alcansaje left WPM, the
remaining directors were compelled to hire the respondent as
BRION, J.: manager; that the respondent had entered intothe renovation
agreement with CLN in her own personal capacity; that when he found
the amount quoted by CLN too high, he instructed the respondent to
We review in this petition for review on certiorari1 the decision2 dated
either renegotiate for a lower price or to look for another contractor;
September 28, 2007 and the resolution3 dated April 28, 2008 of the that since the respondent had exceeded her authority as agent of
Court of Appeals (CA) in CA-G.R. CV No. 68289 that affirmed with WPM, the renovation agreement should only bind her; and that since
modification the decision4 of the Regional Trial Court (RTC), Branch
WPM has a separate and distinct personality, Manlapaz cannot be
77, Quezon City. made liable for the respondent’s claim.

The Factual Background


Manlapaz prayed for the dismissal of the complaint for lack of cause of
action, and by way of counterclaim, for the award of ₱350,000.00 as
The respondent, Fe Corazon Labayen, is the owner of H.B.O. Systems moral and exemplary damages and ₱50,000.00 attorney’s fees.
Consultants, a management and consultant firm. The petitioner, WPM
International Trading, Inc. (WPM), is a domestic corporation engaged The RTC, through an order dated March 2, 1993 declared WPM in
in the restaurant business, while Warlito P. Manlapaz (Manlapaz) is its default for its failure to file a responsive pleading.
president.

The Decision of the RTC


Sometime in 1990, WPM entered into a management agreement with
the respondent, by virtue of which the respondent was authorized to
operate, manage and rehabilitate Quickbite, a restaurant owned and In its decision, the RTC held that the respondent is entitled to
operated by WPM. As part of her tasks, the respondent looked for a indemnity from Manlapaz. The RTC found that based on the records,
contractor who would renovate the two existing Quickbite outlets in there is a clear indication that WPM is a mere instrumentality or
Divisoria, Manila and Lepanto St., University Belt, Manila. Pursuant to business conduit of Manlapaz and as such, WPM and Manlapaz are
the agreement, the respondent engaged the services of CLN considered one and the same. The RTC also found that Manlapaz had
Engineering Services (CLN) to renovate Quickbite-Divisoria at the cost complete control over WPM considering that he is its chairman,
of ₱432,876.02. president and treasurer at the same time. The RTC thus concluded
that Manlapaz is liable in his personal capacity to reimburse the
respondent the amount she paid to CLN inconnection with the
On June 13, 1990, Quickbite-Divisoria’s renovation was finally renovation agreement.
completed, and its possession was delivered to the respondent.
However, out of the ₱432,876.02 renovation cost, only the amount of
₱320,000.00 was paid to CLN, leaving a balance of ₱112,876.02. The petitioners appealed the RTC decision with the CA. There, they
argued that in view of the respondent’s act of entering into a renovation
agreement with CLN in excess of her authority as WPM’s agent, she is
Complaint for Sum of Money (Civil Case No. Q-90-7013)
not entitled to indemnity for the amount she paid. Manlapaz also
contended that by virtue ofWPM’s separate and distinct personality, he
On October 19, 1990, CLN filed a complaint for sum of money and cannot be madesolidarily liable with WPM.
damages before the RTC against the respondent and Manlapaz, which
was docketed as Civil Case No. Q-90-7013. CLN later amended the The Ruling of the Court of Appeals
complaint to exclude Manlapaz as defendant. The respondent was
declared in default for her failure to file a responsive pleading.
On September 28, 2007, the CA affirmed, with modification on the
award of attorney’s fees, the decision of the RTC.The CA held that the
The RTC, in its January 28, 1991 decision, found the respondent liable petitioners are barred from raising as a defense the respondent’s
to pay CLN actual damages inthe amount of ₱112,876.02 with 12%
alleged lack of authority to enter into the renovation agreement in view
interest per annum from June 18,1990 (the date of first demand) and of their tacit ratification of the contract.
20% of the amount recoverable as attorney’s fees.

The CA likewise affirmed the RTC ruling that WPM and Manlapaz are
Complaint for Damages (Civil Case No. Q-92-13446) one and the same based on the following: (1) Manlapaz is the principal
stockholder of WPM; (2) Manlapaz had complete control over WPM
Thereafter, the respondent instituted a complaint for damages against because he concurrently held the positions of president, chairman of
the petitioners, WPM and Manlapaz. The respondent alleged that in the board and treasurer, in violation of the Corporation Code; (3) two of
Civil Case No. Q-90-7013, she was adjudged liable for a contract that the four other stockholders of WPM are employed by Manlapaz either
she entered into for and in behalf of the petitioners, to which she directly or indirectly; (4) Manlapaz’s residence is the registered
should be entitled to reimbursement; that her participation in the principal office of WPM; and (5) the acronym "WPM" was derived from
management agreement was limited only to introducing Manlapaz to Manlapaz’s initials. The CA applied the principle of piercing the veil of
Engineer Carmelo Neri (Neri), CLN’s general manager; that it was corporate fiction and agreed with the RTC that Manlapaz cannot evade
actually Manlapaz and Neri who agreed on the terms and conditions of his liability by simply invoking WPM’s separate and distinct personality.
the agreement; that when the complaint for damages was filed against
her, she was abroad; and that she did not know of the case until she
After the CA's denial of their motion for reconsideration, the petitioners
returned to the Philippines and received a copy of the decision of the filed the present petition for review on certiorari under Rule 45 of the
RTC. Rules of Court.

76
The Petition Piercing the corporate veil based on the alter ego theory requires the
concurrence of three elements, namely:
The petitioners submit that the CA gravely erred in sustaining the
RTC’s application of the principle of piercing the veil of corporate (1) Control, not mere majority or complete stock control, but
fiction. They argue that the legal fiction of corporate personality could complete domination, not only of finances but of policy and
only be discarded upon clear and convincing proof that the corporation business practice in respect to the transaction attacked so
is being used as a shield to avoid liability or to commit a fraud. Since that the corporate entity as to this transaction had at the time
the respondent failed to establish that any of the circumstances that no separate mind, will or existence of its own;
would warrant the piercing is present, Manlapaz claims that he cannot
be made solidarily liable with WPM to answerfor damages allegedly
(2) Such control must have beenused by the defendant to
incurred by the respondent.
commit fraud or wrong, to perpetuate the violation of a
statutory or other positive legal duty, or dishonest and unjust
The petitioners further argue that, assuming they may be held liable to act in contravention of plaintiff’s legal right; and
reimburse to the respondentthe amount she paid in Civil Case No. Q-
90-7013, such liability is only limited to the amount of ₱112,876.02,
(3) The aforesaid control and breach of duty must have
representing the balance of the obligation to CLN, and should not
proximately caused the injury or unjust loss complained of.
include the twelve 12% percent interest, damages and attorney’s fees.

The absence of any ofthese elements prevents piercing the corporate


The Issues
veil.12

The core issues are: (1) whether WPM is a mere instrumentality, alter-
In the present case, the attendantcircumstances do not establish that
ego, and business conduit of Manlapaz; and (2) whether Manlapaz is
WPM is a mere alter ego of Manlapaz.
jointly and severally liable with WPM to the respondent for
reimbursement, damages and interest.
Aside from the fact that Manlapaz was the principal stockholder of
WPM, records do not show that WPM was organized and controlled,
Our Ruling
and its affairs conducted in a manner that made it merely an
instrumentality, agency, conduit or adjunct ofManlapaz. As held in
We find merit in the petition. Martinez v. Court of Appeals,13 the mere ownership by a
singlestockholder of even all or nearly all of the capital stocks ofa
corporation is not by itself a sufficient ground to disregard the separate
We note, at the outset, that the question of whether a corporation is a
corporate personality. To disregard the separate juridical personality of
mere instrumentality or alter-ego of another is purely one of fact.5 This
a corporation, the wrongdoing must be clearly and convincingly
is also true with respect to the question of whether the totality of the
established.14
evidence adduced by the respondentwarrants the application of the
piercing the veil of corporate fiction doctrine.6
Likewise, the records of the case do not support the lower courts’
finding that Manlapaz had control or domination over WPM or its
Generally, factual findings of the lower courts are accorded the highest
finances. That Manlapaz concurrentlyheld the positions of president,
degree of respect, if not finality. When adopted and confirmed by the
chairman and treasurer, or that the Manlapaz’s residence is the
CA, these findings are final and conclusive and may not be reviewed
registered principal office of WPM, are insufficient considerations to
on appeal,7 save in some recognized exceptions8 among others, when
prove that he had exercised absolutecontrol over WPM.
the judgment is based on misapprehension of facts.

In this connection, we stress thatthe control necessary to invoke the


We have reviewed the records and found that the application of the
instrumentality or alter ego rule is not majority or even complete stock
principle of piercing the veil of corporate fiction is unwarranted in the
control but such domination of finances, policies and practices that the
present case.
controlled corporation has, so tospeak, no separate mind, will or
existence of its own, and is but a conduit for its principal. The control
On the Application ofthe Principle of Piercing the Veil of Corporate must be shown to have been exercised at the time the acts complained
Fiction of took place. Moreover, the control and breach of duty must
proximately cause the injury or unjust loss for which the complaint is
made.
The rule is settled that a corporation has a personality separate and
distinct from the persons acting for and in its behalf and, in general,
from the people comprising it.9 Following this principle, the obligations Here, the respondent failed to prove that Manlapaz, acting as
incurred by the corporate officers, orother persons acting as corporate president, had absolute control over WPM.1âwphi1 Even granting that
agents, are the direct accountabilities ofthe corporation they represent, he exercised a certain degree of control over the finances, policies and
and not theirs. Thus, a director, officer or employee of a corporation is practices of WPM, in view of his position as president, chairman and
generally not held personally liable for obligations incurred by the treasurer of the corporation, such control does not necessarily warrant
corporation;10 it is only in exceptional circumstances that solidary piercing the veil of corporate fiction since there was not a single proof
liability will attach to them. that WPM was formed to defraud CLN or the respondent, or that
Manlapaz was guilty of bad faith or fraud.
Incidentally, the doctrine of piercing the corporate veil applies only in
three (3) basic instances, namely: a) when the separate and distinct On the contrary, the evidence establishes that CLN and the respondent
corporate personality defeats public convenience, as when the knew and acted on the knowledgethat they were dealing with WPM for
corporate fiction is used as a vehicle for the evasion of an existing the renovation of the latter’s restaurant, and not with Manlapaz. That
obligation; b) in fraud cases, or when the corporate entity is used to WPM later reneged on its monetary obligation to CLN, resulting to the
justify a wrong, protect a fraud, or defend a crime; or c) is used in alter filing of a civil case for sum of money against the respondent, does not
ego cases, i.e., where a corporation is essentially a farce, since it is a automatically indicate fraud, in the absence of any proof to support it.
mere alter ego or business conduit of a person, or where the
corporation is so organized and controlled and its affairs so conducted
This Court also observed that the CA failed to demonstrate how the
as to make it merely aninstrumentality, agency, conduit or adjunct of
separate and distinct personalityof WPM was used by Manlapaz to
another corporation.11
7
defeat the respondent’s right for reimbursement. Neither was there any
showing that WPM attempted to avoid liability or had no property
against which to proceed.

Since no harm could be said to have been proximately caused by


Manlapaz for which the latter could be held solidarily liable with WPM,
and considering that there was no proof that WPM had insufficient
funds, there was no sufficient justification for the RTC and the CA to
have ruled that Manlapaz should be held jointly and severally liable to
the respondent for the amount she paid to CLN. Hence, only WPM is
liable to indemnify the respondent.

Finally, we emphasize that the piercing of the veil of corporate fiction is


frowned upon and thus, must be done with caution.15 It can only be
done if it has been clearly established that the separate and distinct
personality of the corporation is used to justify a wrong, protect fraud,
or perpetrate a deception. The court must be certain that the corporate
fiction was misused to such an extent that injustice, fraud, or crime was
committed against another, in disregard of its rights; it cannot be
presumed.

On the Award of Moral Damages

On the award of moral damages, we find the same in order in view of


WPM's unjustified refusal to pay a just debt. Under Article 2220 of the
New Civil Code,16 moral damages may be awarded in cases of a
breach of contract where the defendant acted fraudulently or in bad
faith or was guilty of gross negligence amounting to bad faith.

In the present case, when payment for the balance of the renovation
cost was demanded, WPM, instead of complying with its obligation,
denied having authorized the respondent to contract in its behalf and
accordingly refused to pay. Such cold refusal to pay a just debt
amounts to a breach of contract in bad faith, as contemplated by Article
2220. Hence, the CA's order to pay moral damages was in order.

WHEREFORE, in light of the foregoing, the decision dated September


28, 2007 of the Court of Appeals in CA-G.R. CV No. 68289 is
MODIFIED and.that petitioner Warlito P. Manlapaz is ABSOLVED from
any liability under the renovation agreement.

SO ORDERED.

78
G.R. No. L-13119 September 22, 1959 . . . There was only management for the business of gawgaw
and coffee with whom the laborers are dealing regarding
their work. Hence, the filing of action against the La
RICARDO TANTONGCO, petitioner,
Campana Starch and Coffee Factory is proper and
vs.
justified.1âwphïl.nêt
KAISAHAN NG MGA MANGGAGAWA SA LA CAMPAN (KKM) AND
THE HONORABLE COURT OF INDUSTRIAL
RELATIONS, respondents. The order of denial was appealed to this Tribunal
through certiorari under G.R. No. L-5677. In disposing of the case, we
held:
Ernesto C. Estrella for petition for petitioner.
Carlos E. Santiago for respondent Union.
Pedro M. Ligaya for respondent CIR. As to the first ground, petitioners obviously do not question
the fact that the number of employees of the La Campana
Gaugau Packing involved in the case is more than the
MONTEMAYOR, J.:
jurisdictional number (31) required by law, but they contend
that the industrial court has no jurisdiction to try case against
This is a petition for certiorari and prohibition with prayer for issuance La Campana Coffee Factory Co. Inc. because the latter has
of a writ of preliminary injunction to prohibit respondent Court of allegedly only 14 laborers and only five of these are
Industrial Relations from proceeding with the hearing of the contempt members of respondent Kaisahan. This contention loses
proceedings for which petitioner Ricardo Tantongco was cited to force when it is noted that, as found by the industrial court —
appear the present his evidence. The contempt proceedings which and this finding is conclusive upon us — La Campana
petitioner seeks to stop are based on the order of the Court of Gaugau Packing and La Campana Coffee Factory Co. Inc.,
Industrial Relations, dated September 30, 1957, which reads as are operating under one single management, that is, one
follows: business though with two trade names. True, the coffee
factory is a corporation , and, by legal fiction, an entity
existing separate and part from the persons composing it,
It appearing that the Order of this Court, in the above-entitled
that is, Tan Tong and his family. But is settled this fiction of
case, dated February 18, 1957 (folios 134-166), has become law, which has been introduced as a matter of convenience
final and executory and the respondents have failed to and to subserve the ends of justice cannot be invoke to
comply with the same, the said respondents, namely, the La further an end subversive of that purpose.
Campana Starch and Coffee Factory or its manager or the
person who has charge of the management, and the
administrator of the Estate of Ramon Tantongco are hereby ... The attempt to make the two factories appear as two
ordered to comply with said order, within five days from separate business, when in reality, they are but one is but a
receipt hereof, particularly the following, to wit: device to defeat the ends of the law (the Act governing
capital and labor relations) and should not be permitted to
prevail. (La Campana Coffee Factory, et al., vs. Kaisahan ng
(a) To reinstate the persons named in the said mga Manggagawa, etc. et al., 93 Phil., 160; 49 Off. Gaz., [6]
Order of February 18, 1957;
2300.)

(b) To deposit the amount of P65,534.01 with this Upon the return of the case to the Court of Industrial Relations, the
Court.
latter proceeded with the hearing. In the meantime incidental cases
involving the same parties came up and were filed before the Court of
With respect to possible back wages from August 28, 1957 Industrial Relations in the following cases:1âwphïl.nêt
as mentioned in the petition for contempt of August 30, 1957,
the same shall first be determined. Case No. 584-V(1) — petition for contempt against the La
Campana Starch and Coffee Factory for having employed 21
Failure to comply with this Order shall be directly dealt with new laborers in violation of the order of July 21, 1951, filed
accordingly. on July 25, 1951;

It would appear that petitioner Ricardo Tantongco failed to comply with Case No. 584-V(2) — petition of La Campana for authority
said order and so, as already stated, he was cited to appear and to for authority to dismiss Loreto Bernabe, filed on July 25,
adduce evidence on his behalf to show why he should not be punished 19651;
for indirect contempt.
Case No. 584-V(3) — petition of Union to reinstate Bonifacio
The facts in this case may be briefly narrated thus: Sometime in June, Calderon with backpay, filed on August 3, 1951;
1951, members of the Kaisahan ng mga Manggagawa sa La
Campana, a labor union to which were affiliated workers in the La Case No. 584-V(5) — petition of Union to reinstate Marcelo
Campana Starch Factory and La Campana Coffee Factory, two Estrada and Exequiel Rapiz with back pay and to punish
separate entities but under the one management, presented demands
officials of the company for contempt, filed on February 13,
for higher wages, and more privileges and benefits in connection with 1952; and
their work. When the management failed and refused to grant the
demands, the Department of Labor intervened; but failing to settle the
controversy, it certified the dispute to the Court of Industrial Relations Case No. 584-V(6) — petition of union for reinstatement of
on July 17, 1951, where it was docketed as Case No. 584–V. On the Ibardolaza and seven other member-laborers and to punish
theory that the laborers presenting the demands were only the ones the officers of the company for contempt, filed on July 15,
working in the coffee factory, said company filed through the 1953.
management a motion to dismiss claiming that inasmuch as there were
only 14 of them in said factory, the Court of Industrial Relations had no
These five cases were heard jointly. In the meantime Ramon
jurisdiction to entertain and decide the case. The motion was denied by
Tantongco supposed to be the owner and manager of the La Campana
the Court of Industrial Relations, which said:
Starch Factory and the person in charge of the La Campana Coffee
Factory died on May 16, 1956. On motion of the labor union, the Court
of Industrial Relations order the inclusion as party respondent of the
79
administrator of the estate of Ramon Tantongco who was Ricardo did not deprive the CIR of its jurisdiction over the cases
Tantongco. aforementioned. Moreover, the money claims of the laborers were
merely incidental to their demands for reinstatement for having been
unjustly dismissed, and for better working conditions.
Ricardo Tantongco, as administrator, under a special appearance filed
a motion to dismiss all the cases including the main case, that is to
say, Cases No. 584-(V) to 584-V(6), on the ground that said cases Petitioner, however, contends that in G.R. No. L-5677, we "pierced the
involved claims for sums of money and consequently should be filed veil of corporate existence", and held that the La Campana Starch and
before the probate court having jurisdiction over the estate, pursuant to Coffee Factory and its owner, Ramon Tantongco, were one; so that
the provisions of Rule 3, Section 21, and Rule 88, Section 1 of the with the death of Ramon, the La Campana entities ceased to exist,
Rules of Court. On August 23, 1956, the Court of Industrial Relations resulting in the loss of jurisdiction of the CIR to enforce its order
denied the motion to dismiss and proceed to hear the incidental cases against said entities. The reason we applied the so-called "piercing the
against the La Campana entities. veil of corporate existence" in G.R. No. L-5677 was to avoid the
technicality therein advanced in order to defeat the jurisdiction of the
CIR. We there found that although there were ostensibly two separate
On June 12, 1956, a partial decision was rendered in the main case
companies or entities, they were managed by the same person or
No. 584-V, which partial decision was elevated to us and is still
persons and the workers in both were used interchangeably so that in
pending appeal. On February 18, 1957, the Court of Industrial
order to determine whether or not the CIR had jurisdiction, the number
Relations issued an order in incidental Cases No. 584-V(1), V(2), V(5)
of workers in both entitles, not in only one, was to be considered.
and V(6), directing the "management of the respondent company and
However, we still believe that although the family of Ramon Tantongco
or the administrator of the Estate of Ramon Tantongco", to reinstate
was practically the owner of both the coffee factory and the starch
the dismissed laborers mentioned therein with back wages. This order
factory, nevertheless these entities are separate from the personality of
of February 18, 1957, as well as the order directing the inclusion of the
Ramon. The coffee factory is a stock corporation and the shares are
administrator of the estate of Ramon Tantongco as additional
owned not only by Ramon but also by others, such as petitioner
respondent in the incidental cases, and the order denying the petition
Ricardo who not only is a stockholder and director and treasurer but
of the administrator to dismiss said incidental cases were appealed to
also the management of the same Furthermore, petitioner is now
this tribunal though certiorari. The appeal, however, was summarily
estopped from claiming that the two entities in question and Ramon are
dismissed by this Court in its resolution of June 12, 1957, as follows:
one. Thus in Annex 3-CIR (par. 1 thereof) which is a complaint for
injunction filed by La Campana Food Products, et al and La Campana
This Court, deliberating upon the allegations of the petition Starch Packing against the consolidated Labor Organization of the
filed in case l-12355 (La Campana Starch Coffee Factory et Philippines, in civil Case No. P-25482 in the Court of First Instance of
al. vs. Kaisahan ng Mga Manggagawa sa la Campana, KKM, Rizal, petitioner admitted the existence and operation of said entities;
et al) for review, on certiorari of the decision of the Court of in Annex 4—CIR where petitioner appeared as General Manager
Industrial Relations referred to therein, and finding that there representing the two entities in its agreement with the La Campana
is no merit in the petition, RESOLVE TO DISMISS the same. Workers Union to resolve the dispute between the two entities and the
laborers in case Nos. 1072-V and 1371-ULP, the existence of the two
entities appears to have been admitted; and in Annex 5-A-CIR, an
The CIR order of February 18, ,1957, in the incidental cases Nos. 584-
answer to the complaint of La Campana Workers Union in case No.
V to V(6), having become final and executory , the laborers involved 1471-ULP (Annex 5-CIR), petitioner admitted the allegation that said
reported for work on August 28, 1957, but they were not admitted by two factories were in existence and doing business with petitioner as
the management. Consequently, the union filed a petition dated August
manager of the same.
30, 1957, to hold respondents in said cases for contempt. After hearing
the CIR issued the order of September 30, 1957, subject of this
petition, ordering "the La Campana Starch and Coffee Factory or its In relation to the order of the CIR requiring petitioner to appear in the
manager or the person who has charge of its management and the contempt proceedings instituted against him, petitioner contends that
administrator of the estate of Ramon Tantongco" to "reinstate the after he ceased to be the administrator of the estate of Ramon
persons named in the order of February 18, 1957" and "to deposit the Tantongco, he may not now be compelled to comply with the order of
amount of P65,534.01." For refusal or failure to comply with said order, the court. In answer, it is enough to bear in mind the jurisdiction and
petitioner Ricardo Tantongco was required to appear before the authority of the CIR as to compliance with and violations of its orders
attorney of the CIR in contempt proceedings. Petitioner now seeks to under section 6, Commonwealth Act No. 143, which we quote below:
prohibit the CIR from proceeding with the trial for contempt and to
enjoin respondent CIR from enforcing its order of September 30, 1957.
. . . The Court or any Judge thereof shall have furthermore,
all the inherent powers of a court of justice provided in
Petitioner contends that upon the death of Ramon Tantongco, the paragraph 5 of Rule 124 of the Supreme Court, as well as
claims of the laborers should have been dismissed and that said claims the power to punish direct and indirect contempt as provided
should have been filed with the probate court having jurisdiction over in Rule 64 of the same Court, under the same procedure and
the administration proceedings of the estate of Ramon Tantongco, penalties provided therein.
pursuant to the provisions of Rule 3, Section 21 of the Rules of Court
and that the failure to file claims with the administrator forever barred
Any violation of any order, award, or decision of the Court of
said claims as provided in Rule 87, Section 5 of the Rules of Court,
Industrial Relations shall, after such order, award or decision
especially after the assets of the estate had been distributed among
has become final, conclusive, and executory, constitute
the heirs, and petitioner had ceased to be the administrator of the
contempt of court: . . .
estate. As already stated this same question was raised by petitioner in
G.R. No. L-12355, entitled "La Campana Starch and Coffee Factory
and Ricardo Tantongco, etc. vs. Kaisahan ng mga Manggagawa sa La In case the employer (or landlord) committing any such
Campana (KKM)," which, as already stated, was summarily dismissed violation or contempt is an association or corporation, the
by this Court in a resolution dated June 12, 1957. Consequently, said manager or the person who has the charge of the
question may not again be raised in the present case. Furthermore, it management of the business of the association or
may be recalled that both in the main case in the incidental cases No. corporation and the officers of directors thereof who have
584-V to 584-V(6), Ramon Tantongco was never a party. The party ordered or authorized the violation of contempt shall be
there was the La Campana Starch and Coffee Factory by which name liable. . . .
it was sought to designate the two entities La Campana Starch Packing
and the La Campana Coffee Factory. Naturally, the claims contained in
In conclusion, we find and hold that the La Campana Starch and Food
said cases were not the claims contemplated by law to be submitted
Products Company which stands for the La Campana Starch and
before the administrator. In other words the death of Ramon Tantongco
Coffee Factory are entities distinct from the personality of Ramon
80
Tantongco; that after the death of Ramon these two entities continued
to exist and to operate under the management of petitioner and that
consequently he is the proper person and official to which the orders of
the CIR are addressed and who is in duty bound to comply with the
same. We further find that the CIR acted with in its jurisdiction in
issuing its order of September 30, 1957 and in requiring petitioner to
appear to give his evidence if any in relation with the contempt
proceedings instituted against him.1âwphïl.nêt

In view of the foregoing, the petition for certiorari is .hereby denied and
the writ of preliminary injunction dissolved, with costs.

81
Adm. Matter No. R-181-P July 31, 1987 discharged employees and pay them full backwages. Respondent,
however, chose to "pierce the veil of corporate entity" usurping a power
belonging to the court and assumed improvidently that since the
ADELIO C. CRUZ, complainant,
complainant is the owner/president of Qualitrans Limousine Service,
vs.
Inc., they are one and the same. It is a well-settled doctrine both in law
QUITERIO L. DALISAY, Deputy Sheriff, RTC, Manila, respondents.
and in equity that as a legal entity, a corporation has a personality
distinct and separate from its individual stockholders or members. The
RESOLUTION mere fact that one is president of a corporation does not render the
property he owns or possesses the property of the corporation, since
the president, as individual, and the corporation are separate entities. 3

Anent the charge that respondent exceeded his territorial jurisdiction,


FERNAN, J.:
suffice it to say that the writ of execution sought to be implemented
was dated July 9, 1984, or prior to the issuance of Administrative
In a sworn complaint dated July 23, 1984, Adelio C. Cruz charged Circular No. 12 which restrains a sheriff from enforcing a court writ
Quiterio L. Dalisay, Senior Deputy Sheriff of Manila, with "malfeasance outside his territorial jurisdiction without first notifying in writing and
in office, corrupt practices and serious irregularities" allegedly seeking the assistance of the sheriff of the place where execution shall
committed as follows: take place.

1. Respondent sheriff attached and/or levied the money belonging to ACCORDINGLY, we find Respondent Deputy Sheriff Quiterio L.
complainant Cruz when he was not himself the judgment debtor in the Dalisay NEGLIGENT in the enforcement of the writ of execution in
final judgment of NLRC NCR Case No. 8-12389-91 sought to be NLRC Case-No. 8-12389-91, and a fine equivalent to three [3] months
enforced but rather the company known as "Qualitrans Limousine salary is hereby imposed with a stern warning that the commission of
Service, Inc.," a duly registered corporation; and, the same or similar offense in the future will merit a heavier penalty.
Let a copy of this Resolution be filed in the personal record of the
respondent.
2. Respondent likewise caused the service of the alias writ of execution
upon complainant who is a resident of Pasay City, despite knowledge
that his territorial jurisdiction covers Manila only and does not extend to SO ORDERED.
Pasay City.

In his Comments, respondent Dalisay explained that when he


garnished complainant's cash deposit at the Philtrust bank, he was
merely performing a ministerial duty. While it is true that said writ was
addressed to Qualitrans Limousine Service, Inc., yet it is also a fact
that complainant had executed an affidavit before the Pasay City
assistant fiscal stating that he is the owner/president of said
corporation and, because of that declaration, the counsel for the
plaintiff in the labor case advised him to serve notice of garnishment on
the Philtrust bank.

On November 12, 1984, this case was referred to the Executive Judge
of the Regional Trial Court of Manila for investigation, report and
recommendation.

Prior to the termination of the proceedings, however, complainant


executed an affidavit of desistance stating that he is no longer
interested in prosecuting the case against respondent Dalisay and that
it was just a "misunderstanding" between them. Upon respondent's
motion, the Executive Judge issued an order dated May 29, 1986
recommending the dismissal of the case.

It has been held that the desistance of complainant does not preclude
the taking of disciplinary action against respondent. Neither does it
dissuade the Court from imposing the appropriate corrective sanction.
One who holds a public position, especially an office directly connected
with the administration of justice and the execution of judgments, must
at all times be free from the appearance of impropriety.1

We hold that respondent's actuation in enforcing a judgment against


complainant who is not the judgment debtor in the case calls for
disciplinary action. Considering the ministerial nature of his duty in
enforcing writs of execution, what is incumbent upon him is to ensure
that only that portion of a decision ordained or decreed in the
dispositive part should be the subject of execution.2 No more, no less.
That the title of the case specifically names complainant as one of the
respondents is of no moment as execution must conform to that
directed in the dispositive portion and not in the title of the case.

The tenor of the NLRC judgment and the implementing writ is clear
enough. It directed Qualitrans Limousine Service, Inc. to reinstate the

82
G.R. No. 165442 August 25, 2010 execute a new CBA incorporating therein his dispositions regarding
benefits of the employees as to wage increase, productivity bonus,
vacation and sick leave, medical allowances and signing bonus.
NASECO GUARDS ASSOCIATION-PEMA (NAGA-PEMA), Petitioner,
Respondent was further ordered to negotiate, for purposes of collective
vs.
bargaining agreement, with NEMU-PEMA led by its president, Ligaya
NATIONAL SERVICE CORPORATION (NASECO), Respondent.
Valencia. The charge of unfair labor practice against respondent and
PNB was dismissed.14
DECISION
Respondent promptly filed a petition for certiorari before the CA
VILLARAMA, JR., J.: questioning the DOLE Secretary’s order and arguing that the ruling of
the DOLE Secretary in favor of the unions and awarding them
monetary benefits totaling five hundred thirty-one million four hundred
This petition for review on certiorari under Rule 45 assails the forty-six thousand six hundred sixty-six and 67/100 (₱531,446,666.67)
Decision1 dated May 27, 2004 of the Court of Appeals (CA) in CA-G.R.
was inimical and deleterious to its financial standing and will result in
SP No. 76667. The appellate court set aside the January 15, closure and cessation of business for the company.
20032 and March 11, 20033 Orders of the Department of Labor and
Employment (DOLE) and ordered the latter to allow the parties to
adduce evidence in support of their respective positions. By Decision15 dated March 19, 2001 (first CA Decision), the CA partly
granted the petition and ruled that a recomputation and reevaluation of
the benefits awarded was in order.
The facts follow.

WHEREFORE, the instant petition is partly GRANTED in that the case


Respondent National Service Corporation (NASECO) is a wholly- is remanded to the Secretary of Labor for purposes of recomputation
owned subsidiary of the Philippine National Bank (PNB) organized
and reevaluation of the CBA benefits.
under the Corporation Code in 1975. It supplies security and
manpower services to different clients such as the Securities and
Exchange Commission, the Philippine Deposit Insurance Corporation, SO ORDERED.16
Food Terminal Incorporated, Forex Corporation and PNB. Petitioner
NASECO Guards Association-PEMA (NAGA-PEMA) is the collective
In compliance with the CA directive, then DOLE Secretary Patricia A.
bargaining representative of the regular rank and file security guards of
Sto. Tomas conducted several clarificatory hearings. On January 15,
respondent. NASECO Employees Union-PEMA (NEMU-PEMA) is the
2003, Secretary Sto. Tomas issued an Order which provides:
collective bargaining representative of the regular rank and file (non-
security) employees of respondent such as messengers, janitors,
typists, clerks and radio-telephone operators.4 From the above, it is indubitable that the total cost to NASECO of our
questioned award would amount to only ₱322,725,000, not
₱531,446,666.67 as claimed by the company. Thus, our November 19,
On December 2, 1993, respondent entered into a memorandum of
1999 Order is hereby affirmed en toto.
agreement5 with petitioner. The terms of the agreement covered the
monetary claims of the petitioner such as salary adjustments,
conversion of salary scheme under Republic Act (R.A.) No. 67586 to WHEREFORE, judgment is hereby rendered:
R.A. No. 6727,7 signing bonus, leaves and other benefits. A year after,
petitioner demanded full negotiation for a collective bargaining
1. [D]irecting NAGA-PEMA and NASECO to execute a new
agreement (CBA) with the respondent and submitted its proposals
collective bargaining agreement effective November 1, 1993,
thereto.
incorporating therein the dispositions contained in our
November 19, 1999 Order as well as all other items agreed
On June 8, 1995, petitioner and respondent agreed to sign a CBA on upon by the parties.
non-economic terms.8
2. Ordering NASECO to negotiate with NEMA-PEMA for a
On September 24, 1996, petitioner filed a notice of strike because of new collective bargaining agreement.
respondent’s refusal to bargain for economic benefits in the CBA.
Following conciliation hearings, the parties again commenced CBA
The charges of unfair labor practice against NASECO and PNB are
negotiations and started to resolve the issues on wage increase,
dismissed for lack of merit.
productivity bonus, incentive bonus, allowances, and other benefits but
failed to reach an agreement.
SO ORDERED.17
Meanwhile, respondent and NEMU-PEMA entered into a CBA on non-
economic terms.9 Unfortunately, a dispute among the leaders of Respondent filed a motion for reconsideration with the DOLE Secretary
NEMU-PEMA arose and at a certain point, leadership of the which was denied on March 11, 2003.
organization was unclear. Hence, the negotiations concerning the
economic terms of the CBA were put on hold until the internal dispute
could be resolved. Respondent thus filed a petition for certiorari with the CA arguing that
the DOLE Secretary, in issuing the January 15, 2003 Order deprived
respondent of due process of law for there was no reevaluation that
On April 29, 1997, petitioner filed a notice of strike before the National took place in the DOLE. It also argued that the order merely
Conciliation and Mediation Board (NCMB) against respondent and recomputed the DOLE Secretary’s initial award of ₱531,446,666.67
PNB due to a bargaining deadlock. The following day, NEMU-PEMA and reduced it to ₱322,725,000.00, contrary to the ruling of the CA to
likewise filed a notice of strike against respondent and PNB on the recompute and reevaluate. Respondent claimed that what the DOLE
ground of unfair labor practices.10 Efforts by the NCMB to conciliate Secretary should have done was to let the parties introduce evidence
failed and pursuant to Article 263(g) of the Labor Code,11 as amended, to show the proper computation of the monetary awards under the
then DOLE Secretary Cresenciano B. Trajano assumed jurisdiction approved CBA.
over the strike notices on June 25, 1998.12
In its second Decision dated May 27, 2004, the CA granted the
On November 19, 1999, then DOLE Secretary Bienvenido E. petition, thus:
Laguesma issued a Resolution13 directing petitioner and respondent to
83
WHEREFORE, the orders dated 15 January 2003 and 11 March 2003 The respondent’s right to due process in this case has not been
are hereby SET ASIDE and the case remanded to the public denied. The order in the first CA decision to recompute and reevaluate
respondent to allow the parties to adduce evidence in support of their was satisfied when the DOLE Secretary reexamined their initial
respective positions. findings and adjusted the awarded benefits. A reevaluation, contrary to
what the respondent claims, is a process by which a person or office
(in this case the DOLE secretary) revisits its own initial pronouncement
SO ORDERED.18
and makes another assessment of its findings. In simple terms, to
reevaluate is to take another look at a previous matter in issue. A
A motion for reconsideration was filed by herein petitioner but the same reevaluation does not necessitate the introduction of new materials for
was denied by the CA on September 22, 200419 finding no reason to review nor does it require a full hearing for new arguments.
reverse and set aside its earlier decision.
From a procedural standpoint, a reevaluation is a continuation of the
Petitioner now comes to this Court for relief by way of a petition for original case and not a new proceeding. Hence, the evidence, financial
review on certiorari seeking to set aside and reverse the May 27, 2004 reports and other documents submitted by the parties in the course of
Decision and the September 22, 2004 Resolution of the CA. the original proceeding are to be visited and reviewed again. In this
light, the respondent has been given the opportunity to be heard by the
DOLE Secretary.
The main issue in this case is whether or not the respondent’s right to
due process was violated. A side issue raised by the petitioner is
whether or not PNB, being the undisputed owner of and exercising Also, contrary to the claim of the respondent that it was barred by the
control over respondent, should be made liable to pay the CBA DOLE Secretary to introduce supporting documents during the
benefits awarded to the petitioner. recomputation and reevaluation, the records show that an Order by
then Secretary of Labor Patricia A. Sto. Tomas dated July 11, 2002
specifically allowed both parties to submit their respective
Petitioner argues first that there was no violation of due process computations as regards the awarded benefits. To wit:
because respondent was never prohibited by the DOLE Secretary to
submit supporting documents when the instant case was pending on
remand. Petitioner contends that due process is properly observed WHEREFORE, the Bureau of Working Conditions is hereby directed to
when there is an opportunity to be heard, to present evidence and to submit to this Office a detailed computation of the CBA benefits
file pleadings, which was never denied to respondent. indicated in the resolution of November 19, 2001 within twenty (20)
days from receipt of this Order. The parties may submit their own
computations to the Bureau for validation.
Second, petitioner argues that the CA erred in stating that respondent
was a company operating at a loss and therefore cannot be expected
to act generously and confer upon its employees additional benefits SO ORDERED.24 (Italics supplied.)
exceeding what is mandated by law. It is the petitioner’s position that
based on the "no loss, no profit" policy of respondent with PNB,
It is thus inaccurate for the respondent to claim that it was denied due
respondent in truth has no "pocket" of its own and is, in effect, one (1)
process because it had all the opportunity to introduce any supporting
and the same with PNB with regard to financial gains and/or liabilities.
document in the course of the recomputation and reevaluation of the
Thus, petitioners contend that the CBA benefits should be shouldered
DOLE Secretary. Respondent admits that it did attach the financial
by PNB considering the poor financial condition of respondent. To
statements and other documents in support of its alleged financial
support such claim, petitioner submitted evidence20 to show that PNB
incapacity to pay the CBA awarded benefits, the same evidence it had
is in superb financial condition and is very much capable of shouldering
earlier submitted before the CA (Memorandum in the first CA decision)
the CBA award.21
in the motion for reconsideration of the DOLE Secretary’s January 15,
2003 Order.25 There is thus no showing that the DOLE Secretary
Respondent on the other hand maintains that the DOLE Secretary denied respondent this basic constitutional right.
violated its right to due process when she merely recomputed the CBA
award instead of reevaluating the entire case and allowing it to present
On the issue of liability, petitioner contends that PNB should be held
supporting documents in accordance with the first CA decision. 22 It
liable to shoulder the CBA benefits awarded to them by virtue of it
claims that the order of the CA to reevaluate included and required a
being a company having full financial, managerial and functional
full assessment of the case together with reception of evidence such
control over respondent as its subsidiary, and by reason of the unique
as financial statements, and the omission of such is a violation of its
"no loss, no profit" scheme implemented between respondent and
right to due process.
PNB.

As to the petitioner’s argument that respondent and PNB are


We are not persuaded.
essentially the same when it comes to financial condition, respondent
contends that although a subsidiary, it has a separate and distinct
personality from PNB with its own charter. Hence, the issue of PNB’s Verily, what the petitioner is asking this Court to do is to pierce the veil
financial well-being is immaterial in this case. of corporate fiction of respondent and hold PNB (being the mother
company) liable for the CBA benefits.
The petition is partly meritorious.
In Concept Builders, Inc. v. NLRC,26 we explained the doctrine of
piercing the corporate veil, as follows:
In simple terms, the constitutional guarantee of due process requires
that a litigant be given "a day in court." It is the availability of the
opportunity to be heard that determines whether or not due process It is a fundamental principle of corporation law that a corporation is an
was violated. A litigant may or may not avail of the opportunity to be entity separate and distinct from its stockholders and from other
heard but as long as such was made available to him/her, there is no corporations to which it may be connected. But, this separate and
violation of the due process clause. In the case of Lumiqued v. distinct personality of a corporation is merely a fiction created by law
Exevea,23 this Court declared that "[a]s long as a party was given the for convenience and to promote justice. So, when the notion of
opportunity to defend his interests in due course, he cannot be said to separate juridical personality is used to defeat public convenience,
have been denied due process of law, for this opportunity to be heard justify wrong, protect fraud or defend crime, or is used as a device to
is the very essence of due process. Moreover, this constitutional defeat the labor laws, this separate personality of the corporation may
mandate is deemed satisfied if a person is granted an opportunity to be disregarded or the veil of corporate fiction pierced. This is true
seek reconsideration of the action or ruling complained of."
84
likewise when the corporation is merely an adjunct, a business conduit
or an alter ego of another corporation.

Also in Pantranco Employees Association (PEA-PTGWO) v. National


Labor Relations Commission,27 this Court ruled:

Whether the separate personality of the corporation should be pierced


hinges on obtaining facts appropriately pleaded or proved. However,
any piercing of the corporate veil has to be done with caution, albeit the
Court will not hesitate to disregard the corporate veil when it is misused
or when necessary in the interest of justice. After all, the concept of
corporate entity was not meant to promote unfair objectives.

Applying the doctrine to the case at bar, we find no reason to pierce


the corporate veil of respondent and go beyond its legal personality.
Control, by itself, does not mean that the controlled corporation is a
mere instrumentality or a business conduit of the mother company.
Even control over the financial and operational concerns of a
subsidiary company does not by itself call for disregarding its corporate
fiction. There must be a perpetuation of fraud behind the control or at
least a fraudulent or illegal purpose behind the control in order to justify
piercing the veil of corporate fiction. Such fraudulent intent is lacking in
this case.

Petitioner argues that the appreciation, analysis and inquiry of this


case may go beyond the presentation of respondent, and therefore
must include the PNB, the bank being the undisputed whole owner of
respondent and the sole provider of funds for the company’s
operations and for the payment of wages and benefits of the
employees, under the "no loss, no profit" scheme.28

We disagree. There is no showing that such "no loss, no profit"


scheme between respondent and PNB was implemented to defeat
public convenience, justify wrong, protect fraud or defend crime, or is
used as a device to defeat the labor laws, nor does the scheme show
that respondent is a mere business conduit or alter ego of PNB. Absent
proof of these circumstances, respondent’s corporate personality
cannot be pierced.1âwphi1

It is apparent that petitioner wants the Court to disregard the corporate


personality of respondent and directly go after PNB in order for it to
collect the CBA benefits. On the same breath, however, petitioner
argues that ultimately it is PNB, by virtue of the "no loss, no profit"
scheme, which shoulders and provides the funds for financial liabilities
of respondent including wages and benefits of employees. If such
scheme was indeed true as the petitioner presents it, then there was
absolutely no need to pierce the veil of corporate fiction of respondent.
Moreover, the Court notes the pendency of a separate suit for
absorption or regularization of NASECO employees filed by petitioner
and NEMU-PEMA against PNB and respondent, docketed as NLRC
NCR Case No. 06-03944-96), which is still on appeal with the National
Labor Relations Commission (NLRC), as per manifestation by
respondent. In the said case, petitioner submitted for resolution by the
labor tribunal the issues of whether PNB is the employer of NASECO’s
work force and whether NASECO is a labor-only contractor.29

WHEREFORE, the petition is PARTLY GRANTED. The Decision


dated May 27, 2004 and Resolution dated September 22, 2004 in CA-
G.R. SP No. 76667 are hereby REVERSED and SET ASIDE as to the
order to remand the case to the Secretary of Labor for introduction of
supporting evidence. Accordingly, the Orders of the Secretary of Labor
dated January 15, 2003 and March 11, 2003 are REINSTATED and
UPHELD.

No costs.

SO ORDERED.

85
G.R. No. 199687 trial court brushed aside E-Securities’ claim of denial of due process on
petitioner as "xxx case records show that notices regarding these
proceedings had been tendered to the latter, which refused to even
PACIFIC REHOUSE CORPORATION, Petitioners,
receive them. Clearly, [petitioner] had been sufficiently put on notice
vs.
and afforded the chance to give its side[,] yet[,] it chose not to." Thus,
COURT OF APPEALS and EXPORT AND INDUSTRY BANK,
the RTC disposed as follows:
INC., Respondents.

WHEREFORE, xxx,
x-----------------------x

Let an Alias Writ of Execution be issued relative to the above-entitled


G.R. No. 201537
case and pursuant to the RESOLUTION dated October 18, 2005 and
to this Order directing defendant EIB Securities, Inc., and/or Export
PACIFIC REHOUSE CORPORATION, PACIFIC CONCORDE and Industry Bank, Inc., to fully comply therewith.
CORPORATION, MIZPAH HOLDINGS, INC., FORUM HOLDINGS
CORPORATION and EAST ASIA OIL COMPANY, INC., Petitioners,
The Branch Sheriff of this Court is directed to cause the immediate
vs.
implementation of the given alias writ in accordance with the Order of
EXPORT AND INDUSTRY BANK, INC., Respondent.
Execution to be issued anew by the Branch Clerk of Court.

DECISION
SO ORDERED. x x x

REYES, J.:
With this development, petitioner filed an Omnibus Motion (Ex
Abundanti Cautela) questioning the alias writ because it was not
On the scales of justice precariously lie the right of a prevailing party to impleaded as a party to the case. The RTC denied the motion in its
his victor's cup, no more, no less; and the right of a separate entity Order dated August 26, 2011 and directed the garnishment of
from being dragged by the ball and chain of the vanquished party. P1,465,799,000.00, the total amount of the 32,180,000 DMCI shares at
P45.55 per share, against petitioner and/or E-Securities.2 x x x.
(Citations omitted)
The facts of this case as garnered from the Decision1 dated April 26,
2012 of the Court of Appeals (CA) in CA-G.R. SP No. 120979 are as
follows: The Regional Trial Court (RTC) ratiocinated that being one and the
same entity in the eyes of the law, the service of summons upon EIB
Securities, Inc. (E-Securities) has bestowed jurisdiction over both the
We trace the roots of this case to a complaint instituted with the Makati
parent and wholly-owned subsidiary.3 The RTC cited the cases of Sps.
City Regional Trial Court (RTC), Branch 66, against EIB Securities Inc. Violago v. BA Finance Corp. et al.4 and Arcilla v. Court of
(E-Securities) for unauthorized sale of 32,180,000 DMCI shares of Appeals5 where the doctrine of piercing the veil of corporate fiction was
private respondents Pacific Rehouse Corporation, Pacific Concorde applied notwithstanding that the affected corporation was not brought
Corporation, Mizpah Holdings, Inc., Forum Holdings Corporation, and to the court as a party. Thus, the RTC held in its Order6 dated August
East Asia Oil Company, Inc. In its October 18, 2005 Resolution, the 26, 2011:
RTC rendered judgment on the pleadings. The fallo reads:

WHEREFORE, premises considered, the Motion for Reconsideration


WHEREFORE, premises considered, judgment is hereby rendered
with Motion to Inhibit filed by defendant EIB Securities, Inc. is denied
directing the defendant [E-Securities] to return the plaintiffs’ [private
for lack of merit. The Omnibus Motion Ex Abundanti C[au]tela is
respondents herein] 32,180,000 DMCI shares, as of judicial demand.
likewise denied for lack of merit.

On the other hand, plaintiffs are directed to reimburse the defendant Pursuant to Rule 39, Section 10 (a) of the Rules of Court, the Branch
the amount of [P]10,942,200.00, representing the buy back price of the Clerk of Court or the Branch Sheriff of this Court is hereby directed to
60,790,000 KPP shares of stocks at [P]0.18 per share.
acquire 32,180,000 DMCI shares of stock from the Philippine Stock
Exchange at the cost of EIB Securities, Inc. and Export and Industry
SO ORDERED. x x x Bank[,] Inc. and to deliver the same to the plaintiffs pursuant to this
Court’s Resolution dated October 18, 2005.
The Resolution was ultimately affirmed by the Supreme Court and
attained finality. To implement this Order, let GARNISHMENT issue against ALL
THOSE HOLDING MONEYS, PROPERTIES OF ANY AND ALL
KINDS, REAL OR PERSONAL BELONGING TO OR OWNED BY
When the Writ of Execution was returned unsatisfied, private DEFENDANT EIB SECURITIES, INC. AND/OR EXPORT AND
respondents moved for the issuance of an alias writ of execution to INDUSTRY BANK[,] INC., [sic] in such amount as may be sufficient to
hold Export and Industry Bank, Inc. liable for the judgment obligation acquire 32,180,000 DMCI shares of stock to the Philippine Stock
as E- Securities is "a wholly-owned controlled and dominated Exchange, based on the closing price of Php45.55 per share of DMCI
subsidiary of Export and Industry Bank, Inc., and is[,] thus[,] a mere shares as of August 1, 2011, the date of the issuance of the Alias Writ
alter ego and business conduit of the latter. E-Securities opposed the of Execution, or the total amount of PhP1,465,799,000.00.
motion[,] arguing that it has a corporate personality that is separate
and distinct from petitioner. On July 27, 2011, private respondents filed
their (1) Reply attaching for the first time a sworn statement executed SO ORDERED.7
by Atty. Ramon F. Aviado, Jr., the former corporate secretary of
petitioner and E-Securities, to support their alter ego theory; and CA-G.R. SP No. 120979
(2) Ex-Parte Manifestation alleging service of copies of the Writ of
Execution and Motion for Alias Writ of Execution on petitioner.
Export and Industry Bank, Inc. (Export Bank) filed before the CA a
petition for certiorari with prayer for the issuance of a temporary
On July 29, 2011, the RTC concluded that E-Securities is a mere
restraining order (TRO)8 seeking the nullification of the RTC Order
business conduit or alter ego of petitioner, the dominant parent dated August 26, 2011 for having been made with grave abuse of
corporation, which justifies piercing of the veil of corporate fiction. The discretion amounting to lack or excess of jurisdiction. In its petition,
86
Export Bank made reference to several rulings 9 of the Court upholding On December 22, 2011, the CA, through a Special Division of Five,
the separate and distinct personality of a corporation. issued another Resolution,21 which reiterated the Resolution dated
October 25, 2011 granting the issuance of a writ of preliminary
injunction.
In a Resolution10 dated September 2, 2011, the CA issued a 60-day
TRO enjoining the execution of the Orders of the RTC dated July 29,
2011 and August 26, 2011, which granted the issuance of an alias writ On January 2, 2012, one of the petitioners herein, Pacific Rehouse
of execution and ordered the garnishment of the properties of E- filed before the Court a petition for certiorari22 under Rule 65, docketed
Securities and/or Export Bank. The CA also set a hearing to determine as G.R. No. 199687, demonstrating its objection to the Resolutions
the necessity of issuing a writ of injunction, viz: dated October 25, 2011 and December 22, 2011 of the CA.

Considering the amount ordered to be garnished from petitioner Export On April 26, 2012, the CA rendered the assailed Decision23 on the
and Industry Bank, Inc. and the fiduciary duty of the banking institution merits of the case, granting Export Bank’s petition. The CA disposed of
to the public, there is grave and irreparable injury that may be caused the case in this wise:
to [Export Bank] if the assailed Orders are immediately implemented.
We thus resolve to GRANT the Temporary Restraining Order effective
We GRANT the petition. The Orders dated July 29, 2011 and August
for a period of sixty (60) days from notice, restraining/enjoining the
26, 2011 of the Makati City Regional Trial Court, Branch 66, insofar as
Sheriff of the Regional Trial Court of Makati City or his deputies,
[Export Bank] is concerned, are NULLIFIED. The Writ of Preliminary
agents, representatives or any person acting in their behalf from
Injunction (WPI) is rendered PERMANENT.
executing the July 29, 2011 and August 26, 2011 Orders. [Export
Bank] is DIRECTED to POST a bond in the sum of fifty million pesos
(P50,000,000.00) within ten (10) days from notice, to answer for any SO ORDERED.24
damage which private respondents may suffer by reason of this
Temporary Restraining Order; otherwise, the same shall automatically
become ineffective. The CA explained that the alter ego theory cannot be sustained
because ownership of a subsidiary by the parent company is not
enough justification to pierce the veil of corporate fiction. There must
Let the HEARING be set on September 27, 2011 at 2:00 in the be proof, apart from mere ownership, that Export Bank exploited or
afternoon at the Paras Hall, Main Building, Court of Appeals, to misused the corporate fiction of E-Securities. The existence of
determine the necessity of issuing a writ of preliminary injunction. The interlocking incorporators, directors and officers between the two
Division Clerk of Court is DIRECTED to notify the parties and their corporations is not a conclusive indication that they are one and the
counsel with dispatch. same.25 The records also do not show that Export Bank has complete
control over the business policies, affairs and/or transactions of E-
Securities. It was solely E-Securities that contracted the obligation in
xxxx
furtherance of its legitimate corporate purpose; thus, any fall out must
be confined within its limited liability.26
SO ORDERED.11
The petitioners, without filing a motion for reconsideration, filed a
Pacific Rehouse Corporation (Pacific Rehouse), Pacific Concorde Petition for Review27 under Rule 45 docketed as G.R. No.
Corporation, Mizpah Holdings, Inc., Forum Holdings Corporation and 201537,28 impugning the Decision dated April 26, 2012 of the CA.
East Asia Oil Company, Inc. (petitioners) filed their Comment 12 to
Export Bank’s petition and proffered that the cases mentioned by
Considering that G.R. Nos. 199687 and 201537 originated from the
Export Bank are inapplicable owing to their clearly different factual
same set of facts, involved the same parties and raised intertwined
antecedents. The petitioners alleged that unlike the other cases, there
issues, the cases were then consolidated per Resolution dated
are circumstances peculiar only to E-Securities and Export Bank such
September 26, 2012, for a thorough discussion of the merits of the
as: 499,995 out of 500,000 outstanding shares of stocks of E-
case.
Securities are owned by Export Bank;13 Export Bank had actual
knowledge of the subject matter of litigation as the lawyers who
represented E-Securities are also lawyers of Export Bank.14 As an alter Issues
ego, there is no need for a finding of fraud or illegality before the
doctrine of piercing the veil of corporate fiction can be applied.15
In précis, the issues for resolution of this Court are the following:

After oral arguments before the CA, the parties were directed to file
In G.R. No. 199687,
their respective memoranda.16

On October 25, 2011, the CA issued a Resolution,17 granting Export WHETHER THE CA COMMITTED GRAVE ABUSE OF DISCRETION
Bank’s application for the issuance of a writ of preliminary injunction, IN GRANTING EXPORT BANK’S APPLICATION FOR THE
ISSUANCE OF A WRIT OF PRELIMINARY INJUNCTION.
viz:

In G.R. No. 201537,


WHEREFORE, finding [Export Bank’s] application for the ancillary
injunctive relief to be meritorious, and it further appearing that there is
urgency and necessity in restraining the same, a Writ of Preliminary I.
Injunction is hereby GRANTED and ISSUED against the Sheriff of the
Regional Trial Court of Makati City, Branch 66, or his deputies, agents,
representatives or any person acting in their behalf from executing the WHETHER THE CA COMMITTED A REVERSIBLE ERROR IN
July 29, 2011 and August 26, 2011 Orders. Public respondents are RULING THAT EXPORT BANK MAY NOT BE HELD LIABLE FOR A
ordered to CEASE and DESIST from enforcing and implementing the FINAL AND EXECUTORY JUDGMENT AGAINST E-SECURITIES IN
subject orders until further notice from this Court.18 AN ALIAS WRIT OF EXECUTION BY PIERCING ITS VEIL OF
CORPORATE FICTION; and

The petitioners filed a Manifestation19 and Supplemental


Manifestation20 challenging the above-quoted CA resolution for lack of II.
concurrence of Associate Justice Socorro B. Inting (Justice Inting), who
was then on official leave.
87
WHETHER THE CA COMMITTED A REVERSIBLE ERROR IN immediately clarified and remedied by the CA, the lack of the
RULING THAT THE ALTER EGO DOCTRINE IS NOT APPLICABLE. Chairperson’s signature on the copies sent to the parties has already
become a non-issue.
Ruling of the Court
It must be emphasized that the instant cases sprang from Pacific
Rehouse Corporation v. EIB Securities, Inc.45 which was decided by
G.R. No. 199687
this Court last October 13, 2010. Significantly, Export Bank was not
impleaded in said case but was unexpectedly included during the
The Resolution dated October 25, 2011 was initially challenged by the execution stage, in addition to E-Securities, against whom the writ of
petitioners in its Manifestation29 and Supplemental Manifestation30 due execution may be enforced in the Order46 dated July 29, 2011 of the
to the lack of concurrence of Justice Inting, which according to the RTC. In including Export Bank, the RTC considered E-Securities as a
petitioners rendered the aforesaid resolution null and void. mere business conduit of Export Bank.47 Thus, one of the arguments
interposed by the latter in its Opposition48 that it was never impleaded
as a defendant was simply set aside.
To the petitioners’ mind, Section 5, Rule VI of the Internal Rules of the
CA (IRCA)31 requires the submission of the resolution granting an
application for TRO or preliminary injunction to the absent Justice/s This action by the RTC begs the question: may the RTC enforce the
when they report back to work for ratification, modification or recall, alias writ of execution against Export Bank?
such that when the absent Justice/s do not agree with the issuance of
the TRO or preliminary injunction, the resolution is recalled and without
The question posed before us is not novel.
force and effect.32 Since the resolution which granted the application
for preliminary injunction appears short of the required number of
consensus, owing to the absence of Justice Inting’s signature, the The Court already ruled in Kukan International Corporation v.
petitioners contest the validity of said resolution. Reyes49 that compliance with the recognized modes of acquisition of
jurisdiction cannot be dispensed with even in piercing the veil of
corporate fiction, to wit:
The petitioners also impugn the CA Resolution dated December 22,
2011 rendered by the Special Division of Five. The petitioners maintain
that pursuant to Batas Pambansa Bilang 12933 and the IRCA,34 such The principle of piercing the veil of corporate fiction, and the resulting
division is created only when the three members of a division cannot treatment of two related corporations as one and the same juridical
reach a unanimous vote in deciding a case on the person with respect to a given transaction, is basically applied only to
merits.35 Furthermore, for petitioner Pacific Rehouse, this Resolution is determine established liability; it is not available to confer on the court
likewise infirm because the purpose of the formation of the Special a jurisdiction it has not acquired, in the first place, over a party not
Division of Five is to decide the case on the merits and not to grant impleaded in a case. Elsewise put, a corporation not impleaded in a
Export Bank’s application for a writ of preliminary injunction. 36 suit cannot be subject to the court’s process of piercing the veil
of its corporate fiction. In that situation, the court has not acquired
jurisdiction over the corporation and, hence, any proceedings taken
We hold that the opposition to the CA resolutions is already nugatory
against that corporation and its property would infringe on its right to
because the CA has already rendered its Decision on April 16, 2012,
due process. Aguedo Agbayani, a recognized authority on Commercial
which disposed of the substantial merits of the case. Consequently, the
Law, stated as much:
petitioners’ concern that the Special Division of Five should have been
created to resolve cases on the merits has already been addressed by
the rendition of the CA Decision dated April 16, 2012. "23. Piercing the veil of corporate entity applies to determination of
liability not of jurisdiction. x x x
"It is well-settled that courts will not determine questions that have
become moot and academic because there is no longer any justiciable This is so because the doctrine of piercing the veil of corporate
controversy to speak of. The judgment will not serve any useful fiction comes to play only during the trial of the case after the
purpose or have any practical legal effect because, in the nature of court has already acquired jurisdiction over the
things, it cannot be enforced."37 In such cases, there is no actual corporation. Hence, before this doctrine can be applied, based on the
substantial relief to which the petitioners would be entitled to and which evidence presented, it is imperative that the court must first have
would be negated by the dismissal of the petition.38 Thus, it would be jurisdiction over the corporation. x x x"50 (Citations omitted)
futile and pointless to address the issue in G.R. No. 199687 as this has
become moot and academic.
From the preceding, it is therefore correct to say that the court must
first and foremost acquire jurisdiction over the parties; and only then
G.R. No. 201537 would the parties be allowed to present evidence for and/or against
piercing the veil of corporate fiction. If the court has no jurisdiction over
the corporation, it follows that the court has no business in piercing its
The petitioners bewail that the certified true copy of the CA Decision
veil of corporate fiction because such action offends the corporation’s
dated April 26, 2012 along with its Certification at the bottom portion
right to due process.
were not signed by the Chairperson39 of the Special Division of Five;
thus, it is not binding upon the parties.40 The petitioners quoted this
Court’s pronouncement in Limkaichong v. Commission on "Jurisdiction over the defendant is acquired either upon a valid service
Elections,41 that a decision must not only be signed by the Justices of summons or the defendant’s voluntary appearance in court. When
who took part in the deliberation, but must also be promulgated to be the defendant does not voluntarily submit to the court’s jurisdiction or
considered a Decision.42 when there is no valid service of summons, ‘any judgment of the court
which has no jurisdiction over the person of the defendant is null and
void.’"51 "The defendant must be properly apprised of a pending action
A cursory glance on a copy of the signature page43 of the decision
against him and assured of the opportunity to present his defenses to
attached to the records would show that, indeed, the same was not
the suit. Proper service of summons is used to protect one’s right to
signed by CA Associate Justice Magdangal M. De Leon. However, it
due process."52
must be noted that the CA, on May 7, 2012, issued a
Resolution44 explaining that due to inadvertence, copies of the decision
not bearing the signature of the Chairperson were sent to the parties As Export Bank was neither served with summons, nor has it
on the same day of promulgation. The CA directed the Division Clerk of voluntarily appeared before the court, the judgment sought to be
Court to furnish the parties with copies of the signature page with the enforced against E-Securities cannot be made against its parent
Chairperson’s signature. Consequently, as the mistake was company, Export Bank. Export Bank has consistently disputed the RTC
8
jurisdiction, commencing from its filing of an Omnibus Motion53 by way non-party liable. In short, liabilities attached only to those who are
of special appearance during the execution stage until the filing of its parties. None of the non-party corporations (VMSC and CMRI) were
Comment54 before the Court wherein it was pleaded that "RTC [of] made liable for the judgment award against Avelino and Arcilla.
Makati[, Branch] 66 never acquired jurisdiction over Export [B]ank.
Export [B]ank was not pleaded as a party in this case. It was never
The Alter Ego Doctrine is not applicable
served with summons by nor did it voluntarily appear before RTC [of]
Makati[, Branch] 66 so as to be subjected to the latter’s jurisdiction." 55
"The question of whether one corporation is merely an alter ego of
another is purely one of fact. So is the question of whether a
In dispensing with the requirement of service of summons or voluntary
corporation is a paper company, a sham or subterfuge or whether
appearance of Export Bank, the RTC applied the cases of Violago and
petitioner adduced the requisite quantum of evidence warranting the
Arcilla. The RTC concluded that in these cases, the Court decided that
piercing of the veil of respondent’s corporate entity."59
the doctrine of piercing the veil of corporate personality can be applied
even when one of the affected parties has not been brought to the
Court as a party.56 As a rule, the parties may raise only questions of law under Rule 45,
because the Supreme Court is not a trier of facts. Generally, we are
not duty-bound to analyze again and weigh the evidence introduced in
A closer perusal on the rulings of this Court in Violago and Arcilla,
and considered by the tribunals below.60 However, justice for all is of
however, reveals that the RTC misinterpreted the doctrines on these
primordial importance that the Court will not think twice of reviewing the
cases. We agree with the CA that these cases are not congruent to the
facts, more so because the RTC and the CA arrived in contradicting
case at bar. In Violago, Spouses Pedro and Florencia Violago
conclusions.
(Spouses Violago) filed a third party complaint against their cousin
Avelino Violago (Avelino), who is also the president of Violago Motor
Sales Corporation (VMSC), for selling them a vehicle which was "It is a fundamental principle of corporation law that a corporation is an
already sold to someone else. VMSC was not impleaded as a third entity separate and distinct from its stockholders and from other
party defendant. Avelino contended that he was not a party to the corporations to which it may be connected. But, this separate and
transaction personally, but VMSC. The Court ruled that "[t]he fact that distinct personality of a corporation is merely a fiction created by law
VMSC was not included as defendant in [Spouses Violago’s] third party for convenience and to promote justice. So, when the notion of
complaint does not preclude recovery by Spouses Violago from separate juridical personality is used to defeat public convenience,
Avelino; neither would such non-inclusion constitute a bar to the justify wrong, protect fraud or defend crime, or is used as a device to
application of the piercing-of-the-corporate-veil doctrine."57 It should be defeat the labor laws, this separate personality of the corporation may
pointed out that although VMSC was not made a third party defendant, be disregarded or the veil of corporate fiction pierced. This is true
the person who was found liable in Violago, Avelino, was properly likewise when the corporation is merely an adjunct, a business conduit
made a third party defendant in the first instance. The present case or an alter ego of another corporation."61
could not be any more poles apart from Violago, because Export Bank,
the parent company which was sought to be accountable for the
judgment against E-Securities, is not a party to the main case. "Where one corporation is so organized and controlled and its affairs
are conducted so that it is, in fact, a mere instrumentality or adjunct of
the other, the fiction of the corporate entity of the "instrumentality" may
In Arcilla, meanwhile, Calvin Arcilla (Arcilla) obtained a loan in the be disregarded. The control necessary to invoke the rule is not majority
name of Csar Marine Resources, Inc. (CMRI) from Emilio Rodulfo. A or even complete stock control but such domination of finances,
complaint was then filed against Arcilla for non-payment of the loan. policies and practices that the controlled corporation has, so to speak,
CMRI was not impleaded as a defendant. The trial court eventually no separate mind, will or existence of its own, and is but a conduit for
ordered Arcilla to pay the judgment creditor for such loan. Arcilla its principal. It must be kept in mind that the control must be shown to
argued that he is not personally liable for the adjudged award because have been exercised at the time the acts complained of took place.
the same constitutes a corporate liability which cannot even bind the Moreover, the control and breach of duty must proximately cause the
corporation as the latter is not a party to the collection suit. The Court injury or unjust loss for which the complaint is made."62
made the succeeding observations:
The Court has laid down a three-pronged control test to establish when
[B]y no stretch of even the most fertile imagination may one be able to the alter ego doctrine should be operative:
conclude that the challenged Amended Decision directed Csar Marine
Resources, Inc. to pay the amounts adjudged. By its clear and
unequivocal language, it is the petitioner who was declared liable (1) Control, not mere majority or complete stock control, but
therefor and consequently made to pay. x x x, even if We are to complete domination, not only of finances but of policy and
business practice in respect to the transaction attacked so
assume arguendo that the obligation was incurred in the name of the
corporation, the petitioner would still be personally liable therefor that the corporate entity as to this transaction had at the time
because for all legal intents and purposes, he and the corporation are no separate mind, will or existence of its own;
one and the same. Csar Marine Resources, Inc. is nothing more than
his business conduit and alter ego. The fiction of a separate juridical (2) Such control must have been used by the defendant to
personality conferred upon such corporation by law should be commit fraud or wrong, to perpetuate the violation of a
disregarded. x x x.58 (Citation omitted) statutory or other positive legal duty, or dishonest and unjust
act in contravention of plaintiff’s legal right; and
It is important to bear in mind that although CMRI was not a party to
the suit, it was Arcilla, the defendant himself who was found ultimately (3) The aforesaid control and breach of duty must [have]
liable for the judgment award. CMRI and its properties were left proximately caused the injury or unjust loss complained of.63
untouched from the main case, not only because of the application of
the alter ego doctrine, but also because it was never made a party to
that case. The absence of any one of these elements prevents ‘piercing the
corporate veil’ in applying the ‘instrumentality’ or ‘alter ego’ doctrine,
the courts are concerned with reality and not form, with how the
The disparity between the instant case and those of Violago and Arcilla corporation operated and the individual defendant’s relationship to that
is that in said cases, although the corporations were not impleaded as operation.64 Hence, all three elements should concur for the alter ego
defendant, the persons made liable in the end were already parties doctrine to be applicable.
thereto since the inception of the main case. Consequently, it cannot
be said that the Court had, in the absence of fraud and/or bad faith,
applied the doctrine of piercing the veil of corporate fiction to make a
89
In its decision, the RTC maintained that the subsequently enumerated All the foregoing circumstances, with the exception of the admitted
factors betray the true nature of E-Securities as a mere alter ego of stock ownership, were however not properly pleaded and proved in
Export Bank: accordance with the Rules of Court.66 These were merely raised by the
petitioners for the first time in their Motion for Issuance of an Alias Writ
of Execution67 and Reply,68 which the Court cannot consider. "Whether
1. Defendant EIB Securities, a subsidiary corporation 100%
the separate personality of the corporation should be pierced hinges on
totally owned by Export and Industry Bank, Inc., was only re-
obtaining facts appropriately pleaded or proved."69
activated by the latter in 2002-2003 and the continuance of
its operations was geared for no other reason tha[n] to serve
as the securities brokerage arm of said parent Albeit the RTC bore emphasis on the alleged control exercised by
corporation bank; Export Bank upon its subsidiary E-Securities, "[c]ontrol, by itself, does
not mean that the controlled corporation is a mere instrumentality or a
business conduit of the mother company. Even control over the
2. It was the parent corporation bank that provided and
financial and operational concerns of a subsidiary company does not
infused the fresh working cash capital needed by defendant
by itself call for disregarding its corporate fiction. There must be a
EIB Securities which prior thereto was non-operating and
perpetuation of fraud behind the control or at least a fraudulent or
severely cash-strapped. [This was so attested by the then
illegal purpose behind the control in order to justify piercing the veil of
Corporate Secretary of both corporations, Atty. Ramon
corporate fiction. Such fraudulent intent is lacking in this case."70
Aviado, Jr., in his submitted Sworn Statement which is
deemed allowable "evidence on motion", under Sec. 7, Rule
133, Rules on Evidence; Bravo vs. Borja, 134 SCRA 438]; Moreover, there was nothing on record demonstrative of Export Bank’s
wrongful intent in setting up a subsidiary, E-Securities. If used to
perform legitimate functions, a subsidiary’s separate existence shall be
3. For effective control purposes, defendant EIB Securities
respected, and the liability of the parent corporation as well as the
and its operating office and staff are all housed in
subsidiary will be confined to those arising in their respective
Exportbank Plaza located at Chino Roces cor. Sen. Gil
business.71 To justify treating the sole stockholder or holding company
Puyat Avenue, Makati City which is the same building
as responsible, it is not enough that the subsidiary is so organized and
w[h]ere the bank parent corporation has its headquarters;
controlled as to make it "merely an instrumentality, conduit or adjunct"
of its stockholders. It must further appear that to recognize their
4. As shown in the General Information Sheets annually filed separate entities would aid in the consummation of a wrong.72
with the S.E.C. from 2002 to 2011, both defendant EIB
Securities and the bank parent corporation share common
As established in the main case73 and reiterated by the CA, the subject
key Directors and corporate officers. Three of the 5-man
32,180,000 DMCI shares which E-Securities is obliged to return to the
Board of Directors of defendant EIB Securities are Directors
petitioners were originally bought at an average price of P0.38 per
of the bank parent corporation, namely: Jaime C. Gonzales,
share and were sold for an average price of P0.24 per share. The
Pauline C. Tan and Dionisio E. Carpio, Jr. In addition, Mr.
proceeds were then used to buy back 61,100,000 KPP shares earlier
Gonzales is Chairman of the Board of both corporations,
sold by E-Securities. Quite unexpectedly however, the total amount of
whereas Pauline C. Tan is concurrently President/General
these DMCI shares ballooned to P1,465,799,000.00.74 It must be taken
Manager of EIB Securities, and Dionisio Carpio Jr., is not
into account that this unexpected turnabout did not inure to the benefit
only director of the bank, but also Director Treasurer of
of E-Securities, much less Export Bank.
defendant EIB Securities;

Furthermore, ownership by Export Bank of a great majority or all of


5. As admitted by the bank parent corporation in its
stocks of E-Securities and the existence of interlocking directorates
consolidated audited financial statements[,] EIB Securities is
may serve as badges of control, but ownership of another corporation,
a CONTROLLED SUBSIDIARY, and for which reason its
per se, without proof of actuality of the other conditions are insufficient
financial condition and results of operations are included and
to establish an alter ego relationship or connection between the two
integrated as part of the group’s consolidated financial
corporations, which will justify the setting aside of the cover of
statements, examined and audited by the same auditing
corporate fiction. The Court has declared that "mere ownership by a
firm;
single stockholder or by another corporation of all or nearly all of the
capital stock of a corporation is not of itself sufficient ground for
6. The lawyers handling the suits and legal matters of disregarding the separate corporate personality." The Court has
defendant EIB Securities are the same lawyers in the Legal likewise ruled that the "existence of interlocking directors, corporate
Department of the bank parent corporation.1âwphi1 The officers and shareholders is not enough justification to pierce the veil of
Court notes that in [the] above-entitled suit, the lawyers who corporate fiction in the absence of fraud or other public policy
at the start represented said defendant EIB Securities and considerations."75
filed all the pleadings and filings in its behalf are also the
lawyers in the Legal Services Division of the bank parent
While the courts have been granted the colossal authority to wield the
corporation. They are Attys. Emmanuel A. Silva, Leonardo
sword which pierces through the veil of corporate fiction, concomitant
C. Bool, Riva Khristine E. Maala and Ma. Esmeralda R.
to the exercise of this power, is the responsibility to uphold the doctrine
Cunanan, all of whom worked at the Legal Services Division
of separate entity, when rightly so; as it has for so long encouraged
of Export Industry Bank located at 36/F, Exportbank Plaza,
businessmen to enter into economic endeavors fraught with risks and
Don Chino Roces Avenue, cor. Sen. Gil Puyat Avenue,
where only a few dared to venture.
Makati City.

Hence, any application of the doctrine of piercing the corporate veil


7. Finally[,] and this is very significant, the control and sway
should be done with caution. A court should be mindful of the milieu
that the bank parent corporation held over defendant EIB
where it is to be applied. It must be certain that the corporate fiction
Securities was prevailing in June 2004 when the very act
was misused to such an extent that injustice, fraud, or crime was
complained of in plaintiff’s Complaint took place, namely the
committed against another, in disregard of its rights. The wrongdoing
unauthorized disposal of the 32,180,000 DMCI shares of
must be clearly and convincingly established; it cannot be presumed.
stock. Being then under the direction and control of the bank
Otherwise, an injustice that was never unintended may result from an
parent corporation, the unauthorized disposal of those
erroneous application.76
shares by defendant EIB Securities is attributable to, and the
responsibility of the former.65

90
In closing, we understand that the petitioners are disgruntled at the
turnout of this case-that they cannot enforce the award due them on its
entirety; however, the Court cannot supplant a remedy which is not
sanctioned by our laws and prescribed rules.

WHEREFORE, the petition in G.R. No. 199687 is hereby DISMISSED


for having been rendered moot and academic. The petition in G.R. No.
201537, meanwhile, is hereby DENIED for lack of merit. Consequently,
the Decision dated April 26, 2012 of the Court of Appeals in CA-G.R.
SP No. 120979 is AFFIRMED.

SO ORDERED.

91
G.R. No. 182729 September 29, 2010 3. to pay the sum of TWENTY THOUSAND PESOS,
(P20,000.00) as reasonable attorney’s fees; and
KUKAN INTERNATIONAL CORPORATION, Petitioner,
vs. 4. to pay the sum of SEVEN THOUSAND NINE HUNDRED
HON. AMOR REYES, in her capacity as Presiding Judge of the SIXTY PESOS and SIX CENTAVOS (P7,960.06) as litigation
Regional Trial Court of Manila, Branch 21, and ROMEO M. expenses.
MORALES, doing business under the name and style "RM
Morales Trophies and Plaques," Respondents.
For lack of factual foundation, the counterclaim is DISMISSED.

DECISION
IT IS SO ORDERED.7

VELASCO, JR., J.:


After the above decision became final and executory, Morales moved
for and secured a writ of execution8 against Kukan, Inc. The sheriff
The Case then levied upon various personal properties found at what was
supposed to be Kukan, Inc.’s office at Unit 2205, 88 Corporate Center,
Salcedo Village, Makati City. Alleging that it owned the properties thus
This Petition for Review on Certiorari under Rule 45 seeks to nullify
levied and that it was a different corporation from Kukan, Inc., Kukan
and reverse the January 23, 2008 Decision1 and the April 16, 2008
International Corporation (KIC) filed an Affidavit of Third-Party Claim.
Resolution2 rendered by the Court of Appeals (CA) in CA-G.R. SP No.
Notably, KIC was incorporated in August 2000, or shortly after Kukan,
100152.
Inc. had stopped participating in Civil Case No. 99-93173.

The assailed CA decision affirmed the March 12, 20073 and June 7,
In reaction to the third party claim, Morales interposed an Omnibus
20074 Orders of the Regional Trial Court (RTC) of Manila, Branch 21,
Motion dated April 30, 2003. In it, Morales prayed, applying the
in Civil Case No. 99-93173, entitled Romeo M. Morales, doing
principle of piercing the veil of corporate fiction, that an order be issued
business under the name and style RM Morales Trophies and Plaques
for the satisfaction of the judgment debt of Kukan, Inc. with the
v. Kukan, Inc. In the said orders, the RTC disregarded the separate
properties under the name or in the possession of KIC, it being alleged
corporate identities of Kukan, Inc. and Kukan International Corporation
that both corporations are but one and the same entity. KIC opposed
and declared them to be one and the same entity. Accordingly, the
Morales’ motion. By Order of May 29, 20039 as reiterated in a
RTC held Kukan International Corporation, albeit not impleaded in the
subsequent order, the court denied the omnibus motion.
underlying complaint of Romeo M. Morales, liable for the judgment
award decreed in a Decision dated November 28, 20025 in favor of
Morales and against Kukan, Inc. In a bid to establish the link between KIC and Kukan, Inc., and thus
determine the true relationship between the two, Morales filed a Motion
for Examination of Judgment Debtors dated May 4, 2005. In this
The Facts
motion Morales sought that subponae be issued against the primary
stockholders of Kukan, Inc., among them Michael Chan, a.k.a. Chan
Sometime in March 1998, Kukan, Inc. conducted a bidding for the Kai Kit. This too was denied by the trial court in an Order dated May
supply and installation of signages in a building being constructed in 24, 2005.10
Makati City. Morales tendered the winning bid and was awarded the
PhP 5 million contract. Some of the items in the project award were
Morales then sought the inhibition of the presiding judge, Eduardo B.
later excluded resulting in the corresponding reduction of the contract
Peralta, Jr., who eventually granted the motion. The case was re-
price to PhP 3,388,502. Despite his compliance with his contractual
raffled to Branch 21, presided by public respondent Judge Amor
undertakings, Morales was only paid the amount of PhP 1,976,371.07,
Reyes.
leaving a balance of PhP 1,412,130.93, which Kukan, Inc. refused to
pay despite demands. Shortchanged, Morales filed a Complaint 6 with
the RTC against Kukan, Inc. for a sum of money, the case docketed as Before the Manila RTC, Branch 21, Morales filed a Motion to Pierce the
Civil Case No. 99-93173 and eventually raffled to Branch 17 of the Veil of Corporate Fiction to declare KIC as having no existence
court. separate from Kukan, Inc. This time around, the RTC, by Order dated
March 12, 2007, granted the motion, the dispositive portion of which
reads:
Following the joinder of issues after Kukan, Inc. filed an answer with
counterclaim, trial ensued. However, starting November 2000, Kukan,
Inc. no longer appeared and participated in the proceedings before the WHEREFORE, premises considered, the motion is hereby GRANTED.
trial court, prompting the RTC to declare Kukan, Inc. in default and The Court hereby declares as follows:
paving the way for Morales to present his evidence ex parte.
1. defendant Kukan, Inc. and newly created Kukan
On November 28, 2002, the RTC rendered a Decision finding for International Corp. as one and the same corporation;
Morales and against Kukan, Inc., disposing as follows:
2. the levy made on the properties of Kukan International
WHEREFORE, consistent with Section 5, Rule 18 of the 1997 Rules of Corp. is hereby valid;
Civil Procedure, and by preponderance of evidence, judgment is
hereby rendered in favor of the plaintiff, ordering Kukan, Inc.:
3. Kukan International Corp. and Michael Chan are jointly
and severally liable to pay the amount awarded to plaintiff
1. to pay the sum of ONE MILLION TWO HUNDRED ONE pursuant to the decision of November [28], 2002 which has
THOUSAND SEVEN HUNDRED TWENTY FOUR PESOS long been final and executory.
(P1,201,724.00) with legal interest at 12% per annum from
February 17, 1999 until full payment;
SO ORDERED.

2. to pay the sum of FIFTY THOUSAND PESOS


From the above order, KIC moved but was denied reconsideration in
(P50,000.00) as moral damages;
another Order dated June 7, 2007.

92
KIC went to the CA on a petition for certiorari to nullify the aforesaid In Carpio v. Doroja,13 the Court ruled that the deciding court has
March 12 and June 7, 2007 RTC Orders. supervisory control over the execution of its judgment:

On January 23, 2008, the CA rendered the assailed decision, the A case in which an execution has been issued is regarded as still
dispositive portion of which states: pending so that all proceedings on the execution are proceedings in
the suit. There is no question that the court which rendered the
judgment has a general supervisory control over its process of
WHEREFORE, premises considered, the petition is hereby DENIED
execution, and this power carries with it the right to determine every
and the assailed Orders dated March 12, 2007 and June 7, 2007 of the
question of fact and law which may be involved in the execution.
court a quo are both AFFIRMED. No costs.

We reiterated the above holding in Javier v. Court of Appeals14 in this


SO ORDERED.11
wise: "The said branch has a general supervisory control over its
processes in the execution of its judgment with a right to determine
The CA later denied KIC’s motion for reconsideration in the assailed every question of fact and law which may be involved in the execution."
resolution.
The court’s supervisory control does not, however, extend as to
Hence, the instant petition for review, with the following issues KIC authorize the alteration or amendment of a final and executory
raises for the Court’s consideration: decision, save for certain recognized exceptions, among which is the
correction of clerical errors. Else, the court violates the principle of
finality of judgment and its immutability, concepts which the Court, in
1. There is no legal basis for the [CA] to resolve and declare Tan v. Timbal,15 defined:
that petitioner’s Constitutional Right to Due Process was not
violated by the public respondent in rendering the Orders
dated March 12, 2007 and June 7, 2007 and in declaring As we held in Industrial Management International Development
petitioner to be liable for the judgment obligations of the Corporation vs. NLRC:
corporation "Kukan, Inc." to private respondent – as
petitioner is a stranger to the case and was never made a
It is an elementary principle of procedure that the resolution of the
party in the case before the trial court nor was it ever served
court in a given issue as embodied in the dispositive part of a decision
a summons and a copy of the complaint.
or order is the controlling factor as to settlement of rights of the parties.
Once a decision or order becomes final and executory, it is removed
2. There is no legal basis for the [CA] to resolve and declare from the power or jurisdiction of the court which rendered it to further
that the Orders dated March 12, 2007 and June 7, 2007 alter or amend it. It thereby becomes immutable and unalterable and
rendered by public respondent declaring the petitioner liable any amendment or alteration which substantially affects a final and
to the judgment obligations of the corporation "Kukan, Inc." executory judgment is null and void for lack of jurisdiction, including the
to private respondent are valid as said orders of the public entire proceedings held for that purpose. An order of execution which
respondent modify and/or amend the trial court’s final and varies the tenor of the judgment or exceeds the terms thereof is a
executory decision rendered on November 28, 2002. nullity. (Emphasis supplied.)

3. There is no legal basis for the [CA] to resolve and declare Republic v. Tango16 expounded on the same principle and its
that the Orders dated March 12, 2007 and June 7, 2007 exceptions:
rendered by public respondent declaring the petitioner [KIC]
and the corporation "Kukan, Inc." as one and the same, and,
Deeply ingrained in our jurisprudence is the principle that a decision
therefore, the Veil of Corporate Fiction between them be
that has acquired finality becomes immutable and unalterable. As
pierced – as the procedure undertaken by public respondent
such, it may no longer be modified in any respect even if the
which the [CA] upheld is not sanctioned by the Rules of
modification is meant to correct erroneous conclusions of fact or law
Court and/or established jurisprudence enunciated by this
and whether it will be made by the court that rendered it or by the
Honorable Supreme Court.12
highest court of the land. x x x

In gist, the issues to be resolved boil down to the question of, first,
The doctrine of finality of judgment is grounded on the fundamental
whether the trial court can, after the judgment against Kukan, Inc. has
principle of public policy and sound practice that, at the risk of
attained finality, execute it against the property of KIC; second,
occasional error, the judgment of courts and the award of quasi-judicial
whether the trial court acquired jurisdiction over KIC; and third, whether
agencies must become final on some definite date fixed by law. The
the trial and appellate courts correctly applied, under the premises, the
only exceptions to the general rule are the correction of clerical
principle of piercing the veil of corporate fiction.
errors, the so-called nunc pro tunc entries which cause no prejudice to
any party, void judgments, and whenever circumstances transpire after
The Ruling of the Court the finality of the decision which render its execution unjust and
inequitable. None of the exceptions obtains here to merit the review
sought. (Emphasis added.)
The petition is meritorious.

So, did the RTC, in breach of the doctrine of immutability and


First Issue: Against Whom Can a Final and
inalterability of judgment, order the execution of its final decision in a
Executory Judgment Be Executed
manner as would amount to its prohibited alteration or modification?

The preliminary question that must be answered is whether or not the We repair to the dispositive portion of the final and executory RTC
trial court can, after adjudging Kukan, Inc. liable for a sum of money in
decision. Pertinently, it provides:
a final and executory judgment, execute such judgment debt against
the property of KIC.
WHEREFORE, consistent with Section 5, Rule 18 of the 1997 Rules of
Civil Procedure, and by preponderance of evidence, judgment is
The poser must be answered in the negative. hereby rendered in favor of the plaintiff, ordering Kukan, Inc.:

93
1. to pay the sum of ONE MILLION TWO HUNDRED ONE service of summons or by the latter’s voluntary appearance and
THOUSAND SEVEN HUNDRED TWENTY FOUR PESOS submission to the authority of the former."
(P1,201,724.00) with legal interest at 12% per annum from
February 17, 1999 until full payment;
The court’s jurisdiction over a party-defendant resulting from his
voluntary submission to its authority is provided under Sec. 20, Rule 14
2. to pay the sum of FIFTY THOUSAND PESOS of the Rules, which states:
(P50,000.00) as moral damages;
Section 20. Voluntary appearance. – The defendant’s voluntary
3. to pay the sum of TWENTY THOUSAND PESOS appearance in the actions shall be equivalent to service of summons.
(P20,000.00) as reasonable attorney’s fees; and The inclusion in a motion to dismiss of other grounds aside from lack of
jurisdiction over the person of the defendant shall not be deemed a
voluntary appearance.
4. to pay the sum of SEVEN THOUSAND NINE HUNDRED
SIXTY PESOS and SIX CENTAVOS (P7,960.06) as litigation
expenses. To be sure, the CA’s ruling that any form of appearance by the party or
its counsel is deemed as voluntary appearance finds support in the
kindred Republic v. Ker & Co., Ltd.25 and De Midgely v. Ferandos.26
x x x x (Emphasis supplied.)

Republic and De Midgely, however, have already been modified if not


As may be noted, the above decision, in unequivocal terms, directed
altogether superseded27 by La Naval Drug Corporation v. Court of
Kukan, Inc. to pay the aforementioned awards to Morales. Thus,
Appeals,28 wherein the Court essentially ruled and elucidated on the
making KIC, thru the medium of a writ of execution, answerable for the
current view in our jurisdiction, to wit: "[A] special appearance before
above judgment liability is a clear case of altering a decision, an
the court––challenging its jurisdiction over the person through a motion
instance of granting relief not contemplated in the decision sought to
to dismiss even if the movant invokes other grounds––is not
be executed. And the change does not fall under any of the recognized
tantamount to estoppel or a waiver by the movant of his objection to
exceptions to the doctrine of finality and immutability of judgment. It is
jurisdiction over his person; and such is not constitutive of a voluntary
a settled rule that a writ of execution must conform to the fallo of the
submission to the jurisdiction of the court."29
judgment; as an inevitable corollary, a writ beyond the terms of the
judgment is a nullity.17
In the instant case, KIC was not made a party-defendant in Civil Case
No. 99-93173. Even if it is conceded that it raised affirmative defenses
Thus, on this ground alone, the instant petition can already be granted.
through its aforementioned pleadings, KIC never abandoned its
Nonetheless, an examination of the other issues raised by KIC would
challenge, however implicit, to the RTC’s jurisdiction over its person.
be proper.
The challenge was subsumed in KIC’s primary assertion that it was not
the same entity as Kukan, Inc. Pertinently, in its Comment and
Second Issue: Propriety of the RTC Opposition to Plaintiff’s Omnibus Motion dated May 20, 2003, KIC
Assuming Jurisdiction over KIC entered its "special but not voluntary appearance" alleging therein
that it was a different entity and has a separate legal personality from
Kukan, Inc. And KIC would consistently reiterate this assertion in all its
The next issue turns on the validity of the execution the trial court pleadings, thus effectively resisting all along the RTC’s jurisdiction of
authorized against KIC and its property, given that it was neither made its person. It cannot be overemphasized that KIC could not file before
a party nor impleaded in Civil Case No. 99-93173, let alone served with
the RTC a motion to dismiss and its attachments in Civil Case No. 99-
summons. In other words, did the trial court acquire jurisdiction over 93173, precisely because KIC was neither impleaded nor served with
KIC? summons. Consequently, KIC could only assert and claim through its
affidavits, comments, and motions filed by special appearance before
In the assailed decision, the appellate court deemed KIC to have the RTC that it is separate and distinct from Kukan, Inc.
voluntarily submitted itself to the jurisdiction of the trial court owing to
its filing of four (4) pleadings adverted to earlier, namely: (a) the
Following La Naval Drug Corporation,30 KIC cannot be deemed to have
Affidavit of Third-Party Claim;18 (b) the Comment and Opposition to waived its objection to the court’s lack of jurisdiction over its person. It
Plaintiff’s Omnibus Motion;19 (c) the Motion for Reconsideration of the would defy logic to say that KIC unequivocally submitted itself to the
RTC Order dated March 12, 2007;20 and (d) the Motion for Leave to
jurisdiction of the RTC when it strongly asserted that it and Kukan, Inc.
Admit Reply.21 The CA, citing Section 20, Rule 14 of the Rules of are different entities. In the scheme of things obtaining, KIC had no
Court, stated that "the procedural rule on service of summons can be other option but to insist on its separate identity and plead for relief
waived by voluntary submission to the court’s jurisdiction through any
consistent with that position.
form of appearance by the party or its counsel."22

Third Issue: Piercing the


We cannot give imprimatur to the appellate court’s appreciation of the Veil of Corporate Fiction
thrust of Sec. 20, Rule 14 of the Rules in concluding that the trial court
acquired jurisdiction over KIC.
The third and main issue in this case is whether or not the trial and
appellate courts correctly applied the principle of piercing the veil of
Orion Security Corporation v. Kalfam Enterprises, Inc. 23 explains how corporate entity––called also as disregarding the fiction of a separate
courts acquire jurisdiction over the parties in a civil case: juridical personality of a corporation––to support a conclusion that
Kukan, Inc. and KIC are but one and the same corporation with respect
Courts acquire jurisdiction over the plaintiffs upon the filing of the to the contract award referred to at the outset. This principle finds its
complaint. On the other hand, jurisdiction over the defendants in a civil context on the postulate that a corporation is an artificial being invested
case is acquired either through the service of summons upon them or with a personality separate and distinct from those of the stockholders
through their voluntary appearance in court and their submission to its and from other corporations to which it may be connected or related.31
authority. (Emphasis supplied.)
In Pantranco Employees Association (PEA-PTGWO) v. National Labor
In the fairly recent Palma v. Galvez,24 the Court reiterated its holding Relations Commission,32 the Court revisited the subject principle of
in Orion Security Corporation, stating: "[I]n civil cases, the trial court piercing the veil of corporate fiction and wrote:
acquires jurisdiction over the person of the defendant either by the

94
Under the doctrine of "piercing the veil of corporate fiction," the court The implication of the above comment is twofold: (1) the court must
looks at the corporation as a mere collection of individuals or an first acquire jurisdiction over the corporation or corporations involved
aggregation of persons undertaking business as a group, disregarding before its or their separate personalities are disregarded; and (2) the
the separate juridical personality of the corporation unifying the group. doctrine of piercing the veil of corporate entity can only be raised
Another formulation of this doctrine is that when two business during a full-blown trial over a cause of action duly commenced
enterprises are owned, conducted and controlled by the same parties, involving parties duly brought under the authority of the court by way of
both law and equity will, when necessary to protect the rights of third service of summons or what passes as such service.
parties, disregard the legal fiction that two corporations are distinct
entities and treat them as identical or as one and the same.
The issue of jurisdiction or the lack of it over KIC has already been
discussed. Anent the matter of the time and manner of raising the
Whether the separate personality of the corporation should be principle in question, it is undisputed that no full-blown trial involving
pierced hinges on obtaining facts appropriately pleaded or KIC was had when the RTC disregarded the corporate veil of KIC. The
proved. However, any piercing of the corporate veil has to be done reason for this actuality is simple and undisputed: KIC was not
with caution, albeit the Court will not hesitate to disregard the corporate impleaded in Civil Case No. 99-93173 and that the RTC did not acquire
veil when it is misused or when necessary in the interest of justice. x x jurisdiction over it. It was dragged to the case after it reacted to the
x (Emphasis supplied.) improper execution of its properties and veritably hauled to court, not
thru the usual process of service of summons, but by mere motion of a
party with whom it has no privity of contract and after the decision in
The same principle was the subject and discussed in Rivera v. United
the main case had already become final and executory. As to the
Laboratories, Inc.:
propriety of a plea for the application of the principle by mere motion,
the following excerpts are instructive:
While a corporation may exist for any lawful purpose, the law will
regard it as an association of persons or, in case of two corporations,
Generally, a motion is appropriate only in the absence of remedies by
merge them into one, when its corporate legal entity is used as a cloak
regular pleadings, and is not available to settle important questions of
for fraud or illegality. This is the doctrine of piercing the veil of
law, or to dispose of the merits of the case. A motion is usually a
corporate fiction. The doctrine applies only when such corporate fiction
proceeding incidental to an action, but it may be a wholly distinct or
is used to defeat public convenience, justify wrong, protect fraud, or
independent proceeding. A motion in this sense is not within this
defend crime, or when it is made as a shield to confuse the legitimate
discussion even though the relief demanded is denominated an
issues, or where a corporation is the mere alter ego or business
"order."
conduit of a person, or where the corporation is so organized and
controlled and its affairs are so conducted as to make it merely an
instrumentality, agency, conduit or adjunct of another corporation. A motion generally relates to procedure and is often resorted to in
order to correct errors which have crept in along the line of the principal
action’s progress. Generally, where there is a procedural defect in a
To disregard the separate juridical personality of a corporation, the
proceeding and no method under statute or rule of court by which it
wrongdoing must be established clearly and convincingly. It cannot be
may be called to the attention of the court, a motion is an appropriate
presumed.33 (Emphasis supplied.)
remedy. In many jurisdictions, the motion has replaced the common-
law pleas testing the sufficiency of the pleadings, and various common-
Now, as before the appellate court, petitioner KIC maintains that the law writs, such as writ of error coram nobis and audita querela. In
RTC violated its right to due process when, in the execution of its some cases, a motion may be one of several remedies available. For
November 28, 2002 Decision, the court authorized the issuance of the example, in some jurisdictions, a motion to vacate an order is a remedy
writ against KIC for Kukan, Inc.’s judgment debt, albeit KIC has never alternative to an appeal therefrom.
been a party to the underlying suit. As a counterpoint, Morales argues
that KIC’s specific concern on due process and on the validity of the
Statutes governing motions are given a liberal
writ to execute the RTC’s November 28, 2002 Decision would be
construction.36 (Emphasis supplied.)
mooted if it were established that KIC and Kukan, Inc. are indeed one
and the same corporation.
The bottom line issue of whether Morales can proceed against KIC for
the judgment debt of Kukan, Inc.––assuming hypothetically that he
Morales’ contention is untenable.
can, applying the piercing the corporate veil principle––resolves itself
into the question of whether a mere motion is the appropriate vehicle
The principle of piercing the veil of corporate fiction, and the resulting for such purpose.
treatment of two related corporations as one and the same juridical
person with respect to a given transaction, is basically applied only to
Verily, Morales espouses the application of the principle of piercing the
determine established liability;34 it is not available to confer on the court
corporate veil to hold KIC liable on theory that Kukan, Inc. was out to
a jurisdiction it has not acquired, in the first place, over a party not
defraud him through the use of the separate and distinct personality of
impleaded in a case. Elsewise put, a corporation not impleaded in a
another corporation, KIC. In net effect, Morales’ adverted motion to
suit cannot be subject to the court’s process of piercing the veil of its
pierce the veil of corporate fiction dated January 3, 2007 stated a new
corporate fiction. In that situation, the court has not acquired
cause of action, i.e., for the liability of judgment debtor Kukan, Inc. to
jurisdiction over the corporation and, hence, any proceedings taken
be borne by KIC on the alleged identity of the two corporations. This
against that corporation and its property would infringe on its right to
new cause of action should be properly ventilated in another complaint
due process. Aguedo Agbayani, a recognized authority on Commercial
and subsequent trial where the doctrine of piercing the corporate veil
Law, stated as much:
can, if appropriate, be applied, based on the evidence adduced.
Establishing the claim of Morales and the corresponding liability of KIC
23. Piercing the veil of corporate entity applies to determination of for Kukan Inc.’s indebtedness could hardly be the subject, under the
liability not of jurisdiction. x x x premises, of a mere motion interposed after the principal action against
Kukan, Inc. alone had peremptorily been terminated. After all, a
complaint is one where the plaintiff alleges causes of action.
This is so because the doctrine of piercing the veil of corporate fiction
comes to play only during the trial of the case after the court has
already acquired jurisdiction over the corporation. Hence, before this In any event, the principle of piercing the veil of corporate fiction finds
doctrine can be applied, based on the evidence presented, it is no application to the instant case.
imperative that the court must first have jurisdiction over the
corporation.35 x x x (Emphasis supplied.)

95
As a general rule, courts should be wary of lifting the corporate veil While it is true that a corporation has a separate and distinct
between corporations, however related. Philippine National Bank v. personality from its stockholder, director and officers, the law expressly
Andrada Electric Engineering Company37 explains why: provides for an exception. When Michael Chan, the Managing Director
of defendant Kukan, Inc. (majority stockholder of the newly formed
corporation [KIC]) confirmed the award to plaintiff to supply and install
A corporation is an artificial being created by operation of law. x x x It
interior signages in the Enterprise Center he (Michael Chan, Managing
has a personality separate and distinct from the persons composing it,
Director of defendant Kukan, Inc.) knew that there was no sufficient
as well as from any other legal entity to which it may be related. This is
corporate funds to pay its obligation/account, thus implying bad faith on
basic.
his part and fraud in contracting the obligation. Michael Chan neither
returned the interior signages nor tendered payment to the plaintiff.
Equally well-settled is the principle that the corporate mask may be This circumstance may warrant the piercing of the veil of corporation
removed or the corporate veil pierced when the corporation is just an fiction. Having been guilty of bad faith in the management of corporate
alter ego of a person or of another corporation. For reasons of public matters the corporate trustee, director or officer may be held personally
policy and in the interest of justice, the corporate veil will justifiably be liable. x x x
impaled only when it becomes a shield for fraud, illegality or inequity
committed against third persons.
Since fraud is a state of mind, it need not be proved by direct evidence
but may be inferred from the circumstances of the case. x x x [A]nd the
Hence, any application of the doctrine of piercing the corporate veil circumstances are: the signature of Michael Chan, Managing Director
should be done with caution. A court should be mindful of the milieu of Kukan, Inc. appearing in the confirmation of the award sent to the
where it is to be applied. It must be certain that the corporate fiction plaintiff; signature of Chan Kai Kit, a British National appearing in the
was misused to such an extent that injustice, fraud, or crime was Articles of Incorporation and signature of Michael Chan also a British
committed against another, in disregard of its rights. The wrongdoing National appearing in the Articles of Incorporation [of] Kukan
must be clearly and convincingly established; it cannot be presumed. International Corp. give the impression that they are one and the same
Otherwise, an injustice that was never unintended may result from an person, that Michael Chan and Chan Kai Kit are both majority
erroneous application. stockholders of Kukan International Corp. and Kukan, Inc. holding 40%
of the stocks; that Kukan International Corp. is practically doing the
same kind of business as that of Kukan, Inc.39 (Emphasis supplied.)
This Court has pierced the corporate veil to ward off a judgment credit,
to avoid inclusion of corporate assets as part of the estate of the
decedent, to escape liability arising from a debt, or to perpetuate fraud As is apparent from its disquisition, the RTC brushed aside the
and/or confuse legitimate issues either to promote or to shield unfair separate corporate existence of Kukan, Inc. and KIC on the main
objectives or to cover up an otherwise blatant violation of the argument that Michael Chan owns 40% of the common shares of both
prohibition against forum-shopping. Only in these and similar corporations, obviously oblivious that overlapping stock ownership is a
instances may the veil be pierced and disregarded. (Emphasis common business phenomenon. It must be remembered, however,
supplied.) that KIC’s properties were the ones seized upon levy on execution and
not that of Kukan, Inc. or of Michael Chan for that matter. Mere
ownership by a single stockholder or by another corporation of a
In fine, to justify the piercing of the veil of corporate fiction, it must be
substantial block of shares of a corporation does not, standing alone,
shown by clear and convincing proof that the separate and distinct provide sufficient justification for disregarding the separate corporate
personality of the corporation was purposefully employed to evade a personality.40 For this ground to hold sway in this case, there must be
legitimate and binding commitment and perpetuate a fraud or like
proof that Chan had control or complete dominion of Kukan and KIC’s
wrongdoings. To be sure, the Court has, on numerous finances, policies, and business practices; he used such control to
occasions,38 applied the principle where a corporation is dissolved and commit fraud; and the control was the proximate cause of the financial
its assets are transferred to another to avoid a financial liability of the
loss complained of by Morales. The absence of any of the elements
first corporation with the result that the second corporation should be prevents the piercing of the corporate veil.41 And indeed, the records
considered a continuation and successor of the first entity. do not show the presence of these elements.

In those instances when the Court pierced the veil of corporate fiction On the other hand, the CA held:
of two corporations, there was a confluence of the following factors:

In the present case, the facts disclose that Kukan, Inc. entered into a
1. A first corporation is dissolved; contractual obligation x x x worth more than three million pesos
although it had only Php5,000.00 paid-up capital; [KIC] was
2. The assets of the first corporation is transferred to a incorporated shortly before Kukan, Inc. suddenly ceased to appear and
second corporation to avoid a financial liability of the first participate in the trial; [KIC’s] purpose is related and somewhat akin to
corporation; and that of Kukan, Inc.; and in [KIC] Michael Chan, a.k.a., Chan Kai Kit,
holds forty percent of the outstanding stocks, while he formerly held the
same amount of stocks in Kukan Inc. These would lead to the
3. Both corporations are owned and controlled by the same inescapable conclusion that Kukan, Inc. committed fraudulent
persons such that the second corporation should be representation by awarding to the private respondent the contract with
considered as a continuation and successor of the first full knowledge that it was not in a position to comply with the obligation
corporation. it had assumed because of inadequate paid-up capital. It bears
stressing that shareholders should in good faith put at the risk of the
In the instant case, however, the second and third factors are business, unencumbered capital reasonably adequate for its
conspicuously absent. There is, therefore, no compelling justification prospective liabilities. The capital should not be illusory or trifling
for disregarding the fiction of corporate entity separating Kukan, Inc. compared with the business to be done and the risk of loss.
from KIC. In applying the principle, both the RTC and the CA miserably
failed to identify the presence of the abovementioned factors. Further, it is clear that [KIC] is a continuation and successor of Kukan,
Consider: Inc. Michael Chan, a.k.a. Chan Kai Kit has the largest block of shares
in both business enterprises. The emergence of the former was
The RTC disregarded the separate corporate personalities of Kukan, cleverly timed with the hasty withdrawal of the latter during the trial to
Inc. and KIC based on the following premises and arguments: avoid the financial liability that was eventually suffered by the latter.
The two companies have a related business purpose. Considering
these circumstances, the obvious conclusion is that the creation of

96
Kukan International Corporation served as a device to evade the between Kukan, Inc. and KIC. As illustrated, these circumstances are
obligation incurred by Kukan, Inc. and yet profit from the goodwill insufficient to establish the identity of KIC as the alter ego or successor
attained by the name "Kukan" by continuing to engage in the same line of Kukan, Inc.
of business with the same list of clients.42 (Emphasis supplied.)
It bears reiterating that piercing the veil of corporate fiction is frowned
Evidently, the CA found the meager paid-up capitalization of Kukan, upon. Accordingly, those who seek to pierce the veil must clearly
Inc. and the similarity of the business activities in which both establish that the separate and distinct personalities of the corporations
corporations are engaged as a jumping board to its conclusion that the are set up to justify a wrong, protect fraud, or perpetrate a deception. In
creation of KIC "served as a device to evade the obligation incurred by the concrete and on the assumption that the RTC has validly acquired
Kukan, Inc." The appellate court, however, left a gaping hole by failing jurisdiction over the party concerned, Morales ought to have proved by
to demonstrate that Kukan, Inc. and its stockholders defrauded convincing evidence that Kukan, Inc. was collapsed and thereafter KIC
Morales. In fine, there is no showing that the incorporation, and the purposely formed and operated to defraud him. Morales has not to us
separate and distinct personality, of KIC was used to defeat Morales’ discharged his burden.
right to recover from Kukan, Inc. Judging from the records, no serious
attempt was made to levy on the properties of Kukan, Inc. Morales
WHEREFORE, the petition is hereby GRANTED. The CA’s January
could not, thus, validly argue that Kukan, Inc. tried to avoid liability or
23, 2008 Decision and April 16, 2008 Resolution in CA-G.R. SP No.
had no property against which to proceed.
100152 are hereby REVERSED and SET ASIDE. The levy placed
upon the personal properties of Kukan International Corporation is
Morales further contends that Kukan, Inc.’s closure is evidenced by its hereby ordered lifted and the personal properties ordered returned to
failure to file its 2001 General Information Sheet (GIS) with the Kukan International Corporation. The RTC of Manila, Branch 21 is
Securities and Exchange Commission. However, such fact does not hereby directed to execute the RTC Decision dated November 28,
necessarily mean that Kukan, Inc. had altogether ceased operations, 2002 against Kukan, Inc. with reasonable dispatch.
as Morales would have this Court believe, for it is stated on the face of
the GIS that it is only upon a failure to file the corporate GIS for five (5)
No costs.
consecutive years that non-operation shall be presumed.

SO ORDERED.
The fact that Kukan, Inc. entered into a PhP 3.3 million contract when it
only had a paid-up capital of PhP 5,000 is not an indication of the intent
on the part of its management to defraud creditors. Paid-up capital is
merely seed money to start a corporation or a business entity. As in
this case, it merely represented the capitalization upon incorporation in
1997 of Kukan, Inc. Paid-up capitalization of PhP 5,000 is not and
should not be taken as a reflection of the firm’s capacity to meet its
recurrent and long-term obligations. It must be borne in mind that the
equity portion cannot be equated to the viability of a business concern,
for the best test is the working capital which consists of the liquid
assets of a given business relating to the nature of the business
concern.lawphil

Neither should the level of paid-up capital of Kukan, Inc. upon its
incorporation be viewed as a badge of fraud, for it is in compliance with
Sec. 13 of the Corporation Code,43 which only requires a minimum
paid-up capital of PhP 5,000.1avvphi1

The suggestion that KIC is but a continuation and successor of Kukan,


Inc., owned and controlled as they are by the same stockholders,
stands without factual basis. It is true that Michael Chan, a.k.a. Chan
Kai Kit, owns 40% of the outstanding capital stock of both corporations.
But such circumstance, standing alone, is insufficient to establish
identity. There must be at least a substantial identity of stockholders for
both corporations in order to consider this factor to be constitutive of
corporate identity.

It would not avail Morales any to rely44 on General Credit Corporation


v. Alsons Development and Investment Corporation.45 General Credit
Corporation is factually not on all fours with the instant case. There, the
common stockholders of the corporations represented 90% of the
outstanding capital stock of the companies, unlike here where Michael
Chan merely represents 40% of the outstanding capital stock of both
KIC and Kukan, Inc., not even a majority of it. In that case, moreover,
evidence was adduced to support the finding that the funds of the
second corporation came from the first. Finally, there was proof in
General Credit Corporation of complete control, such that one
corporation was a mere dummy or alter ego of the other, which is
absent in the instant case.

Evidently, the aforementioned case relied upon by Morales cannot


justify the application of the principle of piercing the veil of corporate
fiction to the instant case. As shown by the records, the name Michael
Chan, the similarity of business activities engaged in, and incidentally
the word "Kukan" appearing in the corporate names provide the nexus

97
G.R. No. 167530 March 13, 2013 Subsequent to the filing of the complaint, Hercon, Inc. was acquired by
HRCC in a merger. This prompted the amendment of the complaint to
substitute HRCC for Hercon, Inc.12
PHILIPPINE NATIONAL BANK, Petitioner,
vs.
HYDRO RESOURCES CONTRACTORS Thereafter, on December 8, 1986, then President Corazon C. Aquino
CORPORATION, Respondent. issued Proclamation No. 50 creating the APT for the expeditious
disposition and privatization of certain government corporations and/or
the assets thereof. Pursuant to the said Proclamation, on February 27,
x-----------------------x
1987, DBP and PNB executed their respective deeds of transfer in
favor of the National Government assigning, transferring and
G.R. No. 167561 conveying certain assets and liabilities, including their respective
stakes in NMIC.13 In turn and on even date, the National Government
transferred the said assets and liabilities to the APT as trustee under a
ASSET PRIVATIZATION TRUST, Petitioner,
Trust Agreement.14 Thus, the complaint was amended for the second
vs. time to implead and include the APT as a defendant.
HYDRO RESOURCES CONTRACTORS
CORPORATION, Respondent.
In its answer,15 NMIC claimed that HRCC had no cause of action. It
also asserted that its contract with HRCC was entered into by its then
x-----------------------x President without any authority. Moreover, the said contract allegedly
failed to comply with laws, rules and regulations concerning
G.R. No. 167603 government contracts. NMIC further claimed that the contract amount
was manifestly excessive and grossly disadvantageous to the
government. NMIC made counterclaims for the amounts already paid
DEVELOPMENT BANK OF THE PHILIPPINES, Petitioner, to Hercon, Inc. and attorney’s fees, as well as payment for equipment
vs. rental for four trucks, replacement of parts and other services, and
HYDRO RESOURCES CONTRACTORS damage to some of NMIC’s properties.16
CORPORATION, Respondent.

For its part, DBP’s answer17 raised the defense that HRCC had no
DECISION cause of action against it because DBP was not privy to HRCC’s
contract with NMIC. Moreover, NMIC’s juridical personality is separate
LEONARDO-DE CASTRO, J.: from that of DBP. DBP further interposed a counterclaim for attorney’s
fees.18
These petitions for review on certiorari1 assail the Decision2 dated
November 30, 2004 and the Resolution3 dated March 22, 2005 of the PNB’s answer19 also invoked lack of cause of action against it. It also
Court of Appeals in CA-G.R. CV No. 57553. The said Decision affirmed raised estoppel on HRCC’s part and laches as defenses, claiming that
the Decision4 dated November 6, 1995 of the Regional Trial Court the inclusion of PNB in the complaint was the first time a demand for
(RTC) of Makati City, Branch 62, granting a judgment award of payment was made on it by HRCC. PNB also invoked the separate
₱8,370,934.74, plus legal interest, in favor of respondent Hydro juridical personality of NMIC and made counterclaims for moral
Resources Contractors Corporation (HRCC) with the modification that damages and attorney’s fees.20
the Privatization and Management Office (PMO), successor of
petitioner Asset Privatization Trust (APT),5 has been held solidarily APT set up the following defenses in its answer21: lack of cause of
liable with Nonoc Mining and Industrial Corporation (NMIC)6 and action against it, lack of privity between Hercon, Inc. and APT, and the
petitioners Philippine National Bank (PNB) and Development Bank of National Government’s preferred lien over the assets of NMIC.22
the Philippines (DBP), while the Resolution denied reconsideration
separately prayed for by PNB, DBP, and APT.
After trial, the RTC of Makati rendered a Decision dated November 6,
1995 in favor of HRCC. It pierced the corporate veil of NMIC and held
Sometime in 1984, petitioners DBP and PNB foreclosed on certain DBP and PNB solidarily liable with NMIC:
mortgages made on the properties of Marinduque Mining and Industrial
Corporation (MMIC). As a result of the foreclosure, DBP and PNB
acquired substantially all the assets of MMIC and resumed the On the issue of whether or not there is sufficient ground to pierce the
business operations of the defunct MMIC by organizing NMIC. 7 DBP veil of corporate fiction, this Court likewise finds for the plaintiff.
and PNB owned 57% and 43% of the shares of NMIC, respectively,
except for five qualifying shares.8 As of September 1984, the members
From the documentary evidence adduced by the plaintiff, some of
of the Board of Directors of NMIC, namely, Jose Tengco, Jr., Rolando which were even adopted by defendants and DBP and PNB as their
Zosa, Ruben Ancheta, Geraldo Agulto, and Faustino Agbada, were own evidence (Exhibits "I", "I-1", "I-2", "I-3", "I-4", "I-5", "I5-A", "I-5-B",
either from DBP or PNB.9
"I-5-C", "I-5-D" and submarkings, inclusive), it had been established
that except for five (5) qualifying shares, NMIC is owned by defendants
Subsequently, NMIC engaged the services of Hercon, Inc., for NMIC’s DBP and PNB, with the former owning 57% thereof, and the latter
Mine Stripping and Road Construction Program in 1985 for a total 43%. As of September 24, 1984, all the members of NMIC’s Board of
contract price of ₱35,770,120. After computing the payments already Directors, namely, Messrs. Jose Tengco, Jr., Rolando M. Zosa, Ruben
made by NMIC under the program and crediting the NMIC’s Ancheta, Geraldo Agulto, and Faustino Agbada are either from DBP or
receivables from PNB (Exhibits "I-5", "I-5-C", "I-5-D").

Hercon, Inc., the latter found that NMIC still has an unpaid balance of The business of NMIC was then also being conducted and controlled
₱8,370,934.74.10 Hercon, Inc. made several demands on NMIC, by both DBP and PNB. In fact, it was Rolando M. Zosa, then Governor
including a letter of final demand dated August 12, 1986, and when of DBP, who was signing and entering into contracts with third persons,
these were not heeded, a complaint for sum of money was filed in the on behalf of NMIC.
RTC of Makati, Branch 136 seeking to hold petitioners NMIC, DBP,
and PNB solidarily liable for the amount owing Hercon, Inc.11 The case In this jurisdiction, it is well-settled that "where it appears that the
was docketed as Civil Case No. 15375. business enterprises are owned, conducted and controlled by the
same parties, both law and equity will, when necessary to protect the
98
rights of third persons, disregard legal fiction that two (2) corporations Resources Contractors Corporation in the amount of ₱8,370,934.74
are distinct entities, and treat them as identical." (Phil. Veterans with legal interest from date of demand is hereby AFFIRMED, but the
Investment Development Corp. vs. CA, 181 SCRA 669). dismissal of the case as against Assets Privatization Trust is
REVERSED, and its successor the Privatization and Management
Office is INCLUDED as one of those jointly and severally liable for
From all indications, it appears that NMIC is a mere adjunct, business
such indebtedness. The award of attorney’s fees is DELETED.
conduit or alter ego of both DBP and PNB. Thus, the DBP and PNB
are jointly and severally liable with NMIC for the latter’s unpaid
obligations to plaintiff.23 All other claims and counter-claims are hereby DISMISSED.

Having found DBP and PNB solidarily liable with NMIC, the dispositive Costs against appellants.28
portion of the Decision of the trial court reads:
The respective motions for reconsideration of DBP, PNB, and APT
WHEREFORE, in view of the foregoing, judgment is hereby rendered were denied.29
in favor of the plaintiff HYDRO RESOURCES CONTRACTORS
CORPORATION and against the defendants NONOC
Hence, these consolidated petitions.30

MINING AND INDUSTRIAL CORPORATION, DEVELOPMENT BANK


All three petitioners assert that NMIC is a corporate entity with a
OF THE PHILIPPINES and PHILIPPINE NATIONAL BANK, ordering
juridical personality separate and distinct from both PNB and DBP.
the aforenamed defendants, to pay the plaintiff jointly and severally,
They insist that the majority ownership by DBP and PNB of NMIC is
the sum of ₱8,370,934.74 plus legal interest thereon from date of
not a sufficient ground for disregarding the separate corporate
demand, and attorney’s fees equivalent to 25% of the judgment award.
personality of NMIC because NMIC was not a mere adjunct, business
conduit or alter ego of DBP and PNB. According to them, the
The complaint against APT is hereby dismissed. However, APT, as application of the doctrine of piercing the corporate veil is unwarranted
trustee of NONOC MINING AND INDUSTRIAL CORPORATION is as nothing in the records would show that the ownership and control of
directed to ensure compliance with this Decision. 24 the shareholdings of NMIC by DBP and PNB were used to commit
fraud, illegality or injustice. In the absence of evidence that the stock
control by DBP and PNB over NMIC was used to commit some fraud
DBP and PNB filed their respective appeals in the Court of Appeals.
or a wrong and that said control was the proximate cause of the injury
Both insisted that it was wrong for the RTC to pierce the veil of NMIC’s
sustained by HRCC, resort to the doctrine of "piercing the veil of
corporate personality and hold DBP and PNB solidarily liable with
corporate entity" is misplaced.31
NMIC.25

DBP and PNB further argue that, assuming they may be held solidarily
The Court of Appeals rendered the Decision dated November 30,
liable with NMIC to pay NMIC’s exclusive and separate corporate
2004, affirmed the piercing of the veil of the corporate personality of
indebtedness to HRCC, such liability of the two banks was transferred
NMIC and held DBP, PNB, and APT solidarily liable with NMIC. In
to and assumed by the National Government through the APT, now the
particular, the Court of Appeals made the following findings:
PMO, under the respective deeds of transfer both dated February 27,
1997 executed by DBP and PNB pursuant to Proclamation No. 50
In the case before Us, it is indubitable that [NMIC] was owned by dated December 8, 1986 and Administrative Order No. 14 dated
appellants DBP and PNB to the extent of 57% and 43% respectively; February 3, 1987.32
that said two (2) appellants are the only stockholders, with the
qualifying stockholders of five (5) consisting of its own officers and
For its part, the APT contends that, in the absence of an unqualified
included in its charter merely to comply with the requirement of the law
assumption by the National Government of all liabilities incurred by
as to number of incorporators; and that the directorates of DBP, PNB
NMIC, the National Government through the APT could not be held
and [NMIC] are interlocked.
liable for NMIC’s contractual liability. The APT asserts that HRCC had
not sufficiently shown that the APT is the successor-in-interest of all
xxxx the liabilities of NMIC, or of DBP and PNB as transferors, and that the
adjudged liability is included among the liabilities assigned and
transferred by DBP and PNB in favor of the National Government.33
We find it therefore correct for the lower court to have ruled that:

HRCC counters that both the RTC and the CA correctly applied the
"From all indications, it appears that NMIC is a mere adjunct, business doctrine of "piercing the veil of corporate fiction." It claims that NMIC
conduit or alter ego of both DBP and PNB. Thus, the DBP and PNB
was the alter ego of DBP and PNB which owned, conducted and
are jointly and severally liable with NMIC for the latter’s unpaid controlled the business of NMIC as shown by the following
obligation to plaintiff."26 (Citation omitted.) circumstances: NMIC was owned by DBP and PNB, the officers of
DBP and PNB were also the officers of NMIC, and DBP and PNB
The Court of Appeals then concluded that, "in keeping with the concept financed the operations of NMIC. HRCC further argues that a parent
of justice and fair play," the corporate veil of NMIC should be pierced, corporation may be held liable for the contracts or obligations of its
ratiocinating: subsidiary corporation where the latter is a mere agency,
instrumentality or adjunct of the parent corporation.34
For to treat NMIC as a separate legal entity from DBP and PNB for the
purpose of securing beneficial contracts, and then using such separate Moreover, HRCC asserts that the APT was properly held solidarily
entity to evade the payment of a just debt, would be the height of liable with DBP, PNB, and NMIC because the APT assumed the
injustice and iniquity. Surely that could not have been the intendment obligations of DBP and PNB as the successor-in-interest of the said
of the law with respect to corporations. x x x. 27 banks with respect to the assets and liabilities of NMIC.35 As trustee of
the Republic of the Philippines, the APT also assumed the
responsibility of the Republic pursuant to the following provision of
The dispositive portion of the Decision of the Court of Appeals reads: Section 2.02 of the respective deeds of transfer executed by DBP and
PNB in favor of the Republic:
WHEREFORE, premises considered, the Decision appealed from is
hereby MODIFIED. The judgment in favor of appellee Hydro SECTION 2. TRANSFER OF BANK’S LIABILITIES
9
xxxx In this connection, case law lays down a three-pronged test to
determine the application of the alter ego theory, which is also known
as the instrumentality theory, namely:
2.02 With respect to the Bank’s liabilities which are contingent and
those liabilities where the Bank’s creditors consent to the transfer
thereof is not obtained, said liabilities shall remain in the books of the (1) Control, not mere majority or complete stock control, but
BANK with the GOVERNMENT funding the payment thereof.36 complete domination, not only of finances but of policy and
business practice in respect to the transaction attacked so
that the corporate entity as to this transaction had at the time
After a careful review of the case, this Court finds the petitions
no separate mind, will or existence of its own;
impressed with merit.

(2) Such control must have been used by the defendant to


A corporation is an artificial entity created by operation of law. It
commit fraud or wrong, to perpetuate the violation of a
possesses the right of succession and such powers, attributes, and
statutory or other positive legal duty, or dishonest and unjust
properties expressly authorized by law or incident to its existence. 37 It
act in contravention of plaintiff’s legal right; and
has a personality separate and distinct from that of its stockholders and
from that of other corporations to which it may be connected. 38 As a
consequence of its status as a distinct legal entity and as a result of a (3) The aforesaid control and breach of duty must have
conscious policy decision to promote capital formation,39 a corporation proximately caused the injury or unjust loss complained
incurs its own liabilities and is legally responsible for payment of its of.50 (Emphases omitted.)
obligations.40 In other words, by virtue of the separate juridical
personality of a corporation, the corporate debt or credit is not the debt
The first prong is the "instrumentality" or "control" test. This test
or credit of the stockholder.41 This protection from liability for
requires that the subsidiary be completely under the control and
shareholders is the principle of limited liability.42
domination of the parent.51 It examines the parent corporation’s
relationship with the subsidiary.52 It inquires whether a subsidiary
Equally well-settled is the principle that the corporate mask may be corporation is so organized and controlled and its affairs are so
removed or the corporate veil pierced when the corporation is just an conducted as to make it a mere instrumentality or agent of the parent
alter ego of a person or of another corporation. For reasons of public corporation such that its separate existence as a distinct corporate
policy and in the interest of justice, the corporate veil will justifiably be entity will be ignored.53 It seeks to establish whether the subsidiary
impaled only when it becomes a shield for fraud, illegality or inequity corporation has no autonomy and the parent corporation, though acting
committed against third persons.43 through the subsidiary in form and appearance, "is operating the
business directly for itself."54
However, the rule is that a court should be careful in assessing the
milieu where the doctrine of the corporate veil may be applied. The second prong is the "fraud" test. This test requires that the parent
Otherwise an injustice, although unintended, may result from its corporation’s conduct in using the subsidiary corporation be unjust,
erroneous application.44 Thus, cutting through the corporate cover fraudulent or wrongful.55 It examines the relationship of the plaintiff to
requires an approach characterized by due care and caution: the corporation.56 It recognizes that piercing is appropriate only if the
parent corporation uses the subsidiary in a way that harms the plaintiff
creditor.57 As such, it requires a showing of "an element of injustice or
Hence, any application of the doctrine of piercing the corporate veil
fundamental unfairness."58
should be done with caution. A court should be mindful of the milieu
where it is to be applied. It must be certain that the corporate fiction
was misused to such an extent that injustice, fraud, or crime was The third prong is the "harm" test. This test requires the plaintiff to
committed against another, in disregard of its rights. The wrongdoing show that the defendant’s control, exerted in a fraudulent, illegal or
must be clearly and convincingly established; it cannot be presumed. x otherwise unfair manner toward it, caused the harm suffered. 59 A
x x.45 (Emphases supplied; citations omitted.) causal connection between the fraudulent conduct committed through
the instrumentality of the subsidiary and the injury suffered or the
damage incurred by the plaintiff should be established. The plaintiff
Sarona v. National Labor Relations Commission46 has defined the
must prove that, unless the corporate veil is pierced, it will have been
scope of application of the doctrine of piercing the corporate veil:
treated unjustly by the defendant’s exercise of control and improper
use of the corporate form and, thereby, suffer damages. 60
The doctrine of piercing the corporate veil applies only in three (3)
basic areas, namely: 1) defeat of public convenience as when the
To summarize, piercing the corporate veil based on the alter ego
corporate fiction is used as a vehicle for the evasion of an existing
theory requires the concurrence of three elements: control of the
obligation; 2) fraud cases or when the corporate entity is used to justify
corporation by the stockholder or parent corporation, fraud or
a wrong, protect fraud, or defend a crime; or 3) alter ego cases, where
fundamental unfairness imposed on the plaintiff, and harm or damage
a corporation is merely a farce since it is a mere alter ego or business
caused to the plaintiff by the fraudulent or unfair act of the corporation.
conduit of a person, or where the corporation is so organized and
The absence of any of these elements prevents piercing the corporate
controlled and its affairs are so conducted as to make it merely an
veil.61
instrumentality, agency, conduit or adjunct of another corporation.
(Citation omitted.)
This Court finds that none of the tests has been satisfactorily met in
this case.
Here, HRCC has alleged from the inception of this case that DBP and
PNB (and the APT as assignee of DBP and PNB) should be held
solidarily liable for using NMIC as alter ego.47 The RTC sustained the In applying the alter ego doctrine, the courts are concerned with reality
allegation of HRCC and pierced the corporate veil of NMIC pursuant to and not form, with how the corporation operated and the individual
the alter ego theory when it concluded that NMIC "is a mere adjunct, defendant’s relationship to that operation.62 With respect to the control
business conduit or alter ego of both DBP and PNB."48 The Court of element, it refers not to paper or formal control by majority or even
Appeals upheld such conclusion of the trial court. 49 In other words, complete stock control but actual control which amounts to "such
both the trial and appellate courts relied on the alter ego theory when domination of finances, policies and practices that the controlled
they disregarded the separate corporate personality of NMIC. corporation has, so to speak, no separate mind, will or existence of its
own, and is but a conduit for its principal."63 In addition, the control
must be shown to have been exercised at the time the acts complained
of took place.64

10
Both the RTC and the Court of Appeals applied the alter ego theory of NMIC’s board of directors, four were nominees of either DBP or PNB
and penetrated the corporate cover of NMIC based on two factors: (1) and only one was a nominee of both DBP and PNB.75 Only two
the ownership by DBP and PNB of effectively all the stocks of NMIC, members of the board of directors of NMIC, Jose Tengco, Jr. and
and (2) the alleged interlocking directorates of DBP, PNB and Rolando Zosa, were established to be members of the board of
NMIC.65 Unfortunately, the conclusion of the trial and appellate courts governors of DBP and none was proved to be a member of the board
that the DBP and PNB fit the alter ego theory with respect to NMIC’s of directors of PNB.76 No director of NMIC was shown to be also sitting
transaction with HRCC on the premise of complete stock ownership simultaneously in the board of governors/directors of both DBP and
and interlocking directorates involved a quantum leap in logic and law PNB.
exposing a gap in reason and fact.
In reaching its conclusion of an alter ego relationship between DBP
While ownership by one corporation of all or a great majority of stocks and PNB on the one hand and NMIC on the other hand, the Court of
of another corporation and their interlocking directorates may serve as Appeals invoked Sibagat Timber Corporation v. Garcia,77 which it
indicia of control, by themselves and without more, however, these described as "a case under a similar factual milieu."78 However, in
circumstances are insufficient to establish an alter ego relationship or Sibagat Timber Corporation, this Court took care to enumerate the
connection between DBP and PNB on the one hand and NMIC on the circumstances which led to the piercing of the corporate veil of Sibagat
other hand, that will justify the puncturing of the latter’s corporate Timber Corporation for being the alter ego of Del Rosario & Sons
cover. This Court has declared that "mere ownership by a single Logging Enterprises, Inc. Those circumstances were as follows:
stockholder or by another corporation of all or nearly all of the capital holding office in the same building, practical identity of the officers and
stock of a corporation is not of itself sufficient ground for disregarding directors of the two corporations and assumption of management and
the separate corporate personality."66 This Court has likewise ruled that control of Sibagat Timber Corporation by the directors/officers of Del
the "existence of interlocking directors, corporate officers and Rosario & Sons Logging Enterprises, Inc.
shareholders is not enough justification to pierce the veil of corporate
fiction in the absence of fraud or other public policy considerations." 67
Here, DBP and PNB maintain an address different from that of
NMIC.79 As already discussed, there was insufficient proof of
True, the findings of fact of the Court of Appeals are conclusive and interlocking directorates. There was not even an allegation of similarity
cannot be reviewed on appeal to this Court, provided they are borne of corporate officers. Instead of evidence that DBP and PNB assumed
out of the record or are based on substantial evidence.68 It is equally and controlled the management of NMIC, HRCC’s evidence shows that
true that the question of whether one corporation is merely an alter ego NMIC operated as a distinct entity endowed with its own legal
of another is purely one of fact. So is the question of whether a personality. Thus, what obtains in this case is a factual backdrop
corporation is a paper company, a sham or subterfuge or whether the different from, not similar to, Sibagat Timber Corporation.
requisite quantum of evidence has been adduced warranting the
piercing of the veil of corporate personality.69 Nevertheless, it has been
In relation to the second element, to disregard the separate juridical
held in Sarona v. National Labor Relations Commission70 that this
personality of a corporation, the wrongdoing or unjust act in
Court has the power to resolve a question of fact, such as whether a
contravention of a plaintiff’s legal rights must be clearly and
corporation is a mere alter ego of another entity or whether the
convincingly established; it cannot be presumed. Without a
corporate fiction was invoked for fraudulent or malevolent ends, if the
demonstration that any of the evils sought to be prevented by the
findings in the assailed decision are either not supported by the
doctrine is present, it does not apply.80
evidence on record or based on a misapprehension of facts.

In this case, the Court of Appeals declared:


In this case, nothing in the records shows that the corporate finances,
policies and practices of NMIC were dominated by DBP and PNB in
such a way that NMIC could be considered to have no separate mind, We are not saying that PNB and DBP are guilty of fraud in forming
will or existence of its own but a mere conduit for DBP and PNB. On NMIC, nor are we implying that NMIC was used to conceal fraud. x x
the contrary, the evidence establishes that HRCC knew and acted on x.81
the knowledge that it was dealing with NMIC, not with NMIC’s
stockholders. The letter proposal of Hercon, Inc., HRCC’s
predecessor-in-interest, regarding the contract for NMIC’s mine Such a declaration clearly negates the possibility that DBP and PNB
stripping and road construction program was addressed to and exercised control over NMIC which DBP and PNB used "to commit
fraud or wrong, to perpetuate the violation of a statutory or other
accepted by NMIC.71 The various billing reports, progress reports,
statements of accounts and communications of Hercon, Inc./HRCC positive legal duty, or dishonest and unjust act in contravention of
regarding NMIC’s mine stripping and road construction program in plaintiff’s legal rights." It is a recognition that, even assuming that DBP
and PNB exercised control over NMIC, there is no evidence that the
1985 concerned NMIC and NMIC’s officers, without any indication of or
reference to the control exercised by DBP and/or PNB over NMIC’s juridical personality of NMIC was used by DBP and PNB to commit a
affairs, policies and practices.72 fraud or to do a wrong against HRCC.

HRCC has presented nothing to show that DBP and PNB had a hand There being a total absence of evidence pointing to a fraudulent, illegal
in the act complained of, the alleged undue disregard by NMIC of the or unfair act committed against HRCC by DBP and PNB under the
guise of NMIC, there is no basis to hold that NMIC was a mere alter
demands of HRCC to satisfy the unpaid claims for services rendered
by HRCC in connection with NMIC’s mine stripping and road ego of DBP and PNB. As this Court ruled in Ramoso v. Court of
construction program in 1985. On the contrary, the overall picture Appeals82:
painted by the evidence offered by HRCC is one where HRCC was
dealing with NMIC as a distinct juridical person acting through its own As a general rule, a corporation will be looked upon as a legal entity,
corporate officers.73 unless and until sufficient reason to the contrary appears. When the
notion of legal entity is used to defeat public convenience, justify
Moreover, the finding that the respective boards of directors of NMIC, wrong, protect fraud, or defend crime, the law will regard the
DBP, and PNB were interlocking has no basis. HRCC’s Exhibit "I- corporation as an association of persons. Also, the corporate entity
may be disregarded in the interest of justice in such cases as fraud that
5,"74 the initial General Information Sheet submitted by NMIC to the
Securities and Exchange Commission, relied upon by the trial court may work inequities among members of the corporation internally,
and the Court of Appeals may have proven that DBP and PNB owned involving no rights of the public or third persons. In both instances,
there must have been fraud, and proof of it. For the separate juridical
the stocks of NMIC to the extent of 57% and 43%, respectively.
However, nothing in it supports a finding that NMIC, DBP, and PNB personality of a corporation to be disregarded, the wrongdoing must be
had interlocking directors as it only indicates that, of the five members clearly and convincingly established. It cannot be presumed.

10
As regards the third element, in the absence of both control by DBP
and PNB of NMIC and fraud or fundamental unfairness perpetuated by
DBP and PNB through the corporate cover of NMIC, no harm could be
said to have been proximately caused by DBP and PNB on HRCC for
which HRCC could hold DBP and PNB solidarily liable with
NMIC.1âwphi1

Considering that, under the deeds of transfer executed by DBP and


PNB, the liability of the APT as transferee of the rights, titles and
interests of DBP and PNB in NMIC will attach only if DBP and PNB are
held liable, the APT incurs no liability for the judgment indebtedness of
NMIC. Even HRCC recognizes that "as assignee of DBP and PNB 's
loan receivables," the APT simply "stepped into the shoes of DBP and
PNB with respect to the latter's rights and obligations" in NMIC. 83 As
such assignee, therefore, the APT incurs no liability with respect to
NMIC other than whatever liabilities may be imputable to its assignors,
DBP and PNB.

Even under Section 2.02 of the respective deeds of transfer executed


by DBP and PNB which HRCC invokes, the APT cannot be held liable.
The contingent liability for which the National Government, through the
APT, may be held liable under the said provision refers to contingent
liabilities of DBP and PNB. Since DBP and PNB may not be held
solidarily liable with NMIC, no contingent liability may be imputed to the
APT as well. Only NMIC as a distinct and separate legal entity is liable
to pay its corporate obligation to HRCC in the amount of
₱8,370,934.74, with legal interest thereon from date of demand.

As trustee of the. assets of NMIC, however, the APT should ensure


compliance by NMIC of the judgment against it. The APT itself
acknowledges this.84

WHEREFORE, the petitions are hereby GRANTED.

The complaint as against Development Bank of the Philippines, the


Philippine National Bank, and the Asset Privatization Trust, now the
Privatization and Management Office, is DISMISSED for lack of merit.
The Asset Privatization Trust, now the Privatization and Management
Office, as trustee of Nonoc Mining and Industrial Corporation, now the
Philnico Processing Corporation, is DIRECTED to ensure compliance
by the Nonoc Mining and Industrial Corporation, now the Philnico
Processing Corporation, with this Decision.

SO ORDERED.

102
G.R. No. 156759 June 5, 2013 personally upon said defendant were made, but the same
were ineffectual and unavailing on the ground that per
information of Ms. Quijano said defendant is always out and
ALLEN A. MACASAET, NICOLAS V. QUIJANO, JR., ISAIAS
not available, thus, substituted service was applied;
ALBANO, LILY REYES, JANET BAY, JESUS R. GALANG, AND
RANDY HAGOS, Petitioners,
vs. 2. Defendant Nicolas V. Quijano, at the same address, thru
FRANCISCO R. CO, JR., Respondent. his wife Lu-Ann Quijano, who signed to acknowledge receipt
thereof. That effort (sic) to serve the said summons
personally upon said defendant were made, but the same
DECISION
were ineffectual and unavailing on the ground that per
information of (sic) his wife said defendant is always out and
BERSAMIN, J.: not available, thus, substituted service was applied;

To warrant the substituted service of the summons and copy of the 3. Defendants Isaias Albano, Janet Bay, Jesus R. Galang,
complaint, the serving officer must first attempt to effect the same upon Randy Hagos and Lily Reyes, at the same address, thru
the defendant in person. Only after the attempt at personal service has Rene Esleta, Editorial Assistant of defendant AbanteTonite,
become futile or impossible within a reasonable time may the officer a person of sufficient age and discretion working therein who
resort to substituted service. signed to acknowledge receipt thereof. That effort (sic) to
serve the said summons personally upon said defendants
were made, but the same were ineffectual and unavailing on
The Case the ground that per information of (sic) Mr. Esleta said
defendants is (sic) always roving outside and gathering
Petitioners – defendants in a suit for libel brought by respondent – news, thus, substituted service was applied.
appeal the decision promulgated on March 8, 20021 and the resolution
promulgated on January 13, 2003,2 whereby the Court of Appeals (CA) Original copy of summons is therefore, respectfully returned duly
respectively dismissed their petition for certiorari, prohibition and
served.
mandamus and denied their motion for reconsideration. Thereby, the
CA upheld the order the Regional Trial Court (RTC), Branch 51, in
Manila had issued on March 12, 2001 denying their motion to dismiss Manila, September 22, 2000.
because the substituted service of the summons and copies of the
complaint on each of them had been valid and effective. 3
On October 3, 2000, petitioners moved for the dismissal of the
complaint through counsel’s special appearance in their behalf,
Antecedents alleging lack of jurisdiction over their persons because of the invalid
and ineffectual substituted service of summons. They contended that
the sheriff had made no prior attempt to serve the summons personally
On July 3, 2000, respondent, a retired police officer assigned at the on each of them in accordance with Section 6 and Section 7, Rule 14
Western Police District in Manila, sued Abante Tonite, a daily tabloid of of the Rules of Court. They further moved to drop Abante Tonite as a
general circulation; its Publisher Allen A. Macasaet; its Managing
defendant by virtue of its being neither a natural nor a juridical person
Director Nicolas V. Quijano; its Circulation Manager Isaias Albano; its that could be impleaded as a party in a civil action.
Editors Janet Bay, Jesus R. Galang and Randy Hagos; and its
Columnist/Reporter Lily Reyes (petitioners), claiming damages
because of an allegedly libelous article petitioners published in the At the hearing of petitioners’ motion to dismiss, Medina testified that he
June 6, 2000 issue of Abante Tonite. The suit, docketed as Civil Case had gone to the office address of petitioners in the morning of
No. 00-97907, was raffled to Branch 51 of the RTC, which in due September 18, 2000 to personally serve the summons on each
course issued summons to be served on each defendant, including defendant; that petitioners were out of the office at the time; that he
Abante Tonite, at their business address at Monica Publishing had returned in the afternoon of the same day to again attempt to
Corporation, 301-305 3rd Floor, BF Condominium Building, Solana serve on each defendant personally but his attempt had still proved
Street corner A. Soriano Street, Intramuros, Manila.4 futile because all of petitioners were still out of the office; that some
competent persons working in petitioners’ office had informed him that
Macasaet and Quijano were always out and unavailable, and that
In the morning of September 18, 2000, RTC Sheriff Raul Medina
Albano, Bay, Galang, Hagos and Reyes were always out roving to
proceeded to the stated address to effect the personal service of the gather news; and that he had then resorted to substituted service upon
summons on the defendants. But his efforts to personally serve each realizing the impossibility of his finding petitioners in person within a
defendant in the address were futile because the defendants were then
reasonable time.
out of the office and unavailable. He returned in the afternoon of that
day to make a second attempt at serving the summons, but he was
informed that petitioners were still out of the office. He decided to On March 12, 2001, the RTC denied the motion to dismiss, and
resort to substituted service of the summons, and explained why in his directed petitioners to file their answers to the complaint within the
sheriff’s return dated September 22, 2005,5 to wit: remaining period allowed by the Rules of Court,6 relevantly stating:

SHERIFF’S RETURN Records show that the summonses were served upon Allen A.
Macasaet, President/Publisher of defendant AbanteTonite, through
LuAnn Quijano; upon defendants Isaias Albano, Janet Bay, Jesus R.
This is to certify that on September 18, 2000, I caused the service of
Galang, Randy Hagos and Lily Reyes, through Rene Esleta, Editorial
summons together with copies of complaint and its annexes attached Assistant of defendant Abante Tonite (p. 12, records). It is apparent in
thereto, upon the following: the Sheriff’s Return that on several occasions, efforts to served (sic)
the summons personally upon all the defendants were ineffectual as
1. Defendant Allen A. Macasaet, President/Publisher of they were always out and unavailable, so the Sheriff served the
defendant AbanteTonite, at Monica Publishing Corporation, summons by substituted service.
Rooms 301-305 3rd Floor, BF Condominium Building,
Solana corner A. Soriano Streets, Intramuros, Manila, thru Considering that summonses cannot be served within a reasonable
his secretary Lu-Ann Quijano, a person of sufficient age and time to the persons of all the defendants, hence substituted service of
discretion working therein, who signed to acknowledge
summonses was validly applied. Secretary of the President who is duly
receipt thereof. That effort (sic) to serve the said summons
103
authorized to receive such document, the wife of the defendant and the We find petitioners’ argument without merit. The rule is that certiorari
Editorial Assistant of the defendant, were considered competent will prosper only if there is a showing of grave abuse of discretion or an
persons with sufficient discretion to realize the importance of the legal act without or in excess of jurisdiction committed by the respondent
papers served upon them and to relay the same to the defendants Judge. A judicious reading of the questioned orders of respondent
named therein (Sec. 7, Rule 14, 1997 Rules of Civil Procedure). Judge would show that the same were not issued in a capricious or
whimsical exercise of judgment. There are factual bases and legal
justification for the assailed orders. From the Return, the sheriff
WHEREFORE, in view of the foregoing, the Motion to Dismiss is
certified that "effort to serve the summons personally xxx were made,
hereby DENIED for lack of merit..
but the same were ineffectual and unavailing xxx.

Accordingly, defendants are directed to file their Answers to the


and upholding the trial court’s finding that there was a substantial
complaint within the period still open to them, pursuant to the rules.
compliance with the rules that allowed the substituted service.

SO ORDERED.
Furthermore, the CA ruled:

Petitioners filed a motion for reconsideration, asserting that the sheriff


Anent the issue raised by petitioners that "Abante Tonite is neither a
had immediately resorted to substituted service of the summons upon
natural or juridical person who may be a party in a civil case," and
being informed that they were not around to personally receive the
therefore the case against it must be dismissed and/or dropped, is
summons, and that Abante Tonite, being neither a natural nor a
untenable.
juridical person, could not be made a party in the action.

The respondent Judge, in denying petitioners’ motion for


On June 29, 2001, the RTC denied petitioners’ motion for
reconsideration, held that:
reconsideration.7 It stated in respect of the service of summons, as
follows:
xxxx
The allegations of the defendants that the Sheriff immediately resorted
to substituted service of summons upon them when he was informed Abante Tonite’s newspapers are circulated nationwide, showing
that they were not around to personally receive the same is untenable. ostensibly its being a corporate entity, thus the doctrine of corporation
During the hearing of the herein motion, Sheriff Raul Medina of this by estoppel may appropriately apply.
Branch of the Court testified that on September 18, 2000 in the
morning, he went to the office address of the defendants to personally
An unincorporated association, which represents itself to be a
serve summons upon them but they were out. So he went back to
corporation, will be estopped from denying its corporate capacity in a
serve said summons upon the defendants in the afternoon of the same
suit against it by a third person who relies in good faith on such
day, but then again he was informed that the defendants were out and
representation.
unavailable, and that they were always out because they were roving
around to gather news. Because of that information and because of the
nature of the work of the defendants that they are always on field, so There being no grave abuse of discretion committed by the respondent
the sheriff resorted to substituted service of summons. There was Judge in the exercise of his jurisdiction, the relief of prohibition is also
substantial compliance with the rules, considering the difficulty to serve unavailable.
the summons personally to them because of the nature of their job
which compels them to be always out and unavailable. Additional
matters regarding the service of summons upon defendants were WHEREFORE, the instant petition is DENIED. The assailed Orders of
sufficiently discussed in the Order of this Court dated March 12, 2001. respondent Judge are AFFIRMED.

Regarding the impleading of Abante Tonite as defendant, the RTC SO ORDERED.9


held, viz:
On January 13, 2003, the CA denied petitioners’ motion for
"Abante Tonite" is a daily tabloid of general circulation. People all over reconsideration.10
the country could buy a copy of "Abante Tonite" and read it, hence, it is
for public consumption. The persons who organized said publication Issues
obviously derived profit from it. The information written on the said
newspaper will affect the person, natural as well as juridical, who was
stated or implicated in the news. All of these facts imply that "Abante Petitioners hereby submit that:
Tonite" falls within the provision of Art. 44 (2 or 3), New Civil Code.
Assuming arguendo that "Abante Tonite" is not registered with the 1. THE COURT OF APPEALS COMMITTED AN ERROR OF
Securities and Exchange Commission, it is deemed a corporation by LAW IN HOLDING THAT THE TRIAL COURT ACQUIRED
estoppels considering that it possesses attributes of a juridical person, JURISDICTION OVER HEREIN PETITIONERS.
otherwise it cannot be held liable for damages and injuries it may inflict
to other persons.
2. THE COURT OF APPEALS COMMITTED REVERSIBLE
ERROR BY SUSTAINING THE INCLUSION OF ABANTE
Undaunted, petitioners brought a petition for certiorari, prohibition, TONITE AS PARTY IN THE INSTANT CASE.11
mandamusin the CA to nullify the orders of the RTC dated March 12,
2001 and June 29, 2001.
Ruling

Ruling of the CA
The petition for review lacks merit.

On March 8, 2002, the CA promulgated its questioned


decision,8 dismissing the petition for certiorari, prohibition, mandamus, Jurisdiction over the person, or jurisdiction in personam –the power of
to wit: the court to render a personal judgment or to subject the parties in a
particular action to the judgment and other rulings rendered in the

104
action – is an element of due process that is essential in all actions, As the initiating party, the plaintiff in a civil action voluntarily submits
civil as well as criminal, except in actions in rem or quasi in rem. himself to the jurisdiction of the court by the act of filing the initiatory
Jurisdiction over the defendantin an action in rem or quasi in rem is not pleading. As to the defendant, the court acquires jurisdiction over his
required, and the court acquires jurisdiction over an actionas long as it person either by the proper service of the summons, or by a voluntary
acquires jurisdiction over the resthat is thesubject matter of the action. appearance in the action.15
The purpose of summons in such action is not the acquisition of
jurisdiction over the defendant but mainly to satisfy the constitutional
Upon the filing of the complaint and the payment of the requisite legal
requirement of due process.12
fees, the clerk of court forthwith issues the corresponding summons to
the defendant.16 The summons is directed to the defendant and signed
The distinctions that need to be perceived between an action in by the clerk of court under seal. It contains the name of the court and
personam, on the one hand, and an action inrem or quasi in rem, on the names of the parties to the action; a direction that the defendant
the other hand, are aptly delineated in Domagas v. Jensen, 13 thusly: answers within the time fixed by the Rules of Court; and a notice that
unless the defendant so answers, the plaintiff will take judgment by
default and may be granted the relief applied for.17 To be attached to
The settled rule is that the aim and object of an action determine its
the original copy of the summons and all copies thereof is a copy of the
character. Whether a proceeding is in rem, or in personam, or quasi in
complaint (and its attachments, if any) and the order, if any, for the
rem for that matter, is determined by its nature and purpose, and by
appointment of a guardian ad litem.18
these only. A proceeding in personam is a proceeding to enforce
personal rights and obligations brought against the person and is
based on the jurisdiction of the person, although it may involve his right The significance of the proper service of the summons on the
to, or the exercise of ownership of, specific property, or seek to compel defendant in an action in personam cannot be overemphasized. The
him to control or dispose of it in accordance with the mandate of the service of the summons fulfills two fundamental objectives, namely: (a)
court. The purpose of a proceeding in personam is to impose, through to vest in the court jurisdiction over the person of the defendant; and
the judgment of a court, some responsibility or liability directly upon the (b) to afford to the defendant the opportunity to be heard on the claim
person of the defendant. Of this character are suits to compel a brought against him.19 As to the former, when jurisdiction in personam
defendant to specifically perform some act or actions to fasten a is not acquired in a civil action through the proper service of the
pecuniary liability on him. An action in personam is said to be one summons or upon a valid waiver of such proper service, the ensuing
which has for its object a judgment against the person, as trial and judgment are void.20 If the defendant knowingly does an act
distinguished from a judgment against the property to determine its inconsistent with the right to object to the lack of personal jurisdiction
state. It has been held that an action in personam is a proceeding to as to him, like voluntarily appearing in the action, he is deemed to have
enforce personal rights or obligations; such action is brought against submitted himself to the jurisdiction of the court.21 As to the latter, the
the person. As far as suits for injunctive relief are concerned, it is well- essence of due process lies in the reasonable opportunity to be heard
settled that it is an injunctive act in personam. In Combs v. Combs, the and to submit any evidence the defendant may have in support of his
appellate court held that proceedings to enforce personal rights and defense. With the proper service of the summons being intended to
obligations and in which personal judgments are rendered adjusting afford to him the opportunity to be heard on the claim against him, he
the rights and obligations between the affected parties is in personam. may also waive the process.21 In other words, compliance with the
Actions for recovery of real property are in personam. rules regarding the service of the summons is as much an issue of due
process as it is of jurisdiction.23
On the other hand, a proceeding quasi in rem is one brought against
persons seeking to subject the property of such persons to the Under the Rules of Court, the service of the summons should firstly be
discharge of the claims assailed. In an action quasi in rem, an effected on the defendant himself whenever practicable. Such personal
individual is named as defendant and the purpose of the proceeding is service consists either in handing a copy of the summons to the
to subject his interests therein to the obligation or loan burdening the defendant in person, or, if the defendant refuses to receive and sign for
property. Actions quasi in rem deal with the status, ownership or it, in tendering it to him.24 The rule on personal service is to be rigidly
liability of a particular property but which are intended to operate on enforced in order to ensure the realization of the two fundamental
these questions only as between the particular parties to the objectives earlier mentioned. If, for justifiable reasons, the defendant
proceedings and not to ascertain or cut off the rights or interests of all cannot be served in person within a reasonable time, the service of the
possible claimants. The judgments therein are binding only upon the summons may then be effected either (a) by leaving a copy of the
parties who joined in the action. summons at his residence with some person of suitable age and
discretion then residing therein, or (b) by leaving the copy at his office
or regular place of business with some competent person in charge
As a rule, Philippine courts cannot try any case against a defendant
thereof.25 The latter mode of service is known as substituted service
who does not reside and is not found in the Philippines because of the
because the service of the summons on the defendant is made through
impossibility of acquiring jurisdiction over his person unless he
his substitute.
voluntarily appears in court; but when the case is an action in rem or
quasi in rem enumerated in Section 15, Rule 14 of the Rules of Court,
Philippine courts have jurisdiction to hear and decide the case because It is no longer debatable that the statutory requirements of substituted
they have jurisdiction over the res, and jurisdiction over the person of service must be followed strictly, faithfully and fully, and any substituted
the non-resident defendant is not essential. In the latter instance, service other than that authorized by statute is considered
extraterritorial service of summons can be made upon the defendant, ineffective.26 This is because substituted service, being in derogation of
and such extraterritorial service of summons is not for the purpose of the usual method of service, is extraordinary in character and may be
vesting the court with jurisdiction, but for the purpose of complying with used only as prescribed and in the circumstances authorized by
the requirements of fair play or due process, so that the defendant will statute.27 Only when the defendant cannot be served personally within
be informed of the pendency of the action against him and the a reasonable time may substituted service be resorted to. Hence, the
possibility that property in the Philippines belonging to him or in which impossibility of prompt personal service should be shown by stating the
he has an interest may be subjected to a judgment in favor of the efforts made to find the defendant himself and the fact that such efforts
plaintiff, and he can thereby take steps to protect his interest if he is so failed, which statement should be found in the proof of service or
minded. On the other hand, when the defendant in an action in sheriff’s return.28 Nonetheless, the requisite showing of the
personam does not reside and is not found in the Philippines, our impossibility of prompt personal service as basis for resorting to
courts cannot try the case against him because of the impossibility of substituted service may be waived by the defendant either expressly or
acquiring jurisdiction over his person unless he voluntarily appears in impliedly.29
court.14
There is no question that Sheriff Medina twice attempted to serve the
summons upon each of petitioners in person at their office address, the

105
first in the morning of September 18, 2000 and the second in the
afternoon of the same date. Each attempt failed because Macasaet
and Quijano were "always out and not available" and the other
petitioners were "always roving outside and gathering news." After
Medina learned from those present in the office address on his second
attempt that there was no likelihood of any of petitioners going to the
office during the business hours of that or any other day, he concluded
that further attempts to serve them in person within a reasonable time
would be futile. The circumstances fully warranted his conclusion. He
was not expected or required as the serving officer to effect personal
service by all means and at all times, considering that he was
expressly authorized to resort to substituted service should he be
unable to effect the personal service within a reasonable time. In that
regard, what was a reasonable time was dependent on the
circumstances obtaining. While we are strict in insisting on personal
service on the defendant, we do not cling to such strictness should the
circumstances already justify substituted service instead. It is the spirit
of the procedural rules, not their letter, that governs. 30

In reality, petitioners’ insistence on personal service by the serving


officer was demonstrably superfluous. They had actually received the
summonses served through their substitutes, as borne out by their
filing of several pleadings in the RTC, including an answer with
compulsory counterclaim ad cautelam and a pre-trial brief ad cautelam.
They had also availed themselves of the modes of discovery available
under the Rules of Court. Such acts evinced their voluntary
appearance in the action.

Nor can we sustain petitioners’ contention that Abante Tonite could not
be sued as a defendant due to its not being either a natural or a
juridical person. In rejecting their contention, the CA categorized
Abante Tonite as a corporation by estoppel as the result of its having
represented itself to the reading public as a corporation despite its not
being incorporated. Thereby, the CA concluded that the RTC did not
gravely abuse its discretion in holding that the non-incorporation of
Abante Tonite with the Securities and Exchange Commission was of
no consequence, for, otherwise, whoever of the public who would
suffer any damage from the publication of articles in the pages of its
tabloids would be left without recourse. We cannot disagree with the
CA, considering that the editorial box of the daily tabloid disclosed that
basis, nothing in the box indicated that Monica Publishing Corporation
had owned Abante Tonite.

WHEREFORE, the Court AFFIRMS the decision promulgated on


March 8, 2002; and ORDERS petitioners to pay the costs of suit.

SO ORDERED.

106
G.R. No. 181416 November 11, 2013 delinquent, plaintiff was also barred from exercising his right
to vote in the election of new members of the Board of
Directors x x x;
MEDICAL PLAZA MAKATI CONDOMINIUM
CORPORATION, Petitioner,
vs. 9. x x x Again, prior to the said election date, x x x counsel
ROBERT H. CULLEN, Respondent. for the defendant [MPMCC] sent a demand letter to plaintiff,
anent the said delinquency, explaining that the said unpaid
amount is a carry-over from the obligation of defendant
DECISION
Meridien. x x x;

PERALTA, J.:
10. Verification with the defendant [MPMCC] resulted to the
issuance of a certification stating that Condominium Unit
This is a petition for review on certiorari under Rule 45 of the Rules of 1201 has an outstanding unpaid obligation in the total
Court assailing the Court of Appeals (CA) Decision1 dated July 10, amount of ₱145,567.42 as of November 30, 2002, which
2007 and Resolution2 dated January 25, 2008 in CA-G.R. CV No. again, was attributed by defendant [MPMCC] to defendant
86614. The assailed decision reversed and set aside the September 9, Meridien. x x x;
2005 Order3 of the Regional Trial Court (RTC) of Makati, Branch 58 in
Civil Case No. 03-1018; while the assailed resolution denied the
11. Due to the seriousness of the matter, and the feeling that
separate motions for reconsideration filed by petitioner Medical Plaza
defendant Meridien made false representations considering
Makati Condominium Corporation (MPMCC) and Meridien Land
that it fully warranted to plaintiff that condominium unit 1201
Holding, Inc. (MLHI).
is free and clear from all liens and encumbrances, the matter
was referred to counsel, who accordingly sent a letter to
The factual and procedural antecedents are as follows: defendant Meridien, to demand for the payment of said
unpaid association dues and other assessments imposed on
the condominium unit and being claimed by defendant
Respondent Robert H. Cullen purchased from MLHI condominium Unit
[MPMCC]. x x x;
No. 1201 of the Medical Plaza Makati covered by Condominium
Certificate of Title No. 45808 of the Register of Deeds of Makati. Said
title was later cancelled and Condominium Certificate of Title No. 12. x x x defendant Meridien claimed however, that the
64218 was issued in the name of respondent. obligation does not exist considering that the matter was
already settled and paid by defendant Meridien to defendant
[MPMCC]. x x x;
On September 19, 2002, petitioner, through its corporate secretary, Dr.
Jose Giovanni E. Dimayuga, demanded from respondent payment for
alleged unpaid association dues and assessments amounting to 13. Plaintiff thus caused to be sent a letter to defendant
₱145,567.42. Respondent disputed this demand claiming that he had [MPMCC] x x x. The said letter x x x sought an explanation
been religiously paying his dues shown by the fact that he was on the fact that, as per the letter of defendant Meridien, the
previously elected president and director of petitioner.4 Petitioner, on delinquency of unit 1201 was already fully paid and settled,
the other hand, claimed that respondent’s obligation was a carry-over contrary to the claim of defendant [MPMCC]. x x x;
of that of MLHI.5 Consequently, respondent was prevented from
exercising his right to vote and be voted for during the 2002 election of
14. Despite receipt of said letter on April 24, 2003, and to
petitioner’s Board of Directors.6 Respondent thus clarified from MLHI
date however, no explanation was given by defendant
the veracity of petitioner’s claim, but MLHI allegedly claimed that the
[MPMCC], to the damage and prejudice of plaintiff who is
same had already been settled.7 This prompted respondent to demand
again obviously being barred from voting/participating in the
from petitioner an explanation why he was considered a delinquent
election of members of the board of directors for the year
payer despite the settlement of the obligation. Petitioner failed to make
2003;
such explanation. Hence, the Complaint for Damages 8 filed by
respondent against petitioner and MLHI, the pertinent portions of which
read: 15. Clearly, defendant [MPMCC] acted maliciously by
insisting that plaintiff is a delinquent member when in fact,
defendant Meridien had already paid the said delinquency, if
xxxx
any. The branding of plaintiff as delinquent member was
willfully and deceitfully employed so as to prevent plaintiff
6. Thereafter, plaintiff occupied the said condominium unit from exercising his right to vote or be voted as director of the
no. 1201 and religiously paid all the corresponding monthly condominium corporation; 16. Defendant [MPMCC]’s
contributions/association dues and other assessments ominous silence when confronted with claim of payment
imposed on the same. For the years 2000 and 2001, plaintiff made by defendant Meridien is tantamount to admission that
served as President and Director of the Medical Plaza indeed, plaintiff is not really a delinquent member;
Makati Condominium Corporation;
17. Accordingly, as a direct and proximate result of the said
7. Nonetheless, on September 19, 2002, plaintiff was acts of defendant [MPMCC], plaintiff experienced/suffered
shocked/surprised to receive a letter from the incumbent from mental anguish, moral shock, and serious anxiety.
Corporate Secretary of the defendant Medical Plaza Makati, Plaintiff, being a doctor of medicine and respected in the
demanding payment of alleged unpaid association dues and community further suffered from social humiliation and
assessments arising from plaintiff’s condominium unit no. besmirched reputation thereby warranting the grant of moral
1201. The said letter further stressed that plaintiff is damages in the amount of ₱500,000.00 and for which
considered a delinquent member of the defendant Medical defendant [MPMCC] should be held liable;
Plaza Makati.
18. By way of example or correction for the public good, and
x x x; as a stern warning to all similarly situated, defendant
[MPMCC] should be ordered to pay plaintiff exemplary
damages in the amount of ₱200,000.00;
8. As a consequence, plaintiff was not allowed to file his
certificate of candidacy as director. Being considered a
107
19. As a consequence, and so as to protect his rights and The petition is meritorious.
interests, plaintiff was constrained to hire the services of
counsel, for an acceptance fee of ₱100,000.00 plus
It is a settled rule that jurisdiction over the subject matter is determined
₱2,500.00 per every court hearing attended by counsel;
by the allegations in the complaint. It is not affected by the pleas or the
theories set up by the defendant in an answer or a motion to dismiss.
20. In the event that the claim of defendant [MPMCC] turned Otherwise, jurisdiction would become dependent almost entirely upon
out to be true, however, the herein defendant Meridien the whims of the defendant.18 Also illuminating is the Court’s
should be held liable instead, by ordering the same to pay pronouncement in Go v. Distinction Properties Development and
the said delinquency of condominium unit 1201 in the Construction, Inc.:19
amount of ₱145,567.42 as of November 30, 2002 as well as
the above damages, considering that the non-payment
Basic as a hornbook principle is that jurisdiction over the subject matter
thereof would be the proximate cause of the damages
of a case is conferred by law and determined by the allegations in the
suffered by plaintiff;9
complaint which comprise a concise statement of the ultimate facts
constituting the plaintiff’s cause of action. The nature of an action, as
Petitioner and MLHI filed their separate motions to dismiss the well as which court or body has jurisdiction over it, is determined based
complaint on the ground of lack of jurisdiction.10 MLHI claims that it is on the allegations contained in the complaint of the plaintiff,
the Housing and Land Use Regulatory Board (HLURB) which is vested irrespective of whether or not the plaintiff is entitled to recover upon all
with the exclusive jurisdiction to hear and decide the case. Petitioner, or some of the claims asserted therein. The averments in the complaint
on the other hand, raises the following specific grounds for the and the character of the relief sought are the ones to be consulted.
dismissal of the complaint: (1) estoppel as respondent himself Once vested by the allegations in the complaint, jurisdiction also
approved the assessment when he was the president; (2) lack of remains vested irrespective of whether or not the plaintiff is entitled to
jurisdiction as the case involves an intra-corporate controversy; (3) recover upon all or some of the claims asserted therein. x x x20
prematurity for failure of respondent to exhaust all intra-corporate
remedies; and (4) the case is already moot and academic, the
Based on the allegations made by respondent in his complaint, does
obligation having been settled between petitioner and MLHI.11
the controversy involve intra-corporate issues as would fall within the
jurisdiction of the RTC sitting as a special commercial court or an
On September 9, 2005, the RTC rendered a Decision granting ordinary action for damages within the jurisdiction of regular courts?
petitioner’s and MLHI’s motions to dismiss and, consequently,
dismissing respondent’s complaint.
In determining whether a dispute constitutes an intra-corporate
controversy, the Court uses two tests, namely, the relationship test and
The trial court agreed with MLHI that the action for specific the nature of the controversy test.21
performance filed by respondent clearly falls within the exclusive
jurisdiction of the HLURB.12 As to petitioner, the court held that the
An intra-corporate controversy is one which pertains to any of the
complaint states no cause of action, considering that respondent’s
following relationships: (1) between the corporation, partnership or
obligation had already been settled by MLHI. It, likewise, ruled that the
association and the public; (2) between the corporation, partnership or
issues raised are intra-corporate between the corporation and
association and the State insofar as its franchise, permit or license to
member.13
operate is concerned; (3) between the corporation, partnership or
association and its stockholders, partners, members or officers; and (4)
On appeal, the CA reversed and set aside the trial court’s decision and among the stockholders, partners or associates themselves.22 Thus,
remanded the case to the RTC for further proceedings. Contrary to the under the relationship test, the existence of any of the above intra-
RTC conclusion, the CA held that the controversy is an ordinary civil corporate relations makes the case intra-corporate.23
action for damages which falls within the jurisdiction of regular
courts.14 It explained that the case hinged on petitioner’s refusal to
Under the nature of the controversy test, "the controversy must not
confirm MLHI’s claim that the subject obligation had already been
only be rooted in the existence of an intra-corporate relationship, but
settled as early as 1998 causing damage to respondent.15 Petitioner’s
must as well pertain to the enforcement of the parties’ correlative rights
and MLHI’s motions for reconsideration had also been denied. 16
and obligations under the Corporation Code and the internal and intra-
corporate regulatory rules of the corporation."24 In other words,
Aggrieved, petitioner comes before the Court based on the following jurisdiction should be determined by considering both the relationship
grounds: of the parties as well as the nature of the question involved. 25

I. Applying the two tests, we find and so hold that the case involves intra-
corporate controversy. It obviously arose from the intra-corporate
relations between the parties, and the questions involved pertain to
THE COURT A QUO HAS DECIDED A QUESTION OF SUBSTANCE,
their rights and obligations under the Corporation Code and matters
NOT THERETOFORE DETERMINED BY THE SUPREME COURT,
relating to the regulation of the corporation.26
OR HAS DECIDED IT IN A WAY NOT IN ACCORD WITH LAW OR
WITH THE APPLICABLE DECISIONS OF THE SUPREME COURT
WHEN IT DECLARED THE INSTANT CASE AN ORDINARY ACTION Admittedly, petitioner is a condominium corporation duly organized and
FOR DAMAGES INSTEAD OF AN INTRA-CORPORATE existing under Philippine laws, charged with the management of the
CONTROVERSY COGNIZABLE BY A SPECIAL COMMERCIAL Medical Plaza Makati. Respondent, on the other hand, is the registered
COURT. owner of Unit No. 1201 and is thus a stockholder/member of the
condominium corporation. Clearly, there is an intra-corporate
relationship between the corporation and a stockholder/member.
II.

The nature of the action is determined by the body rather than the title
THE COURT A QUO HAS DECIDED THE INSTANT CASE IN A WAY
of the complaint.1âwphi1 Though denominated as an action for
NOT IN ACCORD WITH LAW OR WITH THE APPLICABLE
damages, an examination of the allegations made by respondent in his
DECISIONS OF THE SUPREME COURT WHEN IT TOOK
complaint shows that the case principally dwells on the propriety of the
COGNIZANCE OF THE APPEAL WHILE RAISING ONLY PURE
assessment made by petitioner against respondent as well as the
QUESTIONS OF LAW.17
validity of petitioner’s act in preventing respondent from participating in
the election of the corporation’s Board of Directors. Respondent

108
contested the alleged unpaid dues and assessments demanded by broad enough to cover a condominium corporation, it does not seem to
petitioner. be the legislative intent. A thorough review of the deliberations of the
bicameral conference committee would show that the lawmakers did
not intend to extend the coverage of the law to such kind of
The issue is not novel. The nature of an action involving any dispute as
association. We quote hereunder the pertinent portion of the Bicameral
to the validity of the assessment of association dues has been settled
Conference Committee’s deliberation, to wit:
by the Court in Chateau de Baie Condominium Corporation v.
Moreno.27 In that case, respondents therein filed a complaint for intra-
corporate dispute against the petitioner therein to question how it THE CHAIRMAN (SEN. ZUBIRI). Let’s go back, Mr. Chair, very quickly
calculated the dues assessed against them, and to ask an accounting on homeowners.
of association dues. Petitioner, however, moved for the dismissal of the
case on the ground of lack of jurisdiction alleging that since the
THE ACTING CHAIRMAN (REP. ZIALCITA). Ang sa akin lang, I think
complaint was against the owner/developer of a condominium whose
our views are similar, Your Honor, Senator Zubiri, the entry of the
condominium project was registered with and licensed by the HLURB,
condominium units might just complicate the whole matters. So we’d
the latter has the exclusive jurisdiction. In sustaining the denial of the
like to put it on record that we’re very much concerned about the plight
motion to dismiss, the Court held that the dispute as to the validity of
of the Condominium Unit Homeowners’ Association. But this could very
the assessments is purely an intra-corporate matter between petitioner
well be addressed on a separate bill that I’m willing to co-sponsor with
and respondent and is thus within the exclusive jurisdiction of the RTC
the distinguished Senator Zubiri, to address in the Condominium Act of
sitting as a special commercial court. More so in this case as
the Philippines, rather than address it here because it might just create
respondent repeatedly questioned his characterization as a delinquent
a red herring into the entire thing and it will just complicate matters,
member and, consequently, petitioner’s decision to bar him from
hindi ba?
exercising his rights to vote and be voted for. These issues are clearly
corporate and the demand for damages is just incidental. Being
corporate in nature, the issues should be threshed out before the RTC THE CHAIRMAN (SEN. ZUBIRI). I also agree with you although I
sitting as a special commercial court. The issues on damages can still sympathize with them---although we sympathize with them and we feel
be resolved in the same special commercial court just like a regular that many times their rights have been also violated by abusive
RTC which is still competent to tackle civil law issues incidental to condominium corporations. However, there are certain things that we
intra-corporate disputes filed before it.28 have to reconcile. There are certain issues that we have to reconcile
with this version.
Moreover, Presidential Decree No. 902-A enumerates the cases over
which the Securities and Exchange Commission (SEC) exercises In the Condominium Code, for example, they just raised a very peculiar
exclusive jurisdiction: situation under the Condominium Code --- Condominium Corporation
Act. It’s five years the proxy, whereas here, it’s three years. So there
would already be violation or there will be already a problem with their
xxxx
version and our version. Sino ang matutupad doon? Will it be our
version or their version?
b) Controversies arising out of intra-corporate or partnership
relations, between and among stockholders, members or
So I agree that has to be studied further. And because they have a law
associates; between any or all of them and the corporation,
pertaining to the condominium housing units, I personally feel that it
partnership or association of which they are stockholders,
would complicate matters if we include them. Although I agree that
members, or associates, respectively; and between such
they should be looked after and their problems be looked into.
corporation, partnership or association and the State insofar
as it concerns their individual franchise or right to exist as
such entity; and Probably we can ask our staff, Your Honor, to come up already with
the bill although we have no more time. Hopefully we can tackle this
again on the 15th Congress. But I agree with the sentiments and the
c) Controversies in the election or appointment of directors,
inputs of the Honorable Chair of the House panel.
trustees, officers, or managers of such corporations,
partnerships, or associations.29
May we ask our resource persons to also probably give comments?
To be sure, this action partakes of the nature of an intra-corporate
controversy, the jurisdiction over which pertains to the SEC. Pursuant Atty. Dayrit.
to Section 5.2 of Republic Act No. 8799, otherwise known as the
Securities Regulation Code, the jurisdiction of the SEC over all cases
MR. DAYRIT.
enumerated under Section 5 of Presidential Decree No. 902-A has
been transferred to RTCs designated by this Court as Special
Commercial Courts.30 While the CA may be correct that the RTC has Yes I agree with you. There are many, I think, practices in their
jurisdiction, the case should have been filed not with the regular court provisions in the Condominium Law that may be conflicting with this
but with the branch of the RTC designated as a special commercial version of ours.
court. Considering that the RTC of Makati City, Branch 58 was not
designated as a special commercial court, it was not vested with
jurisdiction over cases previously cognizable by the SEC. 31 The CA, For instance, in the case of, let’s say, the condominium, the so-called
therefore, gravely erred in remanding the case to the RTC for further common areas and/or maybe so called open spaces that they may
proceedings. have, especially common areas, they are usually owned by the
condominium corporation. Unlike a subdivision where the open spaces
and/or the common areas are not necessarily owned by the
Indeed, Republic Act (RA) No. 9904, or the Magna Carta for association. Because sometimes --- generally these are donated to the
Homeowners and Homeowners’ Associations, approved on January 7, municipality or to the city. And it is only when the city or municipality
2010 and became effective on July 10, 2010, empowers the HLURB to gives the approval or the conformity that this is donated to the
hear and decide inter-association and/or intra-association homeowners’ association. But generally, under PD [Presidential
controversies or conflicts concerning homeowners’ associations. Decree] 957, it’s donated. In the Condominium Corporation, hindi.
However, we cannot apply the same in the present case as it involves Lahat ng mga open spaces and common areas like corridors, the
a controversy between a condominium unit owner and a condominium function rooms and everything, are owned by the corporation. So that’s
corporation. While the term association as defined in the law covers one main issue that can be conflicting.
homeowners’ associations of other residential real property which is

109
THE CHAIRMAN (SEN. ZUBIRI). I’ll just ask for a one-minute owners and the condominium corporation are set forth in the above
suspension so we can talk. Act.

THE ACTING CHAIRMAN (REP. ZIALCITA). Unless you want to put a Clearly, condominium corporations are not covered by the amendment.
catchall phrase like what we did in the Senior Citizen’s Act. Something Thus, the intra-corporate dispute between petitioner and respondent is
like, to the extent --- paano ba iyon? To the extent that it is practicable still within the jurisdiction of the RTC sitting as a special commercial
and applicable, the rights and benefits of the homeowners, are hereby court and not the HLURB. The doctrine laid down by the Court in
extended to the --- mayroon kaming ginamit na phrase eh...to the Chateau de Baie Condominium Corporation v. Moreno35 which in turn
extent that it be practicable and applicable to the unit homeoweners, is cited Wack Wack Condominium Corporation, et al v. CA36 is still a
hereby extended, something like that. It’s a catchall phrase. But then good law.
again, it might create a...
WHEREFORE, we hereby GRANT the petition and REVERSE the
MR. JALANDONI. It will become complicated. There will be a lot of Court of Appeals Decision dated July 10, 2007 and Resolution dated
conflict of laws between the two laws. January 25, 2008 in CA-G.R. CV No. 86614. The Complaint before the
Regional Trial Court of Makati City, Branch 58, which is not a special
commercial court, docketed as Civil Case No. 03-1018 is ordered
THE ACTING CHAIRMAN (REP. ZIALCITA). Kaya nga eh. At saka, I
DISMISSED for lack of jurisdiction. Let the case be REMANDED to the
don’t know. I think the --- mayroon naman silang protection sa ano eh,
Executive Judge of the Regional Trial Court of Makati City for re-raffle
di ba? Buyers decree doon sa Condominium Act. I’m sure there are
purposes among the designated special commercial courts.
provisions there eh. Huwag na lang, huwag na lang.

SO ORDERED.
MR. JALANDONI. Mr. Chairman, I think it would be best if your
previous comments that you’d be supporting an amendment.1âwphi1 I
think that would be --- Well, that would be the best course of action
with all due respect.

THE ACTING CHAIRMAN (REP. ZIALCITA). Yeah. Okay. Thank you.


So iyon na lang final proposal naming ‘yung catchall phrase, "With
respect to the..."32

xxxx

THE CHAIRMAN (SEN. ZUBIRI). xxx And so, what is their final
decision on the definition of homeowners?

THE ACTING CHAIRMAN (REP. ZIALCITA).

We stick to the original, Mr. Chairman. We’ll just open up a whole can
of worms and a whole new ball game will come into play. Besides, I am
not authorized, neither are you, by our counterparts to include the
condominium owners.

THE CHAIRMAN (SEN. ZUBIRI).

Basically that is correct. We are not authorized by the Senate nor –


because we have discussed this lengthily on the floor, actually, several
months on the floor. And we don’t have the authority as well for other
Bicam members to add a provision to include a separate entity that has
already their legal or their established Republic Act tackling on that
particular issue. But we just like to put on record, we sympathize with
the plight of our friends in the condominium associations and we will
just guarantee them that we will work on an amendment to the
Condominium Corporation Code. So with that – we skipped, that is
correct, we have to go back to homeowners’ association definition,
Your Honor, because we had skipped it altogether. So just quickly
going back to Page 7 because there are amendments to the definition
of homeowners. If it is alright with the House Panel, adopt the opening
phrase of Subsection 7 of the Senate version as opening phrase of
Subsection 10 of the reconciled version.

x x x x33

To be sure, RA 4726 or the Condominium Act was enacted to


specifically govern a condominium. Said law sanctions the creation of
the condominium corporation which is especially formed for the
purpose of holding title to the common area, in which the holders of
separate interests shall automatically be members or shareholders, to
the exclusion of others, in proportion to the appurtenant interest of their
respective units.34 The rights and obligations of the condominium unit

10

Vous aimerez peut-être aussi